You are on page 1of 241

‫‪BAC‬‬ ‫‪2020‬‬ ‫‪2020‬‬

‫مواضيع البكالوريا من ‪ 2019‬إلى ‪ + 2008‬الحلول النموذجية المفصلة‬

‫مادة العلوم الفيزيائية‬


‫شعبة‪ :‬علوم تجريبية‬
‫مالحظة ‪:‬‬
‫تم وضع هذه الباقة ألخذ المعلومات و األفكار الطازجة ‪ ،‬فـاغتنموا‬
‫الفرصة أيها التالميذ الشرفاء ‪...‬‬
‫التالميذ النظاميين ‪ :‬خذوا األفكار من الوحدات حسب وتيرة الدروس‬
‫في القسم ‪ ،‬و ال تستعجلوا عن بقية الوحدات األخرى ‪.‬‬
‫التالميذ األحرار ‪ :‬ال مشكلة في تفحص كل المواضيع و االستفادة منها ‪.‬‬

‫‪ -‬ال تنسوا أن‪ :‬تعب المراجعة أفضل من ألم السقوط‬

‫من تجميع و تنظيم ‪ :‬عقبة بن نافع‬


‫ترتيب المواضيع و حلولها النموذجية المفصلة‬
‫‪ ‬امتحان بكالوريا ‪ { 2019‬األستاذ طواهرية عبد العزيز }‬
‫‪ ‬امتحان بكالوريا ‪ { 2018‬األستاذ قزوري عبد القادر }‬
‫‪ ‬امتحان بكالوريا ‪ { 2017‬األستاذ قزوري عبد القادر }‬
‫‪ ‬امتحان بكالوريا ‪ 2017‬دورة ‪ { 2‬تصحيح نموذجي وزاري }‬
‫‪ ‬امتحان بكالوريا ‪ { 2016‬األستاذ قزوري عبد القادر }‬
‫‪ ‬امتحان بكالوريا ‪ 2016‬دورة ‪ { 2‬األستاذ قزوري عبد القادر}‬
‫‪ ‬امتحان بكالوريا ‪ { 2015‬األستاذ قزوري عبد القادر }‬
‫‪ ‬امتحان بكالوريا ‪ { 2014‬األستاذ قزوري عبد القادر }‬
‫‪ ‬امتحان بكالوريا ‪ { 2013‬األستاذ قزوري عبد القادر }‬
‫‪ ‬امتحان بكالوريا ‪ { 2012‬األستاذ فرقاني فارس }‬
‫‪ ‬امتحان بكالوريا ‪ { 2011‬األستاذ قزوري عبد القادر }‬
‫‪ ‬امتحان بكالوريا ‪ { 2010‬األستاذ فرقاني فارس }‬
‫‪ ‬امتحان بكالوريا ‪ { 2009‬األستاذ قزوري عبد القادر }‬
‫‪ ‬امتحان بكالوريا ‪ { 2008‬األستاذ قزوري عبد القادر }‬
‫⋑‬
‫اجلمهورية اجلزائرية الدميقراطية الشعبية‬
‫وزارة الرتبية الوطنية‬
‫الديوان الوطين لالمتحاانت واملسابقات‬
‫دورة‪2019 :‬‬ ‫امتحان بكالوراي التعليم الثانوي‬
‫الشعبة‪ :‬علوم جتريبية‬
‫املدة‪ 03 :‬سا و‪ 03‬د‬ ‫اختبار يف مادة‪ :‬العلوم الفيزايئية‬
‫على المترشح أن يختار أحد الموضوعين اآلتيين‪:‬‬
‫الموضوع األول‬
‫يحتوي الموضوع األول على ‪ 40‬صفحات (من الصفحة ‪ 1‬من ‪ 8‬إلى الصفحة ‪ 0‬من ‪)8‬‬
‫التمرين األول‪ 40( :‬نقاط)‬
‫هل تعلم؟ في ‪ 72‬أكتوبر ‪ ،1998‬قتلت الصاعقة فريق كرة‬
‫قدم بأكمله في جمهورية الكونغو الديمقراطية‪.‬‬
‫أثناء العاصفة الرعدية‪ ،‬تسبب التيارات العنيفة في السحاب‬
‫تصادمات بين جزيئات الماء‪ ،‬ظهور شحنات موجبة وشحنات‬
‫سالبة‪ .‬الشحنتان متعاكستان ومنفصلتان‪ :‬قاعدة السحابة‬
‫سلبا والجزء العلوي إيجاباً‪ .‬في نفس الوقت تكون التربة‬‫مشحونة ً‬
‫مشحونة إيجاباً كما بالشكل ‪ 1‬المنمذج للصورة المقابلة‪.‬‬
‫وبالتالي‪ ،‬فإنها تشكل مكثفة مشحونة‪ ،‬أحد لبوسيها هو‬
‫األرض (اللبوس ‪ A‬الموجب) واآلخر قاعدة السحابة‬
‫(اللبوس ‪ B‬السالب)‪ ،‬سعتها ‪ ، C‬التوتر الكهربائي بين‬
‫‪+++++++++++++++‬‬
‫سحابة‬ ‫لبوس ‪B‬‬ ‫طرفي المكثفة هو ‪. U AB  E 108 V‬‬
‫‪-------------‬‬
‫يهدف هذا التمرين إلى حساب المقاومة الكهربائية‬
‫لبوس ‪A‬‬ ‫للهواء وذاتية وشيعة‪.‬‬
‫األرض‬
‫‪+++++++++++++++++++++‬‬ ‫‪ . 1‬البرق ظاهرة كهربائية طبيعية تحدث نتيجة تفريغ‬
‫كهربائي في الهواء الرطب ما بين األرض وسحابة‪.‬‬
‫الشكل ‪ .1‬رسم تخطيطي للصورة‬
‫نعتبر الهواء الرطب ناقال أوميا مقاومته ‪. R‬‬
‫‪i 103 A ‬‬
‫‪5‬‬ ‫‪‬‬
‫‪t 10-5 s‬‬ ‫‪‬‬ ‫تتطور شدة التيار الكهربائي أثناء التفريغ وفق المنحنى‬
‫‪0‬‬ ‫البياني الشكل ‪.2‬‬
‫‪4‬‬ ‫‪ .1.1‬ارسم شكال تخطيطيا لدارة التفريغ الكهربائية‬
‫المنمذجة للظاهرة الموصوفة بالشكل ‪.1‬‬
‫‪ .2.1‬بتطبيق قانون جمع التوترات الكهربائية‪ ،‬أسس‬
‫المعادلة التفاضلية لتطور شدة التيار ‪. i  t ‬‬
‫‪t‬‬
‫‪‬‬
‫‪ i (t )  I 0  e‬حال للمعادلة التفاضلية‬ ‫‪RC‬‬
‫‪ .1.1‬بين أن‪:‬‬
‫السابقة‪.‬‬
‫الشكل ‪ .2‬تطور شدة التيار الكهربائي بداللة الزمن‬

‫صفحة ‪ 1‬من ‪8‬‬


‫⋑‬
‫اختبار يف مادة‪ :‬العلوم الفيزايئية ‪ //‬الشعبة‪ :‬علوم جتريبية ‪ //‬بكالوراي ‪2019‬‬

‫‪ .1.1‬باستغالل البيان (الشكل ‪:)2‬‬

‫‪ .1.4.1‬استخرج قيمة كل من شدة التيار الكهربائي العظمى ‪ I 0‬وثابت الزمن ‪ ‬لثنائي القطب ‪. R, C‬‬
‫‪ .2.1.1‬احسب قيمة ‪ R‬واستنتج قيمة سعة المكثفة ‪. C‬‬

‫‪ .1.1‬المثالن القائالن «عندما يهدر الرعد‪ ،‬اذهب إلى الداخل» و«إذا كان هناك برق بالقرب من موقعك‪ ،‬فأنت لست‬
‫آمنا بالخارج»‪ .‬على ضوء هذا أعط بعض قواعد الحماية من الصاعقة‪.‬‬
‫)‪uC (V‬‬
‫‪ .2‬نربط مكثفة مشحونة سعتها ‪ C  102  F‬مع وشيعة ذاتيتها ‪ L‬ومقاومتها ‪. r‬‬
‫بواسطة التجريب المدعم بالحاسوب ‪  ExAO ‬تم الحصول على منحنى‬
‫تطور التوتر الكهربائي بين طرفي المكثفة ‪ uC  t ‬الشكل ‪.1‬‬

‫‪0‬‬ ‫‪0, 4‬‬ ‫) ‪t (ms‬‬ ‫‪ .1.2‬حدد نمط االهتزاز واستنتج قيمة شبه الدور ‪. T‬‬

‫‪ِ .2.2‬جد قيمة ذاتية الوشيعة ‪ L‬باعتبار ‪T  T0‬‬


‫حيث‪ T0 :‬الدور الذاتي للدارة المثالية ‪. L, C‬‬

‫الشكل ‪ .0‬تطور التوتر ‪uC  t ‬‬

‫التمرين الثاني‪ 07( :‬نقاط)‬


‫‪ .1‬نقترح ثالثة محاليل مائية ‪  S 2  ،  S1 ‬و ‪  S 3 ‬لألحماض ‪ HA2 ، HA1‬و ‪ HA3‬على الترتيب لها نفس التركيز المولي‬
‫‪ ، c  5 102 mol  L1‬قيم الـ ‪ pH‬للمحاليل الثالث‪ 3, 2 ، 1, 3 :‬و ‪ 2,9‬وترتب هذه األحماض حسب تزايد قوتها‬
‫الحمضية الشكل ‪.1‬‬
‫تزايد القوة الحمضية‬
‫يهدف هذا التمرين إلى مقارنة قوة األحماض‪.‬‬
‫‪HA3‬‬ ‫‪HA2‬‬ ‫‪HA1‬‬
‫الشكل ‪.0‬‬
‫كل المحاليل مأخوذة في الدرجة ‪. 25C‬‬
‫‪ .1.1‬أعط تعريفا للحمض الضعيف‪.‬‬
‫‪ .2.1‬انسب لكل محلول قيمة الـ ‪ pH‬الموافق له مع التبرير‪.‬‬
‫‪pH‬‬
‫‪‬‬ ‫‪ .1.1‬بين أن الحمضين ‪ HA2‬و ‪ HA3‬ضعيفان وأن ‪ HA1‬حمض قوي‪.‬‬
‫‪‬‬
‫‪‬‬ ‫‪ .1.1‬اكتب عبارة ثابت الحموضة ‪ Ka‬للثنائية ‪. HA  aq  / A  aq ‬‬

‫‪ .1.1‬اثبت أن عبارة الـ ‪ pH‬تعطى بالعالقة‪:‬‬

‫‪1‬‬ ‫‪1‬‬ ‫‪1‬‬


‫‪ log  HAéq‬‬ ‫‪pH   log  HAéq  pKa‬‬
‫‪2‬‬ ‫‪2‬‬
‫‪0‬‬
‫‪1‬‬
‫الشكل ‪ .5‬تطور الـ ‪ pH‬بداللة ‪ log  HAéq‬‬

‫صفحة ‪ 2‬من ‪8‬‬


‫⋑‬
‫اختبار يف مادة‪ :‬العلوم الفيزايئية ‪ //‬الشعبة‪ :‬علوم جتريبية ‪ //‬بكالوراي ‪2019‬‬

‫‪ .1.1‬من أجل قيم مختلفة للتركيز المولي ‪  HAéq‬للمحلولين‬


‫الحمضيين الضعيفين السابقين‪ ،‬نقيس قيم ‪ pH‬الموافقة ثم نمثل‬
‫المنحنى البياني لـتطور الـ ‪ pH‬بداللة ‪(  log  HAéq‬الشكل ‪.)1‬‬
‫‪ .1.1.1‬ارفق كل منحنى بالحمض الموافق له مع التعليل‪.‬‬
‫‪ .2.1.1‬حدد قيمة ‪ pKa‬لكل ثنائية ‪ HA  aq  / A  aq ‬من المنحنيين‪ ‬و‪ ‬بالشكل ‪.1‬‬

‫نسخن باالرتداد وبوجود وسيط‪ ،‬مزيجا ستوكيومتريا ألحد الحمضين النقيين السابقين مع االيثانول ‪ C2 H 5 OH ‬‬ ‫‪.2‬‬
‫فينتج المركب العضوي ‪  CH 3COO  C2 H 5 ‬والماء‪.‬‬
‫) ‪n(mol‬‬
‫‪ .1.2‬حدد الوظيفة الكيميائية للمركب العضوي الناتج مع ذكر‬
‫اسمه‪.‬‬

‫‪ .2.2‬المتابعة الزمنية للتحول الكيميائي الحادث عن طريق‬


‫معايرة الحمض المتبقي مكنت من رسم المنحنى البياني‬
‫لتطور كمية مادة الحمض المتبقي بداللة الزمن‬
‫‪ n  f  t ‬الشكل ‪.1‬‬
‫‪0,1‬‬ ‫‪ .1.2.2‬احسب سرعة اختفاء الحمض عند اللحظة‬
‫)‪t (min‬‬ ‫‪ t 10 min‬واستنتج سرعة التفاعل عند نفس‬
‫‪0‬‬
‫‪5‬‬ ‫اللحظة‪.‬‬
‫الشكل ‪ .6‬تطور كمية مادة الحمض المتبقي‬ ‫‪ .2.2.2‬اذكر العوامل التي تؤثر في سرعة هذا التحول‪.‬‬
‫بداللة الزمن‬

‫التمرين التجريبي‪ 42( :‬نقاط)‬


‫تعتبر منطقة تيميمون بوالية أدرار المعروفة بالواحة الحمراء مقصداً‬
‫للسيَّاح لممارسة رياضة التزحلق على الكثبان الرملية‪.‬‬
‫يهدف التمرين الى دراسة الحركة المستقيمة لمتزحلق على الرمل‪.‬‬

‫باستغالل شريط فيديو لمتزحلق (الشخص ‪ +‬لوازمه) تم تصويره من‬


‫طرف أحد زوار منطقة تيميمون‪ ،‬ندرس الجملة {المتزحلق} التي‬
‫مركز عطالتها ‪ G‬المنمذجة بنقطة مادية كتلتها ‪. m‬‬

‫المعطيات‪:‬‬
‫صورة ‪ JPEG‬لمتزحلق على الرمل‬
‫صورة لمتزحلق على الرمل‬
‫‪A‬‬ ‫كتلة الجملة ‪ m  70 kg‬؛‬ ‫‪‬‬
‫شدة تسارع حقل الجاذبية‬ ‫‪‬‬
‫األرضية ‪ g  9,8 m  s 2‬؛‬
‫طول المسار األفقي ‪ BC  12 m‬؛‬ ‫‪‬‬
‫‪C‬‬ ‫‪B‬‬
‫زاوية الميل ‪.   41‬‬ ‫‪‬‬

‫الشكل ‪7‬‬
‫صفحة ‪ 0‬من ‪8‬‬
‫⋑‬
‫اختبار يف مادة‪ :‬العلوم الفيزايئية ‪ //‬الشعبة‪ :‬علوم جتريبية ‪ //‬بكالوراي ‪2019‬‬

‫‪ .1‬المرحلة األولى (المسار ‪:) AB‬‬


‫حركة المتزحلق تتم على مستو مائل انطالقا‬
‫من النقطة ‪ A‬دون سرعة ابتدائية الشكل ‪.7‬‬
‫معالجة شريط الفيديو السابق ببرمجية ‪ Avistep‬مكنتنا‬
‫من تسجيل المواضع المتتالية لمركز عطالة الجملة‬
‫خالل مجاالت زمنية متتالية ومتساوية ‪ t  0,8 s‬الشكل ‪.8‬‬

‫‪G0‬‬ ‫‪G3‬‬ ‫‪G5‬‬ ‫‪G7‬‬


‫‪t0  0‬‬
‫‪4m‬‬
‫الشكل ‪ .8‬تسجيل المواضع المتتالية لمركز عطالة الجملة‬

‫‪ .1.1‬عرف المرجع الغاليلي (العطالي)‪.‬‬


‫‪ .2.1‬احسب قيم السرعة في اللحظات ‪ t5 ، t3‬و ‪ t7‬الموافقة للمواضع ‪ G7 ، G5 ، G3‬على الترتيب‪.‬‬
‫‪ .1.1‬ارسم على ورق ميليمتري المنحنى البياني لتطور السرعة اللحظية بداللة الزمن ) ‪. v  f (t‬‬
‫‪ِ .1.1‬جد بيانياً قيمة تسارع مركز عطالة الجملة ‪ aG‬واستنتج طبيعة الحركة‪.‬‬
‫‪ .1.1‬احسب بيانياً المسافة المقطوعة بين الموضعين ‪ G0‬و ‪. G8‬‬
‫‪ .1.1‬بإهمال قوى االحتكاك على المسار ‪: AB‬‬
‫‪ .1.1.1‬بتطبيق القانون الثاني لنيوتن‪ِ ،‬جد عبارة التسارع ‪ aG‬واحسب قيمته‪.‬‬
‫‪ .2.1.1‬برر االختالف بين قيمتي التسارع المحسوبتين في السؤالين (‪ ).1.1‬و(‪.).1.1.1‬‬

‫‪ .7‬المرحلة الثانية(المسار ‪:) BC‬‬


‫يصل المتزحلق الى النقطة ‪ B‬بسرعة ‪ vB  12 m  s 1‬ويواصل حركته المستقيمة على المستوي األفقي ‪BC‬‬
‫ليتوقف عند الموضع ‪ . C‬تنمذج القوى المعيقة للحركة بقوة وحيدة ‪ f‬مماسية للمسار وثابتة في الشدة‪.‬‬

‫‪ .1.2‬أحص ومثل القوى الخارجية المطبقة على مركز عطالة الجملة ‪. G‬‬

‫‪ِ .2.2‬جد شدة القوة ‪ ، f‬بتطبيق مبدأ إنحفاظ الطاقة للجملة المدروسة‪.‬‬

‫انتهى الموضوع األول‬


‫صفحة ‪ 4‬من ‪8‬‬
‫⋑‬
‫اختبار يف مادة‪ :‬العلوم الفيزايئية ‪ //‬الشعبة‪ :‬علوم جتريبية ‪ //‬بكالوراي ‪2019‬‬

‫الموضوع الثاني‬
‫يحتوي الموضوع الثاني على ‪ 40‬صفحات (من الصفحة ‪ 5‬من ‪ 8‬إلى الصفحة ‪ 8‬من ‪)8‬‬

‫التمرين األول‪ 40( :‬نقاط)‬


‫حدث تكاثر غير طبيعي في الكريات الحمراء‪ .‬لمعالجة هذا المرض يحقن‬ ‫داء الفاكيز يصيب النخاع العظمي وي ِ‬
‫المريض بمحلول يحتوي على نظير الفوسفور ‪ 15 P‬الذي يدمر الكريات الحمراء الزائدة بفعل اإلشعاع المنبعث منه‪.‬‬
‫‪32‬‬

‫يهدف هذا التمرين إلى دراسة النشاط اإلشعاعي لنظير الفوسفور‪.‬‬

‫المعطيات‪:‬‬
‫ثابت أفوغادرو ‪ N A  6, 02 10 mol‬؛‬
‫‪1‬‬
‫‪‬‬
‫‪23‬‬

‫نصف العمر ‪ t1/2 ( 15 P)  14,32 jours‬؛‬


‫‪32‬‬
‫‪‬‬
‫‪ m( 15 P)  31,97391u‬؛‬
‫‪32‬‬
‫‪‬‬
‫‪ m( 15 P)  29,97831u‬؛‬
‫‪30‬‬
‫‪‬‬
‫كتلة البروتون ‪ m p  1, 00728u‬؛‬ ‫‪‬‬
‫كتلة النيترون ‪ mn  1,00866u‬؛‬ ‫‪‬‬
‫‪. 1u  931,5 MeV / c2‬‬ ‫‪‬‬

‫‪ .1‬اذكر أنواع التفككات اإلشعاعية الطبيعية مع تحديد الجسيم المنبعث عن كل تفكك‪.‬‬

‫‪Z‬‬ ‫‪ .2‬اعتمادا على المخطط الممثل في الشكل ‪:1‬‬

‫‪ .1.2‬استنتج قيمة كل من العددين ‪ A‬و ‪ Z‬ثم أعط رمز‬


‫النواة الموافقة‪.‬‬
‫‪17‬‬
‫إلى النواة ‪ ، ZY‬محددا‬ ‫‪ .2.2‬اكتب معادلة تفكك النواة ‪P‬‬
‫‪A‬‬ ‫‪32‬‬
‫‪15‬‬
‫نوع التفكك النووي الحادث‪.‬‬
‫‪A‬‬
‫‪16‬‬ ‫‪Z‬‬‫‪Y‬‬ ‫‪33‬‬
‫‪16‬‬ ‫‪S‬‬
‫‪30‬‬ ‫‪32‬‬ ‫‪ .1‬في اللحظة ‪ t  0‬يحقن مريض بجرعة من محلول‬
‫‪15‬‬ ‫‪15‬‬ ‫‪P‬‬ ‫‪15‬‬ ‫‪P‬‬ ‫يحتوي على كمية قدرها ‪ n0  3,12 1010 mol‬من‬
‫نظير الفوسفور ‪.12‬‬
‫‪14‬‬
‫‪N‬‬ ‫‪ .1.1‬احسب عدد أنوية الفوسفور ‪ 32‬المحتواة في هذه‬
‫‪14‬‬ ‫‪15‬‬ ‫‪16‬‬ ‫‪17‬‬ ‫الجرعة‪.‬‬
‫الشكل ‪ .1‬مستخرج من المخطط ‪ N  Z ‬‬ ‫‪ .2.1‬يزول مفعول الجرعة عندما تتفكك ‪ 99 0 0‬من األنوية‬
‫االبتدائية‪ ،‬بين أن مفعولها يزول بعد ‪ 95 jours‬من‬
‫لحظة الحقن‪.‬‬
‫‪ .4‬لعنصر الفوسفور نظير آخر هو ‪. 15 P‬‬
‫‪30‬‬

‫بالـ ‪. MeV‬‬
‫‪32‬‬
‫‪15‬‬ ‫و‪P‬‬ ‫‪30‬‬
‫‪15‬‬ ‫‪ .1.4‬احسب طاقة الربط النووي ‪ E‬لكل من النواتين ‪P‬‬
‫‪ .2.1‬بين أي النواتين أكثر استق ار ار مع التعليل‪.‬‬

‫صفحة ‪ 5‬من ‪8‬‬


‫⋑‬
‫اختبار يف مادة‪ :‬العلوم الفيزايئية ‪ //‬الشعبة‪ :‬علوم جتريبية ‪ //‬بكالوراي ‪2019‬‬

‫التمرين الثاني‪ 42( :‬نقاط)‬


‫في حياتنا اليومية‪ ،‬أمثلة كثيرة عن النواس الثقلي مثل‪ :‬األرجوحة‪ ،‬رقاص ساعة حائط‪ ،‬ثرية‪...‬‬

‫استوحى فكرة دراسة‬


‫َ‬ ‫يعتبر العالم الفيزيائي والفلكي اإليطالي غاليلو غاليلي‪ ،‬أول من‬
‫النواس الثقلي عندما شاهد الثرية المعلقة في سقف قاعة الحفالت وهي تهتز بعد أن‬
‫حركتها التيارات الهوائية‪.‬‬

‫المعطيات‪:‬‬
‫‪ ‬شدة تسارع حقل الجاذبية األرضية ‪ g  9,8 m  s 2‬؛‬
‫غاليلو غاليلي‬ ‫‪ ‬نهمل تأثير الهواء‪.‬‬
‫(‪1500‬م – ‪1007‬م)‬
‫أوالً‪ :‬دراسة الحركة االهتزازية للنواس البسيط‬

‫يعتبر النواس البسيط نموذجا مثاليا للنواس الثقلي ويتألف من خيط‬


‫‪O‬‬ ‫مهمل الكتلة وعديم االمتطاط طوله مثبت من إحدى نهايتيه‬
‫‪0‬‬ ‫بنقطة ‪ O‬ومعلق بنهايته الحرة كرية كتلتها ‪ m‬مهملة األبعاد بالنسبة‬
‫̛‬ ‫لطول الخيط (جسم نقطي) الشكل ‪.2‬‬
‫نزيح النواس في المستوي الشاقولي عن وضع توازنه المستقر ‪O‬‬
‫‪A‬‬
‫‪h‬‬
‫بزاوية ‪  0  8‬في جهة نعتبرها موجبة‪ ،‬ثم نتركه لحاله من النقطة ‪A‬‬
‫دون سرعة ابتدائية في اللحظة ‪ ، t  0‬فينجز اهت اززات حرة حول‬
‫‪x‬‬
‫‪O‬‬

‫محور أفقي مار بالنقطة ‪ O‬ونقيس بواسطة ميقاتية زمن ‪ 10‬اهت اززات‬
‫كاملة فنجده ‪. t  14 s‬‬
‫‪E‬‬ ‫سطح االرض‬
‫‪ .1‬عرف دور النواس البسيط‪.‬‬
‫الشكل ‪7‬‬
‫‪y‬‬

‫‪ .2‬احسب قيمة الدور الذاتي ‪ T0‬للنواس البسيط‪.‬‬

‫‪ .1‬نقترح أربع عبارات للدور الذاتي للنواس البسيط‪ ،‬اختر العبارة الصحيحة ثم علل إجابتك باستعمال التحليل البعدي‪.‬‬

‫‪0‬‬
‫؛‬ ‫‪  3  T0  2 ‬؛‬ ‫‪  4 T0  2 ‬؛‬
‫‪g‬‬ ‫‪m‬‬
‫‪1 ‬‬ ‫‪T0  2 ‬‬ ‫‪ 2‬‬ ‫‪T0  2 ‬‬
‫‪g‬‬ ‫‪g‬‬

‫‪ .1‬احسب طول النواس البسيط ‪.  ‬‬

‫‪ .1‬ضع اإلشارة (‪ )‬أمام العبارة الصحيحة واالشارة (‪ )‬أمام العبارة الخاطئة لما يلي‪:‬‬

‫‪ -‬الدور ال يتعلق بالكتلة ‪............ m‬‬


‫‪‬‬
‫‪.........‬‬ ‫‪ -‬الدور يتناسب طردا مع‬
‫‪‬‬
‫‪ -‬الدور يتناسب طردا مع ‪......... g‬‬
‫‪‬‬
‫‪ -‬الدور يتعلق بالسعات الصغيرة ‪....  0‬‬
‫‪‬‬

‫صفحة ‪ 6‬من ‪8‬‬


‫⋑‬
‫اختبار يف مادة‪ :‬العلوم الفيزايئية ‪ //‬الشعبة‪ :‬علوم جتريبية ‪ //‬بكالوراي ‪2019‬‬

‫ثانياً‪ :‬دراسة حركة قذيفة‬

‫عند مرور الكرية بوضع التوازن ‪ O‬في االتجاه الموجب بالسرعة ‪ v0  0,3 m  s 1‬ينقطع الخيط فتتحرر الكرية في الهواء‬
‫لتصطدم بسطح األرض الذي يبعد عن المستوي األفقي المار بنقطة التعليق ‪ O‬بارتفاع ‪. h  1,5m‬‬

‫‪ِ .1‬جد‪ ،‬بتطبيق القانون الثاني لنيوتن المعادلتين الزمنيتين للحركة ) ‪ x(t‬و ) ‪ y (t‬في المعلم ) ‪ . (Ox, Oy‬الشكل ‪.2‬‬

‫‪ .2‬استنتج معادلة المسار وحدد احداثيي نقطة االصطدام ‪ E‬بسطح األرض‪.‬‬

‫‪ .1‬عين خصائص شعاع سرعة مركز عطالة الكرية ‪ G‬عند الموضع ‪. E‬‬

‫التمرين التجريبي‪ 42( :‬نقاط)‬


‫تصنف التحوالت الكيميائية إلى تامة وغير تامة‪.‬‬
‫نقترح في هذا التمرين دراسة تحولين أحدهما تام واآلخر غير تام‪.‬‬

‫أوال‪ :‬دراسة تفاعل الكحول )‪ ( B‬ذي الصيغة المجملة ‪ C3 H8O‬مع شوارد البرمنغنات ‪MnO4‬‬

‫المعطيات‪:‬‬
‫‪ ‬الكتلة المولية الجزيئية للكحول )‪. M ( B)  60 g  mol 1 ( B‬‬

‫نضع في إيرلينة ماير موضوعة فوق مخالط مغناطيسي حجما ‪ V0  50 mL‬من محلول برمنغنات البوتاسيوم‬
‫))‪ ( K  (aq)  MnO4 (aq‬تركيزه المولي ‪ ، c0  0,1 mol  L1‬المحمض بحمض الكبريت المركز‪.‬‬
‫في اللحظة ‪ t  0‬نضيف للمزيج كتلة قدرها ‪ m  3, 75 g‬من الكحول )‪ ( B‬ذي الصيغة الجزيئية المجملة ‪، C3 H8O‬‬
‫حيث يصبح حجم الوسط التفاعلي ‪ . VT  60 mL‬التحول الكيميائي الحادث بطيء‪ ،‬ن ِ‬
‫نمذجه بالمعادلة الكيميائية‪:‬‬

‫) ‪5C3 H 8O(l )  2MnO4 (aq)  6 H  (aq)  5C3 H 6O(l )  2Mn 2 (aq)  8H 2O(l‬‬

‫) ‪nB (mmol‬‬
‫‪ .1‬عرف كل من الم ِ‬
‫ؤكسد والمرِجع‪.‬‬

‫‪ .2‬بين أن التفاعل الحادث هو تفاعل أكسدة‪-‬إرجاع‪ ،‬ثم اكتب‬


‫الثنائيتين ‪ Ox / Re d‬المشاركتين في التفاعل‪.‬‬
‫‪ .1‬وضح دور حمض الكبريت المركز في هذا التفاعل‪.‬‬

‫‪ .1‬أنشئ جدوالً لتقدم التفاعل واحسب قيمة التقدم األعظمي ‪. xmax‬‬

‫‪ .1‬المتابعة الزمنية لتطور كمية مادة الكحول )‪ ، ( B‬مكنتنا من‬


‫رسم المنحنى البياني الممثل بالشكل ‪.3‬‬
‫‪0‬‬ ‫‪4‬‬ ‫)‪t (min‬‬ ‫‪ .1.1‬حدد قيمة التقدم النهائي ‪ x f‬ثم أثبت أن هذا التفاعل تام‪.‬‬
‫الشكل ‪ .3‬تطور كمية مادة الكحول )‪ ( B‬بداللة الزمن‬

‫صفحة ‪ 7‬من ‪8‬‬


‫⋑‬
‫اختبار يف مادة‪ :‬العلوم الفيزايئية ‪ //‬الشعبة‪ :‬علوم جتريبية ‪ //‬بكالوراي ‪2019‬‬

‫‪ .2.1‬عرف زمن نصف التفاعل ‪ t1/2‬ثم حدد بيانياً قيمته‪.‬‬

‫‪ .1.1‬احسب السرعة الحجمية الختفاء الكحول )‪ ( B‬في اللحظة ‪. t  0‬‬

‫ثانيا‪ :‬دراسة تفاعل الكحول )‪ ( B‬مع حمض االيثانويك ‪.  CH 3COOH ‬‬

‫صنف الكحول )‪ ، ( B‬نجري تفاعل أسترة لمزيج ابتدائي متساوي الموالت( ‪ 50 mmol‬من الكحول )‪ ( B‬و‬‫لتحديد ِ‬
‫‪ 50 mmol‬من حمض االيثانويك )‪ ) ( A‬مع إضافة قطرات من حمض الكبريت المركز‪.‬‬

‫ن ِ‬
‫سخن المزيج باالرتداد لمدة ساعة‪.‬‬
‫‪ِ .1‬‬
‫وضح دور حمض الكبريت المركز في هذا التفاعل‪.‬‬
‫) ‪nB (mmol‬‬
‫‪ .2‬اكتب معادلة التفاعل الحادث‪.‬‬

‫‪ .1‬أنشئ جدوالً لتقدم التفاعل واحسب قيمة التقدم األعظمي ‪. xmax‬‬


‫مثل تطور كمية مادة‬‫‪ .1‬المنحنى البياني الممثل بالشكل ‪ 4‬ي ِ‬
‫الكحول )‪ ( B‬بداللة الزمن‪:‬‬
‫‪ .1.1‬اكتب بروتوكوال تجريبيا توضح فيه كيفية الحصول‬
‫على المنحنى البياني الشكل ‪.4‬‬
‫)‪t (min‬‬

‫‪0‬‬
‫‪ .2.1‬حدد قيمة التقدم النهائي ‪ x f‬وأثبت أن هذا التفاعل‬
‫‪10‬‬ ‫غير تام‪.‬‬
‫الشكل ‪ .4‬تطور كمية مادة الكحول )‪( B‬‬
‫بداللة الزمن‬ ‫‪ .1.1‬احسب مردود التفاعل واستنتج ِ‬
‫صنف الكحول )‪. ( B‬‬

‫‪ .1‬دعم هذه الجملة بالتفسير أكثر «يمكن الحصول على اإلستر السابق بتفاعل آخر تام‪ ،‬سريع وناشر للح اررة»‪.‬‬

‫انتهى الموضوع الثاني‬

‫صفحة ‪ 8‬من ‪8‬‬


‫العالمة‬
‫عناصر االجابة على الموضوع األول‬
‫مجزأة المجموع‬

‫𝐾‬ ‫𝑅‬ ‫التمرين األول‪:‬‬


‫‪-1‬‬
‫𝐶‬
‫‪ -1.1‬شكل تخطيطي لدارة التفريغ‪:‬‬
‫‪ -1.2‬المعادلة التفاضلية لتطور شدة التيار )𝒕(𝒊‪:‬‬
‫‪𝑈𝐶 + 𝑈𝑅 = 0‬‬
‫𝑞‬
‫‪+ 𝑅𝑖 = 0‬‬
‫𝐶‬
‫باشتقاق الطرفين‪:‬‬
‫𝑞𝑑 ‪1‬‬ ‫𝑖𝑑‬
‫‪+𝑅 =0‬‬
‫𝑡𝑑 𝐶‬ ‫𝑡𝑑‬
‫‪1‬‬ ‫𝑖𝑑‬
‫‪𝑖+𝑅 =0‬‬
‫𝐶‬ ‫𝑡𝑑‬
‫𝒊𝒅‬ ‫𝟏‬
‫‪+‬‬ ‫𝟎=𝒊‬
‫𝑪𝑹 𝒕𝒅‬
‫𝟏‬
‫‪ -1.3‬بيان أنّ‪ 𝒊(𝒕) = −𝑰𝟎 𝒆−𝑹𝑪𝒕 :‬حال للمعادلة التفاضلية السابقة‪:‬‬
‫‪1‬‬ ‫‪1‬‬
‫𝑖𝑑‬ ‫‪1‬‬ ‫‪−‬‬ ‫𝑡‬ ‫‪1‬‬ ‫‪1‬‬ ‫‪−‬‬ ‫𝑡‬
‫=‬ ‫𝑒𝐼‬ ‫𝐶𝑅‬ ‫و‬ ‫‪𝑖=−‬‬ ‫𝑒𝐼‬ ‫𝐶𝑅‬ ‫لدينا‪:‬‬
‫𝑡𝑑‬ ‫‪𝑅𝐶 0‬‬ ‫𝐶𝑅‬ ‫‪𝑅𝐶 0‬‬
‫ومنه‪:‬‬
‫𝑖𝑑‬ ‫‪1‬‬ ‫‪1‬‬ ‫‪1‬‬ ‫‪1‬‬ ‫‪1‬‬
‫‪+‬‬ ‫=𝑖‬ ‫‪𝐼0 𝑒 −𝑅𝐶 𝑡 −‬‬ ‫‪𝐼0 𝑒 −𝑅𝐶 𝑡 = 0‬‬
‫𝐶𝑅 𝑡𝑑‬ ‫𝐶𝑅‬ ‫𝐶𝑅‬
‫وبالتالي هو حل للمعادلة التفاضلية‪.‬‬
‫‪ -1.4‬باستغالل البيان (الشكل ‪:)-2-‬‬
‫‪-1.1.4‬‬
‫شدّة التيار‪.𝑰𝟎 = 𝟐. 𝟏𝟎𝟒 𝑨 :‬‬
‫ثابت الزمن 𝜏 باالسقاط نجد‪𝝉 = 𝟓. 𝟏𝟎−𝟓 𝒔 :‬‬
‫‪ -2.1.4‬قيمة 𝑹 و𝑪‪:‬‬
‫𝐸‬ ‫𝐸‬ ‫‪108‬‬
‫= = 𝑅 ⇒ = ‪𝐼0‬‬
‫𝑅‬ ‫‪𝐼0 2. 104‬‬
‫‪𝑅 = 5. 103 Ω‬‬
‫‪𝜏 5. 10−5‬‬
‫= = 𝐶 ⇒ 𝐶𝑅 = 𝜏‬
‫𝑅‬ ‫‪5. 103‬‬
‫𝑭 𝟖‪𝝉 = 𝟏𝟎−‬‬
‫‪ -1.5‬قواعد الحماية من الصاعقة‪:‬‬
‫‪ -‬الحذر من االمساك بالمعادن واألسالك‪.‬‬
‫𝒎𝒐𝒄 ‪𝒘𝒘𝒘. 𝒕𝒐𝒖𝒂𝒉𝒓𝒊𝒂.‬‬ ‫تصحيح مقترح من طرف‪ :‬األستاذ طواهرية عبد العزيز‬ ‫‪1‬‬
‫‪ -‬اغالق الجواالت وأجهزة االتصاالت‪.‬‬
‫‪ -‬عدم الوقوف في االماكن المكشوفة أو مناطق هطول االمطار الرعدية‪.‬‬
‫‪ -‬تجنب الجلوس تحت أي شجرة مرتفعة ومنفردة حتى ال تتسبب الصاعقة في الوفاة‪.‬‬
‫‪ -‬عندما يكون البرق قريبا يجب االنخفاض ألسفل مسافة ممكنة دون مالمسة االرض‪.‬‬
‫‪ -2‬تفريغ المكثفة في وشيعة‪:‬‬
‫‪ -1.2‬تحديد نمط االهتزاز‪ :‬اهتزازات شبه دورية متخامدة‪.‬‬
‫‪ -‬قيمة شبه الدور 𝑇‪𝑻 = 𝟎, 𝟐𝒎𝒔 :‬‬
‫‪ -2.2‬تحديد ذاتية الوشيعة 𝑳‪:‬‬
‫𝐶𝐿√ ‪𝑇0 = 2π.‬‬
‫‪𝑇0 2‬‬ ‫‪(2. 10−4 )2‬‬
‫‪𝐿= 2 = 2‬‬
‫) ‪4𝜋 𝐶 4𝜋 (10−8‬‬
‫𝑯𝟏 ‪𝑳 = 𝟎,‬‬

‫التمرين الثاني‪:‬‬
‫‪-1‬‬
‫‪ -1.1‬تعريف الحمض الضعيف‪ :‬هو الحمض الذي يتفكك جزئيا في الماء‪.‬‬
‫‪ -2.1‬نسب لكل محلول قيمة الـ 𝑯𝑷 الموافقة له‪:‬‬
‫لدينا معادلة تفكك حمض ضعيف فيالماء بالشكل التالي‪:‬‬
‫‪𝐻𝐴 + 𝐻2 𝑂 = 𝐴− + 𝐻3 𝑂+‬‬
‫اي أن‪:‬‬
‫] ‪𝑃𝐻 = − log[𝐻3 𝑂+‬‬
‫أي أنه كلما زاد تركيز شوارد الهيدرونيوم ] ‪ [𝐻3 𝑂+‬كلما نقص الـ𝐻𝑃 وبالتالي الحمض الذي له 𝐻𝑃‬
‫اقل هو الحمض األقوى‪:‬‬
‫‪𝐻𝐴3‬‬ ‫‪𝐻𝐴2‬‬ ‫‪𝐻𝐴1‬‬ ‫الحمض‬
‫‪2,9‬‬ ‫‪3,2‬‬ ‫‪1,3‬‬ ‫الـ𝑯𝑷 الموافق‬
‫‪ -3.1‬بيان أن الحمضين 𝟑𝑨𝑯 و 𝟐𝑨𝑯 ضعيفين والحمض 𝟏𝑨𝑯 قوي‪:‬‬
‫لدينا‪:‬‬
‫𝐻𝑃‪[𝐻3 𝑂+ ] = 10−‬‬
‫‪ -‬الحمض ‪ 𝐻𝐴3‬ضعيف‪[𝐻3 𝑂+ ]3 < 𝐶. [𝐻3 𝑂+ ]3 = 10−2,9 = 1,25. 10−3 𝑚𝑜𝑙. 𝑙 −1 :‬‬
‫‪ -‬الحمض ‪ 𝐻𝐴2‬ضعيف‪[𝐻3 𝑂+ ]2 < 𝐶. [𝐻3 𝑂+ ]2 = 10−3,2 = 6,3. 10−4 𝑚𝑜𝑙. 𝑙 −1 :‬‬
‫‪ -‬الحمض ‪ 𝐻𝐴1‬قوي‪[𝐻3 𝑂+ ]1 = 𝐶. [𝐻3 𝑂+ ]1 = 10−1.3 = 6,3. 10−4 𝑚𝑜𝑙. 𝑙 −1 :‬‬
‫‪ -4.1‬عبارة ثابت الحموضة 𝒂𝑲 للثنائية ) ‪:(𝑯𝑨⁄𝑨−‬‬
‫𝒇] ‪[𝑨− ]𝒇 . [𝑯𝟑 𝑶+‬‬
‫= 𝒂𝑲‬
‫𝒇]𝑨𝑯[‬

‫𝒎𝒐𝒄 ‪𝒘𝒘𝒘. 𝒕𝒐𝒖𝒂𝒉𝒓𝒊𝒂.‬‬ ‫تصحيح مقترح من طرف‪ :‬األستاذ طواهرية عبد العزيز‬ ‫‪2‬‬
‫‪ -5.1‬اثبات أن عبارة الـ𝑯𝑷 تعطى بالعالقة التالية‪:‬‬
‫‪1‬‬ ‫‪1‬‬
‫𝑎𝐾𝑝 ‪𝑝𝐻 = − log[𝐻𝐴]𝑓 +‬‬
‫‪2‬‬ ‫‪2‬‬
‫لدينا من عبارة ثابت الحموضة 𝑎𝐾 للثنائية ) ‪:(𝐻𝐴⁄𝐴−‬‬
‫𝑓] ‪[𝐴− ]𝑓 . [𝐻3 𝑂+‬‬
‫= 𝑎𝐾‬
‫𝑓]𝐴𝐻[‬
‫‪2‬‬
‫𝑓] ‪[𝐻3 𝑂+‬‬
‫= 𝑎𝐾‬
‫𝑓]𝐴𝐻[‬
‫‪2‬‬
‫𝑓] ‪[𝐻3 𝑂+‬‬
‫‪log 𝐾𝑎 = log‬‬
‫𝑓]𝐴𝐻[‬
‫𝑓]𝐴𝐻[‪log 𝐾𝑎 = 2 log[𝐻3 𝑂+ ]𝑓 − log‬‬
‫𝑓]𝐴𝐻[‪− log 𝐾𝑎 = −2 log[𝐻3 𝑂+ ]𝑓 + log‬‬
‫𝑓]𝐴𝐻[‪𝑝𝐾𝑎 = 2𝑃𝐻 + log‬‬
‫𝟏‬ ‫𝟏‬
‫𝒂𝑲𝒑 ‪𝒑𝑯 = − 𝒍𝒐𝒈[𝑯𝑨]𝒇 +‬‬
‫𝟐‬ ‫𝟐‬
‫‪-6.1‬‬
‫‪ -1.1.6‬ارفاق كل منحنى بالحمض الموافق له مع التعليل‪:‬‬
‫المنحنى (‪ )1‬يوافق الحمض ‪ 𝐻𝐴3‬االضعف ألنه يوافق 𝐻𝑝 أكبر‪.‬‬
‫و بالتالي المنحنى (‪ )2‬يوافق الحمض ‪ 𝐻𝐴2‬ألنه يوافق 𝐻𝑝 أقل‪.‬‬
‫‪ -2.6.1‬تحديد قيمة 𝒂𝑲𝒑 لكل ثنائية‪:‬‬
‫نمدد المنحيين‪:‬‬
‫‪:(𝐴𝐻3 ⁄𝐴−‬‬
‫الثنائية ) ‪3‬‬
‫‪1‬‬
‫المنحنى (‪ )1‬يقطع محور 𝐻𝑝 في النقطة ‪ 𝑝𝐾𝑎 = 2,3‬ومنه 𝟔 ‪.𝒑𝑲𝒂 = 𝟒,‬‬
‫‪2‬‬
‫‪:(𝐴𝐻2 ⁄𝐴−‬‬
‫الثنائية ) ‪2‬‬
‫‪1‬‬
‫المنحنى (‪ )2‬يقطع محور 𝐻𝑝 في النقطة ‪ 𝑝𝐾𝑎 = 1,8‬ومنه 𝟔 ‪.𝒑𝑲𝒂 = 𝟑,‬‬
‫‪2‬‬
‫‪-2‬‬
‫‪ -1.2‬تحديد الوظيفة الكيميائية للمركب الناتج وتسميته‪:‬‬
‫‪-‬أستر ايثانوات االيثيل‪.‬‬
‫‪ -2.2‬المتابعة الزمنية‪:‬‬
‫‪ -1.2.2‬حساب سرعة اختفاء الحمض عند اللحظة 𝒏𝒊𝒎𝟎𝟏 = 𝒕‪:‬‬
‫𝑎𝑛‪d‬‬ ‫‪0,3 − 0.21‬‬
‫‪𝑣𝑎 = −‬‬ ‫‪=−‬‬
‫‪dt‬‬ ‫‪0 − 10‬‬
‫𝟏‪𝒗𝒂 = 𝟗. 𝟏𝟎−𝟑 𝒎𝒐𝒍. 𝒎𝒊𝒏−‬‬

‫𝒎𝒐𝒄 ‪𝒘𝒘𝒘. 𝒕𝒐𝒖𝒂𝒉𝒓𝒊𝒂.‬‬ ‫تصحيح مقترح من طرف‪ :‬األستاذ طواهرية عبد العزيز‬ ‫‪3‬‬
‫سرعة التفاعل‪:‬‬
‫𝟏‪𝒗 = 𝒗𝒂 = 𝟗. 𝟏𝟎−𝟑 𝒎𝒐𝒍. 𝒎𝒊𝒏−‬‬
‫‪ -2.2.2‬العوامل المؤثرة على حركية التفاعل‪:‬‬
‫تأثير درجة الحرارة‪ :‬يؤدي ارتفاع درجة الحرارة الى زيادة سرعة تفاعلي األسترة والحلمأة دون تغيير‬
‫تركيب الخليط عند حالة التوازن‪.‬‬
‫تأثير الوسيط‪ :‬االوسيط هو نوع كيميائي يمكن من تسريع التفاعل دون تغيير تركيب الخليط‪.‬‬

‫التمرين التجرييب‪:‬‬
‫‪ -1‬المرحلة األولى (المسار 𝑩𝑨)‪:‬‬
‫‪ -1.1‬تعريف المرجع العطالي‪:‬‬
‫هو المرجع الذي يكون اما ساكن أو يتحرك حركة مستقيمة منتظمة‪.‬‬
‫‪ -2.1‬حساب السرعات‪:‬‬
‫)‪𝐺2 𝐺4 (1,8). (4‬‬
‫= ‪𝑣3‬‬ ‫=‬ ‫𝟏‪= 𝟒, 𝟓 𝒎. 𝒔−‬‬
‫𝑡‪2Δ‬‬ ‫)‪2(0,8‬‬
‫)‪𝐺4 𝐺6 (3). (4‬‬
‫= ‪𝑣5‬‬ ‫=‬ ‫𝟏‪= 𝟕, 𝟓 𝒎. 𝒔−‬‬
‫𝑡‪2Δ‬‬ ‫)‪2(0,8‬‬
‫)‪𝐺6 𝐺8 (4,2). (4‬‬
‫= ‪𝑣7‬‬ ‫=‬ ‫𝟏‪= 𝟏𝟎, 𝟓 𝒎. 𝒔−‬‬
‫𝑡‪2Δ‬‬ ‫)‪2(0,8‬‬
‫) 𝟏‪𝒗(𝒎. 𝒔−‬‬ ‫‪ -3.1‬رسم المنحنى )𝒕(𝒇 = 𝒗‪:‬‬
‫‪16‬‬
‫‪14‬‬ ‫‪ -1.4‬تسارع مركز عطالة الجملة 𝑮𝒂‪ :‬يمثل‬
‫‪12‬‬ ‫ميل البيان‪.‬‬
‫‪10‬‬
‫𝑦‪Δ‬‬ ‫‪6−0‬‬
‫‪8‬‬ ‫=𝑎‬ ‫=‬
‫‪6‬‬ ‫‪Δ𝑥 3,2 − 0‬‬
‫‪4‬‬
‫𝟐‪𝒂 = 𝟏, 𝟖𝟕𝟓𝒎. 𝒔−‬‬
‫‪2‬‬
‫‪0‬‬ ‫‪ -‬طبيعة الحركة‪:‬‬
‫‪0‬‬ ‫‪0.8 1.6 2.4 3.2‬‬ ‫‪4‬‬ ‫‪4.8 5.6 6.4 7.2‬‬ ‫‪8‬‬
‫)𝒔(𝒕‬ ‫المسار مستقيم ‪ +‬سرعة متزايدة بانتظام من‬
‫الحركة مستقيمة متسارعة بانتظام‪.‬‬
‫‪ -5.1‬حساب بيانيا المسافة المقطوعة من 𝟎𝑮 الى 𝟖𝑮‪:‬‬
‫تمثل مساحة المثلث المحدد من 𝑠‪ 𝑡 = 0‬الى 𝑠‪:𝑡 = 6,4‬‬
‫)‪(12)(6,4‬‬
‫=𝑑‬
‫‪2‬‬
‫𝒎𝟒 ‪𝒅 = 𝟑𝟖,‬‬
‫‪ -6.1‬باهمال قوى االحتكاك على المسار 𝑩𝑨‪:‬‬
‫‪ -1.6.1‬عبارة التسارع 𝑮‪ 𝒂′‬وحساب قيمته‪:‬‬
‫‪ -‬بتطبيق الثقانون الثاني لنيوتن‪:‬‬

‫𝒎𝒐𝒄 ‪𝒘𝒘𝒘. 𝒕𝒐𝒖𝒂𝒉𝒓𝒊𝒂.‬‬ ‫تصحيح مقترح من طرف‪ :‬األستاذ طواهرية عبد العزيز‬ ‫‪4‬‬
‫⃗𝑎𝑚 = ⃗𝐹 ∑‬

‫⃗⃗⃗⃗‬
‫‪𝑃⃗⃗ + 𝑅⃗⃗ = 𝑚𝑎′‬‬
‫باالسقاط على المحور الموازي للحركة‪:‬‬
‫‪𝑚𝑔 sin 𝛼 + 0 = 𝑚𝑎′‬‬
‫)‪𝑎′ = 𝑔 sin 𝛼 = (9,8) sin(41‬‬
‫𝟐‪𝒂′ = 𝟔, 𝟒𝟑𝒎. 𝒔−‬‬
‫‪ -2.6.1‬االختالف‪:‬‬
‫االختالف يعود الى قوى االحتكاك التي ال يمكن اهمالها في هذه الحالة‪.‬‬
‫𝑹⃗‬
‫⃗⃗‬
‫‪ -2‬المرحلة الثانية‪:‬‬
‫‪ -1.2‬احصاء وتمثيل القوى‪:‬‬
‫⃗⃗‬
‫𝒇‬
‫𝑷⃗‬
‫⃗⃗‬ ‫⃗⃗‪:‬‬
‫‪ -2.2‬ايجاد قوى االحتكاك 𝒇‬
‫بتطبيق مبدأ انحفاظ الطاقة للجملة (متزحلق ولوازمه)‪:‬‬
‫𝐶𝐶𝐸 = |)⃗𝑓(𝑊| ‪𝐸𝐶𝐵 −‬‬
‫‪1‬‬
‫‪𝑚𝑣𝐵 2 − 𝑓. 𝐵𝐶 = 0‬‬
‫‪2‬‬
‫‪𝑚𝑣𝐵 2 (70). (12)2‬‬
‫=𝑓‬ ‫=‬
‫𝐶𝐵‪2‬‬ ‫)‪(2)(12‬‬
‫𝑵𝟎𝟐𝟒 = 𝒇‬

‫للمزيد زوروا موقع األستاذ طواهرية عبد العزيز للعلوم الفيزيائية‬


‫𝒎𝒐𝒄 ‪𝒘𝒘𝒘. 𝒕𝒐𝒖𝒂𝒉𝒓𝒊𝒂.‬‬
‫موعدنا بكالوريا ‪ 2020‬باذن اهلل‬

‫𝒎𝒐𝒄 ‪𝒘𝒘𝒘. 𝒕𝒐𝒖𝒂𝒉𝒓𝒊𝒂.‬‬ ‫تصحيح مقترح من طرف‪ :‬األستاذ طواهرية عبد العزيز‬ ‫‪5‬‬
‫العالمة‬
‫عناصر االجابة على الموضوع الثاني‬
‫مجزأة المجموع‬
‫التمرين األول‪:‬‬
‫‪ -1‬أنواع التفككات‪:‬‬
‫‪ -‬التفكك ‪ :α‬اصدار عنصر الهيليوم 𝑒𝐻‪. 42‬‬
‫‪ -‬التفكك ‪ :𝛽 +‬اصدار جسيم بوزتون 𝑒‪. +10‬‬
‫‪ -‬التفكك ‪ :𝛽 −‬اصدار جسيم الكترون 𝑒‪. −10‬‬
‫‪ -‬التفكك 𝛾‪ :‬اصدار اشعاعات‪.‬‬
‫‪-2‬‬
‫‪ -1.2‬استنتاج 𝑨 و𝒁‪:‬‬
‫‪𝑍 = 16‬‬
‫‪𝐴 = 𝑍 + 𝑁 = 16 + 16‬‬
‫‪𝐴 = 32‬‬
‫‪. 32‬‬
‫ومن رمز النواة الموافقة‪ :‬نواة الكبريت 𝑆‪16‬‬

‫‪ -2.2‬معادلة التفكك‪:‬‬
‫بتطبيق قانونا صودي‪:‬‬
‫‪32‬‬ ‫‪32‬‬
‫𝑃‪15‬‬ ‫→‬ ‫𝑆‪16‬‬ ‫) ‪+ −10𝑒(𝛽−‬‬
‫‪-3‬‬
‫‪ -1.3‬عدد أنوية الفوسفور ‪ 32‬المحتواة في العينة‪:‬‬
‫‪𝑁0‬‬
‫= ‪𝑛0‬‬
‫𝐴𝑁‬
‫) ‪𝑁0 = 𝑛0 . 𝑁𝐴 = (3,12. 10−10 )(6,023. 1023‬‬
‫𝑥𝑢𝑎𝑦𝑜𝑛 ‪𝑁0 = 1,879. 1014‬‬
‫‪ -2.3‬بيان أن مفعولها يزول بعد 𝒔𝒓𝒖𝒐𝒋𝟓𝟗 من لحظة الحقن‪:‬‬
‫من قانون التناقص االشعاعي‪𝑁(𝑡) = 𝑁0 𝑒 −𝜆𝑡 :‬‬
‫اذا ‪ 99%‬من األنوية االبتدائية تفككت فان األنوية المشعة المتبقية‪ 𝑁(𝑡) = 1%𝑁0 :‬أي‪𝑁(𝑡) = :‬‬
‫‪𝑁0‬‬
‫‪100‬‬
‫‪𝑁0‬‬
‫𝑡𝜆‪= 𝑁0 𝑒 −‬‬
‫‪100‬‬
‫‪1‬‬
‫𝑡𝜆‪= 𝑒 −‬‬
‫‪100‬‬

‫𝒎𝒐𝒄 ‪𝒘𝒘𝒘. 𝒕𝒐𝒖𝒂𝒉𝒓𝒊𝒂.‬‬ ‫تصحيح مقترح من طرف‪ :‬األستاذ طواهرية عبد العزيز واألستاذ القطبي يوسف‬ ‫‪1‬‬
‫‪1‬‬
‫‪ln‬‬ ‫𝑡𝜆‪= −‬‬
‫‪100‬‬
‫𝑡𝜆 = ‪ln 100‬‬
‫‪ln 100‬‬
‫𝑡=‬
‫𝜆‬
‫‪ln 100‬‬
‫𝑡=‬
‫‪ln 2‬‬
‫‪𝑡1⁄2‬‬
‫‪𝑡1/2 ln 100‬‬
‫𝑡=‬
‫‪ln 2‬‬
‫‪(14,32) ln 100‬‬
‫=𝑡‬
‫‪ln 2‬‬
‫𝑠𝑟𝑢𝑜𝑗‪𝑡 = 95.14‬‬
‫‪-4‬‬
‫‪ -1.4‬طاقة الربط النووي 𝒍𝑬‪:‬‬
‫‪: 30‬‬
‫للنواة 𝑃‪15‬‬

‫‪𝐸𝑙 ( 30‬‬ ‫𝐴‬


‫𝐶 )) 𝑋𝑍 (𝑚 ‪15𝑃 ) = ((𝑍𝑚𝑝 + 𝑁𝑚𝑛 ) −‬‬
‫‪2‬‬

‫‪= ((15. (1,00728) + 15. (1,00866)) − 29,97831) . 931,5‬‬

‫‪𝐸𝑙 ( 30‬‬
‫𝑣𝑒𝑀‪15𝑃) = 242,93‬‬

‫‪: 32‬‬
‫للنواة 𝑃‪15‬‬

‫‪𝐸𝑙 ( 32‬‬ ‫𝐴‬


‫𝐶 )) 𝑋𝑍 (𝑚 ‪15𝑃 ) = ((𝑍𝑚𝑝 + 𝑁𝑚𝑛 ) −‬‬
‫‪2‬‬

‫‪= ((15. (1,00728) + 17. (1,00866)) − 31,9739) . 931,5‬‬

‫‪𝐸𝑙 ( 32‬‬
‫𝑣𝑒𝑀‪15𝑃) = 263,16‬‬

‫‪ -2.4‬النواة األكثر استقرارا‪:‬‬


‫‪𝐸𝑙 30‬‬ ‫‪242,93‬‬
‫= )𝑃‪( 15‬‬ ‫𝑒𝑛𝑜𝑒𝑙𝑐𝑢𝑛‪= 8,072𝑀𝑒𝑣/‬‬
‫𝐴‬ ‫‪30‬‬
‫‪𝐸𝑙 32‬‬ ‫‪263,16‬‬
‫= )𝑃‪( 15‬‬ ‫𝑒𝑛𝑜𝑒𝑙𝑐𝑢𝑛‪= 8,22𝑀𝑒𝑣/‬‬
‫𝐴‬ ‫‪32‬‬
‫‪𝐸𝑙 32‬‬ ‫𝑙𝐸‬
‫‪( 15𝑃) > ( 30‬‬‫)𝑃‬
‫𝐴‬ ‫‪𝐴 15‬‬
‫𝟐𝟑 ‪.‬‬
‫النواة األكثر استقرارا هي‪𝟏𝟓𝑷 :‬‬

‫𝒎𝒐𝒄 ‪𝒘𝒘𝒘. 𝒕𝒐𝒖𝒂𝒉𝒓𝒊𝒂.‬‬ ‫تصحيح مقترح من طرف‪ :‬األستاذ طواهرية عبد العزيز واألستاذ القطبي يوسف‬ ‫‪2‬‬
‫التمرين الثاني‪:‬‬
‫أوال‪ :‬دراسة الحركة االهتزازية للنواس البسيط‪.‬‬
‫‪ -1‬تعريف دور النواس البسيط‪ :‬هو الفترة الزمنية الفاصلة بين مرورين متتاليين للجسم من نفس‬
‫الموضع ومن نفس االتجاه‪.‬‬
‫‪ -2‬قيمة الدور الذاتي 𝟎𝑻‪:‬‬
‫لدينا ‪ 10‬اهتزازات في مدة زمنية قدراها ‪ 14‬ثانية ومنه‪:‬‬
‫‪14‬‬
‫= ‪𝑇0‬‬
‫‪10‬‬
‫𝑠‪𝑇0 = 1,4‬‬
‫‪ -3‬العبارة الصحيحة‪ :‬العبارة (‪:)2‬‬

‫𝑙‬
‫√ ‪𝑇0 = 2𝜋.‬‬
‫𝑔‬

‫التحليل البعدي‪:‬‬
‫]𝑑[‬
‫= ]𝑇[‬ ‫𝑠 = ]𝑇[‬
‫= ]𝑑[ √‬
‫‪[𝑇]2‬‬

‫ومنه العبارة صحيحة‪.‬‬


‫‪ -4‬طول النواس البسيط‪:‬‬

‫𝑙‬
‫√ ‪𝑇0 = 2𝜋.‬‬
‫𝑔‬

‫𝑙‬
‫‪(𝑇0 )2 = 4𝜋 2 .‬‬
‫𝑔‬
‫𝑔 ‪(𝑇0 )2 .‬‬
‫=𝑙‬
‫‪4𝜋 2‬‬
‫)‪(1,4)2 . (9,8‬‬
‫=𝑙‬
‫‪4𝜋 2‬‬
‫𝑚‪𝑙 = 0,49‬‬
‫‪ -5‬العبارة الصحيحة والعبارة الخاطئة‪:‬‬
‫‪ -‬الدور ال يتعلق بالكتلة 𝑚 ‪ .........................‬صحيحة‬
‫‪ -‬الدور يتناسب طردا مع 𝑙√ ‪ ......................‬صحيحة‬
‫‪ -‬الدور يتناسب طردا مع 𝑔√ ‪ ....................‬خاطئة ×‬
‫‪ -‬الدور يتعلق بالسعات الصغيرة ‪ ............. 𝜃0‬خاطئة ×‬

‫𝒎𝒐𝒄 ‪𝒘𝒘𝒘. 𝒕𝒐𝒖𝒂𝒉𝒓𝒊𝒂.‬‬ ‫تصحيح مقترح من طرف‪ :‬األستاذ طواهرية عبد العزيز واألستاذ القطبي يوسف‬ ‫‪3‬‬
‫ثانيا‪ :‬حركة القذيفة‪:‬‬
‫‪ -1‬معادالت الحركة‪:‬‬
‫لدينا حسب القانون الثاني لنيوتن‪:‬‬

‫⃗𝑎𝑚 = ⃗𝐹 ∑‬

‫⃗𝑎𝑚 = ⃗⃗𝑃‬
‫باالسقاط على المحور )𝑥𝑜( نجد‪𝑎𝑥 = 0 :‬‬
‫باالسقاط على المحور )𝑦𝑜( نجد‪𝑎𝑦 = 𝑔 :‬‬
‫‪𝑣𝑥 = 𝑣𝑥0‬‬
‫‪𝑥(𝑡) = 𝑣𝑥0 𝑡 + 𝑥0‬‬
‫من الشروط االبتدائية ‪ 𝑣𝑥0 = 0.3𝑚. 𝑠 −1 :𝑡 = 0‬و𝑚‪ 𝑥0 = 0‬ومنه‪:‬‬
‫)𝟏( … … … … 𝒕𝟑 ‪𝒙(𝒕) = 𝟎,‬‬
‫‪𝑣𝑦 = 𝑔𝑡 + 𝑣𝑥0‬‬
‫‪1‬‬
‫‪𝑦(𝑡) = 𝑔𝑡 2 + 𝑣𝑦0 𝑡 + 𝑦0‬‬
‫‪2‬‬
‫من الشروط االبتدائية ‪ 𝑣𝑦0 = 0𝑚. 𝑠 −1 :𝑡 = 0‬و𝑚‪ 𝑦0 = 0‬ومنه‪:‬‬
‫𝟐 𝟏‬
‫= )𝒕(𝒚‬ ‫)𝟐( ‪𝒈𝒕 … … … … .‬‬
‫𝟐‬
‫‪ -2‬معادلة المسار‪:‬‬
‫)𝑡(𝑥‬
‫= 𝑡 وبالتعويض في (‪ )2‬نجد أن‪:‬‬ ‫من (‪ )1‬نجد أن‪:‬‬
‫‪0,3‬‬

‫‪1 𝑥(𝑡) 2‬‬


‫( 𝑔 = )𝑡(𝑦‬ ‫)‬
‫‪2‬‬ ‫‪0,3‬‬
‫𝟐𝒙𝟒 ‪𝒚(𝒕) = 𝟓𝟒,‬‬
‫نقطة االصطدام 𝐸 تتميز بـ‪𝑦 = 1,5 − 0,49 = 1,01𝑚 :‬‬
‫بالتعويض في معادلة المسار نجد‪:‬‬

‫𝑦‬
‫√=𝑥‬
‫‪54,4‬‬

‫‪1,01‬‬
‫√=𝑥‬
‫‪54,4‬‬

‫𝒎𝟒𝟏 ‪𝒙 = 𝟎,‬‬
‫ومنه‪𝐸(0,14𝑚; 1.01𝑚) :‬‬

‫𝒎𝒐𝒄 ‪𝒘𝒘𝒘. 𝒕𝒐𝒖𝒂𝒉𝒓𝒊𝒂.‬‬ ‫تصحيح مقترح من طرف‪ :‬األستاذ طواهرية عبد العزيز واألستاذ القطبي يوسف‬ ‫‪4‬‬
‫‪ -3‬خصائص شعاع السرعة عند 𝑬 ‪:‬‬
‫‪ 𝑣𝑥 = 0,3𝑚. 𝑠 −1‬و ‪𝑣𝑦 = g = 9,8𝑚. 𝑠 −1‬‬
‫اذا شعاع السرعة عند النقطة 𝐸‪:‬‬
‫⃗𝒋𝟖 ‪⃗⃗𝑬 = 𝟎. 𝟑𝒊⃗ + 𝟗,‬‬
‫𝒗‬
‫التمرين التجريبي‪:‬‬
‫أوال‪ :‬دراسة تفاعل الحمض )𝑩( مع شوارد البرمنغنات‪:‬‬
‫‪ -1‬تعريف المؤكسد والمرجع‪:‬‬
‫‪ -‬المؤكسد‪ :‬هو كل فرد كيميائي قادر على اكتساب الكترون أو أكثر خالل تحول كيميائي‪.‬‬
‫‪ -‬المرجع‪ :‬هو كل فرد كيميائي قادر على فقد الكترون أو أكثر خالل تحول كيميائي‪.‬‬
‫‪ -2‬بيان أن التفاعل تفاعل أكسدة – ارجاع‪:‬‬
‫لدينا من معادلة التفاعل‪:‬‬
‫‪+‬‬ ‫‪−‬‬
‫‪𝐶3 𝐻8 𝑂(𝑙) = 𝐶3 𝐻6 𝑂(𝑙) +‬‬ ‫)𝑞𝑎(𝐻‪2‬‬ ‫𝑒‪+ 2‬‬
‫حدثت عملية أكسدة‪ .‬الثنائية‪(𝐶3 𝐻6 𝑂(𝑙) ⁄𝐶3 𝐻8 𝑂(𝑙) ) :‬‬
‫‪−‬‬ ‫‪+‬‬
‫)𝑞𝑎(‪𝑀𝑛𝑂4‬‬ ‫)𝑞𝑎(𝐻‪+ 8‬‬ ‫)𝑙(𝑂 ‪+ 5𝑒 − = 𝑀𝑛+2 + 4𝐻2‬‬
‫‪−‬‬
‫)𝑞𝑎(‪(Mn𝑂4‬‬ ‫حدثت عملية ارجاع‪ .‬الثنائية‪⁄𝑀𝑛+2 ) :‬‬
‫ومن التفاعل تفاعل أكسدة – ارجاع‪.‬‬
‫‪ -3‬دور حمض الكبريت‪ :‬توفير الوسط الحامضي أي توفير شوارد الهيدروجين ‪ 𝐻+‬المطلوبة لحدوث‬
‫التفاعل‪.‬‬
‫‪ -4‬جدول تقدم التفاعل‪:‬‬
‫معادلة التفاعل‬ ‫‪−‬‬
‫)𝑞𝑎(‪5𝐶3 𝐻8 𝑂(𝑙) + 2𝑀𝑛𝑂4‬‬ ‫‪+‬‬
‫)𝑞𝑎(𝐻‪+ 6‬‬ ‫)𝑙(𝑂 ‪= 5𝐶3 𝐻6 𝑂(𝑙) + 2𝑀𝑛+2 + 8𝐻2‬‬

‫تقدم التفاعل حالة التفاعل‬ ‫كميات المادة بوحدة 𝒍𝒐𝒎‬


‫االبتدائية‬ ‫𝟎 = )𝒕(𝒙‬ ‫‪𝑛1‬‬ ‫‪𝑛0‬‬ ‫بوفرة‬ ‫‪0‬‬ ‫‪0‬‬ ‫بوفرة‬
‫االنتقالية‬ ‫𝒕𝒙 = )𝒕(𝒙‬ ‫𝑡𝑥‪𝑛1 − 5‬‬ ‫بوفرة 𝑡𝑥‪ 5𝑥𝑡 2‬بوفرة 𝑡𝑥‪𝑛0 − 2‬‬
‫النهائية‬ ‫𝒇𝒙 = )𝒕(𝒙‬ ‫𝑓𝑥‪𝑛1 − 5‬‬ ‫بوفرة 𝑓𝑥‪ 5𝑥𝑓 2‬بوفرة 𝑓𝑥‪𝑛0 − 2‬‬
‫‪ -‬حساب التقدم األعظمي‪:‬‬
‫𝑚‬ ‫‪3,75‬‬
‫𝑀 ‪𝑛1‬‬
‫= 𝑥𝑎𝑚𝑥 ⇒ ‪𝑛1 − 5𝑥𝑚𝑎𝑥 = 0‬‬ ‫=‬ ‫𝑙𝑜𝑚 ‪= 60 ⇒ 𝑥𝑚𝑎𝑥 = 1,25. 10−2‬‬
‫‪5‬‬ ‫‪5‬‬ ‫‪5‬‬
‫) ‪𝑛0 𝐶0 . 𝑉0 (0,1)(5. 10−2‬‬
‫= 𝑥𝑎𝑚𝑥 ⇒ ‪𝑛0 − 2𝑥𝑚𝑎𝑥 = 0‬‬ ‫=‬ ‫=‬
‫‪2‬‬ ‫‪2‬‬ ‫‪2‬‬
‫𝑙𝑜𝑚 ‪⇒ 𝑥𝑚𝑎𝑥 = 2,5. 10−3‬‬
‫ومنه التقدم األعظمي‪ 𝒙𝒎𝒂𝒙 = 𝟐, 𝟓. 𝟏𝟎−𝟑 𝒎𝒐𝒍 :‬وبرمنغنات البوتاسيوم هي المتفاعل المحد‪.‬‬
‫‪-5‬‬

‫𝒎𝒐𝒄 ‪𝒘𝒘𝒘. 𝒕𝒐𝒖𝒂𝒉𝒓𝒊𝒂.‬‬ ‫تصحيح مقترح من طرف‪ :‬األستاذ طواهرية عبد العزيز واألستاذ القطبي يوسف‬ ‫‪5‬‬
‫‪ -1.5‬بيان أن التفاعل تام‪:‬‬
‫حتى يكون التفاعل تام يجب أن‪𝑥𝑓 = 𝑥𝑚𝑎𝑥 = 2,5. 10−3 𝑚𝑜𝑙 :‬‬
‫من البيان نجد أن‪𝑛(𝐵)𝑓 = 5. 10−2 𝑚𝑜𝑙 :‬‬
‫ولدينا من جدول التقدم في الحالة النهائية‪:‬‬
‫‪𝑛(𝐵)𝑓 = 𝑛1 − 5𝑥𝑓 = 5. 10−2‬‬
‫‪𝑛(𝐵)𝑓 = 6,25. 10−2 − 5𝑥𝑓 = 5. 10−2‬‬
‫‪6,25. 10−2 − 5. 10−2‬‬
‫= 𝑓𝑥‬
‫‪5‬‬
‫𝒍𝒐𝒎 𝟑‪𝒙𝒇 = 𝒙𝒎𝒂𝒙 = 𝟐, 𝟓. 𝟏𝟎−‬‬
‫ومنه التفاعل تام‪.‬‬
‫‪ -2.5‬تعريف زمن نصف التفاعل 𝟐‪ :𝒕𝟏/‬هو الزمن الالزم لبلوغ التفاعل نصف تقدمه النهائي‬
‫𝑓𝑥‬
‫= ‪.(𝑥𝑡1/2‬‬ ‫)‬
‫‪2‬‬

‫قيمته‪ :‬من البيان وباالسقاط نجد أن‪𝒕𝟏/𝟐 = 𝟐 , 𝟒𝒎𝒊𝒏 :‬‬


‫‪ -3.5‬حساب السرعة الحجمية الختفاء الكحول )𝑩( في اللحظة 𝟎 = 𝒕‪:‬‬
‫𝐵𝑛𝑑 ‪1‬‬
‫‪𝑣𝑣𝑜𝑙(𝐵) = − .‬‬
‫𝑡𝑑 𝑇𝑉‬
‫‪1‬‬ ‫‪62,5. 10−3 − 0‬‬
‫)𝐵(𝑙𝑜𝑣𝑣‬ ‫‪=−‬‬ ‫‪.‬‬
‫‪6. 10−2‬‬ ‫‪0 − 18‬‬
‫𝟏‪𝒗𝒗𝒐𝒍(𝑩) = 𝟓, 𝟕𝟖. 𝟏𝟎−𝟐 𝒎𝒐𝒍. 𝒍−𝟏 . 𝒎𝒊𝒏−‬‬
‫ثانيا‪ :‬دراسة تفاعل الكحول )𝑩( مع حمض االيثانويك 𝑯𝑶𝑶𝑪 𝟑𝑯𝑪‪:‬‬
‫‪ -1‬دور حمض الكبريت المركز في هذا التفاعل‪:‬‬
‫دوره وسيط لتسريع التفاعل (تسريع تشكيل األستر)‬
‫‪ -2‬معادل التفاعل‪:‬‬
‫)𝑙(𝑂 ‪𝐶3 𝐻7 𝑂𝐻(𝑎𝑞) + 𝐶𝐻3 𝐶𝑂𝑂𝐻(𝑎𝑞) = 𝐶𝐻3 𝐶𝑂𝑂𝐶3 𝐻7(𝑎𝑞) + 𝐻2‬‬
‫‪ -3‬جدول تقدم التفاعل‪:‬‬
‫معادلة التفاعل‬ ‫)𝑙(𝑂 ‪𝐶3 𝐻7 𝑂𝐻(𝑎𝑞) + 𝐶𝐻3 𝐶𝑂𝑂𝐻(𝑎𝑞) = 𝐶𝐻3 𝐶𝑂𝑂𝐶3 𝐻7(𝑎𝑞) + 𝐻2‬‬
‫تقدم التفاعل حالة التفاعل‬ ‫كميات المادة بوحدة 𝒍𝒐𝒎𝒎‬
‫‪0‬‬ ‫‪0‬‬
‫االبتدائية‬ ‫𝟎 = )𝒕(𝒙‬ ‫‪50‬‬ ‫‪50‬‬

‫𝑡𝑥‬ ‫𝑡𝑥‬
‫االنتقالية‬ ‫𝒕𝒙 = )𝒕(𝒙‬ ‫𝑡𝑥 ‪50 −‬‬ ‫𝑡𝑥 ‪50 −‬‬

‫𝑓𝑥‬ ‫𝑓𝑥‬
‫النهائية‬ ‫𝒇𝒙 = )𝒕(𝒙‬ ‫𝑓𝑥 ‪50 −‬‬ ‫𝑓𝑥 ‪50 −‬‬

‫𝒎𝒐𝒄 ‪𝒘𝒘𝒘. 𝒕𝒐𝒖𝒂𝒉𝒓𝒊𝒂.‬‬ ‫تصحيح مقترح من طرف‪ :‬األستاذ طواهرية عبد العزيز واألستاذ القطبي يوسف‬ ‫‪6‬‬
‫‪ -‬التقدم األعظمي‪𝒙𝒎𝒂𝒙 = 𝟓. 𝟏𝟎−𝟐 𝒎𝒐𝒍 :‬‬
‫‪ -1.4‬البروتوكول التجريبي‪:‬‬
‫‪ -‬مضع المزيج التفاعلي في حوجلة (𝑙𝑜𝑚𝑚‪ 50‬لك من 𝐴 و𝐵) ونأخذ منه ‪ 10‬أنابيب اختبار بحجم‬
‫ثابت‪.‬‬
‫‪ -‬في كل ‪ 10‬دقائق نأخذ أنبوب أختبار نبرده بواسطة الجليد لتوقيف التفاعل ونضع في بيشر‪.‬‬
‫ثم نعاير كمية الكحول المتبقية بواسطة برمنغنات البوتاسيوم معلومة التركيز (البيشر‪ ،‬الوسط التفاعلي‪،‬‬
‫السحاحة‪)𝐾𝑀𝑛𝑂4 ،‬‬
‫‪ -‬ثم نوقف المعايرة عند الوصول لحجم التكافؤ ونحسب كمية الكحول المتبقية انطالقا من قانون التكافؤ‬
‫𝐵𝑛‬ ‫‪𝐾𝑀𝑛𝑂4‬‬
‫)‬ ‫=‬ ‫(‬
‫‪5‬‬ ‫‪2‬‬

‫‪ -‬نكرر العملية كل مرة ونسجل قيم 𝐵𝑛 بداللة الزمن ونرسم البيان‪.‬‬


‫‪ -2.4‬قيمة التقدم النهائي 𝒇𝒙‪:‬‬
‫من البيان‪𝑛(𝐵)𝑓 = 20𝑚𝑚𝑜𝑙 :‬‬
‫من جدول التقدم السابق في الحالة النهائية‪:‬‬
‫‪𝑛(𝐵)𝑓 = 50 − 𝑥𝑓 = 20‬‬
‫𝒍𝒐𝒎𝒎𝟎𝟑 = 𝒇𝒙‬
‫لدينا‪ 𝒙𝒇 < 𝒙𝒎𝒂𝒙 :‬ومنه التفاعل غير تام‪.‬‬
‫‪ -3.4‬حساب مردود التفاعل‪:‬‬
‫𝑓𝑥‬ ‫‪30‬‬
‫=𝑟‬ ‫=‬
‫𝑥𝑎𝑚𝑥‬ ‫‪50‬‬
‫‪𝒓 = 𝟔𝟎%‬‬
‫ومنه الكحول ثانوي‪.‬‬
‫‪ -5‬نعم يمكن الحصول على األستر بتفاعل آخر تام‪ ،‬سريع وناشر للحرارة وذلك باستعمال كلور األسيل‬
‫𝒍𝑪𝑶𝑪 𝟑𝑯𝑪 بدال من الحمض‪.‬‬

‫𝒎𝒐𝒄 ‪𝒘𝒘𝒘. 𝒕𝒐𝒖𝒂𝒉𝒓𝒊𝒂.‬‬ ‫تصحيح مقترح من طرف‪ :‬األستاذ طواهرية عبد العزيز واألستاذ القطبي يوسف‬ ‫‪7‬‬
‫✍‬
‫اجلمهورية اجلزائرية الدميقراطية الشعبية‬
‫الديوان الوطين لالمتحاانت واملسابقات‬ ‫وزارة الرتبية الوطنية‬
‫دورة‪2018 :‬‬ ‫امتحان بكالوراي التعليم الثانوي‬
‫الشعبة‪ :‬علوم جتريبية‬
‫املدة‪ 30 :‬سا و ‪ 03‬د‬ ‫اختبار يف مادة‪ :‬علوم فيزايئية‬

‫على المترشح أن يختار أحد الموضوعين اآلتيين‪:‬‬


‫الموضوع األول‬
‫يحتوي الموضوع األول على (‪ )40‬صفحات (من الصفحة ‪ 1‬من ‪ 8‬إلى الصفحة ‪ 0‬من ‪)8‬‬

‫الجزء األول‪ 11( :‬نقطة)‬


‫التمرين األول‪ 46( :‬نقاط)‬
‫خالل األلعاب األولمبية التي جرت بالب ارزيل سنة ‪ ،6102‬تحصل األمريكي ريان كروزر)‪(Ryan crouser‬‬

‫‪ v0‬‬ ‫الجّلة أللعاب القوى على إثر رمية قدرها ) ‪. ( D‬‬
‫على الميدالية الذهبية في رياضة رمي ُ‬
‫الجّلة ‪ G‬في المعلم ) ‪(o, x, y‬‬
‫بإهمال تأثير الهواء‪ ،‬تمت دراسة محاكاة حركة مركز عطالة ُ‬
‫‪h‬‬ ‫المرتبط بمرجع أرضي نعتبره غاليليا‪ ،‬ابتداء من لحظة رميها )‪ (t  0‬على ارتفاع ‪h‬‬
‫من سطح األرض إلى غاية ارتطامها به (الشكل‪ )-0-‬فتم الحصول على‬
‫الشكل‪-1-‬‬
‫البيانية التالية‪:‬‬
‫المنحنيات ّ‬
‫) ‪v y (m.s 1‬‬
‫)‪y (m‬‬
‫البيان‪-0-‬‬ ‫)‪x ( m‬‬

‫‪1,96‬‬ ‫)‪t (s‬‬


‫البيان‪-1-‬‬

‫‪0, 25‬‬
‫البيان‪-2-‬‬

‫‪1, 3‬‬ ‫‪5‬‬


‫)‪x ( m‬‬ ‫)‪t (s‬‬

‫‪4,5‬‬ ‫‪0,5‬‬
‫‪-12,25‬‬
‫البيانية‪:‬‬
‫‪ .1‬باالعتماد على المنحنيات ّ‬
‫حدد طبيعة حركة مركز عطالة الجلة ‪ G‬على كل من المحورين )‪ (ox‬و ) ‪ (oy‬مع تبرير إجابتك‪.‬‬ ‫‪ّ .0.0‬‬
‫حدد قيم المقادير التالية‪ :‬مركبتي السرعة االبتدائية ‪ v 0x‬و ‪ ، v 0 y‬مركبتي التسارع ‪ a x‬و ‪ a y‬واالرتفاع ‪. h‬‬
‫‪ّ .6.0‬‬
‫‪ .1.0‬اكتب المعادلتين الزمنيتين ) ‪ x (t‬و ) ‪ y (t‬لحركة ‪ G‬في المعلم ) ‪. (o, x, y‬‬
‫‪ .1.0‬اكتب معادلة البيان ‪ ، 2 ‬ماذا تمثل؟‬

‫صفحة ‪ 1‬من ‪8‬‬


‫✍‬
‫اختبار يف مادة‪ :‬علوم فيزايئية ‪ /‬الشعبة‪ :‬علوم جتريبية‪ /‬بكالوراي ‪8102‬‬

‫‪ .1.0‬ما هي قيمة كل من زاوية القذف ‪ ‬والسرعة التي قذفت بها الجّلة ‪ v 0‬؟‬
‫مكنت الرياضي من الفوز بالميدالية الذهبية ؟‬
‫‪ .2.0‬ما هي قيمة المسافة األفقية ) ‪ ( D‬التي ّ‬
‫‪ .2‬أنجز مخطط الحصيلة الطاقوية للجملة (الجّلة) بين اللحظتين ‪ t  0‬و ‪ t  2, 25s‬ثم اكتب معادلة انحفاظ الطاقة‬
‫واستنتج سرعة مركز عطالة الجّلة عند لحظة ارتطامها بسطح األرض ‪. t  2, 25s‬‬
‫حدد خصائص شعاع سرعة مركز عطالة الجّلة ‪ G‬عند اللحظة ‪. t  2, 25s‬‬ ‫‪ّ .1‬‬
‫‪ .0‬جد عبارة الطاقة الكلية للجملة (جّلة ‪ +‬أرض) عند اللحظتين المذكورتين سابقا بداللة كل من‪:‬‬
‫‪ g ، h ، v0‬و ‪( m‬كتلة الجّلة)‪ .‬ماذا تستنتج ؟ (نعتبر مستوى سطح األرض مرجعا لقياس الطاقة الكامنة الثقالية)‪.‬‬
‫يعطى ‪g  9,8m .s 2 :‬‬

‫التمرين الثاني‪ 40( :‬نقاط)‬


‫الدرقية‪.‬‬
‫الغّدة ّ‬
‫‪  I‬يعتبر اليود من بين العناصر الكيميائية التي تُستخدم في عالج األمراض السرطانية التي تُصيب ُ‬
‫بعينة من النظير ‪53 I‬‬
‫يستخدم نظير اليود المشع ‪ 53 I‬الذي نصف عمره ‪ t 1  8 jours‬في حقن شخص مصاب ّ‬
‫‪131‬‬ ‫‪131‬‬
‫‪2‬‬

‫يوم ‪ 10‬ماي ‪ 8112‬على الساعة الثامنة مساء‪.‬‬ ‫‪m0  1,00 103 mg‬‬ ‫كتلتها‬
‫‪.‬‬ ‫حدد تركيب نواة اليود ‪I‬‬
‫‪131‬‬
‫‪53‬‬ ‫‪ّ .1‬‬
‫أن كتلة نواة واحدة من اليود ‪I‬‬
‫علما ّ‬
‫العينة السابقة ‪ً ،‬‬
‫‪ .2‬احسب قيمة ‪ ، N 0‬عدد األنوية االبتدائية الموجودة في ّ‬
‫‪131‬‬
‫‪53‬‬

‫‪. m(131‬‬‫‪53 I )  2,176  10‬‬


‫‪25‬‬
‫هي ‪kg‬‬
‫فينبعث إلكترون ‪. 10 e‬‬ ‫‪ .0‬تتفكك نواة النظير ‪I‬‬
‫‪131‬‬
‫‪53‬‬

‫تفسر انبعاث إلكترون من النواة؟‬


‫‪ .1.1‬كيف ّ‬
‫‪. 131‬‬
‫المنمذجة لتفكك نواة اليود ‪53 I‬‬
‫‪ .8.1‬اعتمادا على السند اآلتي‪ ،‬اكتب معادلة التفاعل ُ‬
‫‪51‬‬ ‫‪Sb‬‬ ‫‪Te‬‬
‫‪52‬‬ ‫‪53‬‬ ‫‪I‬‬ ‫‪54‬‬ ‫‪Xe‬‬ ‫‪55‬‬ ‫‪Cs‬‬
‫‪ .1.1‬اكتب عبارة قانون التناقص اإلشعاعي‪.‬‬
‫عرف زمن نصف العمر‪ ،‬ثم استنتج العالقة بين ‪ t 1‬و ثابت التفكك ‪. ‬‬
‫‪ّ .3.1‬‬
‫‪2‬‬

‫لعينة السابقة عند اللحظة ‪. t  0‬‬‫‪ .3.1‬احسب قيمة النشاط اإلشعاعي ‪ A 0‬ل ّ‬
‫‪ .4‬يمكث الشخص المصاب في المستشفى تحت المراقبة الطبية لعدة أيام‪ ،‬حتى تصل قيمة التناقص في النشاط‬
‫) ‪E (105 MeV‬‬ ‫اإلشعاعي إلى ‪ 40 0 0‬من قيمته االبتدائية‪.‬‬
‫‪92 p  144 n‬‬ ‫حدد تاريخ وتوقيت خروج المريض من المستشفى‪.‬‬ ‫‪ّ -‬‬
‫‪2,21619‬‬
‫‪ُ  II‬يستعمل اليورانيوم ‪ 813‬كوقود لتوليد الطاقة الكهربائية في مفاعل نووي‪.‬‬
‫‪U  01n‬‬
‫‪235‬‬
‫‪92‬‬ ‫المخطط الطاقوي ألحد التفاعالت النووية الحادثة في هذا المفاعل‬
‫‪2,19836‬‬
‫‪La  35‬‬
‫‪148‬‬ ‫‪85‬‬
‫‪Br  x 01n‬‬ ‫ُممثلة في الشكل‪.-2-‬‬
‫‪2,19669‬‬ ‫‪Z‬‬

‫الشكل ‪-2-‬‬

‫صفحة ‪ 2‬من ‪8‬‬


‫✍‬
‫اختبار يف مادة‪ :‬علوم فيزايئية ‪ /‬الشعبة‪ :‬علوم جتريبية‪ /‬بكالوراي ‪8102‬‬

‫‪ .1‬اكتب معادلة التفاعل النووي الحادث‪ ،‬مع تحديد نوعه‪.‬‬


‫‪ .2‬باستخدام قانوني االنحفاظ‪ ،‬جد قيمة كل من ‪ x‬و ‪. z‬‬
‫المحررة ‪ Elib‬من التفاعل النووي مقدرة بالـ ‪. MeV‬‬
‫ّ‬ ‫‪ .0‬اعتمادا على الشكل ‪ ،-8-‬استنتج الطاقة‬
‫‪. r  30 0 0‬‬ ‫أن المفاعل النووي ينتج استطاعة كهربائية متوسطة مقدارها ‪ Pe  900MW‬بمردود طاقوي‬‫لما ّ‬‫‪ .4‬ع ً‬
‫‪ .1.4‬احسب الطاقة الكهربائية الناتجة ‪ Eelec‬خالل يوم واحد‪.‬‬
‫‪ .8.3‬احسب الطاقة المحررة من المفاعل النووي ‪ Elib‬عندئذ‪.‬‬
‫‪ .1.3‬استنتج مقدار الكتلة ‪ m‬لليورانيوم ‪ 813‬المستهلكة من طرف هذا المفاعل النووي خالل يوم واحد ‪.‬‬
‫‪2‬‬
‫‪1‬‬ ‫‪ .5‬ليكن التفاعل المنمذج بالمعادلة التالية ‪H  13 H  24 He  01n :‬‬
‫الطاقة المحررة لكل نيوكليون(نوية) من هذا التفاعل النووي هي ‪. 3,53Mev / nuc :‬‬
‫حدد نوع هذا التفاعل النووي‪.‬‬
‫‪ّ .1.3‬‬
‫فضل عن التفاعل السابق المذكور في (‪.)1.II‬‬ ‫‪ .8.3‬بالرغم من صعوبة تحقيق هذا التفاعل عمليا إالّ ّأنه ُي ّ‬
‫فضل هذا التفاعل عن التفاعل السابق ؟ ّبرر‪.‬‬‫ب) لماذا ُي ّ‬ ‫أ) أين تكمن هذه الصعوبة؟‬
‫‪92U )  3,9036.10‬‬
‫‪m( 235‬‬ ‫‪22‬‬
‫المعطيات‪ ، 1MW  106 W ، 1Mev  1,6 1013 J :‬كتلة نواة اليورانيوم ‪g : 813‬‬

‫الجزء الثاني‪ 40( :‬نقاط)‬


‫التمرين التجريبي‪ 40( :‬نقاط)‬
‫ظف تجاري يحتوي على حمض الالكتيك ذي الصيغة الجزيئية ‪ C3 H 6O3‬المعلومات التالية‪:‬‬
‫نق أر على لصيقة قارورة من ّ‬
‫‪ -‬الكتلة المولية الجزيئية لحمض الالكتيك ‪M (C3 H 6O3 )  90 g.mol 1 :‬‬
‫‪ -‬الكتلة الحجمية للمنظف التجاري‪  1,13Kg.L1 :‬‬
‫المراد تنظيفه مع التسخين‪.‬‬ ‫ظف التجاري ال ّ‬
‫مركز في الجهاز ُ‬ ‫‪ُ -‬يفرغ المن ّ‬
‫المشكلة أساسا من كربونات الكالسيوم‬
‫سخان مائي و ُ‬
‫ُيستعمل هذا المنظف إلزالة الطبقة الكلسية المترسبة على جدران ّ‬
‫)‪. CaCO3 (s‬‬
‫ظف التجاري وتحديد نسبته المئوية الكتلية ‪ ، P%‬نحّقق التجربتين اآلتيتين‪:‬‬
‫من أجل دراسة فعالية هذا المن ّ‬
‫التجربة األولى‪:‬‬
‫ظف التجاري الذي‬
‫حضر محلوال ) ‪ ( S‬حجمه ‪ Vs  500mL‬وتركيزه المولي ‪ ca‬مخففا ‪ 011‬مرة‪ ،‬انطالقا من المن ّ‬‫‪ُ .1‬ن ّ‬
‫تركيزه المولي ‪. c0‬‬
‫‪ .0.0‬ما هو حجم المحلول التجاري ‪ V0‬الواجب استعماله لتحضير المحلول ) ‪ ( S‬؟‬
‫‪ .6.0‬اذكر البروتوكول التجريبي الالزم لتحضير المحلول ) ‪. ( S‬‬
‫المنمذج بالمعادلة‪:‬‬
‫‪ .2‬لدراسة حركية تفاعل حمض الالكتيك مع كربونات الكالسيوم )‪ُ CaCO3 (s‬‬
‫) ‪CaCO3 ( s)  2C3 H 6O3 (aq)  CO2 ( g )  Ca 2 (aq)  2C3 H 5O3 (aq)  H 2O(l‬‬

‫صفحة ‪ 0‬من ‪8‬‬


‫✍‬
‫اختبار يف مادة‪ :‬علوم فيزايئية ‪ /‬الشعبة‪ :‬علوم جتريبية‪ /‬بكالوراي ‪8102‬‬

‫ُندخل في دورق حجمه ‪ ، V  600mL‬الكتلة ‪ m  0,3g‬من كربونات الكالسيوم )‪ ، CaCO3 (s‬ونسكب فيه عند‬
‫اللحظة ‪ t  0‬حجما ‪ Va  120mL‬من المحلول ) ‪ . ( S‬نقيس في كل لحظة ضغط غاز ثاني أكسيد الفحم ) ‪P(CO2‬‬
‫)‪PCO2 (hPa‬‬ ‫الدورق عند درجة ح اررة ثابتة ‪ . 25C‬بواسطة القط الضغط‬
‫داخل ّ‬
‫لجهاز الـ ‪ ExAO‬تحصلنا على البيان الممثل في الشكل‪. -3-‬‬
‫الشكل(‪)0‬‬
‫‪ .0.6‬في ظروف التجربة يمكن اعتبار الغاز ‪ CO2‬مثالي‪.‬‬
‫باالعتماد على جدول التقدم‪ ،‬أوجد عبارة التقدم ) ‪ x(t‬للتفاعل عند‬
‫لحظة ‪ t‬بداللة ‪ PCO (t ) ، T ، VCO :‬و ‪. R‬‬
‫‪2‬‬ ‫‪2‬‬

‫‪30‬‬ ‫أن هذا التفاعل تام‪.‬‬


‫قيمة التقدم النهائي ‪ ، X f‬ثم أثبت ّ‬ ‫حدد‬
‫‪ّ .6.6‬‬
‫)‪t ( s‬‬
‫بيانيا زمن نصف التفاعل ‪. t1/2‬‬ ‫حدد‬
‫‪ّ .1.6‬‬
‫‪0‬‬ ‫‪60‬‬
‫الترسبات الكلسية ُيطلب استعمال المنظف التجاري ّ‬
‫مركاز مع التسخين‪،‬‬ ‫‪ .1.6‬خالل عملية إزالة ّ‬
‫ما هو أثر هذين العاملين على المدة الزمنية الالزمة إلزالة الراسب؟ عّلل إجابتك‪.‬‬
‫يعطى‪ ، M (CaCO 3 )  100 g .moL :‬ثابت الغازات المثالية ‪. R  8,314SI :‬‬
‫‪1‬‬

‫التجربة الثانية‪:‬‬
‫ظف التجاري‪ ،‬نأخذ حجما ‪ Va'  5mL‬من‬
‫من أجل تحديد النسبة المئوية الكتلية ‪ P%‬لحمض الالكتيك في المن ّ‬
‫نعاير المحلول الناتج عن طريق قياس الـ ‪ pH‬بواسطة‬ ‫المحلول ) ‪ ، ( S‬ونضيف إليه ‪ 100mL‬من الماء المقطر‪ ،‬ثم ّ‬
‫محلول هيدروكسيد الصوديوم ))‪ ( Na  (aq)  OH  (aq‬ذي التركيز المولي ‪. Cb  0, 02mol.L1‬‬
‫‪ .1‬مثّـل برسم تخطيطي التركيب التجريبي للمعايرة معينا أسماء المعدات والمحاليل‪.‬‬
‫المنمذجة للتحول الحادث أثناء المعايرة‪.‬‬‫‪ .2‬اكتب المعادلة الكيميائية ُ‬
‫‪.‬‬ ‫‪ُ .1‬يمثل الشكل ‪ 4 ‬المنحنيين البيانيين‪ pH  f (Vb ) :‬و ) ‪ g (Vb‬‬
‫‪dpH‬‬
‫‪dVb‬‬
‫‪ .0.1‬في رأيك‪ ،‬ما هو سبب‬
‫‪pH‬‬
‫الشكل(‪)3‬‬ ‫إضافة الماء المقطر إلى‬
‫الحجم '‪ Va‬؟ هل يؤثر ذلك‬
‫على حجم األساس‬
‫) ‪pH  f (Vb‬‬ ‫المسكوب عند التكافؤ؟ عّلل‪.‬‬
‫‪ .6.1‬احسب التركيز‬
‫‪dpH‬‬
‫) ‪ g (Vb‬‬
‫‪2‬‬ ‫‪dVb‬‬ ‫المولي ‪ ، ca‬ثم استنتج‬
‫)‪Vb (mL‬‬
‫ظف‬
‫التركيز المولي ‪ c0‬للمن ّ‬
‫‪0‬‬ ‫‪2‬‬ ‫التجاري‪.‬‬
‫ظف التجاري‪ ،‬ثم استنتج النسبة المئوية ‪. P%‬‬
‫‪ .1.1‬احسب كتلة حمض الالكتيك المتواجدة في ‪ 1L‬من المن ّ‬
‫انتهى الموضوع األول‬
‫صفحة ‪ 4‬من ‪8‬‬
‫✍‬
‫اختبار يف مادة‪ :‬علوم فيزايئية ‪ /‬الشعبة‪ :‬علوم جتريبية‪ /‬بكالوراي ‪8102‬‬

‫الموضوع الثاني‬
‫يحتوي الموضوع الثاني على (‪ )40‬صفحات (من الصفحة ‪ 5‬من ‪ 8‬إلى الصفحة ‪ 8‬من ‪)8‬‬

‫الجزء األول‪ 11( :‬نقطة)‬


‫التمرين األول‪ 46( :‬نقاط)‬
‫الكوم سات ‪ -0-‬قمر اصطناعي جزائري تم تركيبه على مستوى مركز تطوير األقمار‬
‫االصطناعية ببئر الجير بوالية وهران‪ ،‬من شأنه توفير خدمة االتصاالت واألنترنت‪،‬‬
‫تم إطالقه بتاريخ ‪ 10‬ديسمبر‪.6102‬‬
‫بث القنوات االذاعية والتلفزيونية‪ّ ،...‬‬
‫قمر اصطناعياً ) ‪ ( S‬كتلته ‪ m‬يدور حول األرض على بعد ‪ r‬من مركزها بحركة دائرية منتظمة‪.‬‬
‫‪ .1‬نعتبر ًا‬
‫لدراسة حركة هذا القمر االصطناعي‪ ،‬نختار معلما مرتبطا بمرجع عطالي مناسب‪.‬‬
‫عرف المعلم المرتبط به‪.‬‬
‫‪ .0.0‬ما هو هذا المرجع؟ ولماذا نعتبره عطاليا؟ ثم ّ‬
‫‪‬‬
‫‪v 2 106  m 2  s 2 ‬‬ ‫طبقها األرض ‪ T‬على القمر االصطناعي ) ‪. ( S‬‬ ‫ّ‬ ‫ت‬
‫ُ‬ ‫التي‬ ‫‪F‬‬ ‫كيفيا شعاع القوة ‪T S‬‬
‫‪ .6.0‬مثّل ً‬
‫‪‬‬
‫عبر عن شدة شعاع القوة ‪ FT S‬بداللة المقادير ‪. r ، m ، MT ، G‬‬
‫‪ّ .1.0‬‬
‫حيث‪ M T :‬كتلة األرض‪.‬‬
‫‪ .1.0‬بتطبيق القانون الثاني لنيوتن في المرجع المختار‪ ،‬جد عبارة مربع‬
‫سرعة مركز عطالة القمر االصطناعي ‪ v 2‬بداللة ‪ MT ، G‬و ‪. r‬‬
‫‪ .2‬يمثل المنحنى البياني المقابل تطور مربع السرعة المدارية للقمر‬
‫‪4,8‬‬
‫االصطناعي ) ‪ ( S‬بداللة مقلوب البعد ‪( v 2  f  ‬الشكل‪.)-0-‬‬
‫‪1‬‬
‫‪r‬‬
‫‪0‬‬ ‫‪2, 4‬‬
‫‪ 108  m 1 ‬‬ ‫‪ .0.6‬اكتب معادلة المنحى البياني‪ ،‬واستنتج قيمة كتلة األرض ‪. M T‬‬
‫‪1‬‬
‫الشكل‪-1-‬‬ ‫‪r‬‬
‫‪ .6.6‬جد عبارة الدور ‪ T‬للقمر االصطناعي ) ‪ ( S‬بداللة ‪ MT ، G‬و ‪. r‬‬
‫‪ .1‬يدور القمر االصطناعي الكوم سات ‪ -0-‬في مسار دائري نصف قطره ‪ ، r  42400 km‬في مستوى خط االستواء‬
‫باتجاه دوران األرض حول محورها‪.‬‬
‫‪ .0.1‬استنتج السرعة المدارية للقمر االصطناعي الكوم سات ‪ -0-‬اعتمادا على الشكل‪.-0-‬‬
‫‪ .6.1‬احسب دور القمر االصطناعي الكوم سات ‪ ،-0-‬وهل يمكن اعتباره جيومستق ار ؟ ّبرر‪.‬‬
‫يعطى‪ :‬ثابت الجذب العام‪G  6, 67 1011SI :‬‬

‫صفحة ‪ 5‬من ‪8‬‬


‫✍‬
‫اختبار يف مادة‪ :‬علوم فيزايئية ‪ /‬الشعبة‪ :‬علوم جتريبية‪ /‬بكالوراي ‪8102‬‬

‫التمرين الثاني‪ 07( :‬نقاط)‬


‫تحتوي العديد من الفواكه على استرات ذات نكهة متميزة‪ ،‬فمثال نكهة فواكه الغابة تعود الى‬
‫ميثانوات اإليثيل الذي يمكن تحضيره في المخبر بتفاعل حمض كربوكسيلي مع كحول‪.‬‬
‫‪ .1‬الدراسة الحركية لتحول إماهة األستر‪.‬‬
‫‪H O  35mS  m2  mol 1‬‬ ‫يعطى‪، HCOO  5, 46 mS  m 2  mol 1 :‬‬
‫‪‬‬
‫صورة ‪ : jpg‬فواكه الغابة‬ ‫‪3‬‬
‫‪‬‬

‫المتابعة الزمنية لتفاعل مزيج ابتدائي متكافئ في كمية المادة يتكون من ‪ 0, 03mol‬لكل‬

‫‪x  mmol ‬‬


‫مكنت من الحصول‬
‫من ميثانوات اإليثيل والماء‪ّ ،‬‬
‫على منحنى الشكل‪.-6-‬‬
‫‪ .0.0‬اكتب معادلة التفاعل المنمذج للتحول الحادث‪.‬‬
‫‪ .6.0‬أنجز جدوال لتقدم التفاعل‪.‬‬
‫‪ .1.0‬استخرج من المنحنى خاصيتين يتميز بهما‬
‫مبر ار إجابتك‪.‬‬
‫التفاعل ّ‬
‫‪2‬‬ ‫‪ .1.0‬احسب مردود التفاعل‪ .‬كيف يمكن جعل هذا‬
‫التفاعل شبه تام؟‬
‫‪t  min ‬‬
‫عين التركيب المولي للمزيج عند التوازن‪.‬‬
‫‪10‬‬
‫‪ّ .1.0‬‬
‫‪0‬‬
‫الشكل ‪-2-‬‬
‫‪ .2.0‬احسب السرعة اللحظية للتفاعل عند اللحظتين‪:‬‬
‫‪ t1  10 min‬و ‪ . t2  30 min‬ماذا تستنتج؟‬
‫‪ .2‬معايرة الحمض الكربوكسيلي بأساس‪.‬‬
‫حضر محلول ( ‪ ) S‬بحل ‪ n  0, 01mol‬من حمض الميثانويك النقي في حجم ‪ V  1L‬من الماء‪.‬‬ ‫ُي ّ‬
‫‪pH-pKA‬‬ ‫قيست ناقليته النوعية في ‪ 25C‬فوجدت ‪.   0,049 S  m1‬‬
‫‪ .0.6‬أنشئ جدوال لتقدم التفاعل الحادث بين الحمض والماء‪.‬‬
‫أن‬
‫‪ .6.6‬احسب التركيز المولي ‪ c A‬للمحلول ( ‪ ) S‬و ّبين ّ‬
‫‪1‬‬
‫حمض الميثانويك ضعيف‪.‬‬
‫‪8mL‬‬
‫‪ .1.6‬احسب قيمة ‪ pH‬المحلول ( ‪. ) S‬‬
‫‪ .1‬معايرة حجم ‪ VA  10mL‬من المحلول ( ‪ ) S‬بمحلول‬
‫‪0‬‬
‫هيدروكسيد الصوديوم ‪  Na  (aq)  OH  (aq) ‬تركيزه‬
‫)‪VB(mL‬‬
‫‪ 0, 9‬‬ ‫مكنت القياسات التجريبية من رسم المنحنى‬
‫المولي ‪ّ . cB‬‬
‫البياني ‪ pH  pK a  f VB ‬الممثل في الشكل ‪.-1-‬‬
‫الشكل‪-0-‬‬ ‫‪HCOOH  aq  HCOO  aq ‬‬ ‫‪ .0.1‬استنتج قيمة ‪ pK a‬للثنائية‬
‫‪ .6.1‬جد التركيز المولي ‪. cB‬‬

‫صفحة ‪ 6‬من ‪8‬‬


‫✍‬
‫اختبار يف مادة‪ :‬علوم فيزايئية ‪ /‬الشعبة‪ :‬علوم جتريبية‪ /‬بكالوراي ‪8102‬‬

‫الجزء الثاني‪ 00( :‬نقاط)‬


‫التمرين التجريبي‪ 00( :‬نقاط)‬
‫بغرض معرفة سلوك ومميزات كل من مكثفة سعتها ‪ C‬ووشيعة مقاومتها ‪ r‬وذاتيتها ‪ ، L‬نحّقق التركيب الكهربائي‬
‫المبين في الشكل‪ -1-‬والذي يتكون من العناصر الكهربائية التالية‪:‬‬
‫ّ‬
‫‪K‬‬ ‫‪ -‬مولد ذي توتر ثابت‪ ،‬قوته المحركة الكهربائية ‪. E‬‬
‫‪1‬‬ ‫‪8‬‬ ‫‪ -‬مكثفة فارغة سعتها ‪. C‬‬
‫‪C‬‬ ‫‪E‬‬ ‫‪ L, r ‬‬ ‫‪ -‬وشيعة مقاومتها ‪ r‬وذاتيتها ‪. L‬‬
‫‪R 10 k‬‬ ‫'‪R‬‬ ‫‪ -‬ناقل أومي مقاومته ‪. R  10K ‬‬
‫مقاومة متغيرة ' ‪. R‬‬ ‫‪-‬‬
‫الشكل‪-4-‬‬
‫‪ -‬بادلة ‪. k‬‬
‫‪ .1‬نضع في اللحظة ‪ t  0‬البادلة ‪ K‬في الوضع (‪.)0‬‬
‫وبين عليه جهة مرور التيار الكهربائي ثم مثّل‪:‬‬ ‫أنقل مخطط الدارة على ورقة اإلجابة‪ّ ،‬‬
‫‪ -‬أسهم التوترين بين طرفي المقاومة ) ‪ (uR‬والمكثفة ) ‪. (uc‬‬
‫‪ -‬كيفية توصيل الدارة براسم اهتزاز ذي ذاكرة لمعاينة التوتر الكهربائي بين طرفي المقاومة ) ‪. uR (t‬‬
‫المدونة في الجدول اآلتي‪:‬‬
‫تمكنا من الحصول على النتائج ّ‬
‫‪ .2‬من القياسات المتحصل عليها وبواسطة برمجية مناسبة‪ّ ،‬‬

‫)‪t (s‬‬ ‫‪0‬‬ ‫‪5‬‬ ‫‪10‬‬ ‫‪15‬‬ ‫‪20‬‬ ‫‪25‬‬ ‫‪30‬‬


‫)‪uR (V‬‬ ‫‪6, 00‬‬ ‫‪3, 63‬‬ ‫‪2, 22‬‬ ‫‪1,34‬‬ ‫‪0,81‬‬ ‫‪0,50‬‬ ‫‪0,30‬‬

‫‪‬‬
‫‪du R‬‬
‫‪dt‬‬
‫‪V s ‬‬ ‫‪1‬‬
‫‪0, 60‬‬ ‫‪0,36‬‬ ‫‪0, 22‬‬ ‫‪0,13‬‬ ‫‪0, 08‬‬ ‫‪0, 05‬‬ ‫‪0, 03‬‬

‫‪ .0.6‬بتطبيق قانون جمع التوترات جد المعادلة التفاضلية التي يحّققها التوتر بين طرفي الناقل األومي ) ‪. uR (t‬‬
‫‪ (‬ثم اكتب معادلته الرياضية‪.‬‬ ‫‪ .6.6‬ارسم البيان الممثل للدالة‪)  f (u R ) :‬‬
‫‪du R‬‬
‫‪dt‬‬
‫‪ .1.6‬استنتج قيمة كل من القوة المحركة الكهربائية ‪ E‬وسعة المكثفة ‪. C‬‬
‫‪ .1.6‬احسب الطاقة الكهربائية المخزنة في المكثفة في اللحظة ‪. t  25s‬‬
‫‪ .3‬نضع اآلن البادلة ‪ K‬في الوضع (‪ )6‬في لحظة نعتبرها مبدأ لقياس األزمنة ‪. t  0‬‬
‫‪ .0.1‬جد المعادلة التفاضلية التي تحققها شدة التيار ) ‪. i (t‬‬
‫أن حل المعادلة التفاضلية السابقة هو من الشكل ) ‪ ، i(t )  A(1  e B.t‬جد العبارة الحرفية لكل من‬
‫‪ .6.1‬علما ّ‬
‫الثابتين ‪ A‬و ‪. B‬‬

‫صفحة ‪ 7‬من ‪8‬‬


‫✍‬
‫اختبار يف مادة‪ :‬علوم فيزايئية ‪ /‬الشعبة‪ :‬علوم جتريبية‪ /‬بكالوراي ‪8102‬‬

‫‪ .0‬يمثل الشكل ‪ -1-‬منحنيات تغيرات شدة التيار المار في الدارة بداللة الزمن‪ ،‬من أجل ثالث قيم مختلفة‬
‫المدونة في الجدول اآلتي‪:‬‬
‫ّ‬ ‫للمقاومة ' ‪R‬‬

‫)‪R '(‬‬ ‫‪8 18 38‬‬

‫‪ .0.1‬أرفق كل منحنى بالمقاومة الموافقة مستعينا بعبارة شدة التيار في النظام الدائم ثم استنتج قيمة مقاومة‬
‫الوشيعة ‪. r‬‬
‫‪ .6.1‬باستغالل المنحنى )‪ : (3‬جد قيمة ذاتية الوشيعة ‪. L‬‬

‫)‪i(A‬‬
‫)‪i (A‬‬
‫)‪(3‬‬

‫)‪(2‬‬
‫)‪(1‬‬
‫‪00,15‬‬
‫‪,15‬‬

‫‪0‬‬ ‫‪0,1‬‬
‫‪0,1‬‬ ‫)‪t(s‬‬
‫)‪t (s‬‬

‫الشكل‪-5-‬‬

‫انتهى الموضوع الثاني‬


‫صفحة ‪ 8‬من ‪8‬‬
‫‪55‬‬

‫‪50‬‬
‫بكـــالوريــا ‪8102‬‬
‫‪45‬‬ ‫العلوم الفيزيــائية ‪ /‬شعبة العلوم التجريبية‬
‫الموضوع األول‬
‫‪40‬‬ ‫التمرين األول‬
‫‪–0‬‬
‫‪35‬‬ ‫‪ : 0 . 0‬على المحور ‪ :  ox ‬من البيان – ‪ : 1‬معادلة البيان من الشكل ‪ ، . x  At :‬إذن الحركة منتظمة ‪( .  x  vxt  x0 ‬أو نقول ‪:‬‬
‫تتناسب الفاصلة مع الزمن ‪ ،‬إذن السرعة ثابتة ‪ ،‬وبالتالي الحركة منتظمة )‬
‫‪30‬‬
‫على المحور ‪ :  oy ‬من البيان – ‪ 3‬معادلة البيان من الشكل ‪ ، v y  Bt  B' :‬إذن الحركة متغيّرة بانتظام ‪vy  ay t  v0 y‬‬
‫‪25‬‬ ‫‪dx 5‬‬
‫‪vx ‬‬ ‫‪‬‬ ‫‪ : 8 . 0‬من البيان – ‪ 10m / s : 1‬‬
‫‪dt 0,5‬‬
‫‪20‬‬
‫من البيــان – ‪v0 y  1,96  5  9 ,8 m / s : 3‬‬
‫‪15‬‬ ‫‪dv y‬‬ ‫‪9,8‬‬ ‫‪dv‬‬
‫‪ay ‬‬ ‫‪‬‬ ‫‪  9,8 m / s 2 ، ax  x  0‬‬
‫‪dt‬‬ ‫‪1‬‬ ‫‪dt‬‬
‫‪10‬‬ ‫‪ : 3 . 0‬المعادلة الزمنية ‪: x t ‬‬

‫‪5‬‬ ‫لدينا ‪ ، vx  10 m / s‬وبالتالي ‪. x t   10 t  C‬‬


‫الشروط االبتدائية ‪ ، x  0  t  0 :‬وبالتالي ‪ ، C  0‬ومنه المعادلة هي ‪)1( x t   10 t‬‬
‫‪0‬‬ ‫‪0,5‬‬
‫‪0,5‬‬ ‫‪1‬‬ ‫‪1,5‬‬ ‫‪2‬‬ ‫‪2,5‬‬ ‫‪3‬‬ ‫‪3,5‬‬ ‫‪4‬‬ ‫‪4,5‬‬ ‫‪5‬‬ ‫‪5,5‬‬ ‫‪6‬‬ ‫‪6,5‬‬ ‫‪7‬‬ ‫‪7,5‬‬ ‫‪:8 y  t8,5‬‬
‫المعادلة‪9‬الزمنية ‪‬‬
‫‪x‬‬
‫‪ 9,8 2‬‬
‫‪y t  ‬‬ ‫لدينا ‪ ، vy   9,8 t 9,8‬وبالتالي '‪t 9,8 t  C‬‬
‫‪2‬‬
‫‪y‬‬ ‫(‪)2‬‬ ‫الشروط االبتدائية ‪ ، y  2 ,6 m  t  0 :‬وبالتالي ‪ ، C'  2,6‬ومنه المعادلة هي ‪y  t    4 ,9 t 2  9 ,8 t  2 ,6‬‬
‫‪ : 4 . 0‬معادلة البيان – ‪: 2‬‬
‫ونعوضها في (‪: )2‬‬
‫ّ‬ ‫نستخرج عبارة الزمن من (‪)1‬‬
‫‪2‬‬
‫‪ x ‬‬ ‫‪ x ‬‬
‫‪y  x    4 ,9    9 ,8   2 ,6‬‬
‫‪ 10 ‬‬ ‫‪ 10 ‬‬

‫‪y  x   4,9 102 x2  0,98x  2,6‬‬


‫‪1,96‬‬
‫‪1,96‬‬ ‫هذه المعادلة هي معادلة مســار الجلّة ‪.‬‬
‫‪0‬‬ ‫‪: 5.0‬‬
‫‪0‬‬ ‫‪0,25‬‬ ‫‪x‬‬
‫‪v0 ‬‬ ‫‪v   v  ‬‬
‫‪x‬‬
‫‪2‬‬
‫‪0y‬‬
‫‪2‬‬
‫‪100  96  14m / s‬‬

‫‪vx 10‬‬
‫‪cos ‬‬ ‫‪‬‬ ‫‪ 0,714‬‬
‫‪v0 14‬‬

‫‪‬‬ ‫‪  44,4 ‬‬


‫‪v0 y‬‬ ‫‪9,8‬‬ ‫‪9 ,8‬‬
‫‪tn ‬‬ ‫‪‬‬ ‫‪ ، sin ‬أو ‪ 0,98‬‬ ‫أو ‪ 0 ,7 :‬‬
‫‪v0‬‬ ‫‪10‬‬ ‫‪14‬‬

‫أو ‪ :‬من البيان – ‪ : 2‬عند الذروة مثال ‪:‬‬


‫‪dy‬‬ ‫‪g‬‬
‫‪ ،‬حيث ‪ 9,9m‬هي فاصلة الذروة ‪.‬‬ ‫‪0 2‬‬ ‫‪ 9 ,9  0 ,98‬‬
‫‪dx‬‬ ‫‪v0 cos 2 ‬‬

‫‪‬‬ ‫نجد ‪44,4 ‬‬

‫‪1‬‬
‫‪14,3‬‬

‫‪13‬‬
‫‪: 6.0‬‬
‫‪11,7‬‬ ‫من البيان – ‪D  22,5 m : 2‬‬

‫‪10,4‬‬ ‫أو ‪ :‬من البيان – ‪ ، D  xM  5  4,5  22,5 m : 1‬حيث ‪ xM‬هي فاصلة أبعد نقطة ‪.‬‬

‫‪9,1‬‬ ‫أو ‪ :‬من البيان – ‪ ، tM  9  0,25  2 ,25 s : 3‬وبالتالي ‪D  xM  vx  tM  10  2,25  22,5 m‬‬

‫‪7,8‬‬ ‫‪G1‬‬ ‫أو ‪ :‬من معادلة المسار ‪ :‬نضع ‪ 4,9 102 x2  0,98x  2,6  0 ، y  0‬‬
‫‪‬‬

‫‪6,5‬‬ ‫‪( x1   2,37‬مرفوض ) ‪ ( x2  22,3 m ،‬مقبول ) ‪.‬‬

‫‪5,2‬‬
‫‪ - 8‬بين اللحظتين ‪ t  0‬و ‪ t  2,25s‬معناه ‪ :‬منذ انطالق الجلة إلى أن ارتطمت باألرض ‪.‬‬

‫بين ‪ G0‬و ‪ : G1‬عمل قوة الثقل '‪W01  P   mgh‬‬


‫‪3,9‬‬

‫‪2,6‬‬
‫‪G0‬‬ ‫‪G22‬‬ ‫بين ‪ G1‬و ‪ : G2‬عمل قوة الثقل '‪W12  P   mgh‬‬

‫‪1,3‬‬
‫‪1,3‬‬ ‫وبالتالي من ‪ G0‬إلى ‪ G2‬عمل قوة الثقل ‪W02  P   0‬‬

‫‪0 y‬‬ ‫إذن بين ‪ G0‬و ‪ : G3‬الطاقة الحركية للجلّة ازدادت من القيمة ‪ Ec 0‬إلى القيمة ‪Ec 3‬‬
‫‪0‬‬ ‫‪4,5‬‬
‫‪4,5‬‬ ‫‪9‬‬ ‫‪13,5‬‬ ‫‪18‬‬ ‫‪G3‬‬
‫‪22,5‬‬ ‫‪27‬‬ ‫‪31,5‬‬ ‫‪36‬‬ ‫‪40,5‬‬ ‫‪45‬‬ ‫‪49,5‬‬ ‫‪54‬‬ ‫‪58,5‬‬ ‫‪63‬‬ ‫‪67,5‬‬ ‫‪72‬‬ ‫‪76,5‬‬ ‫‪81x‬‬
‫‪ Ec 0  Ec 2 ‬‬
‫معادلة انحفاظ الطاقة للجملة (جلّة) ‪EcG0  W03  P   EcG3 :‬‬

‫‪1‬‬ ‫‪1‬‬
‫‪ ،‬ومنه ‪vG3  2 gh  v02  2  9 ,8  2 ,6  196‬‬ ‫‪mv0 2  mgh  mvG23‬‬
‫‪2‬‬ ‫‪2‬‬
‫‪y‬‬
‫‪vG3  15,7 m / s‬‬

‫‪–3‬‬
‫الخصائص هي ‪:‬‬

‫‪ -‬الطويلة ‪ ( :‬محسوبة سابقا ‪) vG3  15,7m / s‬‬

‫‪ -‬الحامل ‪ :‬المماس للمسار في النقطة ‪ ، G3‬حيث نحسب الزاوية ‪‬‬


‫‪‬‬
‫‪vx‬‬ ‫‪10‬‬
‫‪ ، cos  ‬ومنه ‪  50 ,4 ‬‬ ‫‪‬‬ ‫‪ 0,637‬‬
‫‪vG3 15,7‬‬

‫‪1,3‬‬ ‫‪–4‬‬
‫الطاقة الكلية للجملة (جلّة ‪ +‬أرض) ‪:‬‬
‫‪0‬‬ ‫‪4,5‬‬ ‫‪‬‬ ‫‪x‬‬
‫‪1‬‬
‫عند ‪ ، G0‬أي عند ‪ET1  Ec 0  E pp0  mv0 2  mgh : t  0‬‬
‫‪2‬‬
‫‪1,3‬‬
‫‪1,3‬‬ ‫‪1‬‬
‫عند ‪ ، G3‬أي عند ‪ET2  EcG3  mvG32 : t  2,25s‬‬
‫‪2‬‬
‫‪0‬‬ ‫‪4,5‬‬
‫‪4,5‬‬ ‫‪x‬‬

‫‪2‬‬
‫‪1‬‬
‫‪ET1‬‬ ‫‪mv0 2  mgh v 2 2 gh‬‬
‫‪2‬‬ ‫‪ 0 2  2  0,8  0,2  1‬‬
‫‪ET2‬‬ ‫‪1‬‬ ‫‪2‬‬ ‫‪vG3‬‬ ‫‪vG3‬‬
‫‪mvG3‬‬
‫‪2‬‬
‫نستنتج أن الجملة شبه معزولة ‪ ،‬ألن الطاقة الكلية ثابتة ‪.‬‬
‫التمرين الثــاني‬
‫‪-‬‬

‫( ‪ 33‬بروتون ‪ 87 ،‬نوترون )‬ ‫‪131‬‬


‫‪53‬‬ ‫‪I‬‬ ‫‪-0‬‬

‫‪m‬‬ ‫‪110 3 10 3‬‬


‫‪N0 ‬‬ ‫‪‬‬ ‫‪ 4,6 1015 - 8‬‬
‫‪m I  2,176 10‬‬ ‫‪ 22‬‬

‫رغم عدم إدراج قيمة عدد أفوغادرو ‪ ،  N A  6,02 1023 mol 1 ‬إال أن الطريقة التالية مقبولة ‪:‬‬
‫‪10 6‬‬
‫‪N 0  6,02 1023 ‬‬ ‫‪ 4,6 1015‬‬
‫‪131‬‬
‫‪–3‬‬
‫‪1‬‬
‫‪0‬‬ ‫تحول نوترون إلى بروتون ‪n  11 p   10 e ،‬‬
‫‪ : 0 . 3‬السبب هو ّ‬
‫‪131‬‬
‫‪54‬‬ ‫هي ‪Xe‬‬ ‫‪A‬‬
‫‪Z‬‬ ‫‪ ،‬وحسب قانوني صودي لالنحفاظ ‪ A  131 :‬و ‪ ، Z  54‬وبالتالي النواة ‪X‬‬ ‫‪131‬‬
‫‪53‬‬ ‫‪I  ZA X   10 e : 8 . 3‬‬

‫‪ ، m  m0 et : 3 . 3‬أو ‪ N  N 0 e  t‬أو ‪A  A0 et‬‬

‫‪ : 4 . 3‬زمن نصف العمر هو الزمن الالزم لتفكك نصف عدد األنوية االبتدائي ‪.‬‬
‫‪ln 2‬‬ ‫‪N0‬‬
‫‪t1/ 2 ‬‬ ‫‪ ،‬وبالتالي‬ ‫‪ N 0e  t1/ 2‬‬
‫‪‬‬ ‫‪2‬‬
‫‪ln 2‬‬ ‫‪0 ,69‬‬
‫‪A0   N 0 ‬‬ ‫‪ N0 ‬‬ ‫‪ 4 ,6  1015  4 ,6  109 Bq : 5 . 3‬‬
‫‪t1/ 2‬‬ ‫‪8  24  3600‬‬

‫‪ln 0 ,4‬‬ ‫‪ 0 ,916‬‬ ‫‪40‬‬


‫‪t' ‬‬ ‫‪‬‬ ‫'‪ 10 ,62 j ، ln 0,4   t‬‬ ‫‪، 0,4  et' ،‬‬ ‫‪A0  A0e  t' - 4‬‬
‫‪‬‬ ‫‪0 ,69‬‬ ‫‪100‬‬
‫‪8‬‬
‫لدينا ‪ 1 j‬يساوي ‪ 22‬سـاعة ‪ ،‬وبالتالي ‪ 0 ,62 j‬يساوي ‪. 0,62  24 15h‬‬

‫التاريخ والتوقيت ‪ 21 :‬ماي ‪ 2117‬على الساعة ‪ 11‬صباحا ‪.‬‬


‫‪- ‬‬
‫‪235‬‬
‫‪92‬‬‫‪U  01n  148‬‬
‫‪Z La  35 Br  x 0 n‬‬
‫‪85‬‬ ‫‪1‬‬
‫‪-0‬‬

‫نوع التفاعل ‪ :‬انشطار نووي ‪.‬‬


‫‪ - 8‬حسب قانوني صودي لالنحفاظ ‪ ، 236  148  85  x :‬ومنه ‪x  3‬‬
‫‪ ، 92  Z  35‬ومنه ‪Z  57‬‬

‫‪Elib   2,19836  2,19669 105  167 MeV‬‬ ‫‪-3‬‬

‫‪3‬‬
‫‪–4‬‬

‫‪Eelec  P  t  900 106  24  3600  7,77 1013 J : 0 . 4‬‬

‫‪:8.4‬‬

‫‪Eelec 7 ,77  1013‬‬ ‫‪E‬‬


‫‪E'lib ‬‬ ‫‪‬‬ ‫‪ ، elec  100  30‬ومنه ‪ 2 ,59  1014 J‬‬
‫‪0 ,3‬‬ ‫‪0 ,3‬‬ ‫‪E'lib‬‬

‫‪2 ,59 1014‬‬


‫‪E'lib‬‬ ‫‪‬‬ ‫‪ 13‬‬
‫‪ 1,62  10 27 MeV‬‬
‫‪1,6  10‬‬

‫‪: 3.4‬‬

‫(‪)1‬‬ ‫كتلة اليورانيوم المستهلكة ‪m  N  mU  :‬‬

‫‪E'lib 1,62 1027‬‬


‫‪N‬‬ ‫‪‬‬ ‫عدد األنوية المنشطرة ‪ 9 ,7 1024‬‬
‫‪Elib‬‬ ‫‪167‬‬

‫بالتعويض في (‪m  9,7 1024  3,9036 10 22  3,786 103 g : )1‬‬

‫‪m  3,786 kg‬‬

‫‪9 ,7  10 24‬‬
‫‪m  235 ‬‬ ‫حساب الكتلة بالطريق التالية مقبول ‪ 3786 g  3,786 kg :‬‬
‫‪6 ,02  10 23‬‬

‫‪–5‬‬
‫‪ : 0 . 5‬التفاعل هو تفاعل اندماج نووي ‪.‬‬
‫‪ : 8 . 5‬أ ‪ /‬تكمن الصعوبة في توفير الطاقة الكبيرة للتغلّب على قوى التنافر الكهربائي بين األنوية (النواة مشحونة إيجابا) ‪ ،‬ألن في تفاعل‬
‫االندماج يجب أن يكون المزيج كثيفا جدا (مضغوطا) ‪.‬‬
‫‪Elib 176‬‬
‫‪Elib / nucl ‬‬ ‫‪‬‬ ‫المحررة لكل نوكليون في تفاعل االنشطار السابق هي ‪ 0,77 MeV / nucl‬‬
‫ّ‬ ‫ب ‪ /‬الطاقة‬
‫‪236 236‬‬
‫نفضل تفاعل االندماج لألسباب التالية ‪:‬‬
‫‪E fu sion‬‬ ‫‪3,53‬‬
‫‪ ،‬أي حوالي خمسة أضعاف ‪.‬‬ ‫‪‬‬ ‫المحررة لكل نوكليون فيه أكبر مما في تفاعل االنشطار ؛ ‪5‬‬
‫ّ‬ ‫‪ -‬الطاقة‬
‫‪E fission‬‬ ‫‪0,77‬‬

‫ملوث للبيئة بنواتجه المشعّة ‪.‬‬


‫‪ -‬تفاعل االنشطار ّ‬
‫ملوث للبيئة ( ناتجه غير مشع ) ‪.‬‬
‫‪ -‬تفاعل االندماج غير ّ‬
‫التمرين التجريبي‬
‫التجربة األولى ‪:‬‬
‫‪–0‬‬
‫‪500‬‬ ‫‪V‬‬
‫‪V0 ‬‬ ‫‪ : 0 . 0‬معامل التمديد ‪ ، F  100‬ولدينا ‪ ، F  S‬ومنه ‪ 5 mL‬‬
‫‪100‬‬ ‫‪V0‬‬

‫مزودة بإجاصة السحب ‪ ،‬ونضعه في حوجلة عيارية سعتها ‪500mL‬‬


‫صة ّ‬‫‪ : 8 . 0‬نأخذ الحجم ‪ V0  5mL‬من المحلول المر ّكز بواسطة مـا ّ‬
‫طر ‪ ،‬أي نضيف ‪ 495mL‬من الماء المق ّ‬
‫طر ‪ ،‬ثم نر ّج لجعل المحلول متجانسا ‪.‬‬ ‫ونكمل الحجم بالماء المق ّ‬

‫‪4‬‬
‫‪–8‬‬

‫‪y‬‬ ‫‪ : 0 . 8‬جدول التقدّم ‪:‬‬


‫‪0,3‬‬
‫‪n C3 H 6O3   CaVa ، n  CaCO3  ‬‬ ‫كمية المادة االبتدائية للمتفاعلين ‪ 3 10 3 mol :‬‬
‫‪100‬‬
‫‪CaCO3  2C3H6O3  CO2  Ca 2  2C3H5O3  H 2O‬‬ ‫قانون الغازات المثالية ‪PCO2 VCO2   nCO2  RT :‬‬
‫‪3 10 3‬‬ ‫‪CaVa‬‬ ‫‪1‬‬ ‫‪1‬‬ ‫‪1‬‬ ‫‪/‬‬
‫ولدينا من جدول التقدم في اللحظة ‪nCO2   x : t‬‬
‫‪3‬‬
‫‪3 10  x‬‬ ‫‪CaVa  2 x‬‬ ‫‪x‬‬ ‫‪x‬‬ ‫‪2x‬‬ ‫‪/‬‬
‫‪3 103  x f‬‬ ‫‪CaVa  2 x f‬‬ ‫‪xf‬‬ ‫‪xf‬‬ ‫‪2x f‬‬ ‫‪/‬‬ ‫‪PCO2 VCO2 ‬‬
‫(‪)1‬‬ ‫‪x‬‬ ‫وبالتالي‬
‫‪RT‬‬

‫‪PCO2  max  5,2  30 100  15600 Pa‬‬ ‫‪ : 8 . 8‬من البيان‬

‫لدينا ‪ ، VCO2  600  120  480 mL  480 106 m3‬بإهمال كمية ‪ C O2‬المنحلّة في الماء ‪.‬‬

‫‪15600  480 10  6‬‬


‫‪xm ‬‬ ‫‪3  10 3 mol‬‬
‫‪8,31 298‬‬

‫بما أن التقدّم األعظمي يساوي كمية المادة االبتدائية لـ ‪ ، CaCO3‬إذن التفاعل تــام ‪.‬‬

‫‪PCO2  max ‬‬


‫‪t1/ 2  18 s‬‬ ‫‪. PCO2 ‬‬ ‫‪ : 3 . 8‬زمن نصف التفاعل يوافق‬
‫‪2‬‬
‫‪ : 4 . 8‬درجة الحرارة والتراكيز االبتدائية للمتفاعالت عامالن حركيان ‪.‬‬
‫‪78‬‬
‫إن رفع هذين العاملين يؤدّي إلى خفض المدّة الزمنية الالزمة إلزالة الراسب ‪.‬‬
‫التجربة الثانية ‪:‬‬
‫‪30‬‬
‫‪30‬‬
‫‪ - 0‬التركيب في الشكل ‪.‬‬

‫‪0 18 60‬‬
‫‪60‬‬ ‫‪x‬‬
‫‪ – 8‬معادلة تفاعل المعايرة ‪C3 H 6O3   Na  ,OH     Na  ,C3H 5O3   H 2O :‬‬

‫‪–3‬‬
‫‪ : 0 . 3‬سبب إضافة المــاء ‪ :‬هو ج ْعل الحجم الذي نعايره كبيرا نوعا ما ‪ ،‬وذلك لتجنّب اصطدام القضيب المغناطيسي بمسبار مقياس الـ ‪. pH‬‬

‫سحاحة‬ ‫إضافة الماء ال تؤثر على حجم المحلول األساسي المسكوب ‪ ،‬ألن كمية مادة الحمض‬
‫ال تتغير عند إضافة المــاء ‪.‬‬
‫هيدروكسيد الصوديوم‬
‫‪ : 8 . 3‬عند التكافؤ يكون ‪ ، C'a Va  CbVbE‬ولدينا من البيان ‪. VbE  14 mL‬‬
‫حامل‬
‫‪0,02 14‬‬
‫‪C'a ‬‬ ‫وبالتالي ‪ 2,66 10 3 mol / L‬‬
‫‪105‬‬
‫‪105‬‬
‫مقياس ‪pH‬‬ ‫‪Ca  2 ,66 10 3 ‬‬ ‫‪ 5,58 10  2 mol / L‬‬
‫حمض الالكتيك‬ ‫‪5‬‬

‫محرك‬ ‫مخالط‬ ‫‪C0  0,0558 100  5,58 mol / L‬‬

‫كتلة الحمض المنحلة في ‪ 1L‬هي ‪m  5,58  90  502 ,2 g‬‬

‫‪5‬‬
‫‪m‬‬ ‫‪100‬‬
‫‪V‬‬ ‫‪‬‬ ‫الحجم الموافق لكتلة الحمض ‪ 100g‬هو ‪ 0,088L‬‬
‫‪‬‬ ‫‪1130‬‬

‫في الحجم ‪ 502 ,2 g  1L‬من الحمض النقي‬

‫في الحجم ‪ P  g   0,088L‬من الحمض النقي‬

‫‪0 ,088  502 ,2‬‬


‫‪ ، P ‬أي ‪P  44 %‬‬ ‫وبالتالي ‪44‬‬
‫‪1‬‬
‫حساب ‪ P‬بالطريقة التالية مقبول ‪:‬‬
‫‪C0 M‬‬ ‫‪C0 M‬‬ ‫‪5,58  90‬‬ ‫‪10Pd‬‬
‫‪ ، P ‬حيث ‪eau  1kg / L‬‬ ‫‪‬‬ ‫‪‬‬ ‫‪ ، C0 ‬ومنه ‪44‬‬
‫‪10 d 10  ‬‬ ‫‪11,3‬‬ ‫‪M‬‬
‫‪eau‬‬

‫‪6‬‬
‫بكـــالوريــا ‪8102‬‬
‫العلوم الفيزيــائية ‪ /‬شعبة العلوم التجريبية‬
‫الموضوع الثـــاني‬
‫التمرين األول‬
‫‪–0‬‬
‫‪ – 0 . 0‬المرجع ‪ :‬مركزي أرضي ‪ .‬نعتبره عطاليا للتّمكن من تطبيق القانون الثاني لنيوتن فيه ‪ ،‬حيث يكون عطاليا إذا ما اعتبرنا المسافة التي‬
‫يقطعها مركز األرض خالل مدّة الدراسة مستقيما وليس قوسا ‪ ،‬حتى تكون بذلك حركة مركز األرض مستقيمة منتظمة ‪( .‬العبارة باللون األخضر‬
‫كافية لإلجابة على هذا السؤال ) ‪.‬‬
‫المعلم المرتبط به ‪ :‬مبدؤه مركز األرض ومحاوره الثالثة متجهة نحو ثالثة نجوم نعتبرها ثابتة ‪.‬‬
‫القوة التي تطبّقها األرض على القمر الصناعي ‪:‬‬ ‫‪ - 8 . 0‬تمثيل ّ‬
‫‪mM‬‬
‫‪)1( FT / S  G 2 T‬‬ ‫‪-3.0‬‬
‫‪T‬‬ ‫‪r‬‬
‫‪S‬‬ ‫‪ – 4 . 0‬بتطبيق القانون الثـاني لنيوتن في المرجع المركزي أرضي ‪:‬‬
‫‪y‬‬
‫‪FT / S  ma‬‬
‫‪v2‬‬
‫‪FT / S  man  m‬‬ ‫باإلسقاط على المحور المركزي '‪: xx‬‬
‫‪r‬‬
‫‪GM T‬‬ ‫‪1‬‬ ‫‪v2‬‬ ‫‪m M T v2‬‬
‫‪.v ‬‬‫‪2‬‬
‫‪ ، m  G 2 m‬وبالتالي ‪ GM T ‬‬ ‫وباستعمال العالقة (‪: )1‬‬
‫‪r‬‬ ‫‪r‬‬ ‫‪r‬‬ ‫‪r‬‬ ‫‪r‬‬
‫‪–8‬‬
‫‪4 ,8  2  10‬‬ ‫‪6‬‬
‫‪1‬‬
‫‪A  GM T ‬‬ ‫‪8‬‬
‫‪ – 0 . 8‬معــادلة البيان من الشكل ‪ ، v 2  A‬وبالمطابقة نكتب ‪ 4  1014‬‬
‫‪2 ,4  10‬‬ ‫‪r‬‬
‫‪4  1014‬‬
‫‪. MT ‬‬ ‫‪ 11‬‬
‫ومنه ‪ 6  10 24 kg‬‬
‫‪6 ,67  10‬‬
‫‪r3‬‬ ‫‪4 2r 2 4 2 r 3‬‬ ‫‪2 r‬‬
‫‪T  2‬‬ ‫‪ ، T 2 ‬وبالتالي‬ ‫‪‬‬ ‫‪، T‬‬ ‫‪ - 8 . 8‬لدينا‬
‫‪GM T‬‬ ‫‪GM T GM T‬‬ ‫‪v‬‬
‫‪r‬‬
‫‪–3‬‬
‫‪1‬‬ ‫‪1‬‬
‫‪ ، ‬ومن البيان قيمة مربع السرعة الموافق هو‬ ‫‪2 ,4 10 8 m  1 - 0 . 3‬‬
‫‪r 42400 10‬‬ ‫‪3‬‬

‫‪. v  31‬‬
‫‪ ، v2  9,6 106 m2 s 2‬ومنه ‪, 103 m / s‬‬
‫‪2 r 6 ,28  424 105‬‬
‫‪ ، T ‬وهذا الدور يساوي الدور‬ ‫‪‬‬ ‫‪ 85894 s 24h - 8 . 3‬‬
‫‪v‬‬ ‫‪3,1 103‬‬
‫اليومي لألرض ‪ ،‬وبما أن الشروط األخرى محققة ‪ ،‬إذن هذا القمر الصناعي جيومستقر( مستقر أرضيا ) ‪.‬‬
‫‪9,6‬‬

‫‪4,8‬‬
‫التمرين الثــاني‬
‫‪2,4‬‬ ‫لتحول إمــاهة األستر ‪:‬‬
‫ّ‬ ‫‪ x– 0‬الدراسة الحركية‬
‫‪ – 0 . 0‬معادلة التفاعل ‪HCOO  C2 H 5  H 2O  HCOOH  C2 H 5  OH‬‬

‫‪HCOO  C2 H 5  H 2O  HCOOH  C2 H 5  O H‬‬ ‫‪ – 8 . 0‬جدول التقدم‬


‫‪0,03‬‬ ‫‪0,03‬‬ ‫‪0‬‬ ‫‪0‬‬
‫‪0,03  x‬‬ ‫‪0,03  x‬‬ ‫‪x‬‬ ‫‪x‬‬
‫‪0,03  x f‬‬ ‫‪0,03  x f‬‬ ‫‪xf‬‬ ‫‪xf‬‬

‫‪1‬‬
‫‪ – 3 . 0‬الخاصيّة األولى ‪ :‬تفاعل غير تــام ‪ :‬ألن من البيان ‪x f  0,01mol  0,03mol‬‬

‫‪y‬‬ ‫الخاصية الثــانية ‪ :‬تفاعل بطيء ‪ :‬من البيان ‪ ،‬يحتاج التفاعل إلى حوالي ‪ 00‬دقيقة لكي يصل للتوازن ‪.‬‬
‫‪28‬‬ ‫‪xf‬‬ ‫‪0 ,01‬‬
‫‪r‬‬ ‫‪100 ‬‬ ‫‪100  33 % - 4 . 0‬‬
‫‪xm‬‬ ‫‪0 ,03‬‬
‫‪26‬‬
‫نجعل هذا التفاعل شبه تام بسحب أحد الناتجين خالل التفاعل ‪.‬‬
‫‪24‬‬
‫تُقبل اإلجــابة إذا أضاف التلميذ ‪:‬‬
‫‪22‬‬
‫‪ -‬نجعل المزيج االبتدائي غير متساوي الموالت ‪.‬‬
‫‪20‬‬ ‫ألن السؤال ينص على ‪ :‬كيف نجعل هــذا التفاعل شبه تام ‪ .‬يمكن أن يفهم التلميذ أن المقصود هو تفاعل ميثانوات اإليثيل مع الماء ‪ ،‬أو يفهم أن‬
‫المقصود هو تفاعل اإلماهة بشكل عـــام ‪.‬‬
‫‪18‬‬
‫‪n HCOOH  f  n C2 H 5  OH  f  x f  0,01mol‬‬ ‫‪ – 5 . 0‬التركيب المولي للمزيج عند التوازن ‪:‬‬
‫‪16‬‬
‫‪n  H 2O  f  n  HCOO  C2 H 5  f  0,03  0,01  0,02mol‬‬
‫‪14‬‬
‫‪–6.0‬‬
‫‪12‬‬ ‫‪T30‬‬
‫‪T10‬‬ ‫سرعة التفاعل عند اللحظة ‪: t  10mn‬‬
‫‪10‬‬ ‫‪dx 7‬‬
‫‪v10 ‬‬ ‫‪‬‬ ‫‪ 0,35mmol.mn  1‬‬
‫‪dt 20‬‬
‫‪8‬‬
‫‪dx 6‬‬
‫‪v30 ‬‬ ‫‪‬‬ ‫‪ 0,086mmol.mn  1‬‬
‫‪6‬‬ ‫‪dt 70‬‬

‫‪4‬‬ ‫تتناقص السرعة بمرور الزمن ‪ ،‬وذلك لتناقص كمية مادة األستر‬
‫والماء (تراكيز المتفاعالت عامل حركي) ‪.‬‬
‫‪22‬‬

‫‪0‬‬ ‫‪10‬‬ ‫‪20‬‬ ‫‪30‬‬ ‫‪40‬‬ ‫‪50‬‬ ‫‪60‬‬ ‫‪70‬‬ ‫‪80‬‬ ‫‪90‬‬ ‫‪100‬‬ ‫‪110‬‬ ‫‪120‬‬ ‫‪:130‬‬
‫‪140‬بأسـاس‬
‫لكربوكسيلي‬
‫‪ 160‬ا ‪150‬‬
‫‪170‬الحمض‬
‫‪180‬معـايرة‬
‫‪– x8‬‬
‫‪ – 0 . 8‬جدول التقدّم ‪:‬‬
‫‪HCOOH‬‬ ‫‪‬‬ ‫‪H 2O‬‬ ‫‪‬‬ ‫‪HCOO‬‬ ‫‪‬‬ ‫‪H3O‬‬ ‫‪n 0 ,01‬‬
‫‪C‬‬ ‫‪‬‬ ‫‪ 0 ,01mol / L - 8 . 8‬‬
‫‪0,01‬‬ ‫‪/‬‬ ‫‪0‬‬ ‫‪0‬‬ ‫‪V‬‬ ‫‪1‬‬
‫‪0,01  x‬‬ ‫‪/‬‬ ‫‪x‬‬ ‫‪x‬‬
‫‪0 ,01  x f‬‬ ‫‪/‬‬ ‫‪xf‬‬ ‫‪xf‬‬ ‫‪‬‬
‫‪   H 3O   f H O  HCOO‬‬
‫‪3‬‬
‫‪‬‬
‫‪‬‬ ‫‪0,049‬‬
‫‪ H 3O   ‬‬ ‫‪‬‬ ‫‪ 1,21mol / m3  1,2110 3 mol / L‬‬
‫‪f‬‬
‫‪‬‬ ‫‪H3O‬‬
‫‪ HCOO‬‬ ‫‪‬‬ ‫‪40,46 10‬‬ ‫‪3‬‬

‫‪xf‬‬ ‫‪ H 3O   V 1,2110 3‬‬


‫‪f ‬‬ ‫‪‬‬ ‫‪f‬‬
‫‪‬‬ ‫‪ 0 ,12‬‬ ‫لدينا‬
‫‪xm‬‬ ‫‪CV‬‬ ‫‪0 ,01‬‬

‫‪ ،  f  1‬وبالتالي حمض الميثانويك ضعيف في المــاء ‪.‬‬

‫‪pH   Log  H 3O   Log1,21103  2,9‬‬ ‫‪-3.8‬‬

‫‪2‬‬
‫‪–3‬‬

‫‪ – 0 . 3‬قبل إضافة المحلول األساسي ‪VB  0‬‬


‫كان لدينا ‪ ، pH  pK a   0,9‬وبالتالي‬

‫‪pK a  pH  0,9  2,9  0,9  3,8‬‬

‫‪ H 3O  ‬‬
‫‪0‬‬ ‫‪pK a  pH  Log‬‬ ‫‪f‬‬
‫أو‪:‬‬
‫‪11‬‬ ‫‪5‬‬ ‫‪C A   H 3O   f‬‬ ‫‪x‬‬

‫‪–8.3‬‬
‫‪- 0,9‬‬
‫عندما ‪pH  pK a  0‬‬

‫‪VBE‬‬
‫‪ ، VB ‬وبالتالي ‪VBE  10mL‬‬ ‫أي ‪( pH  pK a‬نقطة نصف التكافؤ) يكون ‪ 5mL‬‬
‫‪2‬‬

‫‪ HCOO  ‬‬


‫‪  HCOOH ‬نحو الصفر‬ ‫‪ ، pH  pK a  Log‬هذه النسبة تنتهي إلى ما ال نهاية عندما ينتهي‬ ‫يُقبل الجواب التالي ‪:‬‬
‫‪ HCOOH ‬‬
‫وبالتالي ‪. VBE  10mL‬‬

‫‪C AVA 0 ,01 10‬‬


‫‪CB ‬‬ ‫‪‬‬ ‫عند التكافؤ يكون ‪ ، C AVA  CBVBE‬وبالتالي ‪ 0 ,01mol / L‬‬
‫‪VB‬‬ ‫‪10‬‬

‫التمرين التجريبي‬
‫‪–0‬‬
‫‪1‬‬ ‫‪K 2‬‬
‫‪ -‬جهة التيار وأسهم التوترين بين طرفي المكثّفة والناقل األومي ‪:‬‬
‫‪‬‬

‫‪ -‬توصيل راسم اإلهتزاز بين طرفي الناقل األومي ‪.‬‬


‫‪E‬‬ ‫‪–8‬‬
‫‪du R duC‬‬
‫‪‬‬ ‫‪ ، uR  uC  E - 0 . 8‬وباشتقاق الطرفين بالنسبة للزمن ‪ 0 :‬‬
‫‪dt‬‬ ‫‪dt‬‬
‫‪duR‬‬ ‫‪1‬‬ ‫‪duR 1‬‬ ‫‪du R 1 dq‬‬
‫(‪)1‬‬ ‫‪‬‬ ‫‪uR  0 ،‬‬ ‫‪ i0 ،‬‬ ‫‪‬‬ ‫‪0‬‬
‫‪dt RC‬‬ ‫‪dt C‬‬ ‫‪dt C dt‬‬
‫‪0,7‬‬ ‫‪du R‬‬
‫‪: ‬‬ ‫‪ – 8 . 8‬تمثيل البيان ‪ f  u R ‬‬
‫‪0,6‬‬ ‫‪dt‬‬

‫‪0,5‬‬ ‫‪duR‬‬
‫‪ ، ‬حيث‬ ‫المعادلة الرياضية للبيان من الشكل ‪ A uR‬‬
‫‪0,4‬‬ ‫‪dt‬‬

‫‪0,3‬‬ ‫‪0,6  0,03‬‬


‫‪A‬‬ ‫‪ 0,1‬‬
‫‪0,2‬‬
‫‪6  0,3‬‬

‫‪0,1‬‬ ‫‪–3.8‬‬
‫‪0‬‬ ‫أكبر قيمة للتوتر بين طرفي الناقل األومي تكون عند‬
‫‪0‬‬ ‫‪1‬‬ ‫‪2‬‬ ‫‪3‬‬ ‫‪4‬‬ ‫‪5‬‬ ‫‪6‬‬ ‫‪7‬‬
‫اللحظة ‪ ، t  0‬ألن ‪uC  t  0  0‬‬

‫‪3‬‬
‫وبالتالي ‪uR t  0  E  6V‬‬

‫‪1‬‬ ‫‪du‬‬ ‫‪1‬‬


‫‪A‬‬ ‫‪ ،  R ‬وبالمطابقة ‪:‬‬ ‫من العالقة (‪ )1‬نكتب ‪uR :‬‬
‫‪RC‬‬ ‫‪dt‬‬ ‫‪RC‬‬
‫‪1‬‬
‫‪C‬‬ ‫‪ ،‬وبالتالي ‪ 110 3 F‬‬
‫‪10 103  0,1‬‬

‫‪ - 4 . 8‬في اللحظة ‪ t  25s‬لدينا ‪ ، uR  0,5V‬وبالتالي ‪uC  6  0,5  5,5V‬‬

‫‪1‬‬
‫‪EC  CuC2  0,5 10 3 5,5  1,5110 2 J‬‬
‫‪2‬‬

‫‪2‬‬
‫‪–3‬‬

‫‪di  r  R' ‬‬ ‫‪E‬‬ ‫‪di‬‬ ‫‪di‬‬


‫(‪)2‬‬ ‫‪‬‬ ‫‪i‬‬ ‫‪،‬‬ ‫‪ r  R' i  L‬‬ ‫‪ E ، ri  L  R' i  E ، uB  uR'  E - 0 . 3‬‬
‫‪dt‬‬ ‫‪L‬‬ ‫‪L‬‬ ‫‪dt‬‬ ‫‪dt‬‬

‫(‪)3‬‬ ‫‪ - 8 . 3‬لدينا ‪i t   A1  e Bt ‬‬

‫'‪r  R‬‬ ‫‪r  R'  Bt E‬‬ ‫‪di t ‬‬


‫‪ABe  Bt ‬‬ ‫‪A‬‬ ‫‪Ae ‬‬ ‫‪ ،‬وبالتعويض في (‪: )2‬‬ ‫باشتقاق (‪ )3‬بالنسبة للزمن ‪ ABe Bt :‬‬
‫‪L‬‬ ‫‪L‬‬ ‫‪L‬‬ ‫‪dt‬‬
‫‪y‬‬
‫'‪r  R‬‬ ‫'‪r  R'  r  R‬‬
‫‪ ، Ae  Bt  B ‬ومنه‬
‫‪E‬‬ ‫‪E‬‬
‫‪2,1‬‬ ‫‪. A‬‬ ‫‪ ، B‬و‬ ‫‪‬‬ ‫‪A‬‬
‫'‪r  R‬‬ ‫‪L‬‬ ‫‪‬‬ ‫‪L ‬‬ ‫‪L‬‬ ‫‪L‬‬
‫‪0‬‬
‫‪1,95‬‬
‫‪–4‬‬
‫‪1,8‬‬
‫‪E‬‬
‫‪ ، I ‬وبما أن مقاومة الوشيعة ثابتة في كل تجربة ‪ ،‬إذن أكبر قيمة لشدة التيار تكون في التجربة الموافقة‬ ‫‪ - 0 . 4‬لدينا في النظام الدائم‬
‫‪1,65‬‬ ‫'‪r  R‬‬
‫ألصغر قيمة للمقاومة '‪. R‬‬
‫‪1,5‬‬ ‫البيان (‪R'  38  )1‬‬
‫البيان (‪R'  18  )2‬‬
‫‪1,35‬‬ ‫البيان (‪R'  8  )3‬‬
‫‪E‬‬ ‫‪6‬‬
‫‪1,2‬‬ ‫‪ ، R'  r ‬ومنه ‪r  10  8  2‬‬ ‫‪‬‬ ‫من البيان (‪ )3‬مثال ‪ ، I  0,6 A :‬ولدينا ‪ 10‬‬
‫‪I 0,6‬‬
‫‪1,05‬‬ ‫‪-8.4‬‬

‫التفاضلية يكون‪0,9‬‬ ‫مالحظة ‪ :‬المماس الممثّل في الشكل ‪ ،‬هو مماس لكل البيانات عند ‪ ، t  0‬ألن عند ‪ t  0‬يكون ‪ ، i  0‬وبالتالي من المعادلة‬
‫‪di t  E‬‬
‫( الميل مستقل عن '‪. ) R‬‬ ‫‪‬‬
‫‪0,75‬‬ ‫‪dt‬‬ ‫‪L‬‬

‫‪0,6‬‬
‫‪3‬‬ ‫من المنحني (‪ )3‬مثال ‪.   0,1s :‬‬

‫‪1‬‬ ‫‪L‬‬
‫‪0,45‬‬ ‫‪ ،‬ومنه ‪L    R'  r   0,110  1H‬‬ ‫‪ ‬‬ ‫لدينا‬
‫‪B‬‬ ‫‪R'  r‬‬
‫‪2‬‬
‫‪0,3‬‬ ‫يمكن استخراج ثابت الزمن من أي تجربة ‪ ،‬واستعمال المقاومة الموافقة‬
‫‪1‬‬ ‫ونجد نفس القيمة ‪L  1H‬‬
‫‪0,15‬‬
‫‪0,15‬‬

‫‪0‬‬ ‫‪0,1‬‬
‫‪0,1‬‬ ‫‪0,2‬‬ ‫‪0,3‬‬ ‫‪0,4‬‬ ‫‪0,5‬‬ ‫‪0,6‬‬ ‫‪0,7‬‬ ‫‪0,8‬‬ ‫‪0,9‬‬ ‫‪1‬‬ ‫‪1,1‬‬ ‫‪1,2‬‬ ‫‪1,3‬‬ ‫‪1,4‬‬ ‫‪1,5‬‬ ‫‪1,6‬‬ ‫‪1,7‬‬ ‫‪1,8x‬‬

‫‪4‬‬
1
2
3
1
2
3
4
www.guezouri.org

www.guezouri.org
www.guezouri.org

www.guezouri.org
www.guezouri.org
www.guezouri.org
www.guezouri.org
www.guezouri.org
‫ﺟـﻲ‬
‫هـ (‬
‫)‬

‫ب(‬
‫ﺑﻜﺎﻟﻮرﻳﺎ ‪2013‬‬
‫ﻋﻠﻮم ﻓﻴﺰﻳﺎﺋﻴﺔ – ﺷﻌﺒﺔ اﻟﻌﻠﻮم اﻟﺘﺠﺮﻳﺒﻴﺔ‬
‫اﻟﻤﻮﺿــﻮع اﻷول‬
‫‪www.guezouri.org‬‬
‫‪---------------------------------------‬‬
‫اﻟﺘﻤﺮﻳﻦ اﻷول )‪ 4‬ﻧﻘﻂ(‬
‫‪i‬‬
‫‪ – 1‬ﺗﻤﺜﻴﻞ اﻟﺪارة ‪ :‬اﻟﺸﻜﻞ ‪. 1‬‬
‫‪ - 2‬ﺣﺱﺐ ﻗﺎﻧﻮن ﺟﻤﻊ اﻟﺘﻮﺗﺮات )ﻗـﺎﻧﻮن أوم( ‪uC + u R = E :‬‬
‫‪uC‬‬ ‫‪dq‬‬ ‫‪q‬‬ ‫‪E‬‬ ‫‪q‬‬ ‫‪dq‬‬
‫‪C‬‬ ‫)‪(1‬‬ ‫‪+‬‬ ‫=‬ ‫‪،‬‬ ‫‪+R‬‬ ‫‪=E‬‬
‫‪dt RC R‬‬ ‫‪C‬‬ ‫‪dt‬‬
‫‪E‬‬
‫‪dq‬‬
‫‪R‬‬ ‫‪uR‬‬ ‫‪ - 3‬ﻧﺸﺘﻖ اﻟﻌﺒﺎرة ‪ q = Aeα t + B‬ﺑﺎﻟﻨﺱﺒﺔ ﻟﻠﺰﻣﻦ ‪= Aα eα t :‬‬
‫‪dt‬‬
‫‪Aα eα t +‬‬
‫‪1‬‬
‫‪RC‬‬
‫= ‪Aeα t + B‬‬‫(‬‫‪E‬‬
‫‪R‬‬
‫)‬
‫ﺑﺎﻟﺘﻌﻮﻳﺾ ﻓﻲ اﻟﻤﻌﺎدﻟﺔ )‪: (1‬‬

‫اﻟﺸﻜﻞ ‪1 -‬‬ ‫⎛‬ ‫‪1 ⎞ B‬‬ ‫‪E‬‬


‫‪ ، Aeα t ⎜ α +‬وﺣﺘﻰ ﺗﻜﻮن هﺬﻩ اﻟﻤﻌﺎدﻟﺔ ﻣﺘﺠﺎﻧﺱﺔ ﻳﺠﺐ أن ﻳﻜﻮن ‪:‬‬ ‫‪⎟+‬‬ ‫=‬
‫⎝‬ ‫‪RC ⎠ RC R‬‬
‫‪1‬‬ ‫‪1‬‬
‫‪ ، α = −‬وﺑﺎﻟﺘﺎﻟﻲ ‪. B = CE‬‬ ‫‪ ، α +‬وﻣﻨﻩ‬ ‫‪=0‬‬
‫‪RC‬‬ ‫‪RC‬‬
‫ﻧﺠﺪ ﻋﺒﺎرة ‪ A‬ﺑﺎﻟﺸﺮوط اﻻﺑﺘﺪاﺋﻴﺔ ‪ ،‬أي أن ﺷﺡﻨﺔ اﻟﻤﻜﺜّﻔﺔ ﺗﻜﻮن ﻣﻌﺪوﻣﺔ ﻋﻨﺪ ‪ ) ، t = 0‬وهﺬا ﻟﻢ ﻳُﺸﺎر ﻟﻩ ﻓﻲ اﻟﺘﻤﺮﻳﻦ ‪ ،‬آﺎن ﻣﻦ‬
‫اﻷﺣﺱﻦ أن ﻧﻘﻮل ‪ :‬ﺗﺡﺘﻮي اﻟﺪارة ﻋﻠﻰ ﻣﻜﺜّﻔﺔ ﻓﺎرﻏﺔ(‬
‫ﺑﺎﻟﺘﻌﻮﻳﺾ ﻓﻲ اﻟﻤﻌﺎدﻟﺔ اﻟﺰﻣﻨﻴﺔ ‪ ، 0 = Ae 0 + B :‬وﻣﻨﻩ ‪. A = − B = −CE‬‬
‫‪-4‬‬
‫أ( ﺛﺎﺑﺖ اﻟﺰﻣﻦ ) ‪ (τ‬هﻮ اﻟﺰﻣﻦ اﻟﻤﻮاﻓﻖ ﻟـ ‪. q = 0,63q ( max ) = 0,63 × 4,8 × 10 − 4 ≈ 3 × 10 − 4 C‬‬
‫‪) τ ≈ 40ms‬اﻟﺸﻜﻞ ‪. (2‬‬
‫(‬ ‫‪−4‬‬
‫‪q 10 C‬‬ ‫)‬ ‫ﻧﻘﺘﺮح ﻣﺠﺎﻻ ﻟـ ‪[39ms − 40,5ms] : τ‬‬
‫ﺱﻌﺔ اﻟﻤﻜﺜّﻔﺔ ‪:‬‬
‫‪−3‬‬
‫‪τ‬‬ ‫‪40 × 10‬‬
‫=‪C‬‬ ‫=‬ ‫‪= 40 × 10 − 6 F‬‬
‫‪R‬‬ ‫‪1000‬‬
‫‪q ( max ) 4,8 × 10 − 4‬‬
‫=‪E‬‬ ‫=‬ ‫ب( ‪= 12V‬‬
‫‪3‬‬
‫‪C‬‬ ‫‪40 × 10 − 6‬‬
‫ﺟـ( اﻟﻠﺡﻈﺔ ‪ t = 200ms‬ﺗﻤﺜّﻞ ﻣﺪة ﺗﺱﺎوي‬
‫‪ ، 5τ‬وﺑﺎﻟﺘﺎﻟﻲ ‪:‬‬
‫‪1‬‬
‫‪Ec = CE 2 = 0,5 × 40 × 10 − 6 × 144‬‬
‫‪1‬‬ ‫‪2‬‬
‫‪Ec  2,9 × 10 − 3 J‬‬
‫) ‪t ( ms‬‬
‫‪20‬‬ ‫‪40‬‬ ‫‪1‬‬
‫اﻟﺸﻜﻞ ‪2 -‬‬ ‫أو ‪. E c = q ( max ) E = 0,5 × 4,8 × 10 − 4 × 12 ≈ 2,9 × 10 − 3 J :‬‬
‫‪2‬‬
‫اﻟﺘﻤﺮﻳﻦ اﻟﺜــﺎﻧﻲ )‪ 4‬ﻧﻘﻂ( ‪---------------------------------------------------------------------------------------------------------------------‬‬
‫‪–1‬‬
‫أ( ‪ -‬اﻟﻤﺮﺣﻠﺔ اﻷوﻟﻰ ‪ : t ∈ [0 ; 16 s ] :‬اﻟﻌﺒﺎرة اﻟﺒﻴﺎﻧﻴﺔ ﻟﻠﺱﺮﻋﺔ ﻣﻦ اﻟﺸﻜﻞ ‪ ، v = a t‬ﺣﻴﺚ ‪ a‬ﻳﻤﺜّﻞ اﻟﺘﺱﺎرع ‪ ،‬واﻟﺱﺮﻋﺔ ﺗﺰداد‬
‫‪8‬‬
‫ﺑﻤﺮور اﻟﺰﻣﻦ ‪ ،‬إذن اﻟﺡﺮآﺔ ﻣﺘﺱﺎرﻋﺔ ﺑﺎﻧﺘﻈﺎم ‪= 0,5m / s 2 .‬‬
‫=‪. a‬‬
‫‪16‬‬
‫‪. a=0‬‬ ‫‪ -‬اﻟﻤﺮﺣﻠﺔ اﻟﺜﺎﻧﻴﺔ ‪ ، v = 8 m / s : t ∈ [16 s ; 24 s ] :‬أي اﻟﺱﺮﻋﺔ ﺛﺎﺑﺘﺔ ‪ ،‬وﺑﺎﻟﺘﺎﻟﻲ اﻟﺡﺮآﺔ ﻣﻨﺘﻈﻤﺔ ‪.‬‬

‫= ‪AC‬‬
‫‪(8 + 24) × 8 = 128 m‬‬
‫ب( ﺗﻤﺜّﻞ اﻟﻤﺱﺎﻓﺔ اﻟﻤﻘﻄﻮﻋﺔ ﻣﺱﺎﺣﺔ ﺷﺒﻩ ﻣﻨﺡﺮف ﻗﺎﻋﺪﺗﺎﻩ ‪ 24‬و ‪ 8‬وارﺗﻔﺎﻋﻩ ‪: 8‬‬
‫‪2‬‬
‫أو ﻣﺱﺎﺣﺔ ﻣﺜﻠﺚ زاﺋﺪ ﻣﺱﺎﺣﺔ ﻣﺱﺘﻄﻴﻞ ‪.‬‬
‫‪1‬‬
‫ﻳُﻤﻜﻦ اﻟﺠﻮاب هﻜﺬا ‪ :‬ﺑﻬﺬﻩ اﻟﻄﺮﻳﻘﺔ ‪AC = AB + BC = at12 + v B t 2 2 = 0,5 × 0,5 × (16) + 8 × 8 = 128 m :‬‬
‫‪2‬‬
‫‪2‬‬

‫‪1‬‬
‫‪-2‬‬
‫أ( ﻧﺹ اﻟﻘﺎﻧﻮن اﻟﺜﺎﻧﻲ ﻟﻨﻴﻮﺗﻦ ‪ :‬ﻓﻲ ﻣﻌﻠﻢ ﻏﺎﻟﻴﻠﻲ ‪ ،‬إذا أﺛّﺮت ﻣﺠﻤﻮﻋﺔ ﻣﻦ اﻟﻘﻮى اﻟﺨﺎرﺟﻴﺔ ﻋﻠﻰ ﺟﻤﻠﺔ آﺘﻠﺘﻬﺎ ‪ ، m‬ﻓﺈن هﺬﻩ‬
‫‪G‬‬ ‫‪G‬‬
‫(‬
‫‪. ∑ Fext = ma‬‬ ‫)‬ ‫اﻟﻘﻮى ﺗﻜﻮن ﻣﺘﻨﺎﺱﺒﺔ ﻓﻲ آﻞ ﻟﺡﻈﺔ ﻣﻊ ﺗﺱﺎرع اﻟﺠﻤﻠﺔ ‪.‬‬
‫ب( ﻣﻦ ‪ A‬إﻟﻰ ‪: B‬‬
‫ﺑﺘﻄﺒﻴﻖ اﻟﻘﺎﻧﻮن اﻟﺜـﺎﻧﻲ ﻟﻨﻴﻮﺗﻦ ﻓﻲ ﻣﻌﻠﻢ ﻏﺎﻟﻴﻠﻲ ﻣﺮﺗﺒﻂ ﺑﺱﻄﺢ اﻷرض ‪:‬‬
‫‪G G‬‬ ‫‪G‬‬
‫‪ ،‬وﺑﺈﺱﻘﺎط هﺬﻩ اﻟﻌﻼﻗﺔ ﻋﻠﻰ ﻣﺡﻮر ﻣﻮﺟّﻩ ﻓﻲ ﺟﻬﺔ اﻟﺡﺮآﺔ ‪:‬‬ ‫‪P + F = ma‬‬
‫‪G‬‬ ‫‪G‬‬ ‫‪ma‬‬
‫‪F‬‬ ‫)‪(1‬‬ ‫=‪F‬‬ ‫‪ ، F cos α = ma‬وﻣﻨﻩ‬
‫‪G‬‬ ‫‪F‬‬ ‫‪cos α‬‬
‫‪α‬‬ ‫‪f‬‬
‫‪10 × 0,5‬‬
‫=‪F‬‬ ‫ﺑﺎﻟﺘﻌﻮﻳﺾ ﻓﻲ )‪= 5,77 N : (1‬‬
‫‪A‬‬ ‫‪0,866‬‬
‫‪B‬‬ ‫‪C‬‬ ‫‪G G G‬‬
‫ﺟـ( ﻓﻲ اﻟﻤﺮﺣﻠﺔ اﻟﺜﺎﻧﻴﺔ ) ﻣﻦ ‪ B‬إﻟﻰ ‪P + F + f = 0 : ( C‬‬
‫‪G‬‬ ‫‪G‬‬
‫‪P‬‬ ‫‪.‬‬ ‫‪P‬‬ ‫ﺑﺎﻹﺱﻘﺎط ‪f = F cos α = 5,77 × 0,866 = 5 N ، F cos α − f = 0 :‬‬
‫د( ﻋﻨﺪﻣﺎ ﺗﺼﺒﺢ اﻟﻘﻮى اﻟﻤﺡﺮآﺔ ﻣﺘﺱﺎوﻳﺔ ﻓﻲ اﻟﺸﺪّة ﻟﻠﻘﻮى اﻟﻤﻌﺮﻗﻠﺔ ﺗﺼﺒﺢ اﻟﺱﺮﻋﺔ ﺛﺎﺑﺘﺔ ‪.‬‬
‫أو ‪ :‬ﺣﺱﺐ اﻟﻘﺎﻧﻮن اﻷول ﻟﻨﻴﻮﺗﻦ )ﻣﺒﺪأ اﻟﻌﻄﺎﻟﺔ ( ‪ :‬ﻓﻲ ﻣﻌﻠﻢ ﻏﺎﻟﻴﻠﻲ ‪ ،‬إذا أﺛﺮت ﻣﺠﻤﻮﻋﺔ ﻣﻦ اﻟﻘﻮى ﻋﻠﻰ ﺟﻤﻠﺔ وﺗﺡﺮّآﺖ ﺛﻢ اﻧﻌﺪﻣﺖ ﻓﺠﺄة‬
‫هﺬﻩ اﻟﻘﻮى ﻓﺈن اﻟﺠﻤﻠﺔ ﺗﻮاﺻﻞ اﻟﺡﺮآﺔ ﺑﺱﺮﻋﺔ ﺛﺎﺑﺘﺔ ‪.‬‬

‫اﻟﺘﻤﺮﻳﻦ اﻟﺜﺎﻟﺚ )‪ 4‬ﻧﻘﻂ( ‪----------------------------------------------------------------------------------------------------------------------‬‬


‫‪-1‬‬
‫‪2‬‬
‫‪1H‬‬ ‫= ‪+ 13 H‬‬ ‫‪A‬‬
‫‪Z X‬‬ ‫‪+‬‬ ‫‪1‬‬
‫‪0n‬‬
‫ﺣﺱﺐ ﻗﺎﻧﻮﻧﻲ اﻻﻧﺡﻔﺎظ ‪Z = 2 ، A = 5 − 1 = 4 :‬‬
‫‪ - 2‬اﻻﻧﺪﻣﺎج اﻟﻨﻮوي ‪ :‬ﺗﻔﺎﻋﻞ ﻣﻔﺘﻌﻞ ﻳﺘﻢ ﻓﻴﻩ دﻣﺞ ﻧﻮاﺗﻴﻦ ﺧﻔﻴﻔﺘﻴﻦ واﻟﺡﺼﻮل ﻋﻠﻰ ﻧﻮاة أآﺜﺮ اﺱﺘﻘﺮارا وﺗﺡﺮﻳﺮ اﻟﻄﺎﻗﺔ ‪.‬‬
‫‪El‬‬
‫أآﺒﺮ ‪ ،‬آﻠّﻤﺎ آﺎﻧﺖ اﻟﻨﻮاة أآﺜﺮ اﺱﺘﻘﺮارا ‪،‬‬ ‫‪ - 3‬آﻠﻤﺎ آﺎن‬
‫‪A‬‬
‫‪El‬‬
‫‪A‬‬
‫‪( H ) = 2,223 = 1,15MeV‬‬
‫‪2‬‬
‫‪1‬‬ ‫‪،‬‬
‫‪El‬‬
‫‪A‬‬
‫‪( H ) = 8,57‬‬
‫‪3‬‬
‫‪1‬‬
‫‪3‬‬
‫‪= 2,85MeV‬‬ ‫‪،‬‬
‫‪El‬‬
‫‪A‬‬
‫‪( X ) = 28,441 = 7,10MeV‬‬
‫‪A‬‬
‫‪Z‬‬ ‫ﻟﺪﻳﻨﺎ ‪:‬‬

‫وﺑﺎﻟﺘﺎﻟﻲ‬
‫اﺱﺘﻘﺮار ﻣﺘﺰاﻳﺪ‬
‫‪2‬‬ ‫‪3‬‬ ‫‪A‬‬
‫‪1H‬‬ ‫‪1H‬‬ ‫‪Z X‬‬

‫) ‪E ( MeV‬‬ ‫‪Elib = El‬‬ ‫‪( X ) − E ( H ) − E ( H ) = 28, 41 − 2, 23 − 8,57 = 17,61MeV‬‬


‫‪A‬‬
‫‪Z‬‬
‫‪2‬‬
‫‪l 1‬‬
‫‪3‬‬
‫‪l 1‬‬ ‫‪-4‬‬
‫‪ - 5‬ﻣﺨﻄﻂ اﻟﺡﺼﻴﻠﺔ اﻟﻄﺎﻗﻮﻳﺔ ‪ :‬اﻟﺸﻜﻞ اﻟﻤﻘﺎﺑﻞ‬
‫‪2 p + 3n‬‬

‫‪Δ E1 = 10,8‬‬
‫‪2‬‬
‫‪1‬‬ ‫‪H + 13 H‬‬

‫‪Δ E3 = −28, 41‬‬


‫‪Δ E2 = −17,61‬‬
‫‪A‬‬
‫‪Z‬‬ ‫‪X +‬‬ ‫‪1‬‬
‫‪0‬‬ ‫‪n‬‬

‫اﻟﺘﻤﺮﻳﻦ اﻟﺮاﺑﻊ )‪ 4‬ﻧﻘﻂ( ‪-------------------------------------------------------------------------------------------------------------------------‬‬

‫‪CH 3COOH + H 2O = CH 3COO − + H 3O +‬‬ ‫‪-1‬‬

‫‪σ = λH‬‬
‫‪3O‬‬
‫‪+‬‬
‫‪⎡ H 3O + ⎤ + λ‬‬
‫⎣‬ ‫⎦‬
‫‪⎡CH 3COO − ⎤ = ⎡ H 3O + ⎤ λ‬‬
‫⎣ ‪CH 3COO −‬‬ ‫⎣ ⎦‬ ‫‪⎦ H 3O +‬‬
‫‪+λ‬‬
‫‪CH 3COO −‬‬ ‫(‬ ‫)‬ ‫‪-2‬‬

‫= ⎤ ‪⎡ H 3O +‬‬ ‫‪σ‬‬ ‫‪16 × 10 − 3‬‬


‫=‬ ‫‪= 0, 41mol / m 3 = 4,1 × 10 − 4 mol / L‬‬
‫⎣‬ ‫‪⎦ λ‬‬ ‫‪+‬‬ ‫‪+λ‬‬ ‫‪−‬‬ ‫‪39,1 × 10 − 3‬‬
‫‪H 3O‬‬ ‫‪CH‬‬ ‫‪3COO‬‬

‫⎣‬ ‫⎦‬ ‫(‬


‫‪pH = − Log ⎡ H 3O + ⎤ = − Log 4,1 × 10 − 4 = 3, 4‬‬ ‫)‬ ‫‪-3‬‬

‫‪-4‬‬
‫⎤ ‪⎡ H 3O + ⎤ ⎡CH 3COO −‬‬
‫⎣ = ‪KA‬‬ ‫⎣⎦‬ ‫‪⎦ = 10 − 3,4 × 41, 43 × 10 − 3 = 1,6 × 10 − 5‬‬ ‫أ(‬
‫] ‪[CH 3COOH‬‬
‫‪2‬‬
‫أو ‪:‬‬
‫⎤ ‪⎡CH 3COO −‬‬
‫‪K A = 10 − pK A = 10 − 4,78 = 1,6 × 10 − 5‬‬ ‫‪،‬‬ ‫⎣ ‪pK A = pH − Log‬‬ ‫‪⎦ = 3, 4 − Log 41, 43 × 10 − 3 = 4,78‬‬
‫] ‪[CH 3COOH‬‬
‫ب( ﺣﺠﻢ اﻟﻤﺡﻠﻮل اﻷﺱـﺎﺱﻲ اﻟﻼزم ﻟﻠﺘﻜﺎﻓﺆ هﻮ ‪VbE = 2 × 10 = 20mL‬‬
‫‪CbVbE 2 × 10 − 3 × 20‬‬
‫= ‪Va‬‬ ‫=‬ ‫ﻋﻨﺪ اﻟﺘﻜﺎﻓﺆ ﻳﻜﻮن ‪ ، C Va = CbVbE :‬وﻣﻨﻩ ‪= 4 mL‬‬
‫‪C‬‬ ‫‪10 − 2‬‬
‫اﻟﺘﻤﺮﻳﻦ اﻟﺘﺠﺮﻳﺒﻲ ! )‪ 4‬ﻧﻘﻂ( ‪------------------------------------------------------------------------------------------------------------- ---‬‬

‫‪ – 1‬وﺟﻮد اﻟﺠﻠﻴﺪ ﻓﻲ اﻟﻤﺎء ﻳﺠﻌﻞ درﺟﺔ ﺣﺮارة اﻟﻤﺎء ﻣﺱﺎوﻳﺔ داﺋﻤﺎ ‪ . 0 °C‬واﻟﻐﺮض هﻮ ﺗﺒﺮﻳﺪ اﻟﻤﺰﻳﺞ ﻣﻦ أﺟﻞ إﻳﻘﺎﻓﻩ واﻟﺘﻤﻜّﻦ ﻣﻦ ﻣﻌﺎﻳﺮة‬
‫اﻟﺡﻤﺾ اﻟﻨﺎﺗﺞ وﻣﻨﻊ ﺗﻔﺎﻋﻞ اﻷﺱﺘﺮ ﻣﻊ اﻷﺱﺎس ‪.‬‬

‫اﻟﻜﺎﺷﻒ اﻟﻤﻠﻮّن ﻳﻨﻘﻠﺐ ﻟﻮﻧﻩ ﻋﻨﺪ ﻧﻘﻄﺔ اﻟﺘﻜﺎﻓﺆ ‪ ،‬إذن وﺟﻮدﻩ هﻮ ﻣﻦ أﺟﻞ رﺻْﺪ ﻧﻘﻄﺔ اﻟﺘﻜﺎﻓﺆ ‪.‬‬
‫‪ - 2‬اﻟﺼﻴﻐﺔ اﻟﺠﺰﻳﺌﻴﺔ ﻧﺼﻒ اﻟﻤﻔﺼّﻠﺔ ﻟﻸﺱﺘﺮ ‪HCOO − C 2 H 5 :‬‬
‫‪-3‬‬
‫أ( اﻟﺘﺡﻮّل اﻟﻜﻴﻤﻴﺎﺋﻲ هﻮ إﻣـــﺎهﺔ أﺱﺘﺮ ‪.‬‬
‫ﺧﺼـﺎﺋﺹ اﻟﺘﺡﻮّل اﻟﻜﻴﻤﻴﺎﺋﻲ ﻋﻨﺪ ﺣﺎﻟﺔ اﻟﺘﻮازن ‪ :‬ﻳﺘﻮﻗﻒ ﻇﺎهﺮﻳﺎ ‪ ،‬ﻷن ﺱﺮﻋﺔ اﻹﻣﺎهﺔ ﺗﺼﺒﺢ ﻣﺱﺎوﻳﺔ ﻟﺱﺮﻋﺔ اﻷﺱﺘﺮة ﺑﺎﻟﻘﻴﻤﺔ اﻟﻤﻄﻠﻘﺔ ‪.‬‬

‫آﺎن ﻣﻦ اﻷﻓﻀﻞ ﻃﺮح اﻟﺱﺆال آﻤﺎ ﻳﻠﻲ ‪ :‬اذآﺮ ﺧﺼﺎﺋﺺ هﺬا اﻟﺘﺤﻮّل اﻟﻜﻴﻤﻴﺎﺋﻲ ‪.‬‬
‫ﻓﻴﻜﻮن اﻟﺠﻮاب آﻤﺎ ﻳﻠﻲ ‪ :‬ﺑﻄﻲء ‪ -‬ﻻ ﺣﺮاري – ﻏﻴﺮ ﺗــﺎم ‪.‬‬
‫ب( ‪HCOO − C2 H5 + H 2O = HCOOH + C2 H 5 − OH‬‬
‫‪ - 4‬ﻋﻨﺪ اﻟﺘﻜﺎﻓﺆ ﺗﻜﻮن آﻤﻴﺔ ﻣﺪة اﻟﺡﻤﺾ ﻣﺱﺎوﻳﺔ ﻟﻜﻤﻴﺔ ﻣﺎدة اﻷﺱﺎس ‪ ،‬أي ‪n A = n B = CbVéq‬‬
‫ﻣﻦ اﻟﻤﻔﺮوض إﻋﻄﺎء اﻟﻜﺘﻠﺔ اﻟﺡﺠﻤﻴﺔ ﻟﻠﻤﺎء ) ‪ ( ρ e = 1g / mL‬ﻓﻲ اﻟﺘﻤﺮﻳﻦ ‪.‬‬ ‫آﺘﻠﺔ اﻟﻤـﺎء اﻻﺑﺘﺪاﺋﻴﺔ ‪m = ρ e × V = 1 × 10 = 10 g‬‬
‫‪10‬‬
‫= ) ‪n ( H 2O‬‬ ‫آﻤﻴﺔ ﻣﺎدة اﻟﻤﺎء اﻻﺑﺘﺪاﺋﻴﺔ ‪= 0,55 mol :‬‬
‫‪18‬‬
‫ﺟﺪول اﻟﺘﻘﺪّم ‪:‬‬
‫‪HCOO − C2 H5‬‬ ‫‪+‬‬ ‫‪H 2O‬‬ ‫=‬ ‫‪HCOOH‬‬ ‫‪+ C2 H 5 − OH‬‬
‫‪0,55‬‬ ‫‪0‬‬ ‫‪0‬‬ ‫ﻣﻦ اﻟﺠﺪول ﻟﺪﻳﻨﺎ ‪x f = n A = CbVéq :‬‬
‫‪4,5 × 10−3‬‬
‫‪0,55 − x‬‬ ‫‪x‬‬ ‫‪x‬‬ ‫ﻧﺡﺱﺐ اﻟﺘﻘﺪّم ﺑﺎﺱﺘﻌﻤﺎل هﺬﻩ اﻟﻌﻼﻗﺔ ‪:‬‬
‫‪4,5 × 10−3 − x‬‬
‫‪4,5 × 10−3 − x f‬‬ ‫‪0,55 − x f‬‬ ‫‪xf‬‬ ‫‪xf‬‬

‫‪4,5 × 10−3 − xm‬‬ ‫‪0,55 − xm‬‬ ‫‪xm‬‬ ‫‪xm‬‬

‫) ‪t ( mn‬‬ ‫‪0‬‬ ‫‪10‬‬ ‫‪20‬‬ ‫‪30‬‬ ‫‪40‬‬ ‫‪50‬‬ ‫‪60‬‬ ‫‪70‬‬ ‫‪80‬‬
‫) ‪x ( mmol‬‬ ‫‪0‬‬ ‫‪1,05‬‬ ‫‪1,85‬‬ ‫‪2,50‬‬ ‫‪3,05‬‬ ‫‪3,50‬‬ ‫‪3,80‬‬ ‫‪3,90‬‬ ‫‪3,90‬‬
‫) ‪x ( mmol‬‬
‫‪–5‬‬
‫أ( رﺱﻢ اﻟﺒﻴﺎن ﻋﻠﻰ ورﻗﺔ ﻏﻴﺮ ﻣﻠﻤﺘﺮﻳﺔ ‪.‬‬
‫‪xf‬‬
‫‪ ،‬ﺣﻴﺚ ‪. x m = 4,5 mmol‬‬ ‫=‪r‬‬ ‫ب( اﻟﻤﺮدود ‪× 100 :‬‬
‫‪60 °C‬‬ ‫‪xm‬‬
‫‪3,9 × 10 − 3‬‬
‫=‪r‬‬ ‫‪× 100 = 86,7%‬‬
‫‪40 °C‬‬ ‫‪4,5 × 10 − 4‬‬
‫ﻳﻤﻜﻦ ﻣﺮاﻗﺒﺔ ﻣﺮدود هﺬا اﻟﺘﺡﻮّل ﺑﺈﺣﺪى اﻟﻄﺮﻳﻘﺘﻴﻦ ‪:‬‬
‫‪ -‬ﺟﻌﻞ آﻤﻴﺘﻲ اﻟﻤﺎء واﻷﺱﺘﺮ ﻏﻴﺮ ﻣﺘﻜﺎﻓﺌﺘﻴﻦ ‪ ،‬أي اﻹآﺜﺎر ﻣﻦ‬
‫‪1‬‬ ‫آﻤﻴﺔ أﺣﺪ اﻟﻤﺘﻔﺎﻋﻠﻴﻦ ‪.‬‬
‫‪ -‬ﺱﺡﺐ اﻟﺡﻤﺾ أو اﻟﻜﺡﻮل أﺛﻨﺎء اﻟﺘﻔﺎﻋﻞ ﻟﻤﻨﻊ ﺗﻔﺎﻋﻞ اﻷﺱﺘﺮة‪.‬‬

‫) ‪t ( mn‬‬
‫‪10‬‬
‫‪ - 6‬درﺟﺔ اﻟﺡﺮارة ﻋﺒﺎرة ﻋﻦ ﻋﺎﻣﻞ ﺣﺮآﻲ ‪.‬‬
‫رﻓﻊ درﺟﺔ اﻟﺡﺮارة ﻳﺱﺮّع اﻟﺘﻔﺎﻋﻞ ﺑﺪون اﻟﺘﺄﺛﻴﺮ ﻋﻠﻰ اﻟﻤﺮدود ﻷن اﻟﺘﻔﺎﻋﻞ ﻻ ﺣﺮاري )اﻟﺒﻴﺎن ﻋﻠﻰ ﻧﻔﺲ اﻟﺸﻜﻞ( ‪.‬‬

‫‪3‬‬
‫ﺑﻜﺎﻟﻮرﻳﺎ ‪2013‬‬
‫ﻋﻠﻮم ﻓﻴﺰﻳﺎﺋﻴﺔ – ﺷﻌﺒﺔ اﻟﻌﻠﻮم اﻟﺘﺠﺮﻳﺒﻴﺔ‬
‫اﻟﻤﻮﺿــﻮع اﻟﺜــﺎﻧﻲ‬
‫‪www.guezouri.org‬‬
‫‪---------------------------------------‬‬
‫اﻟﺘﻤﺮﻳﻦ اﻷول )‪ 4‬ﻧﻘﻂ(‬
‫‪ - 1‬دور اﻟﺘﺴﺨﻴﻦ اﻟﻤﺮﺗﺪّ ‪ :‬اﻟﻤﺡﺎﻓﻈﺔ ﻋﻠﻰ آﻤﻴﺔ ﻣﺎدة اﻷﻧﻮاع اﻟﻜﻤﻴﺎﺋﻴﺔ داﺧﻞ اﻟﻤﺰﻳﺞ ‪ ،‬أي ﺗﺒﺮﻳﺪ اﻷﺑﺨﺮة ﻓﻲ اﻟﻤﺒﺮّد اﻟﻤﺎﺋﻲ وإرﺟﺎﻋﻬﺎ‬
‫ﻟﻠﻤﺰﻳﺞ ‪.‬‬
‫دور ﺡﻤﺾ اﻟﻜﺒﺮﻳﺖ ‪ :‬ﺗﺴﺮﻳﻊ اﻟﺘﻔﺎﻋﻞ )وﺳﻴﻂ ( ‪.‬‬
‫‪ – 2‬اﺳﺘﻌﻤﺎل اﻟﻤﺎء اﻟﻤﺎﻟﺡ ‪ :‬ﻟﻌﺰل اﻷﺳﺘﺮ )اﻷﺳﺘﺮ ﻳﻄﻔﻮ ﻓﻮق اﻟﻤﺎء اﻟﻤﺎﻟﺡ وﻻ ﻳﻨﺡﻞ ﻓﻴﻩ( ‪.‬‬
‫‪-3‬‬
‫أ( ﻣﻌﺎدﻟﺔ اﻟﺘﻔﺎﻋﻞ ‪CH 3COOH + C 4 H 9 − OH = CH 3COO − C 4 H 9 + H 2O :‬‬
‫ب( اﻟﺘﻔﺎﻋﻞ اﻟﻜﻴﻤﻴـﺎﺋﻲ ﻟﻴﺲ ﺗﺎﻣّﺎ ‪ ،‬واﻟﺪﻟﻴﻞ ‪:‬‬

‫‪CH 3COOH + C 4 H 9 − OH = CH 3COO − C 4 H 9 + H 2O‬‬ ‫‪xf‬‬ ‫‪n E 0,6‬‬


‫= ‪τf‬‬ ‫=‬ ‫=‬ ‫‪= 0,6‬‬
‫‪1‬‬ ‫‪1‬‬ ‫‪0‬‬ ‫‪0‬‬ ‫‪xm‬‬ ‫‪xm‬‬ ‫‪1‬‬
‫‪1− x‬‬ ‫‪1− x‬‬ ‫‪x‬‬ ‫‪x‬‬
‫‪τ f <1‬‬
‫‪1− xf‬‬ ‫‪1− xf‬‬ ‫‪xf‬‬ ‫‪xf‬‬
‫‪1 − xm‬‬ ‫‪1 − xm‬‬ ‫‪xm‬‬ ‫‪xm‬‬
‫أو ‪:‬‬
‫ﺑﻤﺎ أن آﻤﻴّﺔ ﻣﺎدة اﻷﺳﺘﺮ ﻋﻨﺪ اﻟﺘﻮازن أﻗﻞ ﻣﻦ ‪ ، 1mol‬إذن اﻟﺘﻔﺎﻋﻞ ﻏﻴﺮ ﺗــﺎم ‪.‬‬
‫ﻟﻜﻲ ﻧﺘﺄآﺪ أﻧﻩ ﻏﻴﺮ ﺗﺎم ‪ ،‬ﻧﻘﻮم ﺑﻘﻴﺎس ‪ pH‬اﻟﻤﺰﻳﺞ ﻣﻦ ﺡﻴﻦ ﻵﺧﺮ ‪ ،‬وﻋﻨﺪﻣﺎ ﻧﺡﺼﻞ ﻋﻠﻰ ﻗﻴﻢ ﺛﺎﺑﺘﺔ ‪ ،‬ﻧﺠﺰم أن اﻟﺘﻔﺎﻋﻞ وﺻﻞ ﻟﺡﺎﻟﺔ اﻟﺘﻮازن‬

‫) ‪n E ( mol‬‬ ‫واﻟﺡﻤﺾ ﻟﻢ ﻳﺘﻔﺎﻋﻞ آﻠﻩ ‪.‬‬


‫ﺟـ( ﺳﺮﻋﺔ اﻟﺘﻔﺎﻋﻞ ‪:‬‬

‫•‬

‫•‬

‫•‬

‫‪0,1‬‬

‫‪t ( m) n‬‬
‫‪20‬‬ ‫‪40‬‬ ‫‪60‬‬

‫ﻣﻼﺡﻈﺔ ‪ :‬ﻟﺡﻈﺘﺎن زﻣﻨﻴﺘﺎن آﺎﻓﻴﺘﺎن ﻟﻠﻤﻨﺎﻗﺸﺔ ‪ ،‬إذن ﻟﻤﺎذا ﺗﺸﺘﺘﻴﺖ ﺗﺮآﻴﺰ اﻟﺘﻠﻤﻴﺬ ﺑﻜﻞ هﺬﻩ اﻟﺡﺴﺎﺑﺖ اﻟﻤﺘﺸﺎﺑﻬﺔ ؟؟‬
‫آﺎن ﻣﻦ اﻷﻓﻀﻞ ﻃﻠﺐ ﺡﺴﺎب اﻟﺴﺮﻋﺔ ﻓﻲ اﻟﻠﺡﻈﺔ ‪ t1‬ﻓﻘﻂ ‪ ،‬وﺗُﻌﻄﻰ ﻗﻴﻢ اﻟﺴﺮﻋﺔ ﻓﻲ ‪ t 2‬و ‪ t 3‬ﻋﺪدﻳﺎ ‪.‬‬
‫‪dx dn E‬‬
‫=‪. v‬‬ ‫=‬ ‫ﻣﻦ ﺟﺪول اﻟﺘﻘﺪّم ﻟﺪﻳﻨﺎ ‪ ، n E = x‬وﻟﺪﻳﻨﺎ ﺳﺮﻋﺔ اﻟﺘﻔﺎﻋﻞ ‪:‬‬
‫‪dt‬‬ ‫‪dt‬‬
‫ﺗﻤﺜّﻞ ﺳﺮﻋﺔ اﻟﺘﻔﺎﻋﻞ ﻣﻴﻞ اﻟﻤﻤﺎس ﻓﻲ اﻟﻠﺡﻈﺎت اﻟﻤﻄﻠﻮﺑﺔ ‪.‬‬

‫‪v1 = 1,06 × 10 − 2 mol.mn − 1‬‬ ‫‪ -‬ﻓﻲ اﻟﻠﺡﻈﺔ ‪: t1 = 20mn‬‬


‫‪−3‬‬ ‫‪−1‬‬
‫‪v2 = 5 × 10 mol.mn‬‬ ‫‪ -‬ﻓﻲ اﻟﻠﺡﻈﺔ ‪: t 2 = 40mn‬‬
‫‪v3 = 2 × 10 − 3 mol.mn − 1‬‬ ‫‪ -‬ﻓﻲ اﻟﻠﺡﻈﺔ ‪: t 3 = 60mn‬‬
‫اﻟﺴﺮﻋﺔ ﺗﺘﻨﺎﻗﺹ ﺑﻤﺮور اﻟﺰﻣﻦ ‪ ،‬ﻧﺴﺘﻨﺘﺞ ﻣﻦ هﺬا أن اﻟﺘﺮاآﻴﺰ اﻻﺑﺘﺪاﺋﻴﺔ ﻋﺒﺎرة ﻋﻦ ﻋﺎﻣﻞ ﺡﺮآﻲ ‪.‬‬
‫‪1‬‬
‫‪xf‬‬ ‫‪0,6‬‬
‫‪ .‬ﻳﻤﻜﻦ ﺗﺡﺴﻴﻦ اﻟﻤﺮدود ﺑﻨﺰع اﻟﻤﺎء أﺛﻨﺎء اﻟﺘﻔﺎﻋﻞ ‪ ،‬وذﻟﻚ ﻣﻦ أﺟﻞ ﻣﻨﻊ ﺗﻔﺎﻋﻞ‬ ‫=‪r‬‬ ‫= ‪× 100‬‬ ‫‪× 100 = 60%‬‬ ‫د( اﻟﻤﺮدود ‪:‬‬
‫‪xm‬‬ ‫‪1‬‬
‫اﻹﻣﺎهﺔ )اﻟﺘﻔﺎﻋﻞ ﻏﻴﺮ اﻟﻤﺒﺎﺷﺮ( ‪.‬‬

‫هـ( ﺑﻤﺎ أن اﻟﻤﺰﻳﺞ اﻻﺑﺘﺪاﺋﻲ ﻣﺘﺴﺎوي اﻟﻤﻮﻻت ‪ ،‬وﻣﺮدود اﻟﺘﻔﺎﻋﻞ ‪ ، r = 60%‬إذن اﻟﻜﺡﻮل اﻟﻤﺴﺘﻌﻤﻞ هﻮ آﺡﻮل ﺛـﺎﻧﻮي ‪.‬‬
‫ﺻﻴﻐﺘﻩ اﻟﺠﺰﻳﺌﻴﺔ ﻧﺼﻒ اﻟﻤﻔﺼّﻠﺔ هﻲ ‪ CH 3 − CHOH − CH 2 − CH 3‬واﺳﻤﻩ ﺑﻮﺗــﺎن – ‪ – 2‬أول ‪.‬‬
‫اﻟﺘﻤﺮﻳﻦ اﻟﺜــﺎﻧﻲ )‪ 4‬ﻧﻘﻂ( ‪--------------------------------------------------------------------------------------------------------------------‬‬
‫‪36‬‬ ‫‪35‬‬
‫و ‪Cl‬‬ ‫اﻟﻌﺪدان ‪ 35‬و ‪ 36‬هﻤﺎ اﻟﻌﺪدان اﻟﻜﺘﻠﻴﺎن ﻟﻠﻨﻈﻴﺮﻳﻦ ‪Cl‬‬ ‫‪-1‬‬
‫أو ‪ :‬هﻤﺎ ﻋﺪد اﻟﻨﻮآﻠﻴﻮﻧﺎت ﻓﻲ آﻞ ﻧﻮاة ‪.‬‬
‫أو ‪ :‬ﻳﻤﺜّﻞ آﻞ ﻋﺪد ﻣﺠﻤﻮع اﻟﺒﺮوﺗﻮﻧﺎت واﻟﻨﻮﺗﺮوﻧﺎت ﻓﻲ آﻞ ﻧﻮاة‪.‬‬
‫‪36‬‬
‫‪.‬‬ ‫‪17 Cl‬‬ ‫رﻣﺰ ﻧﻮاة اﻟﻜﻠﻮر ‪: 36‬‬
‫‪El = ⎡⎣ Zm p + ( A − Z ) mn − mCl ⎤⎦ × c 2 = (17 × 1,67262 + 19 × 1,67492 − 59,71128) × 10 − 27 × 9 × 1016 - 2‬‬
‫‪4,92 × 10 − 11‬‬
‫= ‪El = 4,92 × 10 − 11 J‬‬ ‫‪= 307 MeV‬‬
‫‪1,6 × 10 − 13‬‬
‫‪36‬‬ ‫‪36‬‬
‫‪17 Cl‬‬ ‫‪→ 18‬‬ ‫‪Ar + ZA X‬‬ ‫‪–3‬‬
‫‪−‬‬
‫‪. β‬‬ ‫‪ A = 0‬و ‪ ، Z = − 1‬وﻣﻨﻩ ﻧﻤﻂ اﻟﺘﻔﻜﻚ هﻮ‬ ‫ﺡﺴﺐ ﻗﺎﻧﻮﻧﻲ اﻻﻧﺡﻔﺎظ ﻟﺼﻮدي ﻧﺠﺪ ‪:‬‬
‫‪0,69‬‬ ‫‪0,69‬‬ ‫‪0,69‬‬
‫‪−‬‬ ‫‪t‬‬ ‫‪−‬‬ ‫‪t‬‬ ‫‪−‬‬ ‫‪t‬‬
‫‪38‬‬ ‫‪t 1/ 2‬‬ ‫‪38‬‬ ‫‪t 1/ 2‬‬ ‫‪38‬‬ ‫‪t 1/ 2‬‬
‫‪=e‬‬ ‫‪،‬‬ ‫× ‪N0‬‬ ‫‪= N0 e‬‬ ‫× ‪ ، N = N 0‬وﺑﺎﻟﺘﺎﻟﻲ‬ ‫‪ ، N = N 0 e‬وﻟﺪﻳﻨﺎ‬ ‫‪- 4‬‬
‫‪100‬‬ ‫‪100‬‬ ‫‪100‬‬

‫‪0,69‬‬
‫‪ ،‬وﻣﻨﻩ ‪t = 4, 2 × 10 5 ans‬‬ ‫‪ln 0,38 = −‬‬ ‫‪t‬‬
‫‪t1/ 2‬‬

‫اﻟﺘﻤﺮﻳﻦ اﻟﺜــﺎﻟﺚ )‪ 4‬ﻧﻘﻂ( ‪--------------------------------------------------------------------------------------------------------------------‬‬

‫‪ - 1‬ﻃﺮﻳﻘﺔ اﻟﺮﺑﻂ ‪ :‬اﻟﺸﻜﻞ‬


‫‪di‬‬ ‫‪di‬‬
‫‪. (R + r)i + L‬‬ ‫‪=E‬‬ ‫‪،‬‬ ‫‪Ri + ri + L = E‬‬ ‫‪،‬‬ ‫‪ - 2‬ﺡﺴﺐ ﻗﺎﻧﻮن ﺟﻤﻊ اﻟﺘﻮﺗﺮات ‪u R + ub = E :‬‬
‫‪dt‬‬ ‫‪dt‬‬
‫) ‪du R ( R + r‬‬ ‫‪RE‬‬ ‫‪u‬‬
‫)‪(L , r‬‬ ‫)‪(1‬‬‫‪+‬‬ ‫= ‪uR‬‬ ‫ﻧﻌﻮّض ‪ i = R‬وﻧﺠﺪ ‪:‬‬
‫‪dt‬‬ ‫‪L‬‬ ‫‪L‬‬ ‫‪R‬‬
‫‪du R A − τ‬‬
‫‪t‬‬ ‫⎛‬ ‫⎞ ‪−‬‬
‫‪t‬‬
‫‪E‬‬ ‫‪.‬‬ ‫‪= e‬‬ ‫‪ – 3‬ﺑﺎﺷﺘﻘﺎق اﻟﻌﺒﺎرة اﻟﺰﻣﻨﻴﺔ ⎟ ‪: u R = A ⎜1 − e τ‬‬
‫‪X‬‬ ‫‪dt‬‬ ‫‪τ‬‬ ‫⎜‬ ‫⎟‬
‫⎝‬ ‫⎠‬
‫‪R‬‬
‫⎛ )‪A − τ (R + r‬‬ ‫⎞ ‪−‬‬
‫‪t‬‬ ‫‪t‬‬
‫‪uR‬‬ ‫‪RE‬‬
‫‪e +‬‬ ‫= ⎟ ‪⎜1 − e τ‬‬ ‫ﺑﺎﻟﺘﻌﻮﻳﺾ ﻓﻲ )‪: (1‬‬
‫‪τ‬‬ ‫⎜ ‪L‬‬ ‫‪⎟ L‬‬
‫⎝‬ ‫⎠‬

‫‪R + r ⎞ ( R + r ) A RE‬‬
‫‪t‬‬
‫‪L‬‬ ‫‪1 R+r‬‬ ‫‪−‬‬
‫‪τ ⎛1‬‬
‫=‪τ‬‬ ‫‪ ،‬وﻣﻨﻩ‬ ‫‪−‬‬ ‫‪ ،‬وﺡﺘﻰ ﺗﻜﻮن هﺬﻩ اﻟﻤﻌﺎدﻟﺔ ﻣﺘﺠﺎﻧﺴﺔ ﻳﺠﺐ أن ﻳﻜﻮن ‪= 0 :‬‬ ‫‪⎜ −‬‬ ‫‪Ae‬‬ ‫‪⎟+‬‬ ‫=‬
‫‪R+r‬‬ ‫‪τ‬‬ ‫‪L‬‬ ‫‪⎝τ‬‬ ‫⎠ ‪L‬‬ ‫‪L‬‬ ‫‪L‬‬

‫‪. A= R‬‬
‫‪E‬‬
‫‪ ،‬وﻣﻨﻩ‬
‫وﻳﻜﻮن ﻋﻨﺪﺋﺬ ‪( R + r ) A = RE‬‬
‫‪R+r‬‬ ‫‪L‬‬ ‫‪L‬‬

‫]‪[ L‬‬ ‫‪[U ][T ] × [ I ] = T‬‬


‫‪ ،‬وﻣﻨﻩ ‪ τ‬ﻣﺘﺠﺎﻧﺲ ﻣﻊ اﻟﺰﻣﻦ ‪.‬‬ ‫= ] ‪[τ‬‬ ‫=‬ ‫] [‬ ‫‪-4‬‬
‫] ‪[ R + r ] [ I ] [U‬‬

‫‪2‬‬
‫) ‪u R (V‬‬
‫ﺛﺎﺑﺖ اﻟﺰﻣﻦ هﻮ اﻟﺰﻣﻦ اﻟﻤﻮاﻓﻖ ﻟـ ‪u R = 0,63 × ( 6, 4 × 0,5) ≈ 2V‬‬
‫‪τ = 1, 2ms‬‬

‫‪L‬‬
‫= ‪ ، τ‬وﻣﻨﻩ ‪:‬‬ ‫‪-5‬‬
‫‪R+r‬‬
‫‪2‬‬
‫‪L = τ × ( R + r ) = 1, 2 × 10 − 3 × 15 = 18 × 10 − 3 H‬‬
‫) ‪u R ( max‬‬ ‫‪3, 2‬‬
‫= ‪I0‬‬ ‫=‬ ‫‪= 0,32 A‬‬
‫‪R‬‬ ‫‪10‬‬
‫‪E = ( R + r ) I 0 = 15 × 0,32 = 4,8V‬‬
‫‪0,5‬‬

‫) ‪t ( ms‬‬
‫‪1‬‬ ‫‪1,2‬‬

‫اﻟﺘﻤﺮﻳﻦ اﻟﺮاﺑﻊ )‪ 4‬ﻧﻘﻂ( ‪-----------------------------------------------------------------------------------------------------------------------‬‬


‫أوﻻ‬
‫‪ – 1‬ﺑﺘﻄﺒﻴﻖ اﻟﻘﺎﻧﻮن اﻟﺜﺎﻧﻲ ﻟﻨﻴﻮﺗﻦ ﻋﻠﻰ ﺡﺮآﺔ ﺡﺒّﺔ اﻟﺒَﺮَد ) وﻟﻴﺲ ﺡﺒﺔ اﻟﺒَﺮْد آﻤﺎ ﻗﺎل ﻟﻲ أﺡﺪهﻢ ‪ ،‬ﻷن ﺡﺒﺔ اﻟﺒَﺮْد هﻲ ‪( Actifed‬‬
‫ﻧﻌﺘﺒﺮ اﻟﻤﻌﻠﻢ ﻏﺎﻟﻴﻠﻴﺎ ‪.‬‬
‫‪G‬‬ ‫‪G‬‬
‫‪. a=g‬‬ ‫‪ ، P = ma‬وﺑﺎﻹﺳﻘﺎط ﻋﻠﻰ اﻟﻤﺡﻮر ‪ ، P = ma : Oz‬وﻣﻨﻩ‬
‫‪1 2‬‬
‫‪O‬‬ ‫=‪. z‬‬ ‫اﻟﺡﺮآﺔ ﻣﺘﺴﺎرﻋﺔ ﺑﺎﻧﺘﻈﺎم ﻷن اﻟﺘﺴﺎرع ﺛﺎﺑﺖ ‪ ،‬وﺑﺎﻟﺘﺎﻟﻲ ‪ v = gt + v0 = 9,8 t‬و ‪g t + v0 t + z 0 = 4,9 t 2‬‬
‫‪2‬‬
‫‪ - 2‬ﺳﺮﻋﺔ وﺻﻮ ل ﺡﺒﺔ اﻟﺒﺮد ﻟﺴﻄﺡ اﻷرض ‪v = 2 gh = 2 × 9,8 × 1500 = 171,5 m / s ، v 2 − v0 2 = 2 gh :‬‬

‫ﺛــﺎﻧﻴﺎ‬
‫‪G‬‬ ‫‪−2‬‬
‫‪P‬‬ ‫] ‪[ f ] = [ M ][ L][T‬‬ ‫‪−1‬‬
‫‪ ،‬وﻣﻨﻩ وﺡﺪة ‪ k‬هﻲ ‪. kg / m‬‬ ‫= ] ‪[k‬‬ ‫]‪= [ M ][ L‬‬ ‫‪-1‬‬
‫⎤ ‪⎡v 2‬‬
‫⎦ ⎣‬ ‫‪[ L]2 [T ]− 2‬‬
‫‪4‬‬
‫(‬ ‫)‬
‫‪3‬‬
‫‪π = ρ Vg = 1,3 × × 3,14 × 1,5 × 10 − 2‬‬ ‫‪ - 2‬داﻓﻌﺔ أرﺧﻤﻴﺪس ‪× 9,8 = 1,8 × 10 − 4 N :‬‬
‫‪z‬‬ ‫‪3‬‬
‫ﺷﺪة ﻗﻮّة اﻟﺜﻘﻞ ‪P = mg = 13 × 10 − 3 × 9,8 = 0,127 N :‬‬
‫‪P‬‬ ‫‪0,127‬‬
‫‪ ،‬وﺑﺎﻟﺘﺎﻟﻲ ﻧﻬﻤﻞ داﻓﻌﺔ أرﺧﻤﻴﺪس أﻣﺎم ﻗﻮّة اﻟﺜﻘﻞ ‪.‬‬ ‫=‬ ‫اﻟﻤﻘﺎرﻧﺔ ‪= 705 :‬‬
‫‪π 1,8 × 10 − 4‬‬
‫‪-3‬‬
‫‪G G‬‬ ‫‪G‬‬
‫‪P + f = ma‬‬ ‫ﺑﺘﻄﺒﻴﻖ اﻟﻘﺎﻧﻮن اﻟﺜﺎﻧﻲ ﻟﻨﻴﻮﺗﻦ ﻓﻲ ﻣﻌﻠﻢ ﻏﺎﻟﻴﻠﻲ ﻣﺮﺗﺒﻂ ﺑﺴﻄﺡ اﻷرض ‪:‬‬ ‫أ(‬
‫‪dv‬‬ ‫‪k‬‬ ‫‪dv‬‬
‫‪ ،‬وﻧﻜﺘﺐ هﺬﻩ اﻟﻤﻌﺎدﻟﺔ ﺑﺎﻟﺸﻜﻞ‬ ‫‪= g − v2 ،‬‬ ‫‪mg − kv 2 = m‬‬ ‫‪،‬‬ ‫ﺑﺈﺳﻘﺎط هﺬﻩ اﻟﻌﻼﻗﺔ ﻋﻠﻰ اﻟﻤﺡﻮر ‪P − f = ma : Oz‬‬
‫‪dt‬‬ ‫‪m‬‬ ‫‪dt‬‬
‫‪dv‬‬
‫‪.‬‬ ‫اﻟﺘﺎﻟﻲ ‪= A − B v 2 :‬‬
‫‪dt‬‬
‫‪mg‬‬ ‫‪k 2‬‬ ‫‪dv‬‬
‫= ‪vl‬‬ ‫‪ ،‬وﻣﻨﻩ‬ ‫‪g−‬‬ ‫‪ ،‬وﺑﺎﻟﺘﺎﻟﻲ ‪vl = 0‬‬ ‫ب( ﻋﻨﺪ ﺑﻠﻮغ اﻟﺴﺮﻋﺔ اﻟﺡﺪّﻳﺔ ﻳﺼﺒﺡ ‪= 0‬‬
‫‪k‬‬ ‫‪m‬‬ ‫‪dt‬‬
‫ﺟـ( ﻣﻦ اﻟﺒﻴﺎن ‪ ) :‬اﻟﻘﻴﻤﺔ اﻟﺼﺡﻴﺡﺔ اﻟﻮﺡﻴﺪة ﻓﻲ هﺬا اﻟﺒﻴﺎن هﻲ اﻟﺴﺮﻋﺔ اﻟﺡﺪّﻳﺔ ﻓﻘﻂ( ‪vl = 25m / s :‬‬

‫‪mg‬‬ ‫‪0,127‬‬ ‫‪mg‬‬


‫=‪k‬‬ ‫‪2‬‬
‫=‬ ‫‪= 2 × 10 − 4 kg . m − 1 ،‬‬ ‫= ‪vl‬‬ ‫ﻗﻴﻤﺔ ﺛﺎﺑﺖ اﻻﺡﺘﻜﺎك ‪:‬‬
‫‪vl‬‬ ‫‪625‬‬ ‫‪k‬‬

‫د( وﺟﻮد اﻟﻬﻮاء ﻳُﻨﻘﺹ ﻣﻦ اﻟﺴﺮﻋﺔ ‪.‬‬


‫ﻓﻲ اﻟﺴﻘﻮط اﻟﺡﺮ وﺻﻠﺖ ﺡﺒﺔ اﻟﺒﺮد إﻟﻰ ﺳﻄﺡ اﻷرض ﺑﺴﺮﻋﺔ ﻗﺪرهﺎ ‪ ، 171,5m / s‬أﻣﺎ ﺑﻮﺟﻮد اﻟﻬﻮاء )اﻟﺴﻘﻮط اﻟﺡﻘﻴﻘﻲ( ﺗﺼﻞ ﺡﺒﺔ‬

‫اﻟﺒﺮد ﺑﺴﺮﻋﺔ أﻗﻞ ‪ ،‬وهﻲ ‪. 25m / s‬‬

‫‪3‬‬
‫ﺗﻌﻘﻴﺐ ‪:‬‬
‫ﻤﻌﻄﻰ ﻓﻲ اﻟﺘﻤﺮﻳﻦ )اﻟﻤﺮﺳﻮم ﻓﻲ اﻟﺸﻜﻞ ‪. ( 1‬‬
‫اﻟﺒﻴﺎن اﻟ ُ‬

‫‪.‬‬ ‫هﺬا اﻟﺒﻴﺎن ﻣﺮﺳﻮم ﻋﻠﻰ أﺳﺎس أن ﻧﻤﻂ اﻟﺴﻘﻮط هﻮ ‪ ، f = kv‬ﻟﻜﻦ ﺑﺎﺳﺘﻌﻤﺎل ﺧﺼﺎﺋﺹ اﻟﻨﻤﻂ ‪f = kv 2‬‬

‫ﻣﺜﻼ ﻟﻮ أردﻧﺎ اﺳﺘﻨﺘﺎج اﻟﺘﺴﺎرع اﻷرﺿﻲ ) ‪ ( g‬ﻣﻦ هﺬا اﻟﺒﻴﺎن ‪ ،‬واﻟﺬي ﻳﻤﺜّﻞ ﺗﺴﺎرع ﺡﺒﺔ اﻟﺒﺮد ﻋﻨﺪ ‪ t = 0‬ﻧﻀﻊ ‪ ، v = 0‬وﺑﺎﻟﺘﺎﻟﻲ ‪:‬‬
‫)‪v (m / s‬‬
‫‪dv‬‬ ‫‪dv‬‬
‫هﻮ ﻣﻴﻞ اﻟﻤﻤﺎس ‪.‬‬ ‫‪ ،‬وﻧﻌﻠﻢ أن‬ ‫‪=g‬‬
‫‪dt‬‬ ‫‪dt‬‬
‫‪25‬‬
‫=‪g‬‬ ‫‪= 16,66 m / s 2‬‬
‫‪1,5‬‬
‫ﻻ ﻧﻌﻠﻢ ﻓﻲ أي آﻮآﺐ ﺳﻘﻂ هﺬا اﻟﺒﺮَد !!!‬

‫اﻟﺸﻜﻞ ‪1 -‬‬

‫‪5‬‬

‫)‪t (s‬‬
‫اﻟﺒﻴﺎن اﻟﺼﺤﻴﺢ ‪) :‬اﻟﺸﻜﻞ – ‪(2‬‬
‫‪1‬‬
‫)‪v (m / s‬‬
‫‪25‬‬
‫=‪g‬‬ ‫‪= 9,8 m / s 2‬‬
‫‪2,55‬‬

‫اﻟﺸﻜﻞ ‪2 -‬‬

‫‪5‬‬

‫)‪t (s‬‬
‫‪1‬‬ ‫‪2,55‬‬
‫اﻟﺘﻤﺮﻳﻦ اﻟﺘﺠﺮﻳﺒﻲ )‪ 4‬ﻧﻘﻂ( ‪-------------------------------------------------------------------------------------------------------------------‬‬
‫‪ – 1‬اﻟﺘﺮآﻴﺐ اﻟﺘﺠﺮﻳﺒﻲ ‪:‬‬

‫ﺳﺡﺎﺡﺔ ﻣﺪرّﺟﺔ‬

‫‪( Na‬‬ ‫‪+‬‬


‫‪, OH −‬‬ ‫)‬

‫ﻣﻘﻴﺎس ‪pH‬‬
‫ﺑﻴﺸﺮ‬

‫‪C6 H 5COOH‬‬
‫ﻗﻀﻴﺐ ﻣﻐﻨﺎﻃﻴﺴﻲ ﻟﺨﻠﻂ اﻟﻤﺰﻳﺞ‬
‫ﻣﺡﺮك ﻟﺘﺪوﻳﺮ اﻟﻘﻀﻴﺐ‬

‫‪4‬‬
‫‪ - 2‬آﻴﻔﻴّﺔ ﻗﻴﺎس ‪ pH‬اﻟﻤﺡﻠﻮل ‪:‬‬

‫‪ -‬ﻧﻤﺴﺡ ﺧﻠﻴﺔ اﻟﻘﻴﺎس ﺑﻮرق ﺟﺎف وﻧﻈﻴﻒ ‪.‬‬


‫‪ -‬ﻧﻐﻤﺮ اﻟﺨﻠﻴﺔ ﻓﻲ ﻣﺡﻠﻮل ﻣﺮﺟﻌﻲ ) ‪ ، ( pH ≈ 7‬وﻧﻨﺘﻈﺮ إﻟﻰ أن ﻳُﺴﺠّﻞ اﻟﻤﻘﻴﺎس اﻟﻘﻴﻤﺔ ‪ ، 7‬ﺛﻢ ﻧﺨﺮﺟﻬﺎ ﻣﻨﻩ ‪.‬‬

‫‪ -‬ﻧﻐﻤﺮ اﻟﺨﻠﻴﺔ ﻓﻲ ﻣﺡﻠﻮل ﻣﻮﻗﻲ ) ‪ ، ( pH ≈ 4‬وﻧﻨﺘﻈﺮ إﻟﻰ أن ﻳُﺴﺠّﻞ اﻟﻤﻘﻴﺎس اﻟﻘﻴﻤﺔ ‪ ، 4‬ﺛﻢ ﻧﺨﺮﺟﻬﺎ ﻣﻨﻩ ‪.‬‬

‫‪ -‬ﻧﻐﻤﺮ اﻟﺨﻠﻴﺔ ﻓﻲ اﻟﺒﻴﺸﺮ ﺑﺡﻴﺚ ُﺗﺜﺒّﺖ ﻓﻲ وﺿﻊ ﻳُﺠﻨّﺒﻬﺎ اﻟﺘﻼﻣﺲ ﻣﻊ اﻟﻘﻀﻴﺐ اﻟﻤﻐﻨﺎﻃﻴﺴﻲ ‪.‬‬
‫ﻧﻔﺘﺡ ﺑﻌﺪ ذﻟﻚ اﻟﺴﺡّﺎﺡﺔ وﻧﻘﺮأ ﻗﻴﻤﺔ اﻟـ ‪ pH‬ﺑﻌﺪ آﻞ إﺿﺎﻓﺔ ‪ ،‬ﺑﺡﻴﺚ ﻻ ﻧُﺨﺮج اﻟﺨﻠﻴﺔ ﻣﻦ اﻟﺒﻴﺸﺮ أﺛﻨﺎء اﻟﻤﻌﺎﻳﺮة ‪.‬‬

‫آﻼم ﺟﻤﻴﻞ ﺟﺪّا وﺗﻤﺮﻳﻦ ﺗﺠﺮﻳﺒﻲ ﺟﺪّا ‪ ....‬ﻣﻦ اﻟﻤﻔﺮوض أن ﻳﻜﻮن اﻟﺘﻼﻣﻴﺬ ﻗﺪ ﻗﺎﻣﻮا ﺑﺎﻟﺨﻄﻮات اﻟﺴﺎﺑﻘﺔ ﻓﻲ اﻟﻤﺨﺒﺮ ﻣﻊ أﺳﺘﺎذهﻢ ‪...‬‬
‫ﺨﺒﺮ اﻟﺪﻳﻮان اﻟﻮﻃﻨﻲ ﻟﻼﻣﺘﺡﺎﻧﺎت آﻢ ﻣﻦ ﺛﺎﻧﻮﻳﺔ ﺗﻤﻠﻚ هﺬا اﻟﺠﻬﺎز اﻟﻐﺮﻳﺐ ﺑﺎﻟﻨﺴﺒﺔ ﻟﻠﺘﻼﻣﻴﺬ ؟‬
‫ﺳﻴﺎدة وزﻳﺮ اﻟﺘﺮﺑﻴﺔ اﻟﻤﻮﻗّﺮ ‪ ..‬أﻟﻢ ُﺗ ْ‬
‫ﻓﻲ اﻧﺘﻈﺎر ﺗﺰوﻳﺪ اﻟﺜﺎﻧﻮﻳﺎت ﺑﻬﺬا اﻟﺠﻬﺎز ‪ ،‬ﻧﻘﺘﺮح أن ﺗُﻘﺒﻞ اﻹﺟﺎﺑﺔ اﻟﺘﺎﻟﻴﺔ ‪:‬‬
‫ﻧﺡﻘّﻖ ﻗﻴﺎس ‪ pH‬اﻟﻤﺡﻠﻮل ﺑﺎﺳﺘﻌﻤﺎل ﻣﻘﻴﺎس ‪ pH‬ﻧﻀﻴﻒ وﻣﻌــﺎﻳَﺮ ‪.‬‬

‫(‬ ‫( )‬
‫‪C 6 H 5COOH + Na + , OH − = C 6 H 5COO − , Na + + H 2O‬‬ ‫)‬ ‫‪ - 3‬ﻣﻌﺎدﻟﺔ ﺗﻔﺎﻋﻞ اﻟﻤﻌﺎﻳﺮة ‪:‬‬
‫‪pH‬‬
‫‪-4‬‬
‫أ( ﻧﻘﻄﺔ اﻟﺘﻜﺎﻓﺆ ‪ :‬ﺑﻄﺮﻳﻘﺔ اﻟﻤﻤﺎﺳﻴﻦ اﻟﻤﺘﻮازﻳﻴﻦ ﻧﺠﺪ ‪:‬‬

‫) ‪E (18, 4mL ; 8, 4‬‬

‫‪8,4‬‬ ‫‪.‬‬ ‫‪[8,3‬‬ ‫ﻧﻘﺘﺮح ﻣﺠﺎﻻ ﻟـ ‪− 8,5] : pH E‬‬

‫ﻋﻨﺪ اﻟﺘﻜﺎﻓﺆ ﻳﻜﻮن ‪ ، C aVa = CbVbE :‬وﻣﻨﻩ‬

‫‪CbVbE 0,1 × 18, 4‬‬


‫‪4,2‬‬ ‫= ‪Ca‬‬ ‫=‬ ‫‪= 9, 2 × 10 − 2 mol / L‬‬
‫‪Va‬‬ ‫‪20‬‬

‫‪1‬‬ ‫ب( ﻗﻴﻤﺔ ‪ pK A‬اﻟﺜﻨﺎﺋﻴﺔ ‪: CH 3COOH / CH 3COO −‬‬


‫) ‪Vb ( mL‬‬
‫‪5‬‬ ‫‪9,2‬‬ ‫‪18,4‬‬ ‫⎤ ‪⎡CH 3COO −‬‬
‫⎣‬ ‫ﻋﻨﺪ ﻧﻘﻄﺔ ﻧﺼﻒ اﻟﺘﻜﺎﻓﺆ ﻳﻜﻮن ‪⎦ = 1‬‬
‫] ‪[CH 3COOH‬‬
‫⎤ ‪⎡CH 3COO −‬‬
‫‪ ،‬وﺑﺎﻟﺘﺎﻟﻲ ‪pK A = 4, 2‬‬ ‫⎣ ‪pK A = pH − Log‬‬ ‫وﻟﺪﻳﻨﺎ ‪⎦ = pH − 0 = pH‬‬
‫] ‪[CH 3COOH‬‬
‫ﻳﻜﻮن ‪ ، pH = 2,6‬وهﻮ ‪ pH‬ﻣﺡﻠﻮل ﺡﻤﺾ اﻟﺒﻨﺰوﻳﻚ ‪.‬‬ ‫)‪(Vb = 0‬‬ ‫ﺟـ( ﻗﺒﻞ ﺑﺪء اﻟﻤﻌﺎﻳﺮة‬

‫‪ ،‬ﺑﻤﺎ أن ‪ ، ⎡ H 3O + ⎤ < C a‬إذن ﺡﻤﺾ اﻟﺒﻨﺰوﻳﻚ ﺿﻌﻴﻒ ‪.‬‬ ‫‪⎡ H 3O + ⎤ = 10 − 2,6 = 2,51 × 10 − 3 mol / L‬‬
‫⎣‬ ‫⎦‬ ‫⎣‬ ‫⎦‬
‫أو ‪ :‬ﻧﻨﺸﻲء ﺟﺪول اﻟﺘﻘﺪم ﻟﺘﻔﺎﻋﻞ ﺡﻤﺾ اﻹﻳﺜﺎﻧﻮﻳﻚ ﻣﻊ اﻟﻤﺎء وﻧﺡﺴﺐ اﻟﻨﺴﺒﺔ اﻟﻨﻬﺎﺋﻴﺔ ﻟﻠﺘﻘﺪّم ‪:‬‬

‫‪ ، τ‬إذن ﺡﻤﺾ اﻟﺒﻨﺰوﻳﻚ ﺿﻌﻴﻒ ‪.‬‬ ‫= ‪ . τ f‬ﺑﻤﺎ أن ‪< 1‬‬


‫‪xf‬‬
‫=‬
‫(‬
‫‪n H 3O +‬‬ ‫‪) = ⎡⎣ H O‬‬ ‫‪3‬‬
‫⎤‪+‬‬
‫⎦‬
‫‪V‬‬ ‫⎤ ‪⎡ H 3O +‬‬
‫⎣=‬ ‫‪⎦ = 10‬‬
‫‪− 2,6‬‬
‫‪= 0,027‬‬
‫‪f‬‬
‫‪xm‬‬ ‫‪Ca V‬‬ ‫‪Ca V‬‬ ‫‪Ca‬‬ ‫‪0,092‬‬
‫ﺗﻌﻘﻴﺐ ‪:‬‬
‫ﻟﻘﺪ ﺗﻨﺒّﻩ اﻷﺳﺘﺎذ أﺡﻤﺪ ﻣﻜﻨﺎﺳﻲ ﻣﻦ اﻟﺸﻠﻒ إﻟﻰ ﻣﺸﻜﻞ ﻋﻠﻤﻲ ﻓﻲ هﺬا اﻟﺘﻤﺮﻳﻦ ‪ ،‬وراﺳﻠﻨﻲ ﻓﻲ ذﻟﻚ ‪ ،‬وهﺎهﻮ ﺗﻌﻘﻴﺒﻲ ‪:‬‬
‫اﻟﺘﺮآﻴﺰ اﻟﻤﻮﻟﻲ ﻟﺡﻤﺾ اﻟﺒﻨﺰوﻳﻚ هﻮ ‪ ، C a = 0,092mol / L‬وهﺬا ﻳﻮاﻓﻖ ﻃﺒﻌﺎ ﺗﺮآﻴﺰا آﺘﻠﻴﺎ ﻗﺪرﻩ ‪:‬‬

‫‪C m = C a × M = 0,092 × 122 = 11, 22 g / L‬‬


‫ﺡﻤﺾ اﻟﺒﻨﺰوﻳﻚ ﻋﺒﺎرة ﻋﻦ ﻣﺮآﺐ ﺻﻠﺐ أﺑﻴﺾ ‪ ،‬وﻟﻤﺎ ﻳﺘﺡﻠّﻞ ﻓﻲ اﻟﻤﺎء ﻳﻌﻄﻴﻨﺎ ﻣﺡﻠﻮﻻ ﻣﺎﺋﻴﺎ ﻟﺡﻤﺾ اﻟﺒﻨﺰوﻳﻚ ‪ ،‬ﻟﻜﻦ ﻓﻲ اﻟﺪرﺟﺔ ‪25 °C‬‬
‫ﻻ ﻳُﻤﻜﻦ أن ﻧﺡﻠﻞ ﻓﻲ ‪ 1L‬ﻣﻦ اﻟﻤﺎء أآﺜﺮ ﻣﻦ ‪ 3g‬ﻣﻦ ﺡﻤﺾ اﻟﺒﻨﺰوﻳﻚ ‪.‬‬

‫إذا ﻗﻠﺖَ ﻟﻲ أن درﺟﺔ اﻟﺡﺮارة ﻏﻴﺮ واردة ﻓﻲ اﻟﺘﻤﺮﻳﻦ ‪ ،‬أﻗﻮل ﻟﻚ إﻧﻬﺎ ‪ 25 °C‬ﻷﻧﻨﺎ وﺟﺪﻧﺎ ‪ ، PK A = 4, 2‬وهﺬﻩ اﻟﻘﻴﻤﺔ ﺗﻜﻮن ﻓﻲ ‪25 °C‬‬
‫ﺷﻜﺮا ﻟﻚ أﺡﻤﺪ‬ ‫إذن اﻹﺳﻢ ‪ ):‬ﺗﻤﺮﻳﻦ ﺗﺠﺮﻳﺒﻲ ( ‪ ...‬ﻻ أﻇﻦ ذﻟﻚ !‬ ‫ﻓﻘﻂ ‪.‬‬
‫‪5‬‬
ƨȈƦǠnjdzơ ƨȈǗơǂǬŻƾdzơ ƨȇǂƟơDŽŪơ ƨȇǁȂȀǸŪơ
ƩƢǬƥƢLjŭơȁ ƩƢǻƢƸƬǷȐdz řǗȂdzơ ǹơȂȇƾdzơ ƨȈǼǗȂdzơ ƨȈƥǂƬdzơ Ƨǁơǃȁ
2012 ƇŒƍŞ :Ƨǁȁƽ ȅȂǻƢưdzơ ǶȈǴǠƬdzơ ƢȇǁȂdzƢǰƥ ǹƢƸƬǷơ
ŗƒŕƒŧŞř ƅƍƄŵ :ƨƦǠnjdzơ
ŻŮƈƍ ŘœŵœŪ 03 :Ƨƾŭơ ŗƒőœƒŨƒŽƃŒ ƅƍƄŶƃŒ :ƧƽƢǷ Ŀ ǁƢƦƬƻơ
:ƇƒƒƃœřƃŒ ƇƒŵƍŰƍƆƃŒ ťšŊ ŧœřŤƒ ƇŊ ŠŬŧřƆƃŒ ƏƄŵ
¾ƍƕŒ ųƍŰƍƆƃŒ
( űœƀƈ 04 ) :¾ƍƕŒ ƇƒŧƆřƃŒ
ŚŕƈƏũƄƔŗ ¿ƏƆţƈƏ H 2C 2O 4 (aq ) ƃƔƅŕŬƄƏƗŔ űƈţ ¿ƏƆţƈ ƉƔŗ ŜŧŕţƅŔ ¿ŷŕſśƅŔ ũƏųś řŬŔũŧƅ
ƑƆŷ ƒƏśţƔ ŕƔƆŷŕſś ŕŠƔŪƈ ŕƊũČŲţ žƉƈŪƅŔ řƅƛŧŗ ( 2K  (aq )  Cr2O 72 (aq ) ) ƇƏƔŬŕśƏŗƅŔ
c1 3, 0 u10-2 mol ˜ L-1 ƓƅƏƈƅŔ ƋŪƔƄũś ƒŨƅŔ ƃƔƅŕŬƄƏƗŔ űƈţ ¿ƏƆţƈ Ɖƈ V 1 100 mL Ƈ»Šţ
c 2 0,8 u10-2 mol ˜ L-1 ƓƅƏƈƅŔ ƋŪƔƄũś ƒŨƅŔ ƇƏƔŬŕśƏŗƅŔ ŚŕƈƏũƄƔŗ ¿ƏƆţƈ Ɖƈ V 2 100 mL ƇŠţƏ
ŧũŔƏŮ ŘũƔŕŸƈ ¿ƜŦ Ɖƈ ƓƆŷŕſśƅŔ şƔŪƈƅŔ ũƏųś ŶŗŕśƊ .ŪƄũƈƅŔ ŚƔũŗƄƅŔ űƈţ Ɖƈ ŚŔũųƁ ŶŲŗƏ
ũƏųś ¿ŝƈƔ ƒŨƅŔ (1-¿ƄŮƅŔ) ƓƊŕƔŗƅŔ ƑƊţƊƈƅŔ ƑƆŷ ¿ŰţƊž ƉƈŪƅŔ řƅƛŧŗ řƆƄŮśƈƅŔ Cr 3 (aq ) ƇƏũƄƅŔ
. t ƉƈŪƅŔ řƅƛŧŗ [Cr 3 (aq )] ƇƏũƄƅŔ ŧũŔƏŮƅ ƓƅƏƈƅŔ ŪƔƄũśƅŔ
  ƌƁŔũżśŬŔ Řŧƈ ŜƔţ Ɖƈ ¿ŷŕſśƅŔ ŔŨƍ ŽƊŰƊ ŽƔƄ -1
.¿ŷŕſśƅŔ ŔŨƎƅ ŪƔƈƈƅŔ ƇŧƂśƅŔ ¿ƏŧŠ ¿ƈƄŌ ƓƊŕƔŗƅŔ ƑƊţƊƈƅŔ Ə ŚŕƔųŸƈƅŔ ƑƆŷ ŔŧŕƈśŷŔ -2
:(řŗŕŠƙŔ řƁũƏ ƑƆŷ ƓśƕŔ ¿ƏŧŠƅŔ ¿ƂƊŔ)
3H 2C 2O 4 (aq ) + Cr2O 72 (aq ) + 8H  (aq ) = 2Cr 3 (aq ) + 6CO 2 (aq ) + 7H 2O (A)
řƅŕţƅŔ ( mmol ) ŘŧŕƈƅŔ řƔƈƄ
řƔœŔŧśŗƛŔ ŘũžƏŗ ŘũžƏŗ
řƔƅŕƂśƊƛŔ ŘũžƏŗ ŘũžƏŗ
řƔœŕƎƊƅŔ ŘũžƏŗ ŘũžƏŗ
[Cr 3 (aq )](mmol ˜ L1 )   ŔŨŕƈƅ   Ƈŕś ũƔŻ ƇŌ Ƈŕś ¿ŷŕſśƅŔ ¿ƍ
.ŕƔƊŕƔŗ ƌśƈƔƁ ũĐŧƁ Ƈŝ ž t 1 ¿ŷŕſśƅŔ ŽŰƊ ƉƈŪ ŽĐũŷ -3
2

ŕƎƊŷ ũŗĐŷ Ƈŝ ž¿ŷŕſśƆƅ v řƔƈŠţƅŔ řŷũŬƅŔ ŽũĐ ŷ -Ō -4


1-¾ƂŬƃŒ
. [Cr 3 (aq )] ƇƏũƄƅŔ ŧũŔƏŮƅ ƓƅƏƈƅŔ ŪƔƄũśƅŔ řƅƛŧŗ
.t=8sƏ t=0 ƉƔśŴţƆƅŔ Ɠž řƔƈŠţƅŔ řŷũŬƅŔ ŖŬţŔ -Ŗ
2 řŷũĉŬƅŔ ƋŨƍ ůƁŕƊś ƒũƎŠƈƅŔ ƐƏśŬƈƅŔ ƑƆŷ ũŬ
Đ ž -Š
t (s) .ƉƈŪƅŔ ũƏũƈ Ŷƈ
0
4

8 ǺǷ 1 ƨƸǨǏ
( űœƀƈ 04 ) :ƑƈœśƃŒ ƇƒŧƆřƃŒ

:řƔśƕŔ ŚŕƈƏƆŸƈƅŔ ŶŮƈ ŶŗƊƈƅ řƂžũƈƅŔ řƔƊƂśƅŔ řƁŕųŗƅŔ Ɖƈ ŕƊŌũƁ žʼnŕƔŪƔſƅŔ ũŗŦƈŗ 2012/04/01 ƇƏƔ Ɠž
J Ə E : ŚŕŷŕŸŮƙŔ - 137
Cs
55 : 137 ƇƏƔŪƔŬƅŔ -
. m0 5, 02 u 102 g : řƔœŔŧśŗƛŔ řƆśƄƅŔ - t 1 2 30,15 ans :ũƈŸƅŔ ŽŰƊ -
.řƁŕųŗƅŔ ƋŨƍ Ɖŷ ŕŗœŕŻ ŶŗƊƈƅŔ ŶƊŰ ťƔũŕś ŕƊŴţƛ ŕƈƊƔŗ
. A 14,97 u1010 Bq ŧŠƊž ŶŗƊƈƆƅ A ųŕŮƊƅŔ Geiger ŧŔŧŷ ¿ŕƈŸśŬŕŗ ūƔƂƊ ŶŗƊƈƅŔ ŔŨƍ ũƈŷ ŧŕŠƔƙ
.J Ə E ƉƔŷŕŸŮƙŔ ŽũĐ ŷ Ƈŝ žƇƏƔŪƔŬƅŔ ŘŔƏƊ ƃƄſś řƅŧŕŸƈ ŖśƄŔ -1
.ƌŸƊŰ
ą řŴţƅ ŶŗƊƈƅŕŗ ŘŧƏŠƏƈ ŚƊŕƄ ƓśƅŔ ƇƏƔŪƔŬƅŔ řƔƏƊƗ N0 ƓœŔŧśŗƛŔ ŧŧŸƅŔ ŖŬţŔ -2
. s 1 »ŗ O ƓŷŕŸŮƙŔ ųŕŮƊƅŔ Śŗŕŝ ŖŬţŔ -3
A 0 ųŕŮƊƅŔ ŖŬţŔ Ƈŝ žŶŗƊƈƅŔ Ɠž řƔƂŗśƈƅŔ řƔƏƊƗŔ ŧŧŸŗ A ųŕŮƊƅŔ ųŗũś ƓśƅŔ řƔžũţƅŔ ŘũŕŗŸƅŔ ŖśƄŔ -4
.ŕƎŸƊŰ
ą řŴţƅ řƊƔŸƆƅ ŪƔƈƈƅŔ
.řƊƔĐ ŸƅŔ ŶƊŰ
ą ťƔũŕś ŖŕŬţƅŕŗ şśƊśŬŔ -5
365,5 jours : řƊŬƅŔ ƇŕƔŌ ŧŧŷ ž N A 6, 02 u 10 23 mol 1 :ƏũŧŕŻƏžŌ Śŗŕŝ :ŘœƒűŶƆƃŒ
. 56 Ba ž 55 Cs ž 54 Xe ž 53 I : ƒũƏŧƅŔ ¿ƏŧŠƅŔ Ɖƈ

( űœƀƈ 04 ) :ŚƃœśƃŒ ƇƒŧƆřƃŒ


. 25 qC Ɠž ¿ƔƅŕţƈƅŔ ¿Ƅ ŨŦŎś
.ʼnŕƈƅŔ Ɠž C 6 H 5COOH ƓƂƊƅŔ ƃƔƏŪƊŗƅŔ űƈţ Ɖƈ m řƆśƄ ¿ţŗ 500 mL ƌƈŠţ S ƛƏƆţƈ ũŲţƊ
.ʼnŕƈƅŔ Ɠž ƃƔƏŪƊŗƅŔ űƈţ ¿ƜţƊŔ řƅŧŕŸƈ ŖśƄŔ -1
.űƈţ/ūŕŬŌ řƔœŕƊŝƆƅ Ka řŲƏƈţƅŔ Śŗŕŝ Řũŕŗŷ ųŷŌ -2
ƇƏƔŧƏŰƅŔ ŧƔŬƄƏũŧƔƍ ¿ƏƆţƈŗ ƃƔƏŪƊŗƅŔ űƈţ ¿ƏƆţƈ Ɖƈ V a 20 mL ŕƈŠţ ũƔŕŸƊ -3
ƓųŸƔ (2-¿ƄŮƅŔ ) ƓƊŕƔŗƅŔ ƑƊţƊƈƅŔ . cb 0, 2 mol ˜ L 1 ƓƅƏƈƅŔ ƋŪƔƄũś (Na  (aq )  HO  (aq ))

.V b ŽŕŲƈƅŔ ūŕŬƗŔ ƇŠţ řƅƛŧŗ şƔŪƈƅŔ pH ũƏųś


.ŘũƔŕŸƈƅŔ ¿ŷŕſś řƅŧŕŸƈ ŖśƄŔ -Ō
 ƓœŕƔƈƔƄƅŔ ŕƈƎƅƏƅŧƈ ŕƈ .(2-¿ƄŮƅŔ ) Ɖƈ Ec Ə E ƉƔśƔųƂƊƅŔ ŚŕƔŝŔŧţŏ ƉƔŷ -Ŗ
.ƃƔƏŪƊŗƅŔ űƈţƅ ca ƓƅƏƈƅŔ ŪƔƄũśƅŔ ŧŃŠ -Š
. S ¿ƏƆţƈƅŔ ũƔŲţśƅ řƆƈŸśŬƈƅŔ ƓƂƊƅŔ ƃƔƏŪƊŗƅŔ űƈţƅ m řƆśƄƅŔ ŖŬţŔ -ŧ

8 ǺǷ 2 ƨƸǨǏ
C 6 H 5COOH (aq ) / C 6 H 5COO  (aq ) řƔœŕƊŝƆƅ Ka řƈƔƁ ŧŃŠ -ƍ
  pH 6, 0 ŧƊŷ ƓƆŷŕſśƅŔ şƔŪƈƅŔ Ɠž řŗƅŕżƅŔ řſŰƅŔ ¿ƄŮƔ ƒŨƅŔ ƓœŕƔƈƔƄƅŔ ŵƏƊƅŔ ŕƈ -Ə

pH
12
10
8 E

6 2-¿ƄŮƅŔ
4
E'
2
Vb(mL)
0
0 2 4 6 8 10 12 14 16 18 20 22 24 26

M (C ) 12 g ˜ mol 1 ž M (H ) 1 g ˜ mol 1 ž M (O ) 16 g ˜ mol 1 :ƏűŶř

( űœƀƈ 04 ) :ŴŕŒŧƃŒ ƇƒŧƆřƃŒ


.ʼnŔƏƎƅŔ Ɠž řƔũƄ ųƏƂŬ řƄũţ ŕƔƆƔƅ௠ƋũŗśŸƊ ƓŲũŌ ƓţųŬ ŶŠũƈ Ɠž ūũŧƊ
. t ƉƈŪƅŔ řƅƛŧŗ v řƔũƄƅŔ řƅŕųŷ ŪƄũƈ řŷũŬ ũƏųś ¿ŝŅƈƔą ( 3-¿ƄŮƅŔ )
: ƉŕƔŗƅŔ Ɖƈ -1
v ( m ˜ s 1 ) .řƄũţƅŔ ƓƈŕŴƊƅ ƓƊƈŪƅŔ ¿ŕŠƈƅŔ ŧŧĐ ţ -Ō
v ( m ˜ s 1 )
.v A řƔŧţƅŔ řŷũŬƅŔ řƈƔƁ ƉƔĐŷ -Ŗ
řƅŕųŷ ŪƄũƈ ŵũŕŬś a0 ŖŬţŔ -Š
.t 0 řŴţƆƅŔ Ɠž řƔũƄƅŔ
 şśƊśŬś ŔŨŕƈ
44
t (st ()s ) ¿ƏŰƏ řŴţƅ ŵũŕŬśƅŔ řƈƔƁ Ɠƍ ŕƈ -ŧ
0 11
 űũƗŔ ŢųŬ Ƒƅŏ řƔũƄƅŔ
0 1-ϞϜθϟ΍
3-¾ƂŬƃŒ

 t 3s řŴţƆƅŔ Ɠž řƔũƄƆƅ řƔƄũţƅŔ řƁŕųƅŔ řƈƔƁ ƉƏƄś ƇƄ -ƍ


.ŹŔũſƅŔ Ɠž řƔũƄƅŔ řƅŕųŷ ŪƄũƈƅ ƓƅƏƁŕŮƅŔ ųƏƂŬƅŔ řƄũţƅ v (t ) řŷũŬƅŔ ųųŦƈ ŕƔſƔƄ ¿ŝƈ -2
m 30 g řƔũƄƅŔ řƆśƄ ž g 9 ,80 m ˜ s 2 :ƏűŶř

8 ǺǷ 3 ƨƸǨǏ
( űœƀƈ 04 ) :ƑŕƒŧŞřƃŒ ƇƒŧƆřƃŒ
R ƉƔũŔŧƂƈƅŔ ũƔŝōśƏ žƉƈŪƅŔ řƅƛŧŗ RL ŖųƂƅŔ
ƓœŕƊŝ Ɠž ũŕƈƅŔ i (t ) ƓœŕŗũƎƄƅŔ ũŕƔśƅŔ ŘŧŮ ũƏųś řŬŔũŧƅ
.(4-¿ƄŮƅŔ) řƔœŕŗũƎƄƅŔ ŘũŔŧƅŔ ŖƄũƊ žũƏųśƅŔ ŔŨƍ ƑƆŷ L Ə
.ŘũƄŔŨ ƒŨ ƓųŗƎƈ ŪŔŪśƍŔ ƇŬŔũ ¿ŕƈŸśŬŕŗ R ƓƈƏƗŔ ¿ƁŕƊƅŔ Ɠžũų ƉƔŗ u R ƓœŕŗũƎƄƅŔ ũśƏśƅŔ ũƏųś ŶŗŕśƊ -1
.ƓųŗƎƈƅŔ ŪŔŪśƍƛŔ ƇŬŔũ ųŗũ řƔſƔƄ ŕƎƔƆŷ ƉƔĐŗ Ƈŝ řŗŕŠƙŔ řƁũƏ ƑƆŷ ŘũŔŧƅŔ ƇŬũ ŧŷŌ -Ŋ
ƓœŕŗũƎƄƅŔ ũśƏśƅŔ ũƏųś řŸŗŕśƈ -Ŗ
K ũƏųś řŸŗŕśƈ Ɖƈ ŕƊśƊƄƈ u R (t )
: Ϧϣ Γέ΍Ϊϟ΍ ϥϮϜΘΗ
L,r ϞϗΎϨϟ΍ Ϧϣ ϥϮϜϤϟ΍ ΐτϘϟ΍ ϲ΋ΎϨΛ - ũŕƈƅŔ ƓœŕŗũƎƄƅŔ ũŕƔśƆƅ i (t ) ŘŧŮƅŔ
.( L , r ) ΔόϴηϮϟ΍ϭ R ϲϣϭϷ΍
E
ϪΗϮϗ ήϤΘδϤϟ΍ ήΗϮΘϠϟ ΪϟϮϣ - .ŘũŔŧƅŔ Ɠž
R E Δϴ΋ΎΑήϬϜϟ΍ ΔϛήΤϤϟ΍ .ƃƅŨ ũĐŬž

4-¾ƂŬƃŒ

:řŸųŕƂƅŔ ƀƆżƊ -2
.ŘũŔŧƅŔ Ɠž ũŕƈƅŔ i (t ) ƓœŕŗũƎƄƅŔ ũŕƔśƅŔ ŘŧŮƅ řƔƆŲŕſśƅŔ řƅŧŕŸƈƅŔ ŧŃŠ -Ō
t
.W Ə A Ɠśũŕŗŷ ŧŃŠ i (t ) A (1  e W ) :¿ƄŮƅŔ Ɖƈ řƅŧŕŸƈƅŔ ƋŨƍ ¿ţ ƉŌ ŕƈƆŷ -Ŗ
  ƉƜŝƈƔ ŔŨŕƈ
řŸƔŮƏ ¿ŕƈŸśŬŕŗ řſƆśŦƈ Ŗũ੶ ŜƜŝ ŪŠƊƊ -3
i (mA )
¿ƁŔƏƊƏ ũƔƔżśƆƅ řƆŗŕƁ L ŕƎśƔśŔŨƏ ŕŗƔũƂś řśŗŕŝ r ŕƎśƈƏŕƂƈ
30 1
řƔ»ƊŕƔŗƅŔ ŚŕƔƊţƊƈƅŔ (5-¿Ƅ»ŮƅŔ) ƉƔĐŗƔ .ř»ſƆśŦƈ řƔƈƏŌ
2 t ƉƈŪƅŔ řƅƛŧŗ i (t ) ƓœŕŗũƎƄƅŔ ũŕƔśƅŔ ŘŧŮ ũƏųśƅ
3 L ƇƔƁ ƀžũƈƅŔ ¿ƏŧŠƅŔ ¿ŝƈƔƏ ŜƜŝƅŔ Ŗũ੶Ɔƅ řŗŬƊƅŕŗ
:řŗũŠś ¿Ƅ Ɠž řƆƈŸśŬƈƅŔ R Ə
t (ms )
0 0,15
1 ŗŕŧŞřƃŒ 2 ŗŕŧŞřƃŒ 3 ŗŕŧŞřƃŒ
5-¾ƂŬƃŒ L (mH) 30 20 40
R (:) 290 190 190

.ƃƅŨ ¿ƆIJŷ .ŕƎƅ ƀžŔƏƈƅŔ ƓƊŕƔŗƅŔ ƑƊţƊƈƅŕŗ řŗũŠś ¿Ƅ ŖŬƊŌ -Ō


. r řƈƏŕƂƈƅŔ řƈƔƁ ŧŃŠ -Ŗ

8 ǺǷ 4 ƨƸǨǏ
ƑƈœśƃŒ ųƍŰƍƆƃŒ
( űœƀƈ 04 ) :¾ƍƕŒ ƇƒŧƆřƃŒ
. 25 qC Ɠž ¿ƔƅŕţƈƅŔ ¿Ƅ ŨŦŎś
c1 1 ,0 u10 2 mol ˜ L 1 ƓƅƏƈƅŔ ƋŪƔƄũś CH 3  COOH ƃƔƏƊŕŝƔƙŔ űƈţƅ S 1 ƛƏƆţƈ ŕƊũŲţ -1
. pH 3 ,4 ƌƅƏ

.ʼnŕƈƅŔ Ŷƈ ƃƔƏƊŕŝƔƙŔ űƈţ ¿ŷŕſś řƅŧŕŸƈ ŖśƄŔ -Ō


.ƓœŕƔƈƔƄƅŔ ¿ŷŕſśƅŔ ƇŧƂśƅ ƛƏŧŠ ŒŮƊŌ -Ŗ
.ʼnŕƈƅŔ Ŷƈ ŕƔƆƄ ¿ŷŕſśƔ ƛ CH 3  COOH ƉŌ ƉƔĐŗ -Š
:řƁƜŸƅŕŗ ƑųŸƔ ¿ŷŕſśƆƅ ƉŪŔƏśƅŔ Śŗŕŝ K 1 ƉŌ ŚŗŝŌ -ŧ
W 12f
.¿ŷŕſśƆƅ ƓœŕƎƊƅŔ ƇŧƂśƅŔ řŗŬƊ W1f :ŜƔţ žƌśƈƔƁ ŖŬţŔ Ƈŝ ž K 1 c1
1  W1 f
 ¿ƏƆţƈƅŔ Ɠž řŗƅŕżƅŔ řſŰƅŔ ¿ƄŮƔ ƒŨƅŔ ƓœŕƔƈƔƄƅŔ ŵƏƊƅŔ ŕƈ -ƍ
.c2 1 ,0 u10 1 mol ˜ L 1 ƓƅƏƈƅŔ ƋŪƔƄũś ƃƔƏƊŕŝƔƙŔ űƈţƅ S 2 ƛƏƆţƈ ŕƊũŲţ řƔƊŕŝ řŗũŠś Ɠž -2
. V 5 ,0 u10 2 mS ˜ m 1 ƌƅ řƔŷƏƊƅŔ řƔƆƁŕƊƅŔ
.¿ƏƆţƈƅŔ Ɠž ŘŧŠŔƏśƈƅŔ řƔŧũŕŮƅŔ ŵŔƏƊƘƅ řƔƅƏƈƅŔ ŪƔƄŔũśƅŔ ŖŬţŔ -Ō
. K 2 Ə W 2 f ŖŬţŔ -Ŗ
 ƓœŕƎƊƅŔ ƇŧƂśƅŔ řŗŬƊ ƑƆŷ řƔœŔŧśŗƛŔ řƔƅƏƈƅŔ ŪƔƄŔũśƅŔ ũƔŝōś ŕƈ -Ō -3
 řƔœŔŧśŗƛŔ řƔƅƏƈƅŔ ŪƔƄŔũśƅŕŗ K ƉŪŔƏśƅŔ Śŗŕŝ ƀƆŸśƔ ¿ƍ -Ŗ
OH
3O
 35 ,9 mS ˜ m 2 ˜ mol 1 ; OCH
3 COO
 4 ,1 mS ˜ m 2 ˜ mol 1 :ƏűŶƒ

( űœƀƈ 04 ) :ƑƈœśƃŒ ƇƒŧƆřƃŒ


.řƔƁũŧƅŔ ŘŧżƅŔ ƉŕųũŬ řŠƅŕŸƈ Ɠž ŕŬŕŬŌ 131
53 I ŧƏƔƅŔ ƇŧŦśŬƔ
53 I ŧƏƔƅŔ ŘŔƏƊ ŖƔƄũś ųŷŌ -1
. 131
. 131
53 I ŧƏƔƅŔ ŘŔƏƊƅ ųŗũƅŔ řƁŕų EA ŖŬţŔ -2
. E  ũŧŰƔ 131 ŧƏƔƅŔ Ɖŏ -3
Ɖƈ ŘŧţŔƏ ƉƏƄś A
Z X řŠśŕƊƅŔ ŚƊŗƅŔ ŘŔƏƊ ƉŌ ŕƈƆŷ ž131 ŧƏƔƅŔ ŘŔƏƊƅ řƆŰŕţƅŔ ƃƄſśƅŔ řƅŧŕŸƈ ŖśƄŔ
127
51 Sb ; 131
52Te ; 132
53 I ; 131
54 X e :řƔƅŕśƅŔ řƔƏƊƗŔ

8 ǺǷ 5 ƨƸǨǏ
ln
lnNN . m0 0 ,696 g ŕƎśƆśƄ 131 ŧƏƔƅŔ Ɖƈ řƊƔŷ -4
.ƓŷŕŸŮƙŔ ůƁŕƊśƅŔ ƉƏƊŕƁ ŖśƄŔ -Ō
řƅƛŧŗ ln N ũƏųś ƑƊţƊƈ (1-¿ƄŮƅŔ) ¿ŝƈƔ -Ŗ
ƃƄſśƅŔ Śŗŕŝ O řƈƔƁ ƌƊƈ şśƊśŬŔ . t ƉƈŪƅŔ
5 .131 ŧƏƔƆƅ ũƈŸƅŔ ŽŰƊ t 1 Ə
2
t ( tjour
( jour))
0 20   16 jours ŧŸŗ řƄƄſśƈƅŔ 131 ŧƏƔƅŔ řƆśƄ ŕƈ -Š
1-¾ƂŬƃŒ
:ŘœƒűŶƆƃŒ
1 131
m( H ) 1 ,00728 u ; m(
1 53 I ) 130 ,97851u ; m( n ) 1 ,00866 u ; 1u 931 ,5 MeV / c 2

( űœƀƈ 04 ) :ŚƃœśƃŒ ƇƒŧƆřƃŒ


Y1 Y2 :Ɖƈ (2-¿ƄŮƅŔ) řƔœŕŗũƎƄ ŘũŔŧ ƉƏƄśś
RR L ,r . E řƔœŕŗũƎƄƅŔ řƄũţƈƅŔ ƌśƏƁ ƓœŕŗũƎƄƅŔ ũśƏśƆƅ ŧƅƏƈ -
K .R 100 : ƌśƈƏŕƂƈ ƓƈƏŌ ¿ƁŕƊ -
. r ŕƎśƈƏŕƂƈƏ L ŕƎśƔśŔŨ řŸƔŮƏ -
2-¾ƂŬƃŒ
1-ϞϜθϟ΍ . K řŸųŕƁ -
K řŸųŕƂƅŔ ƀƆżƊ t 0 řŴţƆƅŔ Ɠž ž(2-¿ƄŮƅŔ) ŘũƄŔŨ ƒŨ ƓųŗƎƈƅŔ ŪŔŪśƍƛŔ ƇŬŔũ ƓƆŦŧƈ ¿ŰƏƊ
.(3-¿ƄŮƅŔ) (2) Ə (1) ƉƔƔƊŕƔŗƅŔ ƉƔƔƊţƊƈƅŔ řŮŕŮƅŔ ƑƆŷ ŧƍŕŮƊž
u (V )
u(V )
.¿ƆIJŷ .ƌƅ ƀžŔƏƈƅŔ ƓƊŕƔŗƅŔ ƑƊţƊƈƅŔ ¿Ŧŧƈ ¿Ƅƅ ŧĐŧţ -Ō-1
1
ŧŃŠ řƔœŕŗũƎƄƅŔ ŚŔũśƏśƅŔ ŶƈŠ ƉƏƊŕƁ ƀƔŗųśŗ -Ŗ
2 . i (t ) ƓœŕŗũƎƄƅŔ ũŕƔśƅŔ ŘŧŮƅ řƔƆŲŕſśƅŔ řƅŧŕŸƈƅŔ
  E ƓœŕŗũƎƄƅŔ ũśƏśƅŔ řƈƔƁ ŕƈ -Ō-2
2
t (t (ms )
. I 0 ƓƈŴŷƗŔ ƓœŕŗũƎƄƅŔ ũŕƔśƅŔ ŘŧŮ řƈƔƁ ŧŃŠ -Ŗ
ms )
0 5 3-¾ƂŬƃŒ .řŸƔŮƏƅŔ řƈƏŕƂƈ r řƈƔƁ ŖŬţŔ -Š
.ƉƈŪƅŔ Ŷƈ ūƊ੶ƈ ƌƊŌ ƒŧŸŗąƅŔ ¿ƔƆţśƅŕŗ ƉƔĐŗƏ .ƉƈŪƅŔ Śŗŕŝ W řƈƔƁ ŕƔƊŕƔŗ ŧŃŠ -Ō-3
.řŸƔŮƏƅŔ řƔśŔŨ L ŖŬţŔ -Ŗ
.řŸƔŮƏƅŔ Ɠž řƊŪŦƈƅŔ řƔƈŴŷƗŔ řƁŕųƅŔ ŖŬţŔ -4

8 ǺǷ 6 ƨƸǨǏ
( űœƀƈ 04 ) :ŴŕŒŧƃŒ ƇƒŧƆřƃŒ
řŠƔśƊƅŔ řŬžŕƊƈƅŔ ƋŨƎŗ Ūŕž ƒŨƅŔ ƓŲŕƔũƅŔ ƀƂţ žƉƔƄŗŗ řƔŗƈƅƏƗŔ ŖŕŸƅƗŔ Ɠž řƆŠƅŔ Ɠƈũ řŬžŕƊƈ ¿ƜŦ
.d 21,51 m

z 4-¾ƂŬƃŒ
¿ŠƗƏ ƓƈũƅŔ řƔƆƈŸƅ ¿ŠŬƈƅŔ ƇƆƔſƅŔ ƑƆŷ ŔŧŕƈśŷŔ
v0 ƇČ śĿ žřƆŠƅŔ ŕƎŗ ŚžŨƁ ƓśƅŔ v 0 řŷũŬƅŔ řƈƔƁ řžũŸƈ
D :ƓƈũƅŔ řŴţƅ ʼnŕƊŝŌ ŚŕƔųŸƈƅŔ űŸŗ ŞŔũŦśŬŔ
A
hA 2, 00 m ŵŕſśũŔ ƑƆŷ řŸƁŔƏƅŔ A řųƂƊƅŔ Ɖƈ řƆŠƅŔ ŚžĊŨƁŁ
G
k
C
řƔƏŔŪƅŔ ŶƊŰś ƓśƅŔ v 0 řŷũŬƅŕŗƏ űũƗŔ ŢųŬƅ řŗŬƊƅŕŗ
O i
x .(4-¿ƄŮƅŔ) ƓƂžƗŔ ųŦƅŔ Ŷƈ D 45q
d xC 21.51 m
ūƊ੶ƈƅŔƏ ŧƈŕŸśƈƅŔ ƇƆŸƈƅŔ Ɠž řƆŠƅŔ řƄũţ ūũŧƊ
G G
. A řųƂƊƅŔ Ɖƈ řƆŠƅŔ ŽŨƁ ŕƎƔž ƇśƔ ƓśƅŔ řŴţƆƅŔ Ɠƍ t 0 řƔœŔŧśŗƛŔ řŴţƆƅŔ ũŕśŦƊƏ (O ; i , k )

.řƆŠƅŔ ¿Ƃŝ ŘƏƂƅ řŗŬƊƅŕŗ ūŧƔƈŦũŌ řŸžŔŧƏ ʼnŔƏƎƅŔ Ŷƈ řƆŠƅŔ ŚŕƄŕƄśţŔ ¿ƈƎƊ
Ƈŝ žũŕśŦƈƅŔ ƇƆŸƈƅŔ Ɠž řƆŠƅŔ řƄũţƅ ƉƔśŪƔƈƈƅŔ z h t Ə x f t ƉƔśƔƊƈŪƅŔ ƉƔśƅŧŕŸƈƅŔ ŧŃŠ -1
. v0 Ə g ž D ž hA ũƔŧŕƂƈƅŔ řƅƛŧŗ z g x řƆŠƅŔ ũŕŬƈ řƅŧŕŸƈ şśƊśŬŔ
.ŕƎśƈƔƁ ŖŬţŔ Ƈŝ ž d Ə g ž D ž hA řƅƛŧŗ v 0 řƔœŔŧśŗƛŔ řŷũŬƅŔ Řũŕŗŷ ŧŃŠ -2
.ʼnŔƏƎƅŔ Ɠž řƆŠƅŔ ŕƎƁũżśŬś ƓśƅŔ řƔƊƈŪƅŔ ŘŧƈƅŔ ŧŃŠ -3
.g 9,8 m ˜ s -2 :ƏűŶř

( űœƀƈ 04 ) :ƑŕƒŧŞřƃŒ ƇƒŧƆřƃŒ

Ɠ»ž ũŮ»Ɣŗ Ɠ»ž ũŲţƊ žƓƊƔŠŬƄƗŔ ʼnŕƈƅŔ Ŷƈ ƇƏƔŬŕśƏŗƅŔ ŧƏƔ ¿ƏƆţƈ ¿ŷŕſśƅ řƔƄũţƅŔ řŬŔũŧƅŔ ¿ŠƗ
ƒŨ»ƅŔ ƇƏƔ»ŬŕśƏŗƅŔ ŧƏƔ ¿ƏƆţƈ Ɖƈ V 1 368 mL ƇŠţƅŔ Ɖƈ ¿ƄŮƈƅŔ s ƓƆŷŕſśƅŔ şƔŪƈƅŔ t 0 řŴţƆƅŔ
ƓƅƏ»ƈƅŔ ƋŪ»ƔƄũś ƒŨƅŔ ƓƊƔŠŬƄƗŔ ʼnŕƈƅŔ Ɖƈ V 2 32 mL ƇŠţƅŔƏ c1 0, 05mol ˜ L1 ƓƅƏƈƅŔ ƋŪƔƄũś
řų»ŬŔƏŗ ƓƊƔŠŬƄƗŔ ʼnŕƈƅŔ ŵŕŠũŏ ƇśƔž žŪƄũƈƅŔ ŚƔũŗƄƅŔ űƈţ Ɖƈ řƔžŕƄ řƔƈƄƏ c 2 0,10 mol ˜ L1

.ŧƏƔƅŔ ƓœŕƊŝ ƌƊŷ şśƊƔ ʼnƓųŗ ¿ŷŕſś ƀžƏ I  (aq ) ŧƏƔƅŔ ŧũŔƏŮ
: řƔśƕŔ řƅŧŕŸƈƅŕŗ ŜŧŕţƅŔ ƓœŕƔƈƔƄƅŔ ¿ŷŕſśƅŔ ŞŨƈƊƊ
H 2O 2 (aq )  2I  (aq )  2H  (aq ) 2H 2O (A)  I 2 (aq )

8 ǺǷ 7 ƨƸǨǏ
řƔƊƈŪ ŚŕŴţƅ Ɠž ¿ƄŮśƈƅŔ ŧƏƔƅŔ ƓœŕƊŝƅ ƓƅƏƈƅŔ ŪƔƄũśƅŔ ūŕƔƁ ¿ƜŦ Ɖƈ ¿ŷŕſśƆƅ ƓƄũţƅŔ ũƏųśƅŔ ŶŗŕśƊ
: řƔśƕŔ řƔƊƏƆƅŔ ŘũƔŕŸƈƅŔ řƂƔũų ¿ŕƈŸśŬŕŗ ƃƅŨƏ žřŗƁŕŸśƈ
ŧƔƆŠƅŔ ƒƏśţƔ ũŮƔŗ Ɠž ŕƎŗƄŬƊƏ s ƓƆŷŕſśƅŔ şƔŪƈƅŔ Ɖƈ V 40, 0 mL ŕƎƈŠţ řƊƔŷ t řŴţƆƅŔ Ɠž ŨŦōƊ
ŕ»Ɣœŕƈ ƛƏ»Ɔţƈ řƊƔŸƅŔ ƋŨƍ Ƒƅŏ ŕƔŠƔũŧś ŽƔŲƊ ƃƅŨ ŧŸŗ žƀũŪƗŕŗ şƔŪƈƅŔ ƉƏƆśƔž žʼnŕŮƊƅŔƏ ũƎŰƊƈƅŔ
ř»Ɣ௠Ƒ»ƅŏ c 3 0,10 mol ˜ L1 ƓƅƏƈƅŔ ƋŪƔƄũś ƒŨƅŔ (2Na  (aq )  S 2O 32  (aq )) ƇƏƔŧƏŰƅŔ ŚŕśƔũŗƄƏƔŝƅ
ŘũƔŕ»ŸƈƅŔ ¿ŷŕſś řƅŧŕŸƈƏ ŽŕŲąƈƅŔ ƇƏƔŧƏŰƅŔ ŚŕśƔũŗƄƏƔŝƅ V E ƇŠţƅŔ ¿ƜżśŬŕŗ .ƀũŪƗŔ ƉƏƆƅŔ ʼnŕſśŦŔ
. t řŴţƆƅŔ Ɠž ŧƏƔƅŔ ƓœŕƊŝƅ ƓƅƏƈƅŔ ŪƔƄũśƅŔ şśƊśŬƊ
řƅƛŧŗ ¿ƄŮśƈƅŔ [I 2 (aq )] ŧƏƔƅŔ ƓœŕƊŝƅ ƓƅƏƈƅŔ ŪƔƄũśƅŔ ũƏųś ƇŬũƊ Ƈŝ žřŗƁŕŸśƈ ŚŕŴţƅ Ɠž řƔƆƈŸƅŔ ŧƔŸƊ
[I 2 (aq )](mmol ˜ L1 ) .(5-¿ƄŮƅŔ) ƓƊŕƔŗƅŔ ƑƊţƊƈƅŔ ƑƆŷ ¿ŰţƊž t ƉƈŪƅŔ
.ŘũƔŕŸƈƅŔ řƔƆƈŷ ƓųƔųŦś ¿ƄŮŗ ƇŬũŔ -Ō -1
40mL ŨŦƗ ŕƎƆƈŸśŬƊ ƓśƅŔ řƆƔŬƏƅŔ Ɠƍ ŕƈ -Ŗ
 ƓƆŷŕſśƅŔ şƔŪƈƅŔ Ɖƈ
.ŘũƔŕŸƈƅŔ ¿ŷŕſś řƅŧŕŸƈ ŖśƄŔ -Š
1
Ɠž ƉŕśƈƍŕŬƈƅŔ ŧŬƄŎƈ/ŶŠũƈ ƉŕśƔœŕƊŝƅŔ
0 100 t (s )
5-¾ƂŬƃŒ I 2 (aq ) / I  (aq ) :ŕƈƍ ¿ƏţśƅŔ ŔŨƍ
S 4O 62  (aq ) / S 2O 32 (aq ) Ə

řƅƛŧŗ [I 2 (aq )] ŧƏƔƅŔ ƓœŕƊŝƅ ƓƅƏƈƅŔ ŪƔƄũśƆƅ řƂžŔƏƈƅŔ řƔžũţƅŔ ŘũŕŗŸƅŔ ŧŃŠ Ƈŝ žŎžŕƄśƅŔ Žĉũŷ -2
.ƇƏƔŧƏŰƅŔ ŚŕśƔũŗƄƏƔŝƅ c 3 ƓƅƏƈƅŔ ŪƔƄũśƅŔƏ V E ƇŠţƅŔƏ V ƇŠţƅŔ
ƓƊƔŠŬƄƗŔ ʼnŕƈƅŔ ƉŌ ƉƔĐŗƏ ƓƊƔŠŬƄƗŔ ʼnŕƈƅŔƏ ƇƏƔŬŕśƏŗƅŔ ŧƏƔ ¿ŷŕſśƅ ŪƔƈƈƅŔ ƇŧƂśƆƅ ƛƏŧŠ ŒŮƊŌ -3
.ŧţƈƅŔ ¿ŷŕſśƈƅŔ Əƍ
.t 100s řŴţƆƅŔ Ɠž ŕƎśƈƔƁ ŖŬţŔ Ƈŝ ž¿ŷŕſśƆƅ řƔƈŠţƅŔ řŷũŬƅŔ v ŽũĐ ŷ -4
. t 1 ¿ŷŕſśƅŔ ŽŰƊ ƉƈŪ ŕƔƊŕƔŗ ŧŃŠ -5
2

8 ǺǷ 8 ƨƸǨǏ
‫ﺍﻟﺼﻔﺤﺔ ‪1 :‬‬ ‫ﻣﻮﺍﺿﻴﻊ ﺍﻟﺒﻜﺎﻟﻮﺭﻳﺎ ‪ -‬ﻋﻠﻮﻡ ﻓﻴﺰﻳﺎﺋﻴﺔ ‪ -‬ﺍﻟﺸﻌﺒﺔ ‪ :‬ﻋﻠﻮﻡ ﲡﺮﻳﺒﻴﺔ ‪ -‬ﺩﻭﺭﺓ ﺟﻮﺍﻥ ‪ - 2012‬ﺍﳌﻮﺿﻮﻉ ‪ - 01‬ﺍﳊﻞ‬

‫ﺍﻷﺳﺘﺎﺫ ‪ :‬ﻓﺮﻗﺎﻧﻲ ﻓﺎﺭﺱ‬ ‫ﻋﻠﻮﻡ ﻓﻴﺰﻳﺎﺋﻴﺔ – ﺛﺎﻟﺜﺔ ﺛﺎﻧﻮﻱ – ﺍﻟﺸﻌﺐ ‪ :‬ﻋﻠﻮﻡ ﲡﺮﻳﺒﻴﺔ ‪ ،‬ﺭﻳﺎﺿﻴﺎﺕ ‪ ،‬ﺗﻘﻨﻲ ﺭﻳﺎﺿﻲ ‪.‬‬
‫ﺍﻟﺼﻔﺤﺔ ‪2 :‬‬ ‫ﻣﻮﺍﺿﻴﻊ ﺍﻟﺒﻜﺎﻟﻮﺭﻳﺎ ‪ -‬ﻋﻠﻮﻡ ﻓﻴﺰﻳﺎﺋﻴﺔ ‪ -‬ﺍﻟﺸﻌﺒﺔ ‪ :‬ﻋﻠﻮﻡ ﲡﺮﻳﺒﻴﺔ ‪ -‬ﺩﻭﺭﺓ ﺟﻮﺍﻥ ‪ - 2012‬ﺍﳌﻮﺿﻮﻉ ‪ - 01‬ﺍﳊﻞ‬

‫ﺍﻷﺳﺘﺎﺫ ‪ :‬ﻓﺮﻗﺎﻧﻲ ﻓﺎﺭﺱ‬ ‫ﻋﻠﻮﻡ ﻓﻴﺰﻳﺎﺋﻴﺔ – ﺛﺎﻟﺜﺔ ﺛﺎﻧﻮﻱ – ﺍﻟﺸﻌﺐ ‪ :‬ﻋﻠﻮﻡ ﲡﺮﻳﺒﻴﺔ ‪ ،‬ﺭﻳﺎﺿﻴﺎﺕ ‪ ،‬ﺗﻘﻨﻲ ﺭﻳﺎﺿﻲ ‪.‬‬
‫ﺍﻟﺼﻔﺤﺔ ‪3 :‬‬ ‫ﻣﻮﺍﺿﻴﻊ ﺍﻟﺒﻜﺎﻟﻮﺭﻳﺎ ‪ -‬ﻋﻠﻮﻡ ﻓﻴﺰﻳﺎﺋﻴﺔ ‪ -‬ﺍﻟﺸﻌﺒﺔ ‪ :‬ﻋﻠﻮﻡ ﲡﺮﻳﺒﻴﺔ ‪ -‬ﺩﻭﺭﺓ ﺟﻮﺍﻥ ‪ - 2012‬ﺍﳌﻮﺿﻮﻉ ‪ - 01‬ﺍﳊﻞ‬

‫ﺍﻷﺳﺘﺎﺫ ‪ :‬ﻓﺮﻗﺎﻧﻲ ﻓﺎﺭﺱ‬ ‫ﻋﻠﻮﻡ ﻓﻴﺰﻳﺎﺋﻴﺔ – ﺛﺎﻟﺜﺔ ﺛﺎﻧﻮﻱ – ﺍﻟﺸﻌﺐ ‪ :‬ﻋﻠﻮﻡ ﲡﺮﻳﺒﻴﺔ ‪ ،‬ﺭﻳﺎﺿﻴﺎﺕ ‪ ،‬ﺗﻘﻨﻲ ﺭﻳﺎﺿﻲ ‪.‬‬
‫ﺍﻟﺼﻔﺤﺔ ‪1 :‬‬ ‫ﻣﻮﺍﺿﻴﻊ ﺍﻟﺒﻜﺎﻟﻮﺭﻳﺎ ‪ -‬ﻋﻠﻮﻡ ﻓﻴﺰﻳﺎﺋﻴﺔ ‪ -‬ﺍﻟﺸﻌﺒﺔ ‪ :‬ﻋﻠﻮﻡ ﲡﺮﻳﺒﻴﺔ ‪ -‬ﺩﻭﺭﺓ ﺟﻮﺍﻥ ‪ - 2012‬ﺍﳌﻮﺿﻮﻉ ‪ - 02‬ﺍﳊﻞ‬

‫ﺍﻷﺳﺘﺎﺫ ‪ :‬ﻓﺮﻗﺎﻧﻲ ﻓﺎﺭﺱ‬ ‫ﻋﻠﻮﻡ ﻓﻴﺰﻳﺎﺋﻴﺔ – ﺛﺎﻟﺜﺔ ﺛﺎﻧﻮﻱ – ﺍﻟﺸﻌﺐ ‪ :‬ﻋﻠﻮﻡ ﲡﺮﻳﺒﻴﺔ ‪ ،‬ﺭﻳﺎﺿﻴﺎﺕ ‪ ،‬ﺗﻘﻨﻲ ﺭﻳﺎﺿﻲ ‪.‬‬
‫ﺍﻟﺼﻔﺤﺔ ‪2 :‬‬ ‫ﻣﻮﺍﺿﻴﻊ ﺍﻟﺒﻜﺎﻟﻮﺭﻳﺎ ‪ -‬ﻋﻠﻮﻡ ﻓﻴﺰﻳﺎﺋﻴﺔ ‪ -‬ﺍﻟﺸﻌﺒﺔ ‪ :‬ﻋﻠﻮﻡ ﲡﺮﻳﺒﻴﺔ ‪ -‬ﺩﻭﺭﺓ ﺟﻮﺍﻥ ‪ - 2012‬ﺍﳌﻮﺿﻮﻉ ‪ - 02‬ﺍﳊﻞ‬

‫ﺍﻷﺳﺘﺎﺫ ‪ :‬ﻓﺮﻗﺎﻧﻲ ﻓﺎﺭﺱ‬ ‫ﻋﻠﻮﻡ ﻓﻴﺰﻳﺎﺋﻴﺔ – ﺛﺎﻟﺜﺔ ﺛﺎﻧﻮﻱ – ﺍﻟﺸﻌﺐ ‪ :‬ﻋﻠﻮﻡ ﲡﺮﻳﺒﻴﺔ ‪ ،‬ﺭﻳﺎﺿﻴﺎﺕ ‪ ،‬ﺗﻘﻨﻲ ﺭﻳﺎﺿﻲ ‪.‬‬
‫ﺍﻟﺼﻔﺤﺔ ‪3 :‬‬ ‫ﻣﻮﺍﺿﻴﻊ ﺍﻟﺒﻜﺎﻟﻮﺭﻳﺎ ‪ -‬ﻋﻠﻮﻡ ﻓﻴﺰﻳﺎﺋﻴﺔ ‪ -‬ﺍﻟﺸﻌﺒﺔ ‪ :‬ﻋﻠﻮﻡ ﲡﺮﻳﺒﻴﺔ ‪ -‬ﺩﻭﺭﺓ ﺟﻮﺍﻥ ‪ - 2012‬ﺍﳌﻮﺿﻮﻉ ‪ - 02‬ﺍﳊﻞ‬

‫ﺍﻷﺳﺘﺎﺫ ‪ :‬ﻓﺮﻗﺎﻧﻲ ﻓﺎﺭﺱ‬ ‫ﻋﻠﻮﻡ ﻓﻴﺰﻳﺎﺋﻴﺔ – ﺛﺎﻟﺜﺔ ﺛﺎﻧﻮﻱ – ﺍﻟﺸﻌﺐ ‪ :‬ﻋﻠﻮﻡ ﲡﺮﻳﺒﻴﺔ ‪ ،‬ﺭﻳﺎﺿﻴﺎﺕ ‪ ،‬ﺗﻘﻨﻲ ﺭﻳﺎﺿﻲ ‪.‬‬
ƨȈƦǠnjdzơƨȈǗơǂǬŻƾdzơƨȇǂƟơDŽŪơƨȇǁȂȀǸŪơ
ƩƢǬƥƢLjŭơȁƩƢǻƢƸƬǷȐdzřǗȂdzơǹơȂȇƾdzơ     ƨȈǼǗȂdzơƨȈƥǂƬdzơƧǁơǃȁ
ƇŒƍŞƧǁȁƽ      ȅȂǻƢưdzơǶȈǴǠƬdzơƢȇǁȂdzƢǰƥǹƢƸƬǷơ
ŗºƒŕƒŧŞřƅƍƄŵƨƦǠnjdzơ
 ŻŮƈƍŘœŵœŪƧƾŭơ    ŗƒőœƒŨƒŽƃŒƅƍƄŶƃŒƧƽƢǷĿǁƢƦƬƻơ

ƇƒƒƃœřƃŒƇƒŵƍŰƍƆƃŒťšŊŧœřŤƒƇŊŠŬŧřƆƃŒƏƄŵ
 ŗűƀƈ ¾ƍƕŒųƍŰƍƆƃŒ

 űœƀƈ4 ¾ƍƕŒƇƒŧƆřƃŒ
 139
54 Xe Ə 38 Sr Ƒƅŏ 92U ƇƏƔƊŔũƏƔƅŔŘŔƏƊũŕųŮƊŔ¿ŷŕſśƅřƔƏƁŕųƅŔřƆƔŰţƅŔ¿ŝƈƔ ¿ƄŮƅŔ ƒƏƁŕųƅŔųųŦƈƅŔ
94 235

 01n ƉƏũśƔƊŗŕƎžŨƁũŝŏ


ΔϗΎτϟ΍
92 p + 144n řƔžũţƅŔŕƎśũŕŗŷŖśƄŔƏŘŔƏƊƆƅ E A ųŗũƅŔřƁŕųŽũĉ ŷŌ 

ΔE 1 řƔƏƊ¿ƄƅųŗũƅŔřƁŕųŘũŕŗŷųŷŌ
 Ŗ 
ΔE 2  235
92U ƇƏƔƊŔũƏƔƅŔŘŔƏƊũŕųŮƊŔřƅŧŕŸƈŖśƄŔŌ 

235
U + 01n
 ŔŨŕƈƅŕƔśŔŨƐŨżƈƓƆŬƆŬś¿ŷŕſśƌƊŌƑƆŷƀŗŕŬƅŔ¿ŷŕſśƅŔŽũŸƔ
 Ŗ
92

 ΔE 139
54 X e + 3894Sr + a 01n  ΔE Ə ΔE 2 Ə ΔE 1 ƉƈƜƄ MeV »ŗŖŬţŔ 


ϞϜθϟ΍
 235
92U Ɖƈ 1 g ũŕųŮƊŔƉŷŘũũţƈƅŔřƁŕųƅŔ ũŔŧƂƈ¿ƏŠƅŕŗŖŬţŔŌ 

 ŘũũţƈƅŔřƁŕųƅŔũƎŴś¿ƄŮƒŌƑƆŷŖ

E A 139 E

( 54 X e ) = 8,34MeV / nucléon ;  A ( 235
92U ) = 7,62 MeV / nucléon  ŘœƒűŶƆƃŒ
A A
E A 94
 N A = 6,02 × 1023 mol -1 ; 1MeV = 1,6 × 10-13 J ; ( 38 Sr ) = 8,62MeV / nucléon
A

 űœƀƈ4 ƑƈœśƃŒƇƒŧƆřƃŒ
řƔƅŕśƅŔřƅŧŕŸƈƅŔƒŨ¿ŷŕſśƅŕŗŞŨƈƊƔƓœŕƔƈƔƄ¿ƏţśƏƍʼnŕƈƅŔƓžCH 3 COOH ƃƔƏƊŕŝƔƛŔűƈţ¿ƜţƊŔ
 CH 3COOH (aq ) + H 2O (A ) = CH 3COO − (aq ) + H 3O + (aq )
 c 0 = 1, 0 × 10-2 mol ⋅ L-1 ƓœŔŧśŗƛŔƓƅƏƈƅŔƋŪƔƄũśƒŨƅŔ¿ƏƆţƈƆƅřƔŷƏƊƅŔřƔƆƁŕƊƅŔ 25 °C ř»ŠũŧƅŔƓžūƔƂƊ
 σ = 1,6 × 10−2 S ⋅ m −1 ŕƍŧŠƊž
¿ƏţśƅŔŔŨƍƓžřƄũŕŮƈƅŔūŕŬŌűƈţŚŕƔœŕƊŝƅŔŧŧĉ ţ
 
 ª¬ H 3O + (aq ) º¼ éq Ə c 0 řƅƛŧŗ K ƓœŕƔƈƔƄƅŔƉŪŔƏśƅŔŚŗŕŝŘũŕŗŷŖśƄŔ
 

ǺǷƨƸǨǏ
řƔƅƏƈƅŔřƔŷƏƊƅŔŚŕƔƆƁŕƊƅŔƏřƔƅƏƈƅŔŪƔƄŔũśƅŔřƅƛŧŗřŴţƅ¿ƄƓžřƔŷƏƊƅŔřƔƆƁŕƊƅŔŘũŕŗŸƅƇŕŸƅŔ¿ƄŮƅŔƑųŸƔ

i =n
 σ (t ) = Σ λi [ χ i ] řżƔŰƅŕŗ¿ƏƆţƈƅŔƓžŘŧŠŔƏśƈƅŔřƔœŕƔƈƔƄƅŔŧŔũžƗŔŽƆśŦƈƅřƔŧũŕŮƅŔ
i =1

 ʼnŕƈƆƅƓśŔŨƅŔƃƄſśƅŔ¿ƈƎƔ žƀŗŕŬƅŔ¿ƏƆţƈƆƅ σ (t ) řƔŷƏƊƅŔřƔƆƁŕƊƆƅřƔžũţƅŔŘũŕŗŸƅŔŖśƄŔ


ŜŧŕţƅŔ¿ŷŕſśƅŔƇŧƂśƅƛƏŧŠŒŮƊŌ
 
řƔœŕƔƈƔƄƅŔřƆƈŠƅŔƉŪŔƏśŧƊŷ¿ƏƆţƈƅŔƓžŘŧŠŔƏśƈƅŔřƔœŕƔƈƔƄƅŔŧŔũžƗŔŽƆśŦƈƅřƔƅƏƈƅŔŪƔƄŔũśƅŔŖŬţŔŌ
 
 K ƓœŕƔƈƔƄƅŔƉŪŔƏśƅŔŚŗŕŝŖŬţŔŖ
 şśƊśŬśŔŨŕƈτ f ƇŧƂśƆƅřƔœŕƎƊƅŔřŗŬƊƅŔƉƔĉ ŷ
 Š


 λH O = 35,9 × 10−3 S ⋅ m 2 ⋅ mol −1 ; λCH


+
COO −
= 4,10 × 10−3 S ⋅ m 2 ⋅ mol −1 ŘœƒűŶƆƃŒ
3 3


 űœƀƈ04 ŚƃœśƃŒƇƒŧƆřƃŒ
ƓƈƏŌ¿ƁŕƊƓžŕƎżƔũſśŗƇƏƂƊ C ŕƎśŸŬřžũŸƈ¿ŠŌƉƈ E = 6V ŚŗŕŝũśƏśŚţśŕƔƆƄŚƊţŮ C ŕƎśŸŬřſŝƄƈ
 R = 4 k Ω .ƌśƈƏŕƂƈ
źƔũſśƅŔŘũŔŧųųŦƈƇŬũŔ

řƔƊƏũśƄƅŏřƔśŕƂƔƈƏƓƈƁũũśƈųƅƏžŪŕƎŠ¿ƈŸśŬƊƉƈŪƅŔ¿ƜŦřſŝƄƈƅŔƓžũųƉƔŗ uC (t ) ũśƏśƅŔũƏųśřŸŗŕśƈƅ 

 ŘũŔŧƅŔƓžũśƈųƅƏſƅŔŪŕƎŠųŗũƇśƔŽƔƄŌ
ƓƅŕśƅŔ¿ƏŧŠƅŔƓžřŸŗŕśƈƅŔşœŕśƊ¿ŠŬƊƏ t = 0 ms řŴţƆƅŔƓžřŸųŕƂƅŔƀƆżƊ
 t (ms )         

 uC ( V )  6, 00  4, 91  4, 02  3, 21  2, 69  1,81  1, 21  0,81  0,54

ƁřŕœŞōŗſŧƍŴƆœƌƀżŧŊžřƔũśƈƔƆƔƈřƁũƏƑƆŷ uC = f (t ) řƅŔŧƆƅ¿ŝƈƈƅŔƓƊŕƔŗƅŔƑƊţƊƈƅŔƇŬũŌŖ
τ ƉƈŪƅŔŚŗŕŝřƈƔƁŕƔƊŕƔŗƉƔĉ ŷ
 ˰Ο
 C řſŝƄƈƅŔřŸŬŖŬţŔŧ
 uC (t ) ƓœŕŗũƎƄƅŔũśƏśƆƅřƔƆŲŕſśƅŔřƅŧŕŸƈƅŔŖśƄŔ žŚŔũśƏśƅŔŶƈŠƉƏƊŕƁƀƔŗųśŗ
 Ō
 
ŕƈƎƊƔƔŸśŖƆųƔƉŕśŗŕŝ A ; α ŜƔţžŕƎƅƜţ uC (t ) = A e − αt ŘũŕŗŸƅŔ¿ŗƂśřƂŗŕŬƅŔřƔƆŲŕſśƅŔřƅŧŕŸƈƅŔŖ

 űœƀƈ ŴŕŒŧƃŒƇƒŧƆřƃŒ
ťƔũŕśŗʼnŕŲſƅŔƑƅŏ¿ŬũŌž m s = 90 kg ƌśƆśƄŚŕƈŔŧŦśŬƛŔŧŧŸśƈƒũœŔŪŠƓŷŕƊųŰŔũƈƁ Alsat1 ŚŕŬƅŌ
T = 98 min ƋũƏŧƏƓŠƆƔƆƍŔũŕŬƈƀžƏűũƗŔ¿ƏţũƏŧƔžřƔŬƏũƅŔʼnŕŲſƅŔřųţƈƉƈũŗƈžƏƊ
ŕŗŬŕƊƈŕŸŠũƈũŕśŦƊƌśƄũţřŬŔũŧ¿ŠƗ 
ƌžũĉ ŷƏűũƗŔ¿ƏţƓŷŕƊųŰƛŔũƈƂƅŔřƄũţřŬŔũŧƅŕŸŠũƈšũśƁŔŌ
ũƆŗƄƅƓƊŕŝƅŔƉƏƊŕƂƅŔůƊŗũƄŅ ŨŖ

ǺǷƨƸǨǏ
ŕƎţųŬƉŷ h ŵŕſśũŔƑƆŷƒũœŔŧũŕŬƈƀžƏűũƗŔ¿ƏţũƏŧƔ(Alsat1)ƓŷŕƊųŰƛŔũƈƂƅŔƉŌűũſŗ 
ƓŷŕƊųŰƛŔũƈƂƆƅřŗŬƊƅŕŗűũƗŔŖŨŠŘƏƁ¿ŝŅ ƈŌ
 RT , h , G , m S , M T řƅƛŧŗƓŷŕƊųŰƛŔũƈƂƆƅűũƗŔŖŨŠŘƏƁŘŧŮƅřƔžũţƅŔŘũŕŗŸƅŔŖśƄŔŖ
ƉƈƓƍřƔũŔŧƈƅŔƓŷŕƊųŰƛŔũƈƂƅŔřŷũŬŘũŕŗŷƉŌƀƂŅ ţśžƉśƏƔƊƅƓƊŕŝƅŔƉƏƊŕƂƅŔƀƔŗųśŗ

GM T
 r = RT + h ŜƔţv = ¿ƄŮƅŔ
r
 r , G , M T řƅƛŧŗƌśũŕŗŷŖśƄŔƏT ũƏŧƅŔŽũĉ ŷŧ
űũƗŔŢųŬϦϋ( Alsat1 ƓŷŕƊųŰƛŔũƈƂƅŔƌƔƆŷŧŠŔƏśƔƒŨƅŔ h ŵŕſśũƛŔŖŬţŔ


ž M T = 6 × 1024 kg űũƗŔřƆśƄŸG = 6, 67 × 10-11 SI ƓƊƏƄƅŔŖŨ੶ƅŔŚŗŕŝ ŘœƒűŶƆƃŒ
 RT = 6,38 × 103 km űũƗŔũųƁŽŰƊ 

 űœƀƈ4 ƑŕƒŧŞřƃŒƇƒŧƆřƃŒ
ŚŕŬŧŸƅŔŽƔŴƊśƏšƏũŠƅŔũƔƎųśƓž¿ƈŸśŬƔƒŨƅŔžƓƊƔŠŬƄƗŔʼnŕƈƅŕŗƉƔŠƏũŧƔƎƅŔŧƔŬƄƏũƔŗ¿ƏƆţƈŽũŸƔ
űƔƔŗśƅŔƓžƃƅŨƄƏřƂŰƜƅŔ
řƔƅŕśƅŔřƔœŕƔƈƔƄƅŔřƅŧŕŸƈƅŕŗŞŨƈƊƈƅŔ¿ŷŕſśƅŔƀžƏŕƔśŔŨƓƊƔŠŬƄƗŔʼnŕƈƅŔƃƄſśƔ
 2H 2O 2 (aq ) = 2H 2O (A ) + O 2 ( g )
ƀŗŕŬƅŔ¿ƏţśƅŔřƔƄũţřŬŔũŧřƔƂƔŗųśƅŔ¿ŕƈŷƗŔřŰţƓžŨƔƈƜśƅŔƑƆŷšũśƁŌ

řƔƅŕśƅŔ¿œŕŬƏƅŔƏŧŔƏƈƅŔƇƎƅƏŕƊśƈƓžŨŕśŬƗŔŶŲƏ
 10 V ƓƊƔŠŬƄŌʼnŕƈŕƎƔƆŷŖśƄŕŝƔŧţşśƊƈ S 0 ƓƊƔŠŬƄƗŔʼnŕƈƅŔƉƈ 500 mL ƑƆŷƒƏśţśŘũƏũŕƁ
 
ƇŠ»ţƅŔžƉƔƔƈŕŴƊƅŔƉƔ»ųũŮƅŔƓžƉƔŠŬƄƗŔƓœŕƊŝŪ໯Ɖƈ10 L ũũţƔƓƊƔŠŬƄƗŔʼnŕƈ»ƅŔƉƈ 1 L ¿Ƅ 
 V M = 22.4L / mol ƓƅƏƈƅŔ
ŚŕƔŠŕŠŪƅŔ
 250 mL ; 200 mL ; 100 mL ; 50 mL řƔũŕƔŷŚƜŠƏţ •
ůƈřŰŕŠŏƏ10 mL ; 5 mL ; 1 mL řƔũŕƔŷŚŕŰŕƈ •
50 mL ŕƎśŸŬřŠũŧƈřţŕţŬ •
250 mL ƌśŸŬũŮƔŗ •
 c ' = 2,0 × 10−3 mol ⋅ L−1 ŚŕƊżƊƈũŗƅŔŧũŔƏŮŗƓƅƏƈƅŔƋŪƔƄũśŕŝƔŧţũŲţƈƇƏƔŬŕśƏŗƅŔŚŕƊżƊƈũŗ¿ƏƆţƈŘũƏũŕƁ 
ũųƂƈʼnŕƈ 
98% .ŪƄũƈƅŔŚƔũŗƄƅŔűƈţŘũƏũŕƁ 
¿ƈŕţ 


ǺǷƨƸǨǏ
ƓƆƔŕƈŗƇŕƔƂƅŔƇƎƊƈŖƆųƇŝ A ž B ž C ž D ŘũżŰƈŚŕŷƏƈŠƈŶŗũŌƑƅŏŨƔƈƜśƅŔşƔƏſśŗŨŕśŬƗŔƇŕƁ
Řũƈ 40  S 0 ¿ƏƆţƈƅŔƉƈřƊƔŷŧƔŧƈśŗƒŌ 200 mL ƇŠţŗ S ¿ƏƆţƈũƔŲţśƙƍŊ
 S ¿ƏƆţƈƅŔũƔŲţśƅŕƔŗƔũŠśƛƏƄƏśƏũŗŶŲ 

 ƓƊƔŠŬƄƗŔʼnŕƈƅŔƃƄſś ¿ŷŕſśƅŔƇŧƂśƅƛƏŧŠŒŮƊŌ 

 S ¿ƏƆţƈƆƅƓƅƏƈƅŔŪƔƄũśƅŔşśƊśŬŔ S 0 ¿ƏƆţƈƆƅƓƅƏƈƅŔŪƔƄũśƅŔŖŬţŔ 

ŧƔŧţƅŔŧũŔƏŮƑƆŷƒƏśţƔ¿ƏƆţƈƉƈŕƊƔŸƈŕƈŠţƌƔƅŏŽƔŲśƏž S ¿ƏƆţƈƅŔƉƈŕƈŠţřŷƏƈŠƈ¿ƄŨŦōśœƒƈœś
ƓƅŕśƅŔ¿ƏŧŠƅŔƀžƏųƔŬƏƄƓŝƜŝƅŔ

D  C  B  A řŷƏƈŠƈƅŔŪƈũ
     (mL ) ŽŕŲƈƅŔųƔŬƏƅŔƇŠţ
     H 2O 2 (mL ) ƇŠţ
     (mL ) ƓƆŷŕſśƅŔųŬƏƅŔƇŠţ
 řųŕŬƏƅŔŵƏƊŕƈ ųƔŬƏƅŔũƏŧŕƈ 

[ H 2O 2 ] (mmol/L) ŕƈŠţžřſƆśŦƈřƔƊƈŪŚŕŴţƅƓžžřŷƏƈŠƈ¿ƄŨŦōś

ƓžŶŲƏƔƏŕƎŗůŕŦƅŔƓƆŷŕſśƅŔųŬƏƅŔƉƈ10 mL ƋũŔŧƂƈ
1 ¿ƏƆţƈŗŘũƔŕŸƈƅŔřƔƆƈŷƌƅƐũŠśƏŧƔƆŠƅŔƏŧũŕŗƅŔʼnŕƈƅŔ
űƈţƉƈŚŔũųƁřžŕŲŐŗ řŲƈţƈƅŔƇƏƔŬŕśƏŗƅŔŚŕƊŕżƊƈũŗ
2
 ŪƄũƈƅŔŚƔũŗƄƅŔ
4   ŧƔƆŠƅŔƏŧũŕŗƅŔʼnŕƈƅŔ¿ŕƈŸśŬŔƉƈűũżƅŔŕƈŌ

 ¿ƄŮƅŔ řƔƊŕƔŗƅŔŚŕƔƊţƊƈƅŔƇŬũŗŘũƔŕŸƈƅŔŚŕƔƆƈŷŚţƈŬ
řŷƏƈŠƈ¿ƄŗůŕŦƅŔƉŕƔŗƅŔŧŧĉ ţ
 Ō
0 10 t (min)

¾ƂŬƃŒ ũĄƔŕŸƈƅŔ S ¿ƏƆţƈƆƅƓƅƏƈƅŔŪƔƄũśƅŔƉŕƔŗƅŔƉƈŧŠƏŔŖ


 S 0 ¿ƏƆţƈƆƅƓƅƏƈƅŔŪƔƄũśƅŔşśƊśŬŔ
 ŘũƏũŕƂƅŔƑƆŷ¿ŠŬƈƏƍŕƈŶƈřƂŗŕųśƈŕƎƔƅŏ¿ŰƏśƈƅŔşœŕśƊƅŔ¿ƍ
 Ο









ǺǷƨƸǨǏ
 ŗűƀƈ ƑºƈœśƃŒųƍºŰƍƆƃŒ

 űœƀƈ ¾ƍƕŒƇƒŧƆřƃŒ
C 2 H 2O 4 (aq ) ƃƔƅŕŬƄƏƗŔűƈţ¿ƏƆţƈƏCr2O 72− (aq ) ŚŕƈƏũƄƔŗƅŔŧũŔƏŮƉƔŗ¿ƏţśƅŔřƔƄũţũƏųśřŬŔũŧƅ
 ( 2K +
(aq ) + Cr2O 72− (aq ) ) ƇƏƔŬŕśƏŗƅŔŚŕƈƏũƄƔŗ¿ƏƆţƈƉƈ V 1 = 40 mL ŕƈŠţ t = 0 s řŴţƆƅŔƓžŞŪƈƊ
ƓƅƏƈƅŔƋŪƔƄũśƃƔƅŕŬƄƏƗŔűƈţ¿ƏƆţƈƉƈ V 2 = 60 mL ƇŠţŶƈ c1 = 0, 2 mol ⋅ L−1 ƓƅƏƈƅŔƋŪƔƄũś
 c2 ¿ƏƎŠƈ
Cr2O 72− (aq ) / Cr 3+ (aq ) Ə : CO 2 (aq ) C 2 H 2O 4 (aq ) ŕƈƍ¿ŷŕſśƅŔƓžƉŕśƄũŕŮƈƅŔƉŕśƔœŕƊŝƅŔŚƊŕƄŔŨŏ 

ŜŧŕţƅŔƓœŕƔƈƔƄƅŔ¿ƏţśƆƅŞŨƈƊƈƅŔŵŕŠũŏŘŧŬƄŌ¿ŷŕſśƅŔƉŷŘũŗŸƈƅŔřƅŧŕŸƈƅŔŖśƄŔŌ
¿ŷŕſśƅŔƇŧƂśƅƛƏŧŠŒŮƊŌŖ
n (Cr 3+ ) mmol řƔƈƄũƏųśƅƓƊŕƔŗƅŔƑƊţƊƈƅŔ 
 ¿ƄŮƅŔ ¿ŝŅ ƈƔ 

ƉƈŪƅŔřƅƛŧŗ Cr 3+ (aq ) Řŧŕƈ
ƉŕƔŗƅŔƉƈŧŠƏŔ
řŴţƆƅŔƓžCr 3+ (aq ) ŧũŔƏŮ¿ƄŅ ŮśřŷũŬŌ
1  t = 20 min
 x f ¿ŷŕſśƆƅƓœŕƎƊƅŔƇŧƂśƅŔŖ
0 5 t (min)
 t 1 ¿ŷŕſśƅŔŽŰƊƉƈŪ
 Š
¿ƄŮƅŒ 2

ŧţƈƅŔ¿ŷŕſśƈƅŔƉƔĉŷŕƈŕś¿ƏţśƅŔũŕŗśŷŕŗŌ

 c2 ƃƔƅŕŬƄƏƗŔűƈţ¿ƏƆţƈƅƓƅƏƈƅŔŪƔƄũśƅŔŧŠƏŔŖ

 űœƀƈ ƑƈœśƃŒƇƒŧƆřƃŒ
 
 ¿ƄŮƅŔ ¿ŬƆŬśƅŔƑƆŷřųƏŗũƈřƔƅŕśƅŔřƔœŕŗũƎƄƅŔũŰŕƊŸƅŔƑƆŷŘũŔŧƒƏśţś
E K  E ŚŗŕŝũśƏśƒŨŧƅƏƈ

- + i
 r ŕƎśƈƏŕƂƈƏ L ŕƎśƔśŔŨřŸƔŮƏ

¿ƄŮƅŒ  R = 100 Ω ƌśƈƏŕƂƈƓƈƏŌ¿ƁŕƊ

L,r
R  K řŸųŕƁ



ŪŔŪśƍŔƇŬŔũ¿ƈŸśŬƊ u R (t ) ƓƈƏƗŔ¿ƁŕƊƅŔƏ u b (t ) řŸƔŮƏƅŔƉƈ¿ƄƓžũųƉƔŗũśƏśƅŔũƏųśƅřƔƊƈŪƅŔřŸŗŕśƈƆƅ


ŘũƄŔŨƒŨƓųŗƎƈ
  u R (t ) Ə u b (t ) Ɖƈ¿ƄŘŧƍŕŮƈƅŘũŔŧƅŕŗƓųŗƎƈƅŔŪŔŪśƍƛŔƇŬŔũųŗũƉƄƈƔŽƔƄƉƔĉŗ
 Ō
 

ǺǷƨƸǨǏ
 ¿ƄŮƅŔ  u R (t ) Ə u b (t ) ƉƔũśƏśƆƅƉƔƆŝƈƈƅŔƉƔƔƊŕƔŗƅŔřŮŕŮƅŔƑƆŷŧƍŕŮƊž t = 0 ms řŴţƆƅŔƓžřŸųŕƂƅŔƀƆżƊŖ

u (V)


ϰϨΤϨϤϟ΍

ôƑƊţƊƈƅŔ 

2

0 5 t (ms) 

u (V)


 ƑƊţƊƈƅŔ 




2 

0 5 t (ms) 
 ϞϜθϟ΍

¿ƔƆŸśƅŔŶƈƌƅƀžŔƏƈƅŔũśƏśƆƅƑƊţƊƈ¿ƄŖŬƊŔ
¿ƄŮƅŔƉƈƉƏƄśŘũŔŧƅŔƓžũŕƈƅŔũŕƔśƅŔŘŧŮƅřƔƆŲŕſśƅŔřƅŧŕŸƈƅŔƉŌŚŗŝŔ
 Ō

di (t )
+ A i (t ) = B
dt
 R Ə r Ə L Ə E řƅƛŧŗ B Ə A Ɖƈ¿ƄŘũŕŗŷųŷŌŖ
řƂŗŕŬƅŔřƔƆŲŕſśƅŔřƅŧŕŸƈƆƅƜţƓƍ i (t ) = (1 − e − At ) ŘũŕŗŸƅŔƉŌƉƈƀƂŅ ţś
B

A
 I 0 ƇœŔŧƅŔƇŕŴƊƅŔƓžũŕƔśƅŔŘŧŮŖŬţŔŧ
 L Əτ Ə r Ə E Ɖƈ¿ƄƇƔƁŖŬţŔ

řŸƔŮƏƅŔƓžřƊŪŦƈƅŔřƔƈŴŷƗŔřƁŕųƅŔŖŬţŔƏ


ǺǷƨƸǨǏ
 űœƀƈ ŚƃœśƃŒƇƒŧƆřƃŒ
Ɖƈ 0,5 mol Ŷƈ A ƒƏŲŷűƈţƉƈ 0,5 mol ŞŪƈƊ E ¿ƔŝƔƛŔŚŔƏƊŕŝƔƈƒƏŲŸƅŔƓœŕƔƈƔƄƅŔŵƏƊƅŔũƔŲţśƅ
ƓœŕƈƇŕƈţƓžƌŸŲƊƏƇŕƄţŐŗƋŧŬƊƇŝũŕŗśŦŔŖƏŗƊŌƓžŪƄũƈƅŔŚƔũŗƄƅŔűƈţƉƈŚŔũųƁŧƏŠƏŗ B ¿ƏţƄ
 1000C řśŗŕŝƌśũŔũţřŠũŧ
 ŽŰƊřƔœƔŪŠƅŔƌśżƔŰƓƍŕƈƏ  E ƓœŕƔƈƔƄƅŔŵƏƊƅŔřŸƔŗųŕƈŌ 
 řƆŰſƈƅŔ 
ŕƎƊƈĻƜƄƇĉ Ŭ ž B Ə A Ɖƈ¿ƄƅřƆŰſƈƅŔ
 ŽŰƊřƔœƔŪŠƅŔřżƔŰƅŔŖśƄŔŖ
 ŜŧŕţƅŔ¿ƏţśƅŔƑƆŷŘũŔũţƅŔřŠũŧƏŪƄũƈƅŔŚƔũŗƄƅŔűƈţƉƈ¿ƄũƔŝōśŕƈ
 ˰Ο
¿ƏţśƅŔŔŨƎƅŞŨƈƊƈƅŔ¿ŷŕſśƅŔƉŷŘũŗŸƈƅŔřƔœŕƔƈƔƄƅŔřƅŧŕŸƈƅŔŖśƄŔ 

ƀžŔƏƈƅŔ K ƓœŕƔƈƔƄƅŔƉŪŔƏśƅŔŚŗŕŝŖŬţŔ¿ŷŕſśƆƅƇŧƂśƅŔ¿ƏŧŠŗŕƊƔŸśŬƈ 

 A ƒƏŲŸƅŔűƈţƅŔƉƈ 0,1 mol şƔŪƈƆƅŽƔŲƊƓœŕƔƈƔƄƅŔƉŪŔƏśƅŔŜƏŧţŧƊŷ 

¿ƆŅŷ ŕƔœŕƂƆśřƔœŕƔƈƔƄƅŔřƆƈŠƅŔũƏųśśƋ੶ŔƒŌƓžŶƁŅ ƏśŌ
řƔœŕƔƈƔƄƅŔřƆƈŠƆƅŧƔŧŠƅŔƉŪŔƏśƅŔřƅŕţŹƏƆŗŧƊŷşƔŪƈƆƅƓƅƏƈƅŔŖƔƄũśƅŔŧŠƏŔŖ

 űœƀƈ ŴŕŒŧƃŒƇƒŧƆřƃŒ
řŠŨƈƊƈƅŔřƅŧŕŸƈƅŔƀžƏ Ra ƇƏƔŧŔũƅŔƃƄſśŗşśƊƔŶŮƈŪ௠222Rn ƉƏŧŔũƅŔũŗśŸƔ
 ZA Ra → 222
86 Rn + 4
2 He
 ƒƏƏƊƅŔ¿ƏţśƅŔŔŨƎƅƀžŔƏƈƅŔŵŕŸŮƙŔųƈƊƏƍŕƈŌ
 
 Z ϭ A Ɖƈ¿ƄŧŠƏŔŖ
 u řƔũŨƅŔ¿śƄƅŔŘŧţƏŗŕƎƊŷŔũŗŸƈ 226
88 Ra ŘŔƏƊƅ Δm ƓƆśƄƅŔůƂƊƅŔŖŬţŔŌ

řƁŕų
 řƆśƄŎžŕƄśƅŔřƁƜŷƉŷũŗŸśƓśƅŔƉƔŕśŮƊƗŘũƔƎŮƅŔřżƔŰƅŔųŷŌŖ
 27,36 × 10−11 J řƈƔƂƅŔƒƏŕŬś 222Rn ƉƏŧŔũƅŔŘŔƏƊƅ E A ųŗũƅŔřƁŕųřƈƔƁƉŌũŕŗśŷŕŗ

ŘŔƏƊƆƅ E A ųŗũƅŔřƁŕųŽũĉ ŷŌ
 222Rn ƉƏŧŔũƅŔŘŔƏƊƅ Δm ƓƆśƄƅŔůƂƊƅŔŖŬţŔŖ
 222Rn ƉƏŧŔũƅŔŘŔƏƊƅřŗŬƊƅŕŗŕƎśƈƔƁşśƊśŬŌƇŝžřƔƏƊ¿ƄƅųŗũƅŔřƁŕųŽũĉ ŷΟ
ŕƎƊƔŗƉƈũŕųŮƊŔŚƜŷŕſśŘŧŷƌƅŜŧţśŜƔţžŧƏƁƏƄŖŰŦƈƅŔƇƏƔƊŔũƏƔƅŔ¿ƈŸśŬƔřƔƏƏƊƅŔŚƜŷŕſƈƅŔƓž

 235
92U +
1
0 n → 94
38 Sr + 139
54 Xe + 3 01 n řƅŧŕŸƈƅŕŗŞŨƈƊƈƅŔ¿ƏţśƅŔ
ũŕųŮƊƛŔ¿ŷŕſśŽũĉ ŷŌ
 (J ) ¿ƏŠƅŔƏ MeV »ƅŕŗŘũŧƂƈ¿ƏţśƅŔŔŨƍʼnŔũŠƉƈŘũũţƈƅŔřƁŕųƅŔŖŬţŔΏ


1 MeV =1,6 × 10 J ž c = 3 × 10 m ⋅ s ž1 u = 1,66 × 10 kg ŘœƒűŶƆƃŒ


−13 8 −1 −27

 m (U ) = 234,994 u ; m (Sr ) = 93,894 u ; m (X e ) = 138,889 u ; m (Rn ) =221,970 u 


 m (Ra ) = 225,977 u ; m ( 11 p ) =1,007 u ; m ( 01n ) =1,009 u 

ǺǷƨƸǨǏ
 űœƀƈ ƑŕƒŧŞřƃŒƇƒŧƆřƃŒ
ƉƏŧʼnŔƏƎƅŔƓžŕƔƅƏƁŕŮřƔųŕųƈřƔũƄųƏƂŬřŬŔũŧƌśŨƈƜśƑƆŷŨŕśŬƗŔšũśƁŔžřƔƂƔŗųśƅŔ¿ŕƈŷƗŔřŰţʼnŕƊŝŌ
řƔƈƁũřƂƔũųŗųƏƂŬƅŔřŠŨƈƊƏv 0 = 0 m ⋅ s −1 řƔœŔŧśŗŔřŷũŬ


 ρair = 1,3 kg ⋅ m -3 ʼnŔƏƎƆƅřƔƈŠţƅŔřƆśƄƅŔŸ r = 1,5 cm ŕƍũųƁŽŰƊŸ m = 3 g řƔũƄƅŔřƆśƄŘœƒűŶƆƃŒ


4
 g = 9,8 m ⋅ s −2 Ÿ f = kv 2 ƃŕƄśţƛŔŘƏƁŸV = π r 3 ŘũƄƅŔƇŠţ
3
ŖƏƆųƈƅŔ
ųƏƂŬƅŔ¿ţŔũƈ¿ƜŦřƔũƄƅŔřƅŕųŷŪƄũƈƓžŘũŝŎƈƅŔřƔŠũŕŦƅŔƐƏƂƅŔ¿ŝŅ ƈ

řƔũƄƅŔřƅŕųŷŪƄũƈƑƆŷƉśƏƔƊƅƓƊŕŝƅŔƉƏƊŕƂƅŔƀƔŗųśŗƏžŕƔƆƔƅŕŻƋũŗśŸƊŖŬŕƊƈřŬŔũŧŶŠũƈũŕƔśŦŕŗ

řŷũŬƆƅřƔƆŲŕſśƅŔřƅŧŕŸƈƅŔŖśƄŔ
ƑƆŷ¿ƏŰţƅŔƉƈŕƊśƊƄƈřƔŠƈũŗŗřųƂśƆƈƅŔũƏŰƅŔųƔũŮşƅƏŷƏřƔũƄƅŔřƄũţřŸŗŕśƈŗřƔƈƁũŔũƔƈŕƄŚţƈŬ

 ¿ƄŮƅŔ v = f (t ) ϭ a = h (t ) ƉƔƊŕƔŗƅŔ
¿ƆŅŷ ƉƈŪƅŔřƅƛŧŗ a (t ) ŵũŕŬśƅŔũƏųś¿ŝŅ ƈƔƉƔƔƊţƊƈƅŔƒŌŌ
v A řƔŧţƅŔřŷũŬƅŔŕƔƊŕƔŗŧŧĉ ţŖ
g
v A = (m − ρair V) ƉŌŕƈƆŷ
 Š
k
 k ƃŕƄśţƛŔ¿ƈŕŸƈřƈƔƁŖŬţŔ»

 1 cm → 2 m ⋅ s −1 Δϋήδϟ΍
Į(m/s2)
v(m/s)

1 cm → 2 m ⋅ s −2 ωέΎδΘϟ΍

 

 0,5 t (s)

¿ƄŮƅŔ

ǺǷƨƸǨǏ
‫ﺗﺼﺤﻴﺢ اﻟﺒﻜﺎﻟﻮرﻳﺎ ‪ - 2011‬اﻟﻌﻠﻮم اﻟﻔﻴﺰﻳﺎﺋﻴﺔ ‪ -‬ﺷﻌﺒﺔ اﻟﻌﻠﻮم اﻟﺘﺠﺮﻳﺒﻴﺔ‬
‫اﻟﻤﻮﺿﻮع اﻷول‬
‫اﻟﺘﻤﺮﻳﻦ اﻷول )‪ 4‬ﻧﻘﻂ(‬
‫‪ - 1‬أ( ﻃﺎﻗﺔ اﻟﺮﺑﻂ ‪ El‬هﻲ أﺻﻐﺮ ﻃﺎﻗﺔ ﻧﻘﺪّﻣﻬﺎ ﻟﻠﻨﻮاة ﻣﻦ أﺟﻞ ﻓﺼﻞ اﻟﻨﻮآﻠﻴﻮﻧﺎت ﻋﻦ ﺑﻌﻀﻬﺎ )ﺗﻔﻜﻴﻚ اﻟﻨﻮاة إﻟﻰ ﻣﻜﻮﻧﺎﺗﻬﺎ( ‪.‬‬

‫أو ‪ :‬هﻲ اﻟﻔﺮق ﺑﻴﻦ ﻃﺎﻗﺔ اﻟﻜﺘﻠﺔ ﻟﻠﻤﻜﻮﻧﺎت وﻃﺎﻗﺔ آﺘﻠﺔ اﻟﻨﻮاة ‪.‬‬
‫أو ‪ :‬اﻟﻄﺎﻗﺔ اﻟﻨﺎﺗﺠﺔ ﻋﻦ ﺗﺠﻤﻴﻊ اﻟﻨﻮآﻠﻴﻮﻧﺎت ﻓﻲ اﻟﻨﻮاة ‪.‬‬

‫‪El = ⎡⎣ Z × m p + ( A − Z ) × mn − mX ⎤⎦ c 2‬‬ ‫ﻋﺒﺎرﺗﻬﺎ اﻟﺤﺮﻓﻴﺔ ‪:‬‬

‫‪⎡⎣ Z × m p + ( A − Z ) × mn − mX ⎤⎦ c 2‬‬
‫ب( ﻋﺒﺎرة ﻃﺎﻗﺔ اﻟﺮﺑﻂ ﻟﻜﻞ ﻧﻮﻳﺔ هﻲ‬
‫‪A‬‬
‫‪ ،‬ﻧﺤﺪّد ﻗﻴﻤﺔ ‪ a‬ﺑﻮاﺳﻄﺔ ﻗﺎﻧﻮن اﻻﻧﺤﻔﺎظ ﻟﺼﻮدي ‪:‬‬ ‫‪235‬‬
‫‪92‬‬ ‫→ ‪U + 01n‬‬ ‫‪94‬‬
‫‪38‬‬ ‫‪Sr + 139‬‬
‫‪54 Xe + a 0 n‬‬
‫‪1‬‬
‫‪ - 2‬أ(‬

‫→ ‪U + 01n‬‬ ‫‪ ،‬وﻣﻨﻪ ‪ ، a = 3‬وﺗﺼﺒﺢ اﻟﻤﻌﺎدﻟﺔ ‪54 Xe + 3 0 n :‬‬


‫‪Sr + 139‬‬ ‫‪236 = 94 + 139 + a‬‬
‫‪235‬‬ ‫‪94‬‬ ‫‪1‬‬
‫‪92‬‬ ‫‪38‬‬

‫ب( ﻳُﺴﻤﻰ اﻟﺘﻔﺎﻋﻞ اﻟﺘﺴﻠﺴﻠﻲ ﻣﻐﺬى ذاﺗﻴﺎ ﻷن اﻟﻨﻮﺗﺮوﻧﺎت اﻟﻨﺎﺗﺠﺔ ﺗُﺴﺘﻌﻤﻞ ﻓﻲ اﻧﺸﻄﺎرات أﺧﺮى ‪.‬‬

‫‪ ،‬ﺣﻴﺚ ‪Δ E1 = 7,62 × 235 = 1790,7 MeV‬‬ ‫‪235‬‬


‫‪92‬‬ ‫‪ Δ E1‬هﻲ ﻃﺎﻗﺔ اﻟﺮﺑﻂ ﻟﻠﻨﻮاة ‪U‬‬ ‫‪-3‬‬

‫‪ ،‬ﺣﻴﺚ ‪Δ E2 = − (1159, 26 + 810, 28 ) = −1969,54 MeV‬‬ ‫‪139‬‬


‫و ‪Xe‬‬ ‫‪94‬‬
‫‪ ΔE2‬هﻲ ﻧﻈﻴﺮ ﻣﺠﻤﻮع ﻃﺎﻗﺘﻲ اﻟﺮﺑﻂ ﻟﻠﻨﻮاﺗﻴﻦ ‪Sr‬‬

‫‪ ،‬ﺣﻴﺚ ‪:‬‬ ‫→ ‪U + 01n‬‬


‫‪235‬‬
‫‪92‬‬
‫‪94‬‬
‫‪38‬‬ ‫‪Sr + 139‬‬
‫‪ Δ E‬هﻲ ﻧﻈﻴﺮ اﻟﻄﺎﻗﺔ اﻟﻤﺤﺮرة ﻓﻲ ﺗﻔﺎﻋﻞ اﻻﻧﺸﻄﺎر ‪54 Xe + 3 0 n :‬‬
‫‪1‬‬

‫(‬ ‫)‬
‫‪Δ E = − El f − El i = − ( 8,34 × 139 + 8,62 × 94 − 1790,7 ) = −178,84 MeV‬‬

‫ﻣﻼﺣﻈﺔ ‪ :‬إﻳﺠﺎد هﺬﻩ اﻟﻄﺎﻗﺎت ﺑﺎﻟﻘﻴﻤﺔ اﻟﻤﻄﻠﻘﺔ ﺗﻌﺘﺒﺮ ﺻﺤﻴﺤﺔ ‪.‬‬


‫‪m‬‬ ‫‪1‬‬
‫=‪N‬‬ ‫= ‪× NA‬‬ ‫‪× 6,02 × 1023 = 2,56 × 1021 :‬‬ ‫‪235‬‬
‫‪ - 4‬أ( ﻧﺤﺴﺐ ﻋﺪد اﻷﻧﻮﻳﺔ اﻟﻤﻮﺟﻮدة ﻓﻲ ‪ 1 g‬ﻣﻦ اﻟﻴﻮراﻧﻴﻮم ‪U‬‬
‫‪M‬‬ ‫‪235‬‬
‫‪Elib = 178,84 × 2,56 × 1021 = 4,58 × 1023 MeV = 7,33 × 1010 J‬‬
‫ب( ﺗﻈﻬﺮ اﻟﻄﺎﻗﺔ اﻟﻤﺤﺮرة ﻋﻠﻰ ﺷﻜﻞ ‪ - :‬ﻃﺎﻗﺔ ﺣﺮآﻴﺔ ﻟﻸﻧﻮﻳﺔ واﻟﻨﻮﺗﺮوﻧﺎت اﻟﻨﺎﺗﺠﺔ ‪ ،‬ﺑﻤﺎ ﻓﻴﻬﺎ ﻃﺎﻗﺔ إرﺗﺪاد اﻷﻧﻮﻳﺔ ‪.‬‬
‫‪ -‬ﻃﺎﻗﺔ اﺷﻌﺎﻋﻴﺔ ) ‪. ( γ‬‬

‫اﻟﺘﻤﺮﻳﻦ اﻟﺜﺎﻧﻲ )‪ 4‬ﻧﻘﻂ( ‪------------------------------------------------------------------------------------------------‬‬

‫‪ - 1‬اﻟﺜﻨﺎﺋﻴﺘﺎن هﻤﺎ ‪H 3O + / H 2O ، CH 3COOH / CH 3COO − :‬‬

‫⎦⎤ ‪⎡⎣CH 3COO − ⎤⎦ ⎡⎣ H 3O +‬‬


‫‪،‬‬ ‫‪[CH 3COOH ]éq = C0 − ⎡⎣ H 3O‬‬ ‫‪+‬‬
‫‪ ،‬وﻟﺪﻳﻨﺎ ⎦⎤ ‪ ⎡⎣CH 3COO ⎤⎦ ≈ ⎡⎣ H 3O‬و ⎦⎤‬
‫‪−‬‬ ‫‪+‬‬
‫=‪K‬‬
‫‪éq‬‬ ‫‪éq‬‬
‫‪-2‬‬
‫‪éq‬‬ ‫‪éq‬‬ ‫‪éq‬‬
‫‪[CH 3COOH ]éq‬‬
‫‪2‬‬
‫⎦⎤ ‪⎡⎣ H 3O +‬‬
‫‪éq‬‬
‫=‪K‬‬ ‫وﺑﺎﻟﺘﺎﻟﻲ‬
‫⎦⎤ ‪C0 − ⎡⎣ H 3O +‬‬
‫‪éq‬‬

‫‪ ، σ t = λ + ⎡⎣ H 3O + ⎤⎦ + λ‬وذﻟﻚ ﺑﺈهﻤﺎل ⎦⎤ ‪⎡⎣OH −‬‬ ‫‪⎡CH 3COO − ⎤⎦ - 3‬‬


‫‪H 3O‬‬ ‫⎣ ‪CH 3COO −‬‬

‫‪ - 4‬ﺟﺪول اﻟﺘﻘﺪّم ‪:‬‬


‫‪CH 3COOH + H 2 O = CH 3COO − + H 3O +‬‬
‫‪C0 V‬‬ ‫ﺑﻜﺜﺮة‬ ‫‪0‬‬ ‫‪0‬‬
‫‪ ، ⎡⎣ H 3O + ⎤⎦ λ + + λ‬وﻣﻨﻪ‬
‫‪éq‬‬ ‫‪H 3O‬‬ ‫(‬
‫‪CH 3COO −‬‬
‫‪= 1,6 × 10−2‬‬ ‫)‬ ‫‪ - 5‬أ(‬

‫‪C0 V − x‬‬ ‫‪----‬‬ ‫‪x‬‬ ‫‪x‬‬


‫‪1,6 × 10−2‬‬ ‫‪1,6 × 10−2‬‬
‫‪C0 V − xéq‬‬ ‫= ⎦⎤ ‪⎡⎣ H 3O +‬‬ ‫=‬ ‫‪= 0, 4 mol / m3‬‬
‫‪(λ‬‬ ‫)‬
‫‪-----‬‬ ‫‪xéq‬‬ ‫‪xéq‬‬ ‫‪éq‬‬
‫‪+λ‬‬ ‫‪40 × 10−3‬‬
‫‪C0 V − xm‬‬ ‫‪xm‬‬ ‫‪xm‬‬ ‫‪H 3O +‬‬ ‫‪CH 3COO −‬‬
‫‪-----‬‬
‫‪⎡⎣ H 3O + ⎤⎦ = 4 × 10−4 mol / L‬‬
‫‪éq‬‬

‫‪1‬‬
‫‪⎡⎣CH 3COO − ⎤⎦ = ⎡⎣ H 3O + ⎤⎦ = 4 × 10−4 mol / L‬‬
‫‪éq‬‬ ‫‪éq‬‬

‫⎦⎤ ‪) ⎡⎣OH −‬ﻓﺎﺋﻘﺔ اﻟﻘﻠﺔ(‬


‫‪éq‬‬
‫‪ ،‬ﻻ داﻋﻲ ﻟﺤﺴﺎب‬ ‫‪[CH 3COOH ]éq = C0 − ⎡⎣ H 3O + ⎤⎦ éq = 10−2 − 4 × 10−4 = 9,6 × 10−3 mol / L‬‬

‫) ‪( 4 ×10‬‬
‫‪2‬‬
‫⎦⎤ ‪⎡⎣ H 3O +‬‬ ‫‪−4 2‬‬
‫‪éq‬‬
‫=‪K‬‬ ‫=‬ ‫‪= 1,67 × 10−5‬‬ ‫ب(‬
‫⎦⎤ ‪C0 − ⎡⎣ H 3O +‬‬ ‫‪9,6 × 10−3‬‬
‫‪éq‬‬

‫⎦⎤ ‪⎡⎣ H 3O +‬‬


‫‪xéq‬‬ ‫‪4 × 10−4‬‬
‫ﺑﻤﺎ أن ‪ τ f < 1‬ﻓﺈن ﺗﺸﺮد ﺣﻤﺾ اﻹﻳﺜﺎﻧﻮﻳﻚ ﻓﻲ اﻟﻤﺎء ﻣﺤﺪود‬ ‫= ‪. τf‬‬
‫‪éq‬‬
‫=‬ ‫=‬ ‫‪= 0,04‬‬ ‫ﺟـ(‬
‫‪xm‬‬ ‫‪C0‬‬ ‫‪10−2‬‬
‫أو ﻧﻘﻮل ‪ :‬ﻧﺴﺘﻨﺘﺞ أن ﺣﻤﺾ اﻹﻳﺜﺎﻧﻮﻳﻚ هﻮ ﺣﻤﺾ ﺿﻌﻴﻒ ﻓﻲ اﻟﻤﺎء ‪.‬‬

‫اﻟﺘﻤﺮﻳﻦ اﻟﺜﺎﻟﺚ )‪ 4‬ﻧﻘﻂ( ‪-----------------------------------------------------------------------------------------------‬‬

‫‪i‬‬ ‫‪ - 1‬ﻣﺨﻄﻂ دارة اﻟﺘﻔﺮﻳﻎ ‪:‬‬


‫‪ - 2‬أ( ﻧﺮﺑﻂ ﻣﻘﻴﺎس اﻟﻔﻮﻟﻂ ﺑﻴﻦ ﻃﺮﻓﻲ اﻟﻤﻜﺜّﻔﺔ ‪.‬‬
‫‪+‬‬ ‫ب( اﻟﺮﺳﻢ اﻟﺒﻴﺎﻧﻲ ‪) :‬ﻓﻲ اﻟﺸﻜﻞ(‬
‫‪V‬‬ ‫‪C‬‬ ‫‪R‬‬
‫‪-‬‬
‫ﺟـ( ﺛﺎﺑﺖ اﻟﺰﻣﻦ هﻮ اﻟﺰﻣﻦ اﻟﻤﻮاﻓﻖ ﻟـ ‪ ، uC = 0,37 E = 2, 2V‬وﺑﺎﻟﺘﺎﻟﻲ ‪τ = 50 ms‬‬
‫‪τ‬‬ ‫‪50 × 10−3‬‬
‫=‪C‬‬ ‫=‬ ‫‪= 12,5 × 10−6 F = 12,5μ F‬‬ ‫د( ﻟﺪﻳﻨﺎ ‪ ، τ = RC‬وﺑﺎﻟﺘﺎﻟﻲ‬
‫‪R‬‬ ‫‪4000‬‬
‫) ‪uC (V‬‬
‫‪ - 3‬أ( ﺣﺴﺐ ﻗﺎﻧﻮن ﺟﻤﻊ اﻟﺘﻮﺗﺮات ‪uC + u R = 0‬‬
‫‪6‬‬
‫‪dq‬‬
‫‪uC + R‬‬ ‫‪=0‬‬
‫‪dt‬‬
‫‪d uC‬‬
‫‪uC + RC‬‬ ‫‪=0‬‬
‫‪dt‬‬
‫‪duC‬‬ ‫‪1‬‬
‫• ‪2,2‬‬ ‫)اﻟﻤﻌﺎدﻟﺔ اﻟﺘﻔﺎﺿﻠﻴﺔ (‬ ‫‪+‬‬ ‫‪uC = 0‬‬
‫‪dt RC‬‬
‫‪duC‬‬
‫ب( ‪= − Aα e −α t‬‬
‫‪0,5‬‬ ‫‪dt‬‬
‫•‬
‫) ‪t ( ms‬‬
‫‪5‬‬ ‫‪50‬‬ ‫‪A −α t‬‬
‫‪− Aα e −α t +‬‬ ‫ﺑﺎﻟﺘﻌﻮﻳﺾ ﻓﻲ اﻟﻤﻌﺎدﻟﺔ اﻟﺘﻔﺎﺿﻠﻴﺔ ‪e = 0 :‬‬
‫‪RC‬‬
‫‪1‬‬ ‫‪1‬‬ ‫‪⎛ 1‬‬ ‫⎞‬
‫=‪α‬‬ ‫‪ ،‬وﻣﻨﻪ‬ ‫⎜ ‪ ، Ae −α t‬وﺣﺘﻰ ﺗﻜﻮن هﺬﻩ اﻟﻤﻌﺎدﻟﺔ ﻣﺘﺠﺎﻧﺴﺔ ﻳﺠﺐ أن ﻳﻜﻮن ‪− α = 0‬‬ ‫‪−α ⎟ = 0‬‬
‫‪RC‬‬ ‫‪RC‬‬ ‫⎝‬ ‫‪RC‬‬ ‫⎠‬
‫ﻣﻦ أﺟﻞ ﺗﻌﻴﻴﻦ ‪ A‬ﻧﺴﺘﻌﻤﻞ اﻟﺸﺮوط اﻻﺑﺘﺪاﺋﻴﺔ ‪ ،‬أي ﻋﻨﺪ ‪ t = 0‬ﻳﻜﻮن ‪ ، uC = E‬وﺑﺎﻟﺘﻌﻮﻳﺾ ﻓﻲ اﻟﻌﺒﺎرة اﻟﺰﻣﻨﻴﺔ ‪:‬‬

‫‪ ، E = Ae0‬وﻣﻨﻪ ‪. A = E‬‬
‫اﻟﺘﻤﺮﻳﻦ اﻟﺮاﺑﻊ )‪ 4‬ﻧﻘﻂ( ‪-----------------------------------------------------------------------------------------------‬‬
‫‪ - 1‬أ( اﻟﻤﺮﺟﻊ اﻟﺬي ﻧﻨﺴﺐ ﻟﻪ دراﺳﺔ ﺣﺮآﺔ اﻟﻘﻤﺮ اﻟﺼﻨﺎﻋﻲ هﻮ اﻟﻤﺮﺟﻊ اﻷرﺿﻲ ﻣﺮآﺰي ‪،‬‬
‫‪r‬‬ ‫‪S‬‬ ‫وهﻮ اﻟﻤﺮﺟﻊ اﻟﻤﺮﺗﺒﻂ ﺑﺎﻟﻤﻌﻠﻢ اﻟﺬي ﻣﺒﺪؤﻩ ﻣﺮآﺰ اﻷرض وﻣﺤﺎورﻩ ﻣﺘﺠﻬﺔ ﻧﺤﻮ ﺛﻼﺛﺔ ﻧﺠﻮم‬
‫‪FT / S r‬‬
‫‪n‬‬ ‫ﺑﻌﻴﺪة ﻧﻌﺘﺒﺮهﺎ ﺛﺎﺑﺘﺔ ‪.‬‬

‫‪r‬‬ ‫ب( اﻟﻘﺎﻧﻮن اﻟﺜﺎﻧﻲ ﻟﻜﺒﻠﺮ ‪ :‬اﻟﻤﺤﻮر اﻟﻮاﺻﻞ ﺑﻴﻦ ﻣﺮآﺰ اﻷرض واﻟﻘﻤﺮ اﻟﺼﻨﺎﻋﻲ ﻳﻤﺴﺢ‬
‫•‪O‬‬ ‫‪u‬‬ ‫ﻣﺴﺎﺣﺎت ﻣﺘﺴﺎوﻳﺔ ﻓﻲ أزﻣﻨﺔ ﻣﺘﺴﺎوﻳﺔ ‪.‬‬
‫‪T‬‬
‫‪ - 2‬أ( ﺗﻤﺜﻴﻞ اﻟﻘﻮة ﻓﻲ اﻟﻤﻌﻠﻢ اﻷرﺿﻲ ﻣﺮآﺰي )اﻟﺸﻜﻞ(‬

‫‪2‬‬
‫‪r‬‬ ‫‪mS M T r‬‬
‫‪FT / S = − G‬‬ ‫ب( ﻗﻮة ﺟﺬب اﻷرض ﻟﻠﻘﻤﺮ اﻟﺼﻨﺎﻋﻲ هﻲ ‪u‬‬
‫) ‪( RT + h‬‬
‫‪2‬‬

‫‪mS M T‬‬
‫‪FT / S = G‬‬ ‫اﻟﻌﺒﺎرة اﻟﺤﺮﻓﻴﺔ ﻟﺸﺪّة هﺬﻩ اﻟﻘﻮة هﻲ‬
‫) ‪( RT + h‬‬
‫‪2‬‬

‫‪r‬‬ ‫‪r‬‬
‫ﺟـ( ﺑﺘﻄﺒﻴﻖ اﻟﻘﺎﻧﻮن اﻟﺜﺎﻧﻲ ﻟﻨﻴﻮﺗﻦ ﻓﻲ اﻟﻤﺮﺟﻊ اﻷرﺿﻲ ﻣﺮآﺰي ‪FT / S = ma :‬‬
‫‪mS M T‬‬ ‫‪r‬‬ ‫‪r‬‬ ‫‪r‬‬ ‫‪r‬‬ ‫‪mS M T r‬‬ ‫‪r‬‬
‫‪G‬‬ ‫‪2‬‬
‫‪ ،‬وﻟﺪﻳﻨﺎ ﻓﻲ ﻣﻌﻠﻢ ﻓﺮﻳﻨﻲ ‪ ، a = an n‬وﻟﺪﻳﻨﺎ آﺬﻟﻚ ‪ ، u = −n‬وﺑﺎﻟﺘﺎﻟﻲ ‪= mS an‬‬ ‫‪−G‬‬ ‫‪2‬‬
‫‪u = mS a‬‬
‫‪r‬‬ ‫‪r‬‬

‫‪GM T‬‬ ‫‪mS M T‬‬ ‫‪v2‬‬


‫=‪. v‬‬ ‫‪ ،‬وﻣﻨﻪ‬ ‫‪G‬‬ ‫=‬ ‫‪m‬‬‫‪S‬‬
‫‪r‬‬ ‫‪r2‬‬ ‫‪r‬‬
‫د( اﻟﺪور هﻮ اﻟﻤﺪة اﻟﻼزﻣﺔ ﻟﻜﻲ ﻳٌﻨﺠﺰ اﻟﻘﻤﺮ اﻟﺼﻨﺎﻋﻲ دورة آﺎﻣﻠﺔ ﺣﻮل اﻷرض ‪.‬‬

‫‪r3‬‬ ‫‪2π r‬‬


‫)‪(1‬‬ ‫‪T = 2π‬‬ ‫= ‪ T‬ﻧﺠﺪ ﻋﺒﺎرة اﻟﺪور‬ ‫ﻣﻦ اﻟﻌﺒﺎرة‬
‫‪G MT‬‬ ‫‪v‬‬

‫‪T 2GM T ( 98 × 60 ) × 4 × 10‬‬


‫‪2‬‬ ‫‪14‬‬
‫= ‪ ، r‬وﻣﻨﻪ‬
‫‪3‬‬
‫=‬ ‫هـ( ﺑﺘﺮﺑﻴﻊ ﻃﺮﻓﻲ اﻟﻌﻼﻗﺔ )‪ (1‬ﻧﺠﺪ ‪= 3,5 × 1020‬‬
‫‪4π 2‬‬ ‫) ‪4 × ( 3,14‬‬
‫‪2‬‬

‫‪r = 3 3,5 × 1020 = 7,047 × 106 m = 7047 km‬‬

‫‪h = r − RT = 7047 − 6380 = 667 km‬‬


‫اﻟﺘﻤﺮﻳﻦ اﻟﺘﺠﺮﻳﺒﻲ )‪ 4‬ﻧﻘﻂ( ‪--------------------------------------------------------------------------------------------‬‬
‫أوﻻ ‪:‬‬
‫‪200‬‬
‫= ‪V0‬‬ ‫هﻮ ‪= 5 mL‬‬ ‫) ‪( S0‬‬ ‫‪ - 1‬ﻣﻌﺎﻣﻞ اﻟﺘﻤﺪﻳﺪ هﻮ ‪ ، F = 40‬إذن اﻟﺤﺠﻢ اﻟﻮاﺟﺐ أﺧﺬﻩ ﻣﻦ اﻟﻤﺤﻠﻮل‬
‫‪40‬‬
‫ﻧﺄﺧﺬ هﺬا اﻟﺤﺠﻢ ﺑﻮاﺳﻄﺔ اﻟﻤﺎﺻﺔ اﻟﺘﻲ ﺳﻌﺘﻬﺎ ‪ 5 mL‬ﻣﻦ اﻟﻤﺤﻠﻮل ) ‪ ( S0‬وﻧﻀﻌﻪ ﻓﻲ اﻟﺤﻮﺟﻠﺔ اﻟﺘﻲ ﺳﻌﺘﻬﺎ ‪ ، 200 mL‬ﺛﻢ ﻧﻜﻤﻞ‬

‫ﺑﺎﻟﻤﺎء اﻟﻤﻘﻄﺮ ﺣﺘﻰ ﺧﻂ اﻟﺤﻮﺟﻠﺔ ‪ ،‬ﻣﻌﻨﺎﻩ ﻧﻀﻴﻒ ‪ 195 mL‬ﻣﻦ اﻟﻤﺎء اﻟﻤﻘﻄﺮ ‪.‬‬
‫‪2H 2 O 2‬‬ ‫=‬ ‫‪2H 2O‬‬ ‫‪+‬‬ ‫‪O2‬‬ ‫‪ - 2‬ﺟﺪول اﻟﺘﻘﺪّم ‪:‬‬
‫‪C0 V‬‬ ‫ﺑﻜﺜﺮة‬ ‫‪0‬‬ ‫‪ - 3‬اﻟﺘﺮآﻴﺰ اﻟﻤﻮﻟﻲ ﻟﻠﻤﺤﻠﻮل ) ‪: ( S0‬‬
‫‪C0 V − 2 x‬‬ ‫‪----‬‬ ‫‪x‬‬
‫‪C0 V − 2 xm‬‬ ‫‪xm‬‬ ‫‪VO2‬‬ ‫‪10‬‬
‫‪-----‬‬ ‫= ‪. xm = nO2‬‬ ‫=‬ ‫ﻣﻦ ﺟﺪول اﻟﺘﻘﺪم ﻟﺪﻳﻨﺎ ‪= 0, 446 mol‬‬
‫‪VM‬‬ ‫‪22, 4‬‬

‫‪2 xm 2 × 0, 446‬‬
‫= ‪C0‬‬ ‫=‬ ‫ﻋﻨﺪ ﻧﻬﺎﻳﺔ اﻟﺘﻔﺎﻋﻞ ﻳﺘﻔﻜﻚ آﻞ اﻟﻤﺎء اﻷآﺴﻮﺟﻴﻨﻲ ‪ ،‬وﻣﻨﻪ ‪ ، C0 V − 2 xm = 0‬وﻧﺴﺘﻨﺘﺞ ‪= 0,892 mol.L−1‬‬
‫‪V‬‬ ‫‪1‬‬
‫‪C0 0,892‬‬
‫=‪C‬‬ ‫=‬ ‫‪= 2, 23 × 10−2 mol.L−1‬‬ ‫اﻟﺘﺮآﻴﺰ اﻟﻤﻮﻟﻲ ﻟﻠﻤﺤﻠﻮل ) ‪ ( S‬هﻮ‬
‫‪F‬‬ ‫‪40‬‬
‫ﺛـﺎﻧﻴﺎ ‪:‬‬
‫‪ - 1‬اﻟﻮﺳﻴﻂ ﻳﺴﺮّع اﻟﺘﻔﺎﻋﻞ ‪ ،‬وﺑﻤﺎ أن اﻟﻮﺳﻴﻂ ﻋﺒﺎرة ﻋﻦ ﻣﺤﻠﻮل ﻣﺎﺋﻲ واﻟﻤﺎء اﻷآﺴﻮﺟﻴﻨﻲ ﻣﺤﻠﻮل ﻣﺎﺋﻲ آﺬﻟﻚ ‪ ،‬إذن اﻟﻮﺳﺎﻃﺔ ﻋﺒﺎرة‬
‫ﻋﻦ وﺳﺎﻃﺔ ﻣﺘﺠﺎﻧﺴﺔ ‪.‬‬
‫‪ - 2‬أ( اﻟﻐﺮض ﻣﻦ اﺳﺘﻌﻤﺎل اﻟﻤﺎء اﻟﺒﺎرد واﻟﺠﻠﻴﺪ هﻮ إﻳﻘﺎف ﺗﻔﻜﻚ اﻟﻤﺎء اﻷآﺴﻮﺟﻴﻨﻲ ﻣﻦ أﺟﻞ ﻣﻌﺎﻳﺮﺗﻪ ﻷن ﺗﻄﻮر اﻟﺘﻔﺎﻋﻞ ﺷﺒﻪ ﻣﻨﻌﺪم‬
‫ﻓﻲ اﻟﺒﺮودة ‪.‬‬
‫ب( ﻻ ﺳﺆال وﻻ ﺟﻮاب‬
‫‪ - 3‬أ( آﻠﻤﺎ آﺎن اﻟﺘﺮآﻴﺰ اﻟﻤﻮﻟﻲ ﻟﻠﻤﺤﻔّﺰ أآﺒﺮ آﻠﻤﺎ آﺎن اﻟﺘﻔﺎﻋﻞ أﺳﺮع وﺑﺎﻟﺘﺎﻟﻲ زﻣﻦ ﻧﺼﻒ اﻟﺘﻔﺎﻋﻞ أﻗﻞ ‪.‬‬
‫)ﻓﻲ هﺬﻩ اﻟﺘﺠﺎرب ﻧﻬﻤﻞ ﺗﺄﺛﻴﺮ اﻟﺘﺮآﻴﺰ اﻟﻤﻮﻟﻲ ﻟﻠﻤﺎء اﻷآﺴﻮﺟﻴﻨﻲ( ‪.‬‬
‫‪3‬‬
‫ﻣﻼﺣﻈﺔ ‪ :‬آﺎن ﻣﻦ اﻷﺣﺴﻦ أن ﻳﻜﻮن اﻟﺴﺆال آﻤﺎ ﻳﻠﻲ ‪ :‬ﻳﺄﺧﺬ آﻞ ﻓﻮج ﻧﻔﺲ اﻟﺤﺠﻢ ‪ V‬ﻣﻦ اﻟﻤﺎء اﻷآﺴﻮﺟﻴﻨﻲ وﻳﻀﻴﻒ ﻟﻪ ﺣﺠﻤﺎ ‪Ve‬‬
‫ﻣﻦ اﻟﻤﺎء اﻟﻤﻘﻄﺮ ‪،‬ﺛﻢ ﺣﺠﻤﺎ ‪ Vc‬ﻣﻦ اﻟﻤﺤﻔﺰ ﺑﺤﻴﺚ ﻳﻜﻮن اﻟﻤﺠﻤﻮع ‪ V + Ve + Vc‬ﻣﺘﺴﺎوﻳﺎ ﻋﻨﺪ آﻞ اﻷﻓﻮاج ‪ .‬وﻓﻲ هﺬﻩ اﻟﺤﺎﻟﺔ ﺳﻴﺘﻤﺪد‬

‫‪V + Ve + Vc‬‬
‫‪.‬‬ ‫اﻟﻤﺎء اﻷآﺴﻮﺟﻴﻨﻲ ﻃﺒﻌﺎ ‪ ،‬وﻟﻤﻌﺮﻓﺔ اﻟﺘﺮآﻴﺰ ﻗﺒﻞ هﺬﻩ اﻟﻌﻤﻠﻴﺔ ﻧﻀﺮب اﻟﺘﺮآﻴﺰ اﻟﻤﻮﺟﻮد ﻓﻲ اﻟﻤﻌﺎﻣﻞ‬
‫‪V‬‬
‫اﻟﺒﻴـﺎن )‪ ← (1‬اﻟﻔﻮج ‪C‬‬
‫اﻟﺒﻴـﺎن )‪ ← (2‬اﻟﻔﻮج ‪A‬‬
‫اﻟﺒﻴـﺎن )‪ ← (3‬اﻟﻔﻮج ‪D‬‬
‫اﻟﺒﻴـﺎن )‪ ← (4‬اﻟﻔﻮج ‪B‬‬

‫‪[ H 2O2 ] = C = 20 × 10−3 = 2 × 10−2 mol / L‬‬ ‫ب( ﻋﻨﺪ اﻟﻠﺤﻈﺔ ‪ t = 0‬ﻧﻘﺮأ ﻋﻠﻰ اﻟﺒﻴﺎن‬

‫ﺣﺴﺐ ﻣﻌﺎﻣﻞ اﻟﺘﻤﺪﻳﺪ ﻳﻜﻮن ‪C0 = 40 × C = 40 × 2 × 10−2 = 0,8mol / L‬‬

‫‪0,892 − 0,800‬‬
‫‪ ،‬أي ‪) 10%‬ﻓﻲ ﺣﺪود اﻟﻤﺴﻤﻮح ﺑﻪ( ‪ .‬ﻳﻮﺟﺪ اﺧﺘﻼف ﻣﺤﺴﻮس ﺑﻴﻦ اﻟﻨﺘﻴﺠﺘﻴﻦ ‪،‬‬ ‫ﺟـ( دﻗﺔ اﻟﻨﺘﻴﺠﺔ هﻲ ‪= 0,1‬‬
‫‪0,892‬‬
‫واﻟﺴﺒﺐ ﻳﺮﺟﻊ أﺳﺎﺳﺎ إﻟﻰ أن ﺟﺰء ﻣﻦ اﻟﻤﺎء اﻷآﺴﻮﺟﻴﻨﻲ ﻗﺪ ﺗﻔﻜّﻚ ﻗﺒﻞ ﻣﻌﺎﻳﺮﺗﻪ ‪.‬‬
‫ﻣﻼﺣﻈﺔ ‪ :‬إﺟﺎﺑﺔ اﻟﻤﺘﺮﺷﺢ ﻻ ﺗﺤﺘﺎج ﻟﻬﺬﻩ اﻟﺘﻔﺎﺻﻴﻞ ‪ ،‬ﺑﻞ ﻳﻜﻔﻲ أن ﺗﻘﻮل ‪ :‬اﻟﻨﺘﻴﺠﺘﺎن ﻣﺨﺘﻠﻔﺘﺎن ‪.‬‬

‫ﻳﺄﺗﻴﻚ ﺳﻠﻢ اﻟﺘﻨﻘﻴﻂ اﻟﻮزاري اﻟﺮﺳﻤﻲ ﻳﻮم اﻟﺨﻤﻴﺲ ﻣﺴــﺎء‬

‫‪4‬‬
‫ﺗﺼﺤﻴﺢ اﻟﺒﻜﺎﻟﻮرﻳﺎ ‪ - 2011‬اﻟﻌﻠﻮم اﻟﻔﻴﺰﻳﺎﺋﻴﺔ ‪ -‬ﺷﻌﺒﺔ اﻟﻌﻠﻮم اﻟﺘﺠﺮﻳﺒﻴﺔ‬
‫اﻟﻤﻮﺿﻮع اﻟﺜﺎﻧﻲ‬
‫اﻟﺘﻤﺮﻳﻦ اﻷول )‪ 4‬ﻧﻘﻂ(‬
‫‪ - 1‬أ( اﻟﻤﻌﺎدﻟﺘﺎن اﻟﻨﺼﻔﻴﺘﺎن ‪:‬‬
‫‪2−‬‬ ‫‪−‬‬ ‫‪+‬‬ ‫‪3+‬‬
‫‪Cr2O7‬‬ ‫‪+ 6 e + 14 H = 2Cr‬‬ ‫‪+ 7 H 2O‬‬

‫×‪3‬‬ ‫‪(C H O‬‬


‫‪2‬‬ ‫‪2‬‬ ‫‪4‬‬ ‫‪= 2CO2 + 2e − + 2 H +‬‬ ‫)‬
‫ﻣﻌﺎدﻟﺔ اﻷآﺴﺪة ‪ -‬ارﺟــﺎع ‪:‬‬

‫‪Cr2O7 2− + 3C2 H 2O4 + 8 H + = 2Cr 3+ + 6CO2 + 7 H 2O‬‬

‫ب( ﺟﺪول اﻟﺘﻘﺪّم ‪:‬‬

‫‪Cr2O7‬‬ ‫‪2−‬‬
‫‪+ 3 C2 H 2O4 + 8H‬‬ ‫‪+‬‬
‫‪= 2Cr‬‬ ‫‪3+‬‬
‫‪+ 6CO2 + 7 H 2O‬‬ ‫آﻤﻴﺔ ﻣﺎدة ‪ Cr2 O7 2 −‬هﻲ‬

‫‪0,008‬‬ ‫‪0,06 × C2‬‬ ‫ﺑﻮﻓﺮة‬ ‫‪0‬‬ ‫‪0‬‬ ‫ﺑﻮﻓﺮة‬ ‫‪C1V1 = 0, 2 × 0,040 = 0,008 mol‬‬

‫‪0,008 − x‬‬ ‫‪0,06 × C2 − 3x‬‬ ‫‪---‬‬ ‫‪2x‬‬ ‫‪6x‬‬ ‫‪---‬‬ ‫‪ - 2‬أ(‬


‫‪1‬‬
‫‪0,008 − x f‬‬ ‫‪0,06 × C2 − 3x f‬‬ ‫‪---‬‬ ‫‪2x f‬‬ ‫‪6x f‬‬ ‫‪---‬‬ ‫‪v‬‬ ‫=‬ ‫‪= 4, 4 × 10−2 mmol.mn −1‬‬
‫‪Cr 3+‬‬ ‫‪22,5‬‬
‫‪n‬‬
‫‪Cr 3 +‬‬
‫) ‪( mmol‬‬ ‫‪ ، n‬وﻣﻦ ﺟﺪول اﻟﺘﻘﺪّم ﻟﺪﻳﻨـﺎ‬
‫‪Cr 3+‬‬
‫ب( ﻟﺪﻳﻨﺎ ﻣﻦ اﻟﺒﻴﺎن ‪= 4 mmol‬‬

‫‪4‬‬
‫= ‪xf‬‬ ‫‪ ، 2 x f = n 3+‬وﻣﻨﻪ ‪= 2 mmol‬‬
‫•‬ ‫‪2‬‬ ‫‪Cr‬‬
‫•‬
‫‪xf‬‬
‫•‬ ‫= ‪ ، x‬أي هﻮ‬ ‫ﺟـ( زﻣﻦ ﻧﺼﻒ اﻟﺘﻔﺎﻋﻞ هﻮ اﻟﺰﻣﻦ اﻟﻤﻮاﻓﻖ ﻟـ‬
‫‪2‬‬
‫‪2‬‬ ‫اﻟﺰﻣﻦ اﻟﻤﻮاﻓﻖ ﻟﺘﺸﻜّﻞ ﻧﺼﻒ آﻤﻴﺔ اﻟﻤﺎدة اﻟﻨﻬﺎﺋﻴﺔ ﻟﺸﻮارد ‪. Cr 3+‬‬

‫‪t1/ 2 = 5,6 mn‬‬


‫‪1‬‬
‫‪ - 3‬أ( ﻟﻮ آﺎن ‪ Cr2O7 2−‬هﻮ اﻟﻤﺘﻔﺎﻋﻞ اﻟﻤﺤﺪّ ﻟﻮﺟﺪﻧﺎ ‪x f = 8 × 10−3 mol‬‬
‫) ‪t ( mn‬‬
‫‪5‬‬ ‫‪20‬‬ ‫أي ‪. x f = 8 mmol‬‬
‫‪5,6‬‬
‫إذن اﻟﻤﺘﻔﺎﻋﻞ اﻟﻤﺤﺪ هﻮ ﺣﻤﺾ اﻷآﺴﺎﻟﻴﻚ ‪.‬‬
‫‪3x f‬‬ ‫‪6 × 10−3‬‬
‫= ‪C2‬‬ ‫=‬ ‫‪ ، 0,06 × C2 − 3x f = 0‬وﻣﻨﻪ ‪= 0,1 mol / L‬‬ ‫ب(‬
‫‪0,06‬‬ ‫‪0,06‬‬
‫اﻟﺘﻤﺮﻳﻦ اﻟﺜﺎﻧﻲ )‪ 4‬ﻧﻘﻂ( ‪-------------------------------------------------------------------------------------------------‬‬

‫‪ - 1‬أ( رﺑﻂ راﺳﻢ اﻻهﺘﺰاز اﻟﻤﻬﺒﻄﻲ ‪ :‬ﻓﻲ اﻟﻤﺪﺧﻞ ‪ X‬ﻧﺸﺎهﺪ ) ‪ub ( t‬‬
‫‪E‬‬
‫‪K‬‬
‫ﻓﻲ اﻟﻤﺪﺧﻞ ‪ Y‬ﻧﺸﺎهﺪ ) ‪ u R ( t‬ﺑﻌﺪ ﻋﻜﺴﻪ‬

‫ﺑﻮاﺳﻄﺔ زر اﻟﻌﻜﺲ ﻋﻠﻰ راﺳﻢ اﻻهﺘﺰاز ‪.‬‬


‫‪i‬‬
‫ب( ﻟﺪﻳﻨﺎ ) ‪ ، u R ( t ) = R i ( t‬وﻧﻌﻠﻢ أن ﻋﻨﺪ اﻟﻠﺤﻈﺔ ‪ t = 0‬ﻳﻜﻮن ‪ i ( t ) = 0‬ﻷن‬
‫) ‪uR ( t‬‬ ‫) ‪ub ( t‬‬ ‫اﻟﻮﺷﻴﻌﺔ ﺗﻤﺎﻧﻊ ﺗﻐﻴّﺮ اﻟﺘﻴﺎر ‪ ،‬وﻳﺰداد هﺬا اﻷﺧﻴﺮ ﺑﻤﺮور اﻟﺰﻣﻦ ‪.‬‬

‫اﻟﺘﻨﺎﺳﺐ ﺑﻴﻦ ) ‪ u R ( t‬و ) ‪ i ( t‬ﻳﺒﻴّﻦ أن اﻟﻤﻨﺤﻨﻲ ‪ 1 -‬هﻮ اﻟﻤﻮاﻓﻖ ﻟـ ) ‪. u R ( t‬‬


‫‪R‬‬ ‫)‪(L , r‬‬
‫ﻋﻨﺪ اﻟﻠﺤﻈﺔ ‪ t = 0‬وﺣﺴﺐ ﻗﺎﻧﻮن ﺟﻤﻊ اﻟﺘﻮﺗﺮات ﻳﻜﻮن ‪ ub ( t ) = E‬ﻷن ‪. u R ( t ) = 0‬‬
‫‪Y‬‬ ‫‪X‬‬
‫اﻟﻤﻨﺤﻨﻰ ‪ 2 -‬ﻳﻮاﻓﻖ ) ‪ub ( t‬‬

‫‪1‬‬
‫‪ - 2‬أ( ﺣﺴﺐ ﻗﺎﻧﻮن ﺟﻤﻊ اﻟﺘﻮﺗﺮات ‪ub ( t ) + uR ( t ) = E‬‬

‫) ‪d i (t‬‬
‫‪( r + R ) × i (t ) + L‬‬ ‫‪=E‬‬
‫‪dt‬‬
‫) ‪d i (t‬‬ ‫‪(r + R) × i‬‬ ‫‪E‬‬
‫‪+‬‬ ‫= ) ‪(t‬‬
‫‪dt‬‬ ‫‪L‬‬ ‫‪L‬‬
‫‪E‬‬ ‫‪r+R‬‬
‫=‪B‬‬ ‫=‪ A‬و‬ ‫ب(‬
‫‪L‬‬ ‫‪L‬‬
‫) ‪d i (t‬‬ ‫‪B‬‬
‫‪ ،‬وﺑﺎﻟﺘﻌﻮﻳﺾ ﻓﻲ اﻟﻤﻌﺎدﻟﺔ اﻟﺘﻔﺎﺿﻠﻴﺔ ‪، Be − A t + B − Be − A t = B :‬‬
‫‪dt‬‬
‫‪= B e− A t‬‬ ‫و‬ ‫= ) ‪i (t‬‬
‫‪A‬‬
‫(‬
‫‪1 − e− A t‬‬ ‫)‬ ‫ﺟـ(‬

‫‪B‬‬
‫= ) ‪ i ( t‬هﻲ ﺣﻞ ﻟﻠﻤﻌﺎدﻟﺔ اﻟﺘﻔﺎﺿﻠﻴﺔ اﻟﺴﺎﺑﻘﺔ ‪.‬‬
‫‪A‬‬
‫(‬ ‫)‬
‫وﺑﺎﻟﺘﺎﻟﻲ ‪ B = B‬أي أن ‪1 − e − A t‬‬

‫‪10‬‬ ‫‪u‬‬
‫= ‪I0‬‬ ‫د( ﻓﻲ اﻟﻨﻈﺎم اﻟﺪاﺋﻢ ‪ ، I 0 = R‬وﻣﻦ اﻟﻤﻨﺤﻨﻲ ‪ 1 -‬ﻟﺪﻳﻨﺎ ﻓﻲ اﻟﻨﻈﺎم اﻟﺪاﺋﻢ ‪ ، uR = 5 × 2 = 10V‬وﺑﺎﻟﺘﺎﻟﻲ ‪= 0,1 A‬‬
‫‪100‬‬ ‫‪R‬‬
‫هـ( ﻣﻦ اﻟﻤﻨﺤﻨﻰ ‪ 2 -‬ﻟﺪﻳﻨﺎ ‪E = 2 × 6 = 12V‬‬
‫‪2‬‬
‫=‪r‬‬ ‫ﻣﻦ اﻟﻤﻨﺤﻨﻰ ‪ 2 -‬ﻟﺪﻳﻨﺎ ‪ ، rI 0 = 2V‬وﻣﻨﻪ ‪= 20 Ω‬‬
‫‪0,1‬‬
‫‪ τ‬هﻮ اﻟﺰﻣﻦ اﻟﻤﻮاﻓﻖ ﻟـ ‪uR = 0,63 × 10 = 6,3V‬‬ ‫ﻣﻦ اﻟﻤﻨﺤﻨﻰ ‪ 1 -‬ﻟﺪﻳﻨﺎ‬
‫) ‪uR (V‬‬ ‫ﻋﻠﻰ اﻟﻤﻨﺤﻨﻲ ‪ 1 -‬ﻧﻘﺮأ ‪τ = 10 ms‬‬
‫ﻳﻤﻜﻦ اﺳﺘﻌﻤﺎل اﻟﻄﺮق اﻷﺧﺮى ‪ ،‬وآﺬﻟﻚ ﻣﻦ اﻟﻤﻨﺤﻨﻲ ‪ 2 -‬ﻣﻦ‬
‫أﺟﻞ ﺣﺴﺎب ﺛﺎﺑﺖ اﻟﺰﻣﻦ ‪.‬‬

‫‪L = τ × ( R + r ) = 10 × 10−3 × 120 = 1, 2 H‬‬ ‫ﻟﺪﻳﻨﺎ‬

‫و( اﻟﻄﺎﻗﺔ اﻷﻋﻈﻤﻴﺔ اﻟﻤﺨﺰّﻧﺔ ﻓﻲ اﻟﻮﺷﻴﻌﺔ ‪:‬‬


‫‪6,3‬‬
‫‪1‬‬
‫‪L I 0 2 = 0,5 × 1, 2 × ( 0,1) = 6 × 10−3 J‬‬
‫‪2‬‬
‫= ‪Eb‬‬
‫‪2‬‬

‫‪2‬‬

‫) ‪t ( ms‬‬
‫‪5‬‬ ‫‪10‬‬
‫اﻟﺘﻤﺮﻳﻦ اﻟﺜﺎﻟﺚ )‪ 4‬ﻧﻘﻂ( ‪-------------------------------------------------------------------------------------------------‬‬

‫‪O‬‬
‫أو ‪HCOO − C2 H 5‬‬ ‫‪HC‬‬ ‫‪O‬‬ ‫‪C2H5‬‬ ‫‪ - 1‬أ( ‪ E‬ﻋﺒﺎرة ﻋﻦ أﺳﺘﺮ ﻋﻀﻮي ‪ .‬ﺻﻴﻐﺘﻪ ﻧﺼﻒ اﻟﻤﻔﺼّﻠﺔ‬

‫‪ : B‬اﻻﻳﺜـﺎﻧﻮل ‪C2 H 5 − OH ،‬‬ ‫‪،‬‬ ‫ب( ‪ : A‬ﺣﻤﺾ اﻟﻤﻴﺜﺎﻧﻮﻳﻚ ‪HCOOH ،‬‬


‫ﺟـ( آﻼهﻤﺎ ﻳﺴﺮّع اﻟﺘﻔﺎﻋﻞ ‪.‬‬
‫‪HCOOH + C2 H 5OH = HCOO − C2 H 5 + H 2O‬‬ ‫‪ - 2‬ﻣﻌﺎدﻟﺔ اﻟﺘﻔﺎﻋﻞ ‪:‬‬

‫‪ - 3‬ﺟﺪول اﻟﺘﻘﺪّم ‪:‬‬


‫‪HCOOH + C2 H 5OH = HCOO − C2 H 5 + H 2O‬‬ ‫ﺑﻤﺎ أن اﻹﻳﺜﺎﻧﻮل هﻮ آﺤﻮل أوﻟﻲ واﻟﻤﺰﻳﺞ‬
‫‪0,5‬‬ ‫‪0,5‬‬ ‫‪0‬‬ ‫‪0‬‬ ‫ﻣﺘﺴﺎوي اﻟﻤﻮﻻت ‪ ،‬إذن‬
‫‪0,5 − x‬‬ ‫‪0,5 − x‬‬ ‫‪x‬‬ ‫‪x‬‬
‫‪0,5 − x f‬‬ ‫‪0,5 − x f‬‬ ‫‪xf‬‬ ‫‪xf‬‬
‫‪67‬‬
‫‪0,5 − xm‬‬ ‫‪0,5 − xm‬‬ ‫‪xm‬‬ ‫‪xm‬‬ ‫× ‪x f = 0,5‬‬ ‫‪= 0,335 mol‬‬
‫‪100‬‬

‫‪2‬‬
‫‪[ HCOO − C2 H 5 ] f [ H 2O ] f‬‬ ‫‪nester × ne‬‬ ‫‪xf 2‬‬ ‫) ‪( 0,335‬‬
‫‪2‬‬

‫=‪K‬‬ ‫=‬ ‫=‬ ‫=‬ ‫‪≈4‬‬


‫‪[ HCOOH ] f [C2 H 5 − OH ] f‬‬ ‫‪nac × nal ( 0,5 − x‬‬
‫‪f‬‬ ‫)‬
‫‪2‬‬
‫)‪( 0,5 − 0,335‬‬
‫‪2‬‬

‫‪nester = 0,335 mol‬‬


‫‪ - 4‬أ( ﻋﻨﺪ اﻟﺘﻮازن آﺎن ﻟﺪﻳﻨﺎ‬
‫‪ne = 0,335 mol‬‬
‫‪nac = 0,5 − 0,335 = 0,165 mol‬‬
‫‪nal = 0,5 − 0,335 = 0,165 mol‬‬

‫‪nester = 0,335 mol‬‬


‫‪ne = 0,335 mol‬‬ ‫ﻋﻨﺪ إﺿﺎﻓﺔ ‪ 0,1mol‬ﻣﻦ اﻟﺤﻤﺾ ﻳﺼﺒﺢ ﻟﺪﻳﻨــﺎ ﻗﺒﻞ ﺑﺪء اﻟﺘﻔﺎﻋﻞ‬
‫‪nac = 0, 265 mol‬‬
‫)‪( 0,335‬‬
‫‪2‬‬

‫‪nal = 0,165 mol‬‬ ‫= ‪Qri‬‬ ‫‪= 2,57‬‬ ‫وﺑﺎﻟﺘﺎﻟﻲ‬


‫)‪( 0, 265 ) × ( 0,165‬‬
‫‪ Qri < K‬وﺑﺎﻟﺘﺎﻟﻲ ﻳﺆول اﻟﺘﻔﺎﻋﻞ ﻓﻲ اﻟﺠﻬﺔ اﻟﻤﺒﺎﺷﺮة ‪ ،‬أي اﺳﺘﻬﻼك اﻟﺤﻤﺾ واﻟﻜﺤﻮل وﻇﻬﻮر اﻷﺳﺘﺮ واﻟﻤﺎء ‪.‬‬

‫ب( ﺟﺪول اﻟﺘﻘﺪم ﻣﻦ أﺟﻞ اﻟﺘﻮازن اﻟﺠﺪﻳﺪ ‪:‬‬


‫‪HCOOH + C2 H 5OH = HCOO − C2 H 5 + H 2O‬‬ ‫ﻋﻨﺪ اﻟﺘﻮازن اﻟﺠﺪﻳﺪ ﻳﻜﻮن ‪:‬‬
‫‪0, 265‬‬ ‫‪0,165‬‬ ‫‪0,335‬‬ ‫‪0,335‬‬
‫‪0, 265 − x‬‬ ‫‪0,165 − x‬‬ ‫‪0,335 + x‬‬ ‫‪0,335 + x‬‬
‫‪0, 265 − x f‬‬ ‫‪0,165 − x f‬‬ ‫‪0,335 + x f‬‬ ‫‪0,335 + x f‬‬

‫) ‪( 0,335 + x‬‬
‫‪2‬‬
‫‪f‬‬
‫‪ ،‬ﻋﻨﺪﻣﺎ ﻧﺤﻞ هﺬﻩ اﻟﻤﻌﺎدﻟﺔ ﻧﺠﺪ اﻟﻘﻴﻤﺘﻴﻦ ‪ x f = 0,027 mol‬و ‪) x f = 0,77 mol‬ﻣﺮﻓﻮﺿﺔ( ‪.‬‬ ‫=‪K‬‬ ‫‪=4‬‬
‫) ‪( 0, 265 − x )( 0,165 − x‬‬
‫‪f‬‬ ‫‪f‬‬

‫‪nester = 0,335 + 0,027 = 0,362 mol‬‬ ‫اﻟﺘﺮآﻴﺐ اﻟﻤﻮﻟﻲ ﻋﻨﺪ اﻟﺘﻮازن اﻟﺠﺪﻳﺪ ﻟﻠﺠﻤﻠﺔ ‪:‬‬
‫‪ne = 0,335 + 0,027 = 0,362 mol‬‬
‫‪nac = 0, 265 − 0,027 = 0, 238 mol‬‬
‫‪nal = 0,165 − 0,027 = 0,138 mol‬‬

‫اﻟﺘﻤﺮﻳﻦ اﻟﺮاﺑﻊ )‪ 4‬ﻧﻘﻂ( ‪--------------------------------------------------------------------------------------------------‬‬


‫‪ - 1‬أ( ﻧﻤﻂ اﻹﺷﻌﺎع اﻟﻤﻮاﻓﻖ ﻟﻬﺬا اﻟﺘﺤﻮل هﻮ اﻟﻨﻤﻂ ‪. α‬‬
‫‪Z = 86 + 2 = 88‬‬ ‫‪،‬‬ ‫ب( ﺣﺴﺐ ﻗﺎﻧﻮن اﻻﻧﺤﻔﺎظ ﻟﺼﻮدي ‪A = 222 + 4 = 226 :‬‬
‫‪Δ m = Z × m p + ( A − Z ) mn − mRa = 88 × 1,007 + 138 × 1,009 − 225,977 = 1,881 u‬‬ ‫‪ - 2‬أ(‬

‫ب( اﻟﻌﻼﻗﺔ هﻲ اﻟﺘﻲ ﺗُﻌﻄﻲ ﻃﺎﻗﺔ اﻟﻜﺘﻠﺔ ‪E = m c 2 ،‬‬


‫‪ - 3‬أ( ﻃﺎﻗﺔ اﻟﺮﺑﻂ ‪ El‬هﻲ أﺻﻐﺮ ﻃﺎﻗﺔ ﻧﻘﺪّﻣﻬﺎ ﻟﻠﻨﻮاة ﻣﻦ أﺟﻞ ﻓﺼﻞ اﻟﻨﻮآﻠﻴﻮﻧﺎت ﻋﻦ ﺑﻌﻀﻬﺎ )ﺗﻔﻜﻴﻚ اﻟﻨﻮاة إﻟﻰ ﻣﻜﻮﻧﺎﺗﻬﺎ( ‪.‬‬

‫‪El‬‬ ‫‪27,36 × 10−11‬‬


‫= ‪Δm‬‬ ‫=‬ ‫‪= 1,836 u‬‬ ‫ب(‬
‫‪931,5 1,6 × 10−13 × 931,5‬‬

‫)ﻳﻤﻜﻨﻚ أن ﺗﺠﺪ ‪ Δ m‬ﺑـ ‪ kg‬ﺑﺎﻟﻌﻼﻗﺔ ‪ ، E = Δm c 2‬أو ﺑﺎﺳﺘﻌﻤﺎل اﻟﻌﻼﻗﺔ ‪. ( Δ m = Z × m p + ( A − Z ) mn − mRn‬‬

‫ﺟـ( ﻃﺎﻗﺔ اﻟﺮﺑﻂ ﻟﻜﻞ ﻧﻮﻳﺔ هﻲ أﺻﻐﺮ ﻃﺎﻗﺔ ﻻزﻣﺔ ﻟﻔﺼﻞ ﻧﻮآﻠﻴﻮن واﺣﺪ ﻣﻦ اﻟﻨﻮاة ‪.‬‬
‫‪El 27,36 × 10−11‬‬
‫=‬ ‫‪= 1, 23 × 10−12 J = 7,68 MeV‬‬
‫‪A‬‬ ‫‪222‬‬
‫‪ - 4‬أ( ﺗﻔﺎﻋﻞ اﻻﻧﺸﻄﺎر هﻮ اﻟﺘﻔﺎﻋﻞ اﻟﻨﻮوي اﻟﺬي ﻳﺘﻢ ﻓﻴﻪ ﺗﻔﺘﻴﺖ ﻧﻮاة ﺛﻘﻴﻠﺔ ﺑﻮاﺳﻄﺔ ﻧﻮﺗﺮون وﻇﻬﻮر ﻧﻮاﺗﻴﻦ أآﺜﺮ اﺳﺘﻘﺮارا ‪.‬‬

‫‪Elib = ( mi − m f ) × 931,5 = ( 234,994 − 93,894 − 138,889 − 2 × 1,009 ) × 931,5 = 179,78 MeV = 2,87 × 10−11 J‬‬ ‫ب(‬

‫‪3‬‬
‫اﻟﺘﻤﺮﻳﻦ اﻟﺘﺠﺮﻳﺒﻲ )‪ 4‬ﻧﻘﻂ(‬
‫‪r‬‬ ‫‪r‬‬
‫‪f‬‬ ‫‪f‬‬ ‫‪-1‬‬
‫‪r‬‬ ‫‪r‬‬
‫‪π •m‬‬ ‫‪π‬‬ ‫ﻣﻼﺣﻈﺔ ‪ :‬ﻳﻤﻜﻦ اﻻﺳﺘﻐﻨﺎء ﻋﻦ ﺗﻤﺜﻴﻞ اﻟﻘﻮى ﻋﻨﺪ ‪t = 0‬‬
‫‪r‬‬ ‫‪•m‬‬
‫‪π‬‬
‫‪•m‬‬

‫‪r‬‬ ‫‪r‬‬
‫‪r‬‬ ‫‪P‬‬ ‫‪P‬‬
‫‪P‬‬
‫اﻟﻨﻈﺎم اﻟﺪاﺋﻢ‬ ‫اﻟﻨﻈﺎم اﻻﻧﺘﻘﺎﻟﻲ‬ ‫ﻋﻨﺪ اﻟﻠﺤﻈﺔ ‪t = 0‬‬

‫‪ - 2‬اﻟﻤﺮﺟﻊ اﻟﺬي ﻧﺨﺘﺎرﻩ هﻮ ﻣﺮﺟﻊ ﺳﻄﺤﻲ أرﺿﻲ ‪.‬‬


‫‪r r r‬‬ ‫‪r‬‬
‫اﻟﻤﻌﺎدﻟﺔ اﻟﺘﻔﺎﺿﻠﻴﺔ ‪ :‬ﺑﺘﻄﺒﻴﻖ اﻟﻘﺎﻧﻮن اﻟﺜﺎﻧﻲ ﻟﻨﻴﻮﺗﻦ ﻓﻲ ﻣﻌﻠﻢ ﺳﻄﺤﻲ أرﺿﻲ ﻧﻌﺘﺒﺮﻩ ﻏﺎﻟﻴﻠﻴﺎ ‪P + π + f = ma :‬‬
‫‪P − π − f = ma‬‬ ‫ﺑﺎﻹﺳﻘﺎط ﻋﻠﻰ اﻟﻤﺤﻮر ‪: Oz‬‬
‫‪r‬‬ ‫‪dv‬‬
‫‪O‬‬ ‫‪f‬‬ ‫‪mg − ρ air Vg − k v 2 = m‬‬
‫‪r‬‬ ‫‪dt‬‬
‫‪π •m‬‬
‫‪dv‬‬ ‫‪⎛ ρ V⎞ k‬‬
‫‪= g ⎜ 1 − air ⎟ − v 2‬‬
‫‪dt‬‬ ‫⎝‬ ‫‪m ⎠ m‬‬

‫‪z‬‬ ‫‪r‬‬
‫‪P‬‬
‫‪dv‬‬ ‫⎞‪⎛ ρ V‬‬
‫‪ ،‬وﺑﺎﻟﺘﺎﻟﻲ اﻟﺒﻴﺎن )‪ (2‬ﻳﻮاﻓﻖ اﻟﺘﺴﺎرع ‪.‬‬ ‫‪ - 3‬أ( ﻋﻨﺪ اﻟﻠﺤﻈﺔ ‪ t = 0‬ﻟﺪﻳﻨـﺎ ‪ ، v = 0‬وﻣﻨﻪ ⎟ ‪= a = g ⎜1 − air‬‬
‫‪dt‬‬ ‫⎝‬ ‫⎠ ‪m‬‬
‫ب( اﻟﺴﺮﻋﺔ اﻟﺤﺪﻳﺔ ‪ vl‬هﻲ اﻟﺴﺮﻋﺔ ﻋﻨﺪﻣﺎ ﻳﺼﺒﺢ ‪ . a = 0‬ﻟﺪﻳﻨﺎ ﻣﻦ اﻟﺒﻴﺎن ‪vl = 2 × 4 = 8 m / s‬‬

‫ﺟـ( ﺑﺘﺮﺑﻴﻊ ﻃﺮﻓﻲ اﻟﻌﻼﻗﺔ ﻧﺠﺪ ‪( m − ρ air V ) = ⎡⎢3 × 10−3 − 1,3 × × 3,14 × 1,5 × 10−2 ⎤⎥ = 4,56 × 10−4 kg. m−1 :‬‬
‫‪g‬‬ ‫‪9,8‬‬ ‫‪4‬‬
‫(‬ ‫)‬
‫‪3‬‬
‫=‪k‬‬ ‫‪2‬‬
‫‪vl‬‬ ‫⎣ ‪64‬‬ ‫‪3‬‬ ‫⎦‬

‫ﻳﺄﺗﻴﻚ ﺳﻠﻢ اﻟﺘﻨﻘﻴﻂ اﻟﻮزاري اﻟﺮﺳﻤﻲ ﻳﻮم اﻟﺨﻤﻴﺲ ﻣﺴــﺎء‬

‫‪4‬‬
‫ﺍﻟﺼﻔﺤﺔ ‪1 :‬‬ ‫ﻣﻮﺍﺿﻴﻊ ﺍﻟﺒﻜﺎﻟﻮﺭﻳﺎ ‪ -‬ﻋﻠﻮﻡ ﻓﻴﺰﻳﺎﺋﻴﺔ ‪ -‬ﺍﻟﺸﻌﺒﺔ ‪ :‬ﻋﻠﻮﻡ ﲡﺮﻳﺒﻴﺔ ‪ -‬ﺩﻭﺭﺓ ﺟﻮﺍﻥ ‪ - 2010‬ﺍﳌﻮﺿﻮﻉ ‪ - 01‬ﺍﳊﻞ‬

‫ﺍﻷﺳﺘﺎﺫ ‪ :‬ﻓﺮﻗﺎﻧﻲ ﻓﺎﺭﺱ‬ ‫ﻋﻠﻮﻡ ﻓﻴﺰﻳﺎﺋﻴﺔ – ﺛﺎﻟﺜﺔ ﺛﺎﻧﻮﻱ – ﺍﻟﺸﻌﺐ ‪ :‬ﻋﻠﻮﻡ ﲡﺮﻳﺒﻴﺔ ‪ ،‬ﺭﻳﺎﺿﻴﺎﺕ ‪ ،‬ﺗﻘﻨﻲ ﺭﻳﺎﺿﻲ ‪.‬‬
‫ﺍﻟﺼﻔﺤﺔ ‪2 :‬‬ ‫ﻣﻮﺍﺿﻴﻊ ﺍﻟﺒﻜﺎﻟﻮﺭﻳﺎ ‪ -‬ﻋﻠﻮﻡ ﻓﻴﺰﻳﺎﺋﻴﺔ ‪ -‬ﺍﻟﺸﻌﺒﺔ ‪ :‬ﻋﻠﻮﻡ ﲡﺮﻳﺒﻴﺔ ‪ -‬ﺩﻭﺭﺓ ﺟﻮﺍﻥ ‪ - 2010‬ﺍﳌﻮﺿﻮﻉ ‪ - 01‬ﺍﳊﻞ‬

‫ﺍﻷﺳﺘﺎﺫ ‪ :‬ﻓﺮﻗﺎﻧﻲ ﻓﺎﺭﺱ‬ ‫ﻋﻠﻮﻡ ﻓﻴﺰﻳﺎﺋﻴﺔ – ﺛﺎﻟﺜﺔ ﺛﺎﻧﻮﻱ – ﺍﻟﺸﻌﺐ ‪ :‬ﻋﻠﻮﻡ ﲡﺮﻳﺒﻴﺔ ‪ ،‬ﺭﻳﺎﺿﻴﺎﺕ ‪ ،‬ﺗﻘﻨﻲ ﺭﻳﺎﺿﻲ ‪.‬‬
‫ﺍﻟﺼﻔﺤﺔ ‪3 :‬‬ ‫ﻣﻮﺍﺿﻴﻊ ﺍﻟﺒﻜﺎﻟﻮﺭﻳﺎ ‪ -‬ﻋﻠﻮﻡ ﻓﻴﺰﻳﺎﺋﻴﺔ ‪ -‬ﺍﻟﺸﻌﺒﺔ ‪ :‬ﻋﻠﻮﻡ ﲡﺮﻳﺒﻴﺔ ‪ -‬ﺩﻭﺭﺓ ﺟﻮﺍﻥ ‪ - 2010‬ﺍﳌﻮﺿﻮﻉ ‪ - 01‬ﺍﳊﻞ‬

‫ﺍﻷﺳﺘﺎﺫ ‪ :‬ﻓﺮﻗﺎﻧﻲ ﻓﺎﺭﺱ‬ ‫ﻋﻠﻮﻡ ﻓﻴﺰﻳﺎﺋﻴﺔ – ﺛﺎﻟﺜﺔ ﺛﺎﻧﻮﻱ – ﺍﻟﺸﻌﺐ ‪ :‬ﻋﻠﻮﻡ ﲡﺮﻳﺒﻴﺔ ‪ ،‬ﺭﻳﺎﺿﻴﺎﺕ ‪ ،‬ﺗﻘﻨﻲ ﺭﻳﺎﺿﻲ ‪.‬‬
‫ﺍﻟﺼﻔﺤﺔ ‪4 :‬‬ ‫ﻣﻮﺍﺿﻴﻊ ﺍﻟﺒﻜﺎﻟﻮﺭﻳﺎ ‪ -‬ﻋﻠﻮﻡ ﻓﻴﺰﻳﺎﺋﻴﺔ ‪ -‬ﺍﻟﺸﻌﺒﺔ ‪ :‬ﻋﻠﻮﻡ ﲡﺮﻳﺒﻴﺔ ‪ -‬ﺩﻭﺭﺓ ﺟﻮﺍﻥ ‪ - 2010‬ﺍﳌﻮﺿﻮﻉ ‪ - 01‬ﺍﳊﻞ‬

‫ﺍﻷﺳﺘﺎﺫ ‪ :‬ﻓﺮﻗﺎﻧﻲ ﻓﺎﺭﺱ‬ ‫ﻋﻠﻮﻡ ﻓﻴﺰﻳﺎﺋﻴﺔ – ﺛﺎﻟﺜﺔ ﺛﺎﻧﻮﻱ – ﺍﻟﺸﻌﺐ ‪ :‬ﻋﻠﻮﻡ ﲡﺮﻳﺒﻴﺔ ‪ ،‬ﺭﻳﺎﺿﻴﺎﺕ ‪ ،‬ﺗﻘﻨﻲ ﺭﻳﺎﺿﻲ ‪.‬‬
‫ﺍﻟﺼﻔﺤﺔ ‪1 :‬‬ ‫ﻣﻮﺍﺿﻴﻊ ﺍﻟﺒﻜﺎﻟﻮﺭﻳﺎ ‪ -‬ﻋﻠﻮﻡ ﻓﻴﺰﻳﺎﺋﻴﺔ ‪ -‬ﺍﻟﺸﻌﺒﺔ ‪ :‬ﻋﻠﻮﻡ ﲡﺮﻳﺒﻴﺔ ‪ -‬ﺩﻭﺭﺓ ﺟﻮﺍﻥ ‪ - 2010‬ﺍﳌﻮﺿﻮﻉ ‪ - 02‬ﺍﳊﻞ‬

‫ﺍﻷﺳﺘﺎﺫ ‪ :‬ﻓﺮﻗﺎﻧﻲ ﻓﺎﺭﺱ‬ ‫ﻋﻠﻮﻡ ﻓﻴﺰﻳﺎﺋﻴﺔ – ﺛﺎﻟﺜﺔ ﺛﺎﻧﻮﻱ – ﺍﻟﺸﻌﺐ ‪ :‬ﻋﻠﻮﻡ ﲡﺮﻳﺒﻴﺔ ‪ ،‬ﺭﻳﺎﺿﻴﺎﺕ ‪ ،‬ﺗﻘﻨﻲ ﺭﻳﺎﺿﻲ ‪.‬‬
‫ﺍﻟﺼﻔﺤﺔ ‪2 :‬‬ ‫ﻣﻮﺍﺿﻴﻊ ﺍﻟﺒﻜﺎﻟﻮﺭﻳﺎ ‪ -‬ﻋﻠﻮﻡ ﻓﻴﺰﻳﺎﺋﻴﺔ ‪ -‬ﺍﻟﺸﻌﺒﺔ ‪ :‬ﻋﻠﻮﻡ ﲡﺮﻳﺒﻴﺔ ‪ -‬ﺩﻭﺭﺓ ﺟﻮﺍﻥ ‪ - 2010‬ﺍﳌﻮﺿﻮﻉ ‪ - 02‬ﺍﳊﻞ‬

‫ﺍﻷﺳﺘﺎﺫ ‪ :‬ﻓﺮﻗﺎﻧﻲ ﻓﺎﺭﺱ‬ ‫ﻋﻠﻮﻡ ﻓﻴﺰﻳﺎﺋﻴﺔ – ﺛﺎﻟﺜﺔ ﺛﺎﻧﻮﻱ – ﺍﻟﺸﻌﺐ ‪ :‬ﻋﻠﻮﻡ ﲡﺮﻳﺒﻴﺔ ‪ ،‬ﺭﻳﺎﺿﻴﺎﺕ ‪ ،‬ﺗﻘﻨﻲ ﺭﻳﺎﺿﻲ ‪.‬‬
‫ﺍﻟﺼﻔﺤﺔ ‪3 :‬‬ ‫ﻣﻮﺍﺿﻴﻊ ﺍﻟﺒﻜﺎﻟﻮﺭﻳﺎ ‪ -‬ﻋﻠﻮﻡ ﻓﻴﺰﻳﺎﺋﻴﺔ ‪ -‬ﺍﻟﺸﻌﺒﺔ ‪ :‬ﻋﻠﻮﻡ ﲡﺮﻳﺒﻴﺔ ‪ -‬ﺩﻭﺭﺓ ﺟﻮﺍﻥ ‪ - 2010‬ﺍﳌﻮﺿﻮﻉ ‪ - 02‬ﺍﳊﻞ‬

‫ﺍﻷﺳﺘﺎﺫ ‪ :‬ﻓﺮﻗﺎﻧﻲ ﻓﺎﺭﺱ‬ ‫ﻋﻠﻮﻡ ﻓﻴﺰﻳﺎﺋﻴﺔ – ﺛﺎﻟﺜﺔ ﺛﺎﻧﻮﻱ – ﺍﻟﺸﻌﺐ ‪ :‬ﻋﻠﻮﻡ ﲡﺮﻳﺒﻴﺔ ‪ ،‬ﺭﻳﺎﺿﻴﺎﺕ ‪ ،‬ﺗﻘﻨﻲ ﺭﻳﺎﺿﻲ ‪.‬‬
‫ﺍﻟﺼﻔﺤﺔ ‪4 :‬‬ ‫ﻣﻮﺍﺿﻴﻊ ﺍﻟﺒﻜﺎﻟﻮﺭﻳﺎ ‪ -‬ﻋﻠﻮﻡ ﻓﻴﺰﻳﺎﺋﻴﺔ ‪ -‬ﺍﻟﺸﻌﺒﺔ ‪ :‬ﻋﻠﻮﻡ ﲡﺮﻳﺒﻴﺔ ‪ -‬ﺩﻭﺭﺓ ﺟﻮﺍﻥ ‪ - 2010‬ﺍﳌﻮﺿﻮﻉ ‪ - 02‬ﺍﳊﻞ‬

‫ﺍﻷﺳﺘﺎﺫ ‪ :‬ﻓﺮﻗﺎﻧﻲ ﻓﺎﺭﺱ‬ ‫ﻋﻠﻮﻡ ﻓﻴﺰﻳﺎﺋﻴﺔ – ﺛﺎﻟﺜﺔ ﺛﺎﻧﻮﻱ – ﺍﻟﺸﻌﺐ ‪ :‬ﻋﻠﻮﻡ ﲡﺮﻳﺒﻴﺔ ‪ ،‬ﺭﻳﺎﺿﻴﺎﺕ ‪ ،‬ﺗﻘﻨﻲ ﺭﻳﺎﺿﻲ ‪.‬‬
‫ﺍﻟﺼﻔﺤﺔ ‪5 :‬‬ ‫ﻣﻮﺍﺿﻴﻊ ﺍﻟﺒﻜﺎﻟﻮﺭﻳﺎ ‪ -‬ﻋﻠﻮﻡ ﻓﻴﺰﻳﺎﺋﻴﺔ ‪ -‬ﺍﻟﺸﻌﺒﺔ ‪ :‬ﻋﻠﻮﻡ ﲡﺮﻳﺒﻴﺔ ‪ -‬ﺩﻭﺭﺓ ﺟﻮﺍﻥ ‪ - 2010‬ﺍﳌﻮﺿﻮﻉ ‪ - 02‬ﺍﳊﻞ‬

‫ﺍﻷﺳﺘﺎﺫ ‪ :‬ﻓﺮﻗﺎﻧﻲ ﻓﺎﺭﺱ‬ ‫ﻋﻠﻮﻡ ﻓﻴﺰﻳﺎﺋﻴﺔ – ﺛﺎﻟﺜﺔ ﺛﺎﻧﻮﻱ – ﺍﻟﺸﻌﺐ ‪ :‬ﻋﻠﻮﻡ ﲡﺮﻳﺒﻴﺔ ‪ ،‬ﺭﻳﺎﺿﻴﺎﺕ ‪ ،‬ﺗﻘﻨﻲ ﺭﻳﺎﺿﻲ ‪.‬‬
4,H1f~J' ~,~, ~')tJ'~'
-.:,~l.ww.l',J -.:,l.a~ ~,Jl' 6t.HJJl ta.a:. 'l .ta.a .-:tt i ,.
.. ~ ",.:..J'M' J JJ
A

~jWt ~I \.tJY~ i.,~ ~~I


l .. J!.I- r# : ~I

: ~ ù:"M,...n J:II.Î ;Ca.; d c:4 >4l1 ~


e
(4l:W 20) : J. ,'il M,..l1
(ltW 04) :J. t'il ù; ;tôill
.b)1 ~ (r).).,:ll .)_;l.~-' (S 2°82-) ~ÜJ~') ~-,~I .))y!i ~ .!J~ 'i.l11 ~4411 J~I {! ~
S 2082-{a:j) + 21-(a:j) = 2S0/-(a:j) + 1 2(a:j) : ~.jb\.a...a rU Jc.tâl: ~WI
~ (t = 0) u.:..ut ~ {!jA.i , CJ.ojll4.l'J~ (e = 35°C) ~lj i)..?- 4y ~ Jc.WlII1\ .J# ~Iyl -1
Cl '~~'I
4,OxlO -2 mol / L (.,!"..r ~.c::..
oJ:r"..>J (2K + +Sps2- ) r~l,j-"I"l:i..s .•C:.\ ~ ..c: ...1 'I.J_I__
~ """-.1 ..~ Y-.J.J:!'t"..r
'
y-- (...).4 VI = lOOmL

~ C z =8,Oxl0-2 moll L ~..,..JI c.jfoji (K+ +r) r~U-*I~.,:l ~lA J~ CJ.o V 2 lOOmL ~ t'" ;;;::

,VT -2
- OOmL ~. . ~..JA U­\...
. ,- Wl\ .:Wl"4 -'.l:I.'
.'t:J\ U""'
.UJOOOQ r • ~
.. ·t IÎ

: 4.l'J~ Jc.WlI J)li.. ~yJl ~ ~~~J~-,~I ~)~ [S20 8 2


- ] ~yJl jfoji1\ i.J4t- ~Î /y
. ~yJI ~ (/2) .l.,:ll ~~ ..,.1..,..J\ jfojill [12] -' JI; ,~ 'Cl
~t~ (1=0) ~1~~~~~-,~IJJI.,.:J..,.1..,..Jljfojil' [S20 82 -Jo ~~t /4

. (1-) .l.J1.J"-'l.J
.. (S 2 ° 2-) .:l.J1 ...
.1_ Wli

.J"-'l ~ U""'
li

.
(.).4 ~W:.
.. Il , ...... " t t , t ~~·j,...i.l:..U
.J '-r . jll4.l'J.l.I·tc:'
.(.).4 • ~ .. -:.tIJ~·'1
~ i.T'~ t..r..r
1--II·,·(.'':II~l:i..J-II
J:r"..JM •
3 2 l

i..wl J)li.. ~I .:l.,:ll ~\Jj..;;W \.A~.J ~\-, .:IJ-:ÙI ç.W\.: i~~ \.A.l~.J V o =lOmL ~ JS. ~ ~yJl
JS. ~-' C' = 1,5 x 10-2 mol / L ..,lyJlojfoji (2Na+ +S2032-) r.J;!J.,-l1 ~ti.:fi.~ ~lA J~ ~I~ li
. 'l:m ~L....aWil\
."
J. a~ 1.... ' - - ~.;. .l . ,II 'tt; ç.ü:ii..'J r'....Y""
..:.r-:-IS""'" ~ ~ ~
',"'Ill r'..;r.;-'~
a.L'l ~ -II ~ÜJ·'< .. J ~\- -
-;.-~.r-~
V' ~..JA
.L ~ i

1 t(min) 0 5 10 15 20 30 45 1 60 1
V'(mL) 0 4,0 6,7 8,7 10,4 13,1 15,3 16,7 :
1
[1 2 ](mmol / L)
1 1

.~L....ayill J.J~ ~i /.l

. [I:d = J(t) ü'+.Jl ~~ ~J-' ~ f""'I) 1..0\

. (t = 20 min) u.:..u\ ~ Jc.\.i:ill ~I ~~I ~~ ~i 1"

:~I ~ 4J~yo ~I jW=IUalI ~ -1- JS..!JI ~ ~I ~4pl i .;loÙl wfo


. E = 6 V ~~ oyiji ~4..,.s oÙyo_
.C=1,2)lF~~-

c::::R~}--JC~ .R = 5 k n <ÛA",lL
.K
~",i Jiu ­
~1.i_
: ~\.ill Jlâ..j
1~ 1

.C ",R 'E ,due(t) 'ue(t) W:H~yi~I~Lil1I~t..JI~",I,~lyjil'~w~1.i~-1


dt
1
---1
.4-1 J=..S ue(t) ==E (l-e Re ) : Ô.)4-l1 J.aij ~ ~I ~Lil11 ~t..J, ûj\S w1 ~-2
.~I ,;S.ll Y~4~1 ô.)1.ill ~4 ~I 4l).l.a LA t R C .;I~I ô~'" J~ -3
:~I J~I ~ tl.foJl ~U:.::...UI ~ ue (t) ~4~' Y jill ~ y..-.l -4
o

.Ue(t) =/ (t) ~Y:l' ~I ~.;I-5


: ~I ~ ~ ~~ , C ,R ,E 4J'J~ i (t) ~~I.)y:a1 ~I i.l.tJJ ~~I ô.;I.~.11 ~",i -6
.(t (1 =0)
-l;(x)) '"

.(t -l; 00) ~ ~ ~\ , ~I ~ ~jâ..J\ ~..,.sJ1 ~, ô.)4t:- ~I -7

. 84 ,:?.;.lll Y.).J Po ~~I oj..)' ~I ~ J~t)' .).lU, ~ y.4ic f'.J:!lt)~1


t.S-""'" ~I _, ,....9 I.L ~.J: 'J Y.'U::..l ô.x:. f'~'-'-·'1·
....,-...-' _~_t ~LAWI ~i 4.F"
..,.~ ~. . -.i f' 1898 ~ ô.)A J..J1~\
~I .• -'10»1.('('..-: ~1 J.....:::ajo '\J;,j~i -"J w~ \.\..-.. . '_t_-'I ~.
- ·.t~.tl
210 f'~""~
.'~ ~ ..) y , ua."',...;- f'.J:J..,.~
:ô.;~ J~\ LA-l
y\J;,j~ _y ~ y.4ic _1
.;Pb ~ 01! ôl~..J a. w~ ~ 210 f'.J:!lt)~\ ,.slSii;_2
. Z ,A ~ JS ~ 1.l.l:It....o J,w:aWI ':?..J~I J~ ~~I Jc,Lil1ll.bt.. ~I
.". t = 0 ~I ~ 4lo ~.l.~ wi", t 1l2 = 138 j ~.~1 0 f'..J:l-lt)~1 ô\p. ~ ~j wÎ w.Jc. \~1-3
:~I 'Ao == 10 Bq
8

. (~\ ~W) ~~'iI.l.Ui1\ ~~ À /'


.t == 0 ~I ~ ~\.) i.lp..,.JI 210 f'.J:!lt)~I ~.J-Îi J.x:. N 0 /y
.f = 0 Ua:a..U1 .) ~ w\S LA if.) y..,t-. ~I ~~i J.x:. ~ ~ ~\ ~j1\ ôJ.J\ /4
(.laW 04) :e=' >1\ ô! >4l1
.'".10'90.0 (h) ttLiJ ~ r.j.J.lI:l.J~ ~ UÂ}il J~ (ms) ~ <rl.lb.....ol ~.)~
.~.lLa ~ <rI.lb.....ol'\ ytoill ~ ~.J ,( R) 1w\ jl:d ~ ô..fi UÂ.J"il .fo..i
.~ o.fo..i r.j~\ ~.J"il r.jy.yJ\ ~I ~ <rl.lb.....ol'I ytoill~.,Po L)oIIjÎÏ
Y~}il r.jy.yal\ ~ .:l~1 La -1
.. ~II r··
.~""t"" • • ~
t ~ .\..<1 ~I ' '\.ill ô l=. ~i-2
U~.J . .

. R .J h '~.J"il ~ MT' f'WI y411 ~U (G) .J. (v 2) ytoill ~~ t;:JA W:!-: ~y=J1 ô.J4tJ1 ~.Ji -3
.(v) .uc~.J (h) ~li:iJ ~i.J ~~I ytoill uJt.-4
.~ ~.J UÂ.J"i1 ~..bi.wa; l' Ij\.J c~l.ytoilll~ UÂ.J"i1 y~;;~ ~i -5
.. ûL..ba.J\
­
T :::= 24h : 1w\.J~ J~ UÂ.J"il ~.,Po .J.J.:l
R 6400km ,ms =2,Oxl0 kg, MT =5,97xl0 24kg , G=6,67xl0· 11 Nm 2 .kg·2
3

(.»w 04) :",ft:!84h tJ:J >431

:~~ (C 2HPH) J"lU:l'I.J (CH3COOH) 1i!L~U:l'1 ~ W:!-: J...,6JI ~~I J~I ~~


CH 3COOH +C 2HpH =CH 3COOC 2H s +H 20
~ 0.0 0, 2mole (j.a lil:,.o 4--;j.a ~I ~b t.,woya ,.ul ~ ~ , (j.ajll AJl'~ Jc,li:l11 .J# .t.,.;1.J.).}
10 ~ ~j.JI ~ ~.J~.:ilI.J ~)I ~ , (C 2HPH) J~\ 0.0 0, 2mole .J (CH3COOH) ~"l~Y\
. ."
~~~.J~
UY\~~ .~J"'"
~.II·UA V' ---'\ " 1... LL.~ ..
0 r-:- ~~ ...-~r.j~~'
10 iS.111'UA U JA.J.Ui.i..\ ~ul
Jo

~4Û 1 '.tA 1...:..


~I·I!. Jo~
.J. .J .,;::-. ~. .J.:lt.r- ,.
'" .. (.J&oIooIIoI

f'~.,-JI ~.J~ 0.o~LaJ):-~I~~~1 ~1..>.l\.;,.j.JJ}il y~'J1 c:::~ t =0 ~I ~


~.J~ 0.o~~L...al.;i~I~t.#,.~, C =1,0 mol.L- 1 ~yJl o~y (Na+ +OH-)
Kli :"UI .._.- '1 ô t-..J ~""lll (V' ) _.........\ (V) .. _II
. .~ ~ ~..>:. f'..J-"'" he \.:..- be f'.J:P"""""
: ~I J~I J
û\.w,\.;ill ~ ,I~.J .Jà.Î Y ~i ~ ~I .Jfo ô.lA ~

t(h) 0 4 8 12 16 20 32 40 48 60
V 'be( mL) 200 168 148 132 118 104 74 66 66 66
~I.i:ill~x(mol)

f~\ ~'J\ f"'"'!1 La Il -1


. (C 2HPH) J~\.J (CH3COOH J'. ~I W:!-: Jc,tLil\ ,..lâ:ill' ~ ~I Iy
(CH3COOH) ~"lU:YI ~ W:!-: J...,6J\ J~ ~1ûJ\ ~, Jc,tLil! ~t...c. ~\ 14

.( Na+ +OH-) ,..J:l.,-JI ~.J~ J):-.J


..;its.:ill ,.yu! L)oII\.w,'JI ~ (V 'he) -' (n) ~I ~I ~ W:!-: ~I ~i It -2
•o~t J~ .,- '1 ~
., C"I ~ U""'-li:l11 ,
Jo.
- ( X ) w. .~i
...lli . ~ J.i:il\ J....\.. . , ~~..
.. U\ ,...r-:-:: Il....l
. x =I(t) ~Y:Jl~\f"'"'!}14
Y~ I~La, t ~l.f.l1I~' ~ ~1/.:l
..u..ui
'" ~I
.. ~
10
.x f '1 • ~II
~...... ,..lill\ AJ"J.lI. U.J
..\,.,.... . ~ Q1/~"""
.-:11 A.lh ...,- U"" ...r- uc- ..>:C­ f.Jt.
'L ~t,,_ tLill ....< •
(4.lii.i 20) : ~ e..~,t4

(J.ti.l 4) :J ,':il tA:! >4l


..\.:Jt.J.l:ur..J\
'"
m n = 1,0087u ; mp ,0073u
1
c=3xI08 mS ; me =O,00055u ; lu=931MeV/C 2
:ul,)~1 ~.",ït ~ CP uyl:aa..J J.J~~) - 1

~t~' 2H
1 3H
1 42 He 14C
6
14 N
7
94Sr
38
140Xe
54
231;
92
(Ot~1 W) M (u) 2,0136 3,0155 4,0015 14,0065 14,0031 93,8945 139,8920 234,9935
E(MeV)
8,57 28,41 99,54 101,44 810,50 1164,75
(ôt.joiJt ~J UU:.)
~ ...................... t ..

E IA(MeV)
JSl ~';I ~l.b) 1,11 ...........
7,10 ...........
7,25 8,62 ............
.......... ....
,.

(ùJ:tlS~

(u) ~I i.l:!..J Iy il"lll~,) ~'1..b li :~\ ul).~a14 ..l~1 Lo -1 - 1


~ ~ y411 A..c. ~.J Z.1 A.1 mp.1 mn .1 il.".ïll ~ (m x ) (.)0 JS. ~~ ~ il.".ïl il.".ïlI.1..U ~'1..b i,)4c- ~, -2
.(C)t\~1
r.JJlI,).,,;U il.".ïll ~J ~'1..b ~\ -3
.. (MeV) Ô.l:!.",l4 235
.~UI J.J..l.:jJ\ u\t.\.;i J..6.i -4
.Jlc. , I)fol fo~~d:UI J~I ~ i,)~\0F. (.)0) il.".ïll r- La-5
:d:U1 J~I (.)0 y..au..ll ~~.J"lll û"zlp.JlI~}-II '
. I~N ""'! l!C J.M 1 1
. , , .. '1 .. \:..:. . "'. 4 H ",. 1
.~.J.J""::!t"' i.:Î..»-" UA W.J...>J-J.J 2 e ~ Y

.~.J.fo.J ' :Sr ' I~Xe ~ 0"fo. 2~;U u:i 14­


.~"j"...J Uots ~.",ï ~~ i.:Î".",ï J~ JS WC y.c- -1
6\..ùj1 ,~
... +
f 4..Jc.~1e , ~..~.
<II>,J.. .1 L"~I· '1 ~UI ~u~
4I'#... ,~tl u"zl ~
,- 0:'1 ~ -2
. (Me V) i.l:!.."l4 ~ \..ùj'll Je.\.i:i ÙA.J J..b:J'I1 Je.\.i:i (.)0 ô,)~I -U\..b.ll ~I -3

.~~ 4.à."......"..l1 il)..l.ll ~ r-) -1


: bb;'pll ~ Ji..-2
• ô,)1..l.l1 ~ ';4_)C.s.U ,)y:ill ,)"JA ~ ­
• U J UR ~
. ~.l:!.."t ­ 3
c
~\.i:ill ~t....J1 ~"i, ulyjill ~ 0.",ïti ~ .llÂic. 'i4-4
'U c 4J'i~
, U
c
= a xe bJ
:JS.,.!JI i.J4 .". ~UI ~li:ill ~t....J1 J,:.. 01-5
•• L.1~W·
. 1.....-I..P .. ~",~·"'t:ib
... ~ ~
.1a",
(lâiJ 4) :~I tif ~,
o 4..bi.l ÛA I~.l} 4l~U lb~ m =100 kg ~ ~ ..uts ~ ~ 1)

'" . ,,-~ . 1·r-­


A...û1..ûJ\. 4..c. ~U.r~.J t_ A' ~W 0 "

~) f =K II JS,!JI ÛA '+i)-}C. "'\ ~I ~-,lÂ..! ;;~ ~l <Ù:a~ ~ltil ~

.( \J.II~) Wb

4..c. yJl .6JY.l} ~I 4.11..bc. .fiJA t.J\......j (a ) ~\~ -2- JS,!JI ù4tl' ~

.( v)

~
._1 U4\.i:il\ 4.1~~1
tt:..AIIl..S~.. u0 i·~ ,0" _.~t 'l:ill'
I.,)J~I..T u~
~ .. L-:'_1
'lill ~

dv =A.v+ B : JS,!JI ~
dt
1 _••-: Le. ,"uuU ull:..J 'lll~ B ,A . t,.
.~.J. o..r.:;r-. .. u. u~
~14..c.yJ' , (g)~}JI~4J1 J4..-i~-: ~Y-i~0P-2
.(Vl)~
.ù~l0.a 4 ~i-,. )~II1\ ôÀ-, ~~ ,(~) .JI~\ 4 4J~ ~U, ~~\ jJ","û-3
.k ~!A.~~t-4
.0S;t .s;7s : ~j11 J4..J1 ~ ÛAj1I4.1Y~~l4..c.~ ~l~ 1J'0Z< ~-5

(latli "':_' Ù:j >4!


.(mol.DI)ô~)4 I.J~ C oJ:foy CH3COOH ~~~'JI ~~'LAJ~
.~WI-, 4l.Jl~YI ~ ~ J-WI ~4411 J~ c::~1 ~~\ Je.\.â:ill .6J.:It-yJSl -1

.. U\ I..T.... 1- \.â:ill - ,:-:, y ...'\.... , .. 0' -2

'u.usJI·uw
.~ r - J-r:'" i.r'ù
.(Je. \.i:ill ~~) 1: ,C 4.1)l.l} [H 30 + ] li) ..~. ~.JÎ- 3

: JS,.ji.l1 ~ (CH3COOH ICH3COO-) ~\.till (Ka) ~~I ~\!i;;j~ ~\:iS~.u' ~-4

K=­
2
1: e
a 1-'1:
:oÜlI J~I ~ ~t::üll ù~-' (C) ~~.JA J:fol3i J:..t ÛA J~ '1: 4.4 .:I~-5
1
C(mol.L- 1) X 10-2 17,8 8,77 1,78 1,08
2 ..•. ~.

r 1: {X 10- ) 1,0 1,4 3,1 4,0


1 A =lIC(L.moZ-1 }
1 B-'1: 2 11-1:

6&-5

·r~1 ~U~fi~(H+ +Cl-) ç.WI.))S~ ~'i:ilrUlI ~~I J~I.)#~~


~.".Jlo~y~WI.))S~.J.o 100 mL Ji.I.l CaC03,.~1 ~u~fi.J.o 2,Og ~~~
.C =1,Ox10-1molL- 1

tes) 20 60 100
P(CÛ2) (Pa) 2280 5560 7170
n(Cû2)(mol)
x(mol)

:~~\ 4.iJ..):J1
J-,~I ~ 4J~\ ~t'iill ûl:ac. i .J.o)1 4.l'i ~ Je, Wli k -' t.} (H +) ~-,.)~I .lJ.,..!l ~y ~ & -II
I~,_ ..
:~till
tes) 20 60 100
[H+ J<mol L- 1
) 0,080 0,056 0,040
n(H+)(mol)
x(mol)

.tl::..:=J JS ~ ~-,.)~I .l.)I..,..!l ô.lLo ~ (n(H+») ~1 -1


~I~'il ô.lWI ~-' (x) rJi:ll\ 4.l'i~ (n(H+» ~ ~\ ~yJl ôJ.~1 ~-,Î , Je,li:ilI,...li:i J-,~ Ü:!.i.tIA-2
.4.".J\ ~-,.)~I .l)y!il (no)
.~ JS t.} (x) f'..li:il\ ~ ~\ -3
f(!iij",j I~Lo x==f(t) ùt;.:lt ~1-4
•.l:r...J\ Je. w.J\ ~ - 5
t_ Wl\ . '.' ~.~ \ 6
.U'"' ~ (.)4.) t
1l2 ~ ­

.t = 50s ~I
t.} Je.li:ill ~I ~ y.J1 ~I -7
M(O)=16glmol ,M(C)=12g/mol • M(Ca)=40g/mol

-"~'''--- ~-- ...­


‫‪ ‬ﺗﺼﺤﻴﺢ‪  ‬ﺍﻣﺘﺤﺎﻥ‪ ‬ﺍﻟﺒﻜﺎﻟﻮﺭﻳﺎ‬
‫‪ ‬ﺍﻟﻌﻠﻮﻡ‪ ‬ﺍﻟﻔﻴﺰﻳــﺎﺋﻴﺔ‪  – ‬ﺷﻌﺒﺔ‪ ‬ﺍﻟﻌﻠﻮﻡ‪ ‬ﺍﻟﺘﺠﺮﻳﺒﻴﺔ‬
‫‪ ‬ﺍﻟﻤﻮﺿﻮﻉ‪ ‬ﺍﻷﻭﻝ‬
‫‪GUEZOURI Abdelkader – Lycée Maraval ­ Oran ‬‬

‫‪ ‬ﺍﻟﺘﻤﺮﻳﻦ‪ ‬ﺍﻷﻭﻝ‪  04 ) ‬ﻧﻘﻂ (‬

‫‪  ­  I‬ﺃ‪  ( ‬ﺟﺪﻭﻝ‪ ‬ﺍﻟﺘﻘﺪﻡ‬


‫‪nS O  2 - = C 1V 1  = 4 ´ 10 ´ 0 ,1 = 4 ´ 10 -3 mol‬‬
‫‪-2‬‬
‫‪2 8 ‬‬

‫‪n I - = C 2V 2  = 8 ´ 10-2 ´ 0 ,1 = 8 ´ 10 -3 mol‬‬

‫‪ ‬ﻣﻌﺎﺩﻟﺔ‪ ‬ﺍﻟﺘﻔﺎ‪ ‬ﻋﻞ‬ ‫‪2 I – (aq) ‬‬ ‫)‪+             S2O8 2 – (aq‬‬ ‫‪® ‬‬ ‫‪I2(aq)  +   2 SO4 2 – ‬‬


‫‪ ‬ﺣﺎﻟﺔ‪ ‬ﺍﻟﺠﻤﻠﺔ‬ ‫‪ ‬ﺍﻟﺘﻘﺪﻡ‬ ‫‪ ‬ﻛﻤﻴﺔ‪ ‬ﺍﻟﻤـــﺎﺩﺓ ‪( mol ) ‬‬
‫‪ ‬ﺍﻟﺤﺎﻟﺔ ‪ ‬ﺍﻻﺑﺘﺪﺍﺋﻴﺔ‬ ‫‪0 ‬‬ ‫‪8 ´ 10 –3 ‬‬ ‫‪4 ´ 10 –3 ‬‬ ‫‪0 ‬‬ ‫‪0 ‬‬

‫‪ ‬ﺍﻟﺤﺎﻟﺔ ‪ ‬ﺍﻹﻧﺘﻘﺎﻟﻴﺔ‬ ‫‪x‬‬ ‫‪8 ´ 10-3  - 2 x‬‬ ‫‪4 ´ 10 -3 1  - x‬‬ ‫‪x ‬‬ ‫‪2x ‬‬

‫‪ ‬ﺍﻟﺤﺎﻟﺔ‪ ‬ﺍﻟﻨﻬﺎﺋﻴﺔ‬ ‫‪x f ‬‬ ‫‪8 ´ 10 -3  - 2 x f ‬‬ ‫‪8 ´ 10 -3  - x f ‬‬ ‫‪x f ‬‬ ‫‪2 x f ‬‬

‫‪C V - x C1V 1 ‬‬ ‫‪x ‬‬


‫‪( 1 ) ‬‬ ‫‪éë S2O 8 2 - ùû = 1 1‬‬ ‫=‬ ‫‪-‬‬ ‫‪ ‬ﺏ‪ ( ‬ﻟﺪﻳﻨﺎ‪  ٬  nS O 2 - = C1 V1  - x : ‬ﺃﻱ‬
‫‪V1 + V2 V1 + V2 V1 + V2 ‬‬ ‫‪2 8 ‬‬

‫‪x ‬‬
‫‪ ٬ ‬ﻭﺑﺎﻟﺘﻌﻮﻳﺾ‪ ‬ﻓﻲ‪ ‬ﺍﻟﻌﻼﻗﺔ ‪ ( 1 ) ‬ﻧﺠﺪ‬ ‫=‪[ I 2 ] ‬‬ ‫‪  ٬‬ﺃﻱ‬ ‫‪nI 2  = x‬‬
‫‪V1 + V2 ‬‬
‫‪C V - x C1V 1 ‬‬
‫‪éë S 2O8 2 - ùû = 1 1‬‬ ‫=‬ ‫‪- [ I 2 ] ‬‬
‫‪V1 + V2 V1 + V2 ‬‬
‫‪C V ‬‬ ‫‪4 ´ 10-2  ´ 0,1 ‬‬
‫= ‪éëS 2O 8 2- ùû = 1 1 ‬‬ ‫‪= 2 ´10 -2 mol . L -1 ‬‬ ‫‪ ‬ﺟـ‪( ‬‬
‫‪V 1 + V 2 ‬‬ ‫‪0, 2 ‬‬

‫‪  ­ II‬ﺃ‪ ( ‬ﻧﺒﺮّﺩ‪ ‬ﺍﻟﻤﺰﻳﺞ‪ ‬ﻗﺒﻞ‪ ‬ﺍﻟﻤﻌﺎﻳ‪ ‬ﺮﺓ‪ ‬ﻟﺘﻮﻗﻴﻒ‪ ‬ﺍﻟﺘﻔﺎﻋﻞ‪  ) ‬ﺃﻱ‪ ‬ﺍﻟﻤﺤﺎﻓﻈﺔ‪ ‬ﻋﻠﻰ‪ ‬ﻛﻤﻴﺔ‪ ‬ﺛﻨﺎﺋﻲ‪ ‬ﺍﻟﻴﻮﺩ ‪ I2 ‬ﻟﺤﻈﺔ‪ ‬ﺃﺧﺬ‪ ‬ﺍﻟﻌﻴّﻨﺔ‪ ٬  ( ‬ﻷﻥ‪ ‬ﻫﺬﺍ‪ ‬ﺍﻟﺘﻔﺎﻋﻞ‪ ‬ﺑﻄﻲء‪ ‬ﺟﺪﺍ‬


‫‪ ‬ﻓﻲ‪ ‬ﺩﺭﺟﺔ‪ ‬ﺣﺮﺍﺭﺓ‪ ‬ﻣﻨﺨﻔ‪ ‬ﻀ‪ ‬ﺔ‪. ‬‬
‫‪ ‬ﺏ‪ ( ‬ﺍﻟﻤﻌﺎﺩﻟﺘﺎﻥ‪ ‬ﺍﻟﻨﺼﻔﻴﺘﺎﻥ ‪: ‬‬

‫‪GUEZOURI ‬‬ ‫‪I2  + 2 e –  =  2 I – ‬‬


‫‪Lycée Maraval ‬‬ ‫‪2 S2O3 2 –  =  S4O6 2 –  + 2 e – ‬‬
‫‪Oran‬‬
‫‪ ‬ﻣﻌﺎﺩﻟﺔ‪ ‬ﺍﻷﻛﺴﺪﺓ‪ – ‬ﺇﺭﺟــﺎﻉ‪: ‬‬
‫‪I2  +  2‬‬ ‫– ‪S2O3 2‬‬ ‫‪®  S4O6 2 –  + 2 I – ‬‬
‫‪ ‬ﻣﻼﺣﻈﺔ ‪  :  1 ‬ﺃﻛﺜﺮ‪ ‬ﺍﻟﻈﻦ‪ ‬ﺃﻥ‪ ‬ﺍﻟﺴﺆﺍﻝ‪ ‬ﻳﻘﺼﺪ ‪ ‬ﻣﻌﺎﺩﻟﺔ‪ ‬ﺍﻷﻛﺴﺪﺓ ‪  – ‬ﺇﺭﺟﺎﻉ ‪ ٬ ‬ﺃﻣﺎ‪ ‬ﺍﻟﻤﻌﺎﺩﻟﺔ ‪ ‬ﺍﻹﺟﻤﺎﻟﻴﺔ ‪ ‬ﻫﻲ‪ ‬ﺍﻟﻤﻌﺎﺩﻟﺔ‪ ‬ﺍﻟﺘﻲ‪ ‬ﻧُﺪﺧﻞ‪ ‬ﻓﻴﻬﺎ‪ ‬ﺍﻟﺸﻮﺍﺭﺩ‪ ‬ﻏﻴﺮ‬
‫‪ ‬ﺍﻟﻔﻌﺎﻟﺔ‪ ٬ ‬ﺣﻴﺚ‪ ‬ﺃﻥ‪ ‬ﻓﻲ‪ ‬ﺣﺎﻟﺘﻨﺎ‪ ‬ﻫﺬﻩ ‪ ‬ﺍﻟﺸﺎﺭﺩﺓ‪ ‬ﻏﻴﺮ‪ ‬ﺍﻟﻔﻌّﺎﻟﺔ‪ ‬ﻫﻲ ‪Na + ‬‬
‫‪I2  +  2 ( 2 Na +  , S2O3 2­ ) ®  ( 2 Na +  , S4O6 2­ )  +  2 ( Na +  , I ­ ) ‬‬
‫‪ ‬ﻣﻼﺣﻈﺔ ‪ : 2 ‬ﺍﻹﺟﺎﺑﺔ ‪ ‬ﺻﺤﻴﺤﺔ‪ ‬ﺳﻮﺍء‪ ‬ﺍﻟﺘﻠﻤﻴﺬ‪ ‬ﻛﺘﺐ‪ ‬ﻣﻌﺎﺩﻟﺔ‪ ‬ﺍﻷﻛﺴﺪﺓ‪ – ‬ﺇﺭﺟﺎﻉ ‪ ‬ﺃﻭ‪ ‬ﺍﻟﻤﻌﺎﺩﻟﺔ‪ ‬ﺍﻹﺟﻤﺎﻟﻴﺔ‪. ‬‬

‫‪1 ‬‬
‫‪ ‬ﺟـ‪  ( ‬ﺟﺪﻭﻝ‪ ‬ﺗﻘﺪّﻡ‪ ‬ﺗﻔﺎﻋﻞ‪ ‬ﺍﻟﻤﻌﺎﻳﺮﺓ ‪: ‬‬
‫‪ ‬ﻣﻌﺎﺩﻟﺔ‪ ‬ﺍﻟﺘﻔﺎﻋﻞ‬ ‫‪2 S2O3 2 – (aq) ‬‬ ‫)‪+              I2(aq‬‬ ‫‪® ‬‬ ‫‪2 I – (aq)  +    S4O6 2 – (aq) ‬‬
‫‪ ‬ﺣﺎﻟﺔ‪ ‬ﺍﻟﺠﻤﻠﺔ‬ ‫‪ ‬ﺍﻟﺘﻘﺪﻡ‬ ‫‪ ‬ﻛﻤﻴﺔ‪ ‬ﺍﻟﻤـــﺎﺩﺓ ‪( mol ) ‬‬

‫‪ ‬ﺍﻟﺤﺎ‪ ‬ﻟﺔ ‪ ‬ﺍﻻﺑﺘﺪﺍﺋﻴﺔ‬ ‫‪0 ‬‬ ‫‪C’V’ ‬‬ ‫‪[ I 2 ] ´ V0 ‬‬ ‫‪0‬‬ ‫‪0 ‬‬

‫‪ ‬ﺍﻟﺤﺎﻟﺔ‪ ‬ﺍﻟﻨﻬﺎﺋﻴﺔ‬ ‫‪xE‬‬ ‫‪C’V’ – 2  x E ‬‬ ‫‪[ I 2 ] ´ V0 –  x E ‬‬ ‫‪= 0 ‬‬ ‫‪2  x E ‬‬ ‫‪x E ‬‬

‫‪C 'V  ' ‬‬


‫‪(1 ) ‬‬ ‫= ‪xE ‬‬ ‫‪ ‬ﻋﻨﺪ‪ ‬ﺍﻟﺘﻜﺎﻓﺆ‪ ‬ﻳﻜﻮﻥ‪ ٬  C 'V '- 2 xE  = 0  : ‬ﻭﻣﻨﻪ‬
‫‪2 ‬‬
‫‪1 C 'V ' ‬‬
‫=‪[ I 2 ] ‬‬ ‫‪ ٬ ‬ﻭﺑﺘﻌﻮﻳﺾ‪ ‬ﻋﺒﺎﺭﺓ ‪  x E ‬ﻣﻦ‪ ‬ﺍﻟﻌﻼﻗﺔ ‪  ( 1 ) ‬ﻧﺠﺪ ‪:‬‬ ‫‪[ I 2 ] ´ V0  - xE  = 0 ‬‬
‫‪2  V0 ‬‬

‫'‪C ‬‬ ‫‪1,5 ´ 10 -2 ‬‬


‫=‪[ I 2 ] ‬‬ ‫= '‪´V‬‬ ‫‪´ V ' = 0, 75 ´ V ' ‬‬ ‫‪ ‬ﺩ(‬
‫‪2V0 ‬‬ ‫‪2 ´ 10 ´10 -3 ‬‬

‫‪ ‬ﺇﺫﺍ‪ ‬ﻋﻮّﺿﻨﺎ ‪ ‬ﻗﻴﻢ ‪  V’ ‬ﺑـ ‪ mL ‬ﻧﺠﺪ ‪  [ I 2 ] ‬ﺑـ ‪.  mmol /L ‬‬

‫‪t (min) ‬‬ ‫‪0 ‬‬ ‫‪5 ‬‬ ‫‪10 ‬‬ ‫‪15 ‬‬ ‫‪20 ‬‬ ‫‪30 ‬‬ ‫‪45 ‬‬ ‫‪60 ‬‬


‫‪V’ (mL) ‬‬ ‫‪0 ‬‬ ‫‪4 ‬‬ ‫‪6,7 ‬‬ ‫‪8,7 ‬‬ ‫‪10,4 ‬‬ ‫‪13,1 ‬‬ ‫‪15,3 ‬‬ ‫‪16,7 ‬‬

‫‪[ I 2 ] ( mmol / L ) ‬‬ ‫‪0‬‬ ‫‪3,0 ‬‬ ‫‪5,0 ‬‬ ‫‪6,5 ‬‬ ‫‪7,8 ‬‬ ‫‪9,8 ‬‬ ‫‪11,5 ‬‬ ‫‪12,5 ‬‬

‫‪[ I 2 ] ( mmol / L ) ‬‬ ‫‪ ‬ﻫـ‪ ( ‬ﺭﺳﻢ‪ ‬ﺍﻟﺒﻴﺎﻥ‪: ‬‬

‫‪ ‬ﺍﻟﺴﺮﻋﺔ‪ ‬ﺍﻟﺤﺠﻤﻴّ‪ ‬ﺔ‪ ‬ﻟﻠﺘﻔﺎﻋﻞ ‪: ‬‬
‫‪ ‬‬ ‫‪ ‬ﻭ‪( ‬‬
‫‪1  d x ‬‬
‫‪N ‬‬ ‫=‪v ‬‬ ‫´‬
‫‪· ‬‬ ‫‪VT  dt‬‬

‫‪5 · ‬‬
‫‪ ‬ﺣﻴﺚ ‪  x ‬ﻫﻮ‪ ‬ﻛﻤﻴﺔ‪ ‬ﻣﺎﺩﺓ‬
‫‪ ‬ﺛﻨ‪ ‬ﺎﺋﻲ‪ ‬ﺍﻟﻴﻮﺩ‪ ‬ﻓﻲ‪ ‬ﺍﻟﻠﺤﻈﺔ ‪. t ‬‬
‫‪ ‬ﻭﻟﺪﻳﻨﺎ ‪:‬‬
‫·‬ ‫·‬ ‫·‬ ‫·‬ ‫·‬
‫‪10‬‬ ‫‪20 ‬‬ ‫‪30 ‬‬ ‫‪40 ‬‬ ‫‪60‬‬ ‫)‪t (mn‬‬
‫‪x = n ( I 2 ) = [ I 2 ] ´ VT ‬‬

‫= ‪v‬‬
‫‪1 d‬‬
‫´‬
‫= ) ‪([ I ] ´V ‬‬
‫‪2 ‬‬ ‫‪T ‬‬ ‫‪1 ‬‬
‫´ ‪´V T ‬‬
‫‪d [ I 2 ] d [ I 2 ] ‬‬
‫=‬ ‫‪ ‬ﻭﺑﺎﻟﺘﺎﻟﻲ ‪:‬‬
‫‪VT‬‬ ‫‪dt‬‬ ‫‪V T ‬‬ ‫‪dt‬‬ ‫‪dt‬‬

‫‪d [ I 2 ]  11 ‬‬ ‫‪d [ I 2 ] ‬‬


‫=‪v‬‬ ‫=‬ ‫‪ ‬ﻫﻮ‪ ‬ﻣﻴﻞ‪ ‬ﺍﻟﻤﻤﺎﺱ‪ ‬ﻓﻲ‪ ‬ﺍﻟﻨﻘﻄﺔ ‪ ٬ N ‬ﺃﻱ ‪= 0, 24 mmol. L-1. mn -1 ‬‬
‫‪dt‬‬ ‫‪46 ‬‬ ‫‪dt ‬‬
‫‪GUEZOURI ‬‬
‫‪Lycée Maraval ‬‬
‫‪Oran‬‬

‫‪2 ‬‬
‫‪ ‬ﺍﻟﺘﻤﺮﻳﻦ‪ ‬ﺍﻟﺜﺎﻧﻲ‪ 4 ) ‬ﻧﻘﻂ‪( ‬‬

‫‪  – 1 ‬ﺍﻟﻤﻌﺎﺩﻟﺔ‪ ‬ﺍﻟﺘﻔﺎﺿﻠﻴﺔ ‪:‬‬


‫)‪E = u C (t ) + U R = u C  (t ) + Ri (t ‬‬
‫)‪dq (t ‬‬ ‫‪d éCu C  (t ) ùû‬‬
‫‪E = u C (t ) + R‬‬ ‫‪= uC  (t ) + R  ë‬‬
‫‪dt‬‬ ‫‪dt ‬‬
‫‪du (t ) ‬‬
‫‪E = u C  (t ) + RC  C ‬‬
‫‪dt‬‬
‫)‪duC  (t ‬‬ ‫‪1 ‬‬ ‫‪E ‬‬
‫‪( 1 ) ‬‬ ‫‪+‬‬ ‫=‪u C  (t ) ‬‬
‫‪dt‬‬ ‫‪RC‬‬ ‫‪RC‬‬

‫‪duC  (t )  E  - RC ‬‬


‫‪1 ‬‬
‫‪t ‬‬ ‫‪æ‬‬ ‫‪-‬‬
‫‪1‬‬
‫‪tö‬‬ ‫‪-‬‬
‫‪1 ‬‬
‫‪t ‬‬
‫=‬ ‫‪e ‬‬ ‫‪ ­ 2 ‬ﻟﺪﻳﻨﺎ ‪ ٬ uC  (t ) = E ç 1 - e RC ÷ = E - Ee RC ‬ﻭﺑﺎﺷﺘﻘﺎﻕ ‪ u C  (t ) ‬ﺑﺎﻟﻨﺴﺒﺔ‪ ‬ﻟﻠﺰﻣﻦ‪ ‬ﻧﺠﺪ‬
‫‪dt‬‬ ‫‪RC‬‬ ‫‪è‬‬ ‫‪ø‬‬
‫‪E‬‬ ‫‪E ‬‬ ‫‪E - RC1 t E‬‬ ‫‪1 ‬‬
‫‪E - RC ‬‬ ‫‪t ‬‬ ‫‪E ‬‬
‫‪  ٬ ‬ﻭﺑﺎﻟﺘﺎﻟﻲ ‪ ‬ﺍﻟﻤﻌﺎﺩﻟﺔ‬ ‫=‬ ‫‪  ٬ ‬ﻭﺑﺎﻻﺧﺘﺰﺍﻝ ‪   ‬ﻧﺠﺪ‬ ‫‪e‬‬ ‫‪+‬‬ ‫‪-‬‬ ‫‪e ‬‬ ‫=‬ ‫‪ ‬ﻧﻌﻮّﺽ ‪ ‬ﻓﻲ ‪ ‬ﺍﻟﻤﻌﺎﺩﻟﺔ ‪:  ( 1 ) ‬‬
‫‪RC RC‬‬ ‫‪RC‬‬ ‫‪RC RC‬‬ ‫‪RC‬‬
‫‪ ‬ﺍﻟﺘﻔﺎﺿﻠﻴﺔ‪ ‬ﺍﻟﺴﺎﺑﻘﺔ‪ ‬ﺗﻘﺒﻞ‪ ‬ﺍﻟﺤﻞ‪ ‬ﺍﻟﻤﻘﺘﺮﺡ ‪. ‬‬
‫· ‪ ‬ﻭﺣﺪﺓ‪ ‬ﺍﻟﻤﻘﺪﺍﺭ ‪ :  RC ‬ﻧﻘﻮﻡ‪ ‬ﺑﺘﺤﻠﻴﻞ‪ ‬ﺑُﻌﺪﻱ‪ ‬ﻟﻬﺬﺍ‪ ‬ﺍﻟﻤﻘﺪﺍﺭ‬ ‫‪­ 3 ‬‬

‫= ] ‪ ٬  [ RC‬ﻭﺑﺎﻟﺘﺎﻟﻲ‪ ‬ﻭﺣﺪﺓ‪ ‬ﺍﻟﻤﻘﺪﺍﺭ‪ ‬ﻫﻲ‪ ‬ﺍﻟﺜﺎﻧﻴﺔ ‪(s ) ‬‬


‫‪[U ] ´ [Q ] = [U ] ´ [ I ] ´ [T  ] = T ‬‬
‫‪[ ] ‬‬
‫] ‪[ I ] [U ] [ I ] [U‬‬
‫‪ ‬ﺍﻟﻤﺪﻟﻮﻝ‪ ‬ﺍﻟﻌﻤﻠﻲ‪ : ‬ﻫﻮ‪ ‬ﻣﺆﺷّﺮ‪ ‬ﻟﻤﺪﺓ‪ ‬ﺍﻟﻨﻈﺎﻡ‪ ‬ﺍﻻﻧﺘﻘﺎﻟﻲ‪ ‬ﺃﺛﻨﺎء‪ ‬ﺷﺤﻦ‪ ‬ﺃﻭ‪ ‬ﺗﻔﺮﻳﻎ‪ ‬ﻣﻜﺜﻔﺔ ‪.‬‬ ‫·‬

‫· ‪ ‬ﺍﺳﻤﻪ‪ : ‬ﺛﺎﺑﺖ‪ ‬ﺍﻟﺰﻣﻦ )‪. ( t‬‬
‫‪  ­ 4 ‬ﻟﺪﻳﻨﺎ ‪t = RC = 5000 ´1, 2 ´10-6  = 6 ms‬‬
‫‪1 ‬‬
‫‪æ‬‬ ‫‪- ´0  ö‬‬
‫‪t = 0 Þ u C  = 6 ç1 - e‬‬
‫‪ç‬‬
‫‪t‬‬
‫‪÷÷ = 6 (1 - 1) = 0 ‬‬
‫‪è‬‬ ‫‪ø ‬‬
‫‪1 ‬‬
‫‪æ‬‬ ‫‪- ´t ö‬‬ ‫‪æ 1 ö‬‬
‫‪t = t Þ u C  = 6 ç1 - e‬‬ ‫‪t‬‬
‫‪    ٬ ‬ﻭﻫﻜﺬﺍ‪... ‬‬
‫‪ç‬‬ ‫‪÷÷ = 6 ç1 - ÷ = 3, 7 V ‬‬
‫‪è‬‬ ‫‪ø‬‬ ‫‪è eø‬‬

‫‪t (ms)  0  6 ‬‬ ‫‪12  18  24 ‬‬

‫‪t (t) ‬‬ ‫‪0 ‬‬ ‫‪1 ‬‬ ‫‪2 ‬‬ ‫‪3 ‬‬ ‫‪4 ‬‬


‫‪uC (V)  0  3,7  5,2  5,7  5,9 ‬‬

‫‪GUEZOURI ‬‬
‫‪Lycée Maraval ‬‬
‫‪Oran‬‬

‫‪3 ‬‬
‫‪ – 5 ‬ﺍﻟﺒﻴﺎﻥ ‪uC  = f (t ) ‬‬
‫‪uC  (V) ‬‬

‫‪1 ‬‬

‫‪3 ‬‬ ‫‪t (ms) ‬‬

‫‪– 6 ‬‬
‫)‪du C  (t ‬‬ ‫‪1 ‬‬
‫‪æ E  - RC ‬‬ ‫‪t  ö‬‬
‫‪i (t ) = C‬‬ ‫‪=C ç‬‬ ‫‪e ‬‬ ‫÷‬
‫‪dt‬‬ ‫‪è RC ‬‬ ‫‪ø‬‬
‫‪1 ‬‬
‫‪E  - RC ‬‬ ‫‪t ‬‬
‫‪i (t ) = e ‬‬
‫‪R‬‬
‫‪E -t1 ´¥ E ‬‬ ‫‪E‬‬ ‫‪E ‬‬
‫‪i = e ‬‬ ‫= ‪    ٬  i = e 0 ‬ﻭﻋﻨﺪﻣﺎ ‪= ´ 0 = 0  :  t ® ¥‬‬ ‫‪ ‬ﻓﻲ‪ ‬ﺍﻟﻠﺤﻈﺔ ‪:  t = 0 ‬‬
‫‪R‬‬ ‫‪R‬‬ ‫‪R‬‬ ‫‪R‬‬
‫‪1 ‬‬ ‫‪2 ‬‬
‫‪ – 7 ‬ﺍﻟﻄﺎﻗﺔ‪ ‬ﺍﻟﻜﻬﺮﺑﺎﺋﻴﺔ ‪E e = C ëé u C  (t ) ûù :‬‬
‫‪2 ‬‬
‫‪1 ‬‬
‫‪1 ‬‬ ‫‪æ‬‬ ‫‪- ´¥ ö‬‬
‫‪E e  = CE 2 = 0,5 ´ 1, 2 ´ 10 -6  ´ 36 ‬‬ ‫‪ ٬  uC  = E ç 1 - e‬ﻭﺑﺎﻟﺘﺎﻟﻲ‬ ‫‪t‬‬
‫‪ ‬ﻋﻨﺪﻣﺎ ‪  t ® ¥‬ﻳﻜﻮﻥ ‪÷ = E‬‬
‫‪2 ‬‬ ‫‪è‬‬ ‫‪ø‬‬
‫‪Ee  = 21,6 mJ ‬‬

‫‪ ‬ﺍﻟﺘﻤﺮﻳﻦ‪ ‬ﺍﻟﺜﺎﻟﺚ‪ 4 ) : ‬ﻧﻘﻂ‪( ‬‬

‫‪  ­ 1 ‬ﺃ‪ ( ‬ﻋﻨﺼﺮ‪ ‬ﻣﺸﻊ‪ : ‬ﻫﻮ‪ ‬ﻋﻨﺼﺮ‪ ‬ﺇﺣﺪﻯ‪ ‬ﺫﺭﺍﺗﻪ‪ ‬ﺃﻭ‪ ‬ﺃﻛﺜﺮ‪ ‬ﻏﻴﺮ‪ ‬ﻣﺴﺘﻘﺮّﺓ‪ ٬ ‬ﺗﺘﺤﻠّﻞ‪ ‬ﻧﻮﺍﺗﻬﺎ‪ ‬ﺗﻠﻘﺎﺋﻴﺎ‪ ‬ﺑﻮﺍﺳﻄﺔ‪ ‬ﺗﺤﻮّﻝ‪ ‬ﻧﻮﻭﻱ‪ ‬ﺇﻟﻰ‪ ‬ﺃﻧﻮﻳﺔ‪ ‬ﺃﺧﺮﻯ‪. ‬‬
‫‪ ‬ﺏ‪ ( ‬ﻟﻠﻌﻨﺼﺮ‪ ‬ﻧﻈﺎﺋﺮ‪ : ‬ﺃﻱ‪ ‬ﺃﻥ‪ ‬ﻫﻨﺎﻙ‪ ‬ﻣﺠﻤﻮﻋﺔ‪ ‬ﻣﻦ‪ ‬ﺍﻟﺬﺭ‪ ‬ﺍﺕ‪ ‬ﺗﻨﺘﻤﻲ‪ ‬ﻟﻨﻔﺲ‪ ‬ﺍﻟﻌﻨﺼﺮ‪ ٬ ‬ﻛﻠﻬﺎ‪ ‬ﻟﻬﺎ‪ ‬ﻧﻔﺲ‪ ‬ﺍﻟﺮﻗﻢ‪ ‬ﺍﻟﺬﺭﻱ ‪ Z ‬ﻭﺗﺨﺘﻠﻒ‪ ‬ﻓﻲ‪ ‬ﺍﻟﻌﺪﺩ‪ ‬ﺍﻟﻜﺘﻠﻲ ‪A ‬‬
‫‪210‬‬ ‫‪A ‬‬
‫‪84‬‬ ‫® ‪Po‬‬ ‫‪Pb +  2 4 He‬‬
‫‪Z ‬‬ ‫‪­  2 ‬‬
‫‪  ٬  210 = A + 4 ‬ﻭﻣﻨﻪ ‪A = 206 ‬‬
‫‪GUEZOURI ‬‬
‫‪Lycée Maraval ‬‬
‫‪Z = 82 ‬‬ ‫‪  ٬  84 = Z + 2 ‬ﻭﻣﻨﻪ‬
‫‪Oran‬‬

‫‪ln 2‬‬ ‫‪0, 69 ‬‬


‫=‪l‬‬ ‫=‬ ‫‪= 5,8 ´10 -8 s -1 ‬‬ ‫‪  ­ 3 ‬ﺃ‪( ‬‬
‫‪t 1/ 2  138 ´ 86400 ‬‬

‫‪4 ‬‬
‫‪A 0 ‬‬ ‫‪10 8 ‬‬
‫= ‪N 0 ‬‬ ‫=‬ ‫‪-8 ‬‬
‫‪= 1,7 ´ 10 15 ‬‬ ‫‪ ‬ﺏ‪( ‬‬
‫‪l‬‬ ‫‪5,8 ´ 10 ‬‬
‫‪1 ‬‬ ‫‪N ‬‬
‫‪e - l t  = ‬‬ ‫‪ ‬ﺟـ‪  ٬  N = N 0 e - l t  ( ‬ﻭﻟﺪﻳﻨﺎ ‪ ٬  N = 0 ‬ﻭﻣﻨﻪ‬
‫‪4 ‬‬ ‫‪4 ‬‬
‫‪ln 4‬‬ ‫‪1,38 ‬‬
‫‪ ‬ﺍﻟﺰﻣﻦ‪ ‬ﺍﻟﻼﺯﻡ‪ ‬ﻫﻮ ‪t = 275 j‬‬ ‫‪. ‬‬ ‫= ‪t‬‬ ‫=‬ ‫‪-8 ‬‬
‫‪= 0, 24 ´ 10 8  s‬‬
‫‪l‬‬ ‫‪5,8 ´ 10 ‬‬

‫‪ ‬ﺍﻟﺘﻤﺮﻳﻦ‪ ‬ﺍﻟﺮﺍﺑﻊ‪ 4 ) ‬ﻧﻘﻂ‪( ‬‬

‫‪  – 1 ‬ﺍﻟﻤﻌﻠﻢ‪ ‬ﺍﻟﻤﺮﻛﺰﻱ‪ ‬ﺍﻷﺭﺿﻲ‪ ‬ﻫﻮ‪ ‬ﺍﻟﻤﻌﻠﻢ‪ ‬ﺍﻟﺬﻱ‪ ‬ﻣﺒﺪﺅﻩ‪ ‬ﻣﺮﻛﺰ‪ ‬ﺍﻷﺭﺽ‪ ‬ﻭﻣﺤﺎﻭﺭﻩ‪ ‬ﺍﻟﺜﻼﺛﺔ‪ ‬ﻣﺘﺠﻬﺔ‪ ‬ﻧﺤﻮ‪ ‬ﺛﻼﺛﺔ‪ ‬ﻧﺠﻮﻡ‪ ‬ﺛﺎﺑﺘﺔ ‪. ‬‬


‫‪T  2 ‬‬
‫‪  ٬ ‬ﺣﻴﺚ ‪  : T ‬ﻫﻮ‪ ‬ﺩﻭﺭ‪ ‬ﺣﺮﻛﺔ‪ ‬ﺍﻟﻘﻤﺮ‪ ‬ﺍﻟﺼﻨﺎﻋﻲ‪ : r  ٬  ‬ﺍﻟﺒﻌﺪ‪ ‬ﺑﻴﻦ‪ ‬ﺍﻟﻘﻤﺮ‪ ‬ﺍﻟﺼﻨﺎﻋﻲ‪ ‬ﻭﻣﺮﻛﺰ‪ ‬ﺍﻷ‪ ‬ﺭﺽ‪. ‬‬ ‫‪  – 2 ‬ﺍﻟﻘﺎﻧﻮﻥ‪ ‬ﺍﻷﻭﻝ‪ ‬ﻟﻜﺒﻠﺮ‪= K  : ‬‬
‫‪r 3 ‬‬
‫‪T  2‬‬ ‫‪4 p 2 ‬‬
‫‪( 1 ) ‬‬ ‫‪3 ‬‬
‫=‬ ‫‪ ‬ﻭﺑﺎﻟﺘﺎﻟﻲ ‪:‬‬
‫‪( R + h )  GM T ‬‬
‫‪2 ‬‬
‫‪4 p 2  ( R + h ) ‬‬
‫‪2 ‬‬ ‫‪2p‬‬ ‫‪2 p‬‬ ‫‪2 p ( R + h ) ‬‬
‫= ‪T ‬‬ ‫‪   ٬‬ﻭﻣﻨﻪ‬ ‫= ‪T ‬‬ ‫=‬ ‫=‬ ‫‪ ­ 3 ‬ﻟﺪﻳﻨﺎ‬
‫‪v 2 ‬‬ ‫‪w‬‬ ‫‪v ‬‬ ‫‪v ‬‬
‫‪R +h‬‬
‫‪2 ‬‬
‫‪2 ‬‬ ‫)‪4 p 2  ( R + h ‬‬ ‫‪4 p 2 ‬‬
‫‪( 2 ) ‬‬ ‫‪v‬‬ ‫‪( R + h ) = G M T ‬‬ ‫‪ ٬ ‬ﻭﺑﺎﻟﺘﺎﻟﻲ‪ ‬ﺍﻟﻌﻼﻗﺔ‪ ‬ﺍﻟﻤﻄﻠﻮﺑﺔ‪ ‬ﻫﻲ ‪:‬‬ ‫‪3 ‬‬
‫=‬ ‫‪ ‬ﺑﺎﻟﺘﻌﻮﻳﺾ‪ ‬ﻓﻲ‪ ‬ﺍﻟ‪ ‬ﻌﻼﻗﺔ ‪: (1 ) ‬‬
‫‪v 2  ( R + h ) ‬‬ ‫‪G M T ‬‬

‫‪  –  4 ‬ﺍﻟﻘﻤﺮ ‪ ‬ﺍﻟﺼﻨﺎﻋﻲ ‪ ‬ﺟﻴﻮ ‪ ‬ﻣﺴﺘﻘﺮ ‪ ) ‬ﺍﻟﻤﺴﺘﻘﺮ ‪ ‬ﺃﺭﺿﻴﺎ‪  ( ‬ﻫﻮ ‪ ‬ﺍﻟﻘﻤﺮ ‪ ‬ﺍﻟﺼﻨﺎﻋﻲ ‪ ‬ﺍﻟﺬﻱ‪ ‬ﻳﺒﺪﻭ‪ ‬ﺛﺎﺑﺘﺎ‪ ‬ﻟﻤﻼﺣﻆ ‪ ‬ﻋﻠﻰ ‪ ‬ﺳﻄﺢ ‪ ‬ﺍﻷﺭﺽ ‪  ٬ ‬ﺣﻴﺚ‪ ‬ﺗﻜﻮﻥ ‪ ‬ﺳﺮﻋﺔ‬
‫‪ ‬ﺩﻭﺭﺍﻧﻪ‪ ‬ﻣﺴﺎﻭﻳﺔ‪ ‬ﻟﺴﺮﻋﺔ‪ ‬ﺩﻭﺭﺍﻥ‪ ‬ﺍﻷﺭﺽ‪ ‬ﻓﻲ‪ ‬ﻣﻌﻠﻢ‪ ‬ﺃﺭﺿﻲ‪ ‬ﻣﺮﻛﺰﻱ‪ ٬ ‬ﻭﻳﺪﻭﺭ‪ ‬ﻓﻲ‪ ‬ﻧﻔﺲ‪ ‬ﺟﻬﺔ‪ ‬ﺩﻭﺭﺍﻥ‪ ‬ﺍﻷﺭﺽ‪ ‬ﻓﻲ‪ ‬ﻣﺴﺘﻮﻱ‪ ‬ﺍﻻﺳﺘﻮﺍء ‪. ‬‬
‫‪ ‬ﺍﺭﺗﻔﺎﻋﻪ‪  : ‬ﻣﻦ‪ ‬ﻗﺎﻧﻮﻥ‪ ‬ﻛﺒﻠﺮ ‪ ‬ﻧﺤﺴﺐ‪ ‬ﺍﻻﺭﺗﻔﺎﻉ‪ ٬  h ‬ﺣﻴﺚ ‪T = 24 h‬‬
‫‪2 ‬‬ ‫‪-11‬‬ ‫‪24 ‬‬
‫‪3 ‬‬ ‫‪T 2 G M T  ( 86400 )  ´ 6, 67 ´ 10 ´ 5,97 ´ 10 ‬‬
‫) ‪(R + h‬‬ ‫=‬ ‫‪2 ‬‬
‫=‬ ‫‪= 75,38 ´ 10 21 ‬‬
‫‪4p‬‬ ‫‪39, 44 ‬‬

‫‪( R + h ) = 3  75,38 ´ 1021 = 4, 22 ´ 10 7 m‬‬


‫‪h = 4, 22 ´ 107 - 0, 64 ´ 107 = 3,58 ´107  m = 35800 km‬‬ ‫‪ ‬ﻭﻣﻨﻪ‬

‫‪GM T ‬‬ ‫‪6,67 ´ 10 -11 ´ 5,97 ´ 10 24 ‬‬


‫= ‪v‬‬ ‫=‬ ‫‪ ‬ﺑﺎﻟﺘﻌﻮﻳﺾ‪ ‬ﻓﻲ‪ ‬ﺍﻟﻌﻼﻗﺔ‪= 3070 m / s  : (2 ) ‬‬
‫‪( R + h ) ‬‬ ‫‪4, 22 ´ 10 7 ‬‬

‫‪m s M T ‬‬ ‫‪2 ´ 103 ´ 5,97 ´ 10 24 ‬‬


‫‪F =G‬‬ ‫‪2‬‬
‫‪= 6,67 ´ 10-11 ‬‬ ‫‪ – 5 ‬ﻗﻮﺓ‪ ‬ﺍﻟﺠﺬﺏ ‪= 447, 2 N  :‬‬
‫‪7  2 ‬‬
‫) ‪(R + h‬‬ ‫‪( 4, 22 ´10  ) ‬‬
‫‪ ‬ﺍﻟﻘﻤﺮ‪ ‬ﺍﻟﺼﻨﺎﻋﻲ‪ ‬ﻻ‪ ‬ﻳﺴﻘﻂ‪ ‬ﻋﻠﻰ‪ ‬ﺍﻷﺭﺽ‪ ‬ﻷﻧﻪ‪ ‬ﻣﺘﻮﺍﺯﻥ‪ ‬ﺑﻴﻦ‪ ‬ﻗﻮﺓ‪ ‬ﺟﺬﺏ‪ ‬ﺍﻷﺭﺽ‪ ‬ﻭﺍﻟﻘﻮﺓ‪ ‬ﺍﻟﻄﺎﺭﺩﺓ‪ ‬ﺍﻟﻤﺮﻛﺰﻳﺔ‪ ‬ﺍﻟﻨﺎﺗﺠﺔ‪ ‬ﻋﻦ‪ ‬ﺩﻭﺭﺍﻧﻪ‪ ‬ﺍﻟﻤﺴﺘﻤﺮ‪. ‬‬

‫‪GUEZOURI ‬‬
‫‪Lycée Maraval ‬‬
‫‪Oran‬‬
‫‪5 ‬‬
‫‪ ‬ﺍﻟﺘﻤﺮﻳﻦ‪ ‬ﺍﻟﺨﺎﻣﺲ‪ 4 ) ‬ﻧﻘﻂ‪( ‬‬

‫‪  ­ 1 ‬ﺃ‪ ( ‬ﺍﻷﺳﺘﺮ‪ ‬ﺍﻟﻤﺘﺸﻜﻞ‪ ‬ﻫﻮ‪  ‬ﺇﺛﺎﻧﻮﺍﺕ‪ ‬ﺍﻹﻳﺜﻴﻞ ‪( CH3COO­C2H5  ) ‬‬


‫‪ ‬ﺏ‪ ( ‬ﺟﺪﻭﻝ‪ ‬ﺍﻟﺘﻘﺪّﻡ ‪: ‬‬
‫‪ ‬ﻣﻌﺎﺩﻟﺔ‪ ‬ﺍﻟﺘﻔﺎﻋﻞ‬ ‫‪C2H5­OH  +  CH3­COOH  =  CH3­COO­C2H5  +  H2O ‬‬
‫‪ ‬ﺣــﺎﻟﺔ‪ ‬ﺍﻟﺠﻤﻠﺔ‬ ‫‪ ‬ﺍﻟﺘﻘﺪﻡ‬ ‫‪( mol ) ‬‬ ‫‪ ‬ﻛﻤﻴﺔ‪ ‬ﺍﻟﻤـــﺎﺩﺓ‬
‫‪ ‬ﺍﻟﺤﺎﻟﺔ‪ ‬ﺍﻻﺑﺘﺪﺍﺋﻴﺔ‬ ‫‪0 ‬‬ ‫‪0,2 ‬‬ ‫‪0,2 ‬‬ ‫‪0 ‬‬ ‫‪0 ‬‬
‫‪ ‬ﺍﻟﺤﺎﻟﺔ‪ ‬ﺍﻻﻧﺘﻘﺎﻟﻴﺔ‬ ‫‪x ‬‬ ‫‪0,2 ­ x ‬‬ ‫‪0,2 ­ x ‬‬ ‫‪x ‬‬ ‫‪x ‬‬
‫‪ ‬ﺍﻟﺤﺎﻟﺔ‪ ‬ﺍﻟﻨﻬﺎﺋﻴﺔ‬ ‫‪xf ‬‬ ‫‪0,2 – xf ‬‬ ‫‪0,2 – xf ‬‬ ‫‪xf ‬‬ ‫‪xf ‬‬

‫‪ ‬ﺟـ‪ ( ‬ﻣﻌﺎﺩﻟﺔ‪ ‬ﺗﻔﺎﻋﻞ‪ ‬ﺍﻟﻤﻌﺎﻳﺮﺓ ‪:‬‬

‫‪CH 3 - COOH +‬‬ ‫‪( Na‬‬ ‫‪+‬‬


‫‪,OH ­ )    =    (CH 3 - COO - ,Na + )  + H 2 O‬‬

‫‪na = C ´V 'be = V ' be ‬‬ ‫‪  ­ 2 ‬ﺃ‪  ( ‬ﻋﻨﺪ‪ ‬ﺍﻟﺘﻜﺎﻓﺆ‪ ‬ﺗﻜﻮﻥ‪ ‬ﻛﻤﻴﺔ‪ ‬ﻣﺎﺩﺓ‪ ‬ﺍﻟﺤﻤﺾ‪ ‬ﺍﻟﺒﺎﻗﻲ ‪  ( n a  ) ‬ﻣﺴﺎﻭﻳﺔ‪ ‬ﻟﻜﻤﻴﺔ‪ ‬ﻣﺎﺩﺓ‪ ‬ﺍﻷﺳﺎﺱ‪ ٬  ( nOH - ) ‬ﺃﻱ ‪: ‬‬

‫‪( 1 ) ‬‬ ‫‪x = 0, 2 - V ' be ‬‬ ‫‪ ‬ﺏ‪  ( ‬ﻣﻦ‪ ‬ﺟﺪﻭﻝ‪ ‬ﺍﻟﺘﻘﺪﻡ‪ ‬ﻟﺪﻳﻨﺎ ‪ ٬  na  = 0, 2 - x‬ﻭﻣﻨﻪ‬


‫‪ ‬ﺗﺘﻤّﺔ‪ ‬ﺍﻟﺠﺪﻭﻝ‪ : ‬ﺑﺎﺳﺘﻌﻤﺎﻝ‪ ‬ﺍﻟﻌﻼﻗﺔ ‪ ( 1 ) ‬ﻧﻜﻤﻞ‪ ‬ﺍﻟﺠﺪﻭﻝ‬
‫‪t (h) ‬‬ ‫‪0 ‬‬ ‫‪4 ‬‬ ‫‪8 ‬‬ ‫‪12 ‬‬ ‫‪16 ‬‬ ‫‪20 ‬‬ ‫‪32 ‬‬ ‫‪40 ‬‬ ‫‪48 ‬‬ ‫‪60 ‬‬
‫‪V’be ‬‬ ‫‪200 ‬‬ ‫‪168 ‬‬ ‫‪148 ‬‬ ‫‪132 ‬‬ ‫‪118 ‬‬ ‫‪104 ‬‬ ‫‪74 ‬‬ ‫‪66 ‬‬ ‫‪66 ‬‬ ‫‪66 ‬‬
‫‪x (mol) ‬‬ ‫‪0 ‬‬ ‫‪0,032  0,052  0,068  0,082  0,096  0,126  0,134  0,134  0,134 ‬‬

‫‪x (mol) ‬‬ ‫‪ ‬ﺟـ‪  ( ‬ﺍﻟﺮ‪ ‬ﺳﻢ‪ ‬ﺍﻟﺒﻴﺎﻧﻲ‪: ‬‬

‫‪0,02 ‬‬

‫‪5 ‬‬
‫‪t (h) ‬‬

‫‪x f ‬‬ ‫‪0,134 ‬‬
‫‪ ‬ﻧﺴﺘﻨﺘﺞ‪ ‬ﺃﻥ‪ ‬ﻫﺬﺍ‪ ‬ﺍﻟﺘﻔﺎﻋﻞ‪ ‬ﻏﻴﺮ‪ ‬ﺗ‪ ‬ﺎ‪ ‬ﻡ‪. ‬‬ ‫=‪t‬‬ ‫=‬ ‫‪ ‬ﺩ‪= 0, 67  ( ‬‬
‫‪x max ‬‬ ‫‪0, 2 ‬‬

‫= ‪Q rf ‬‬
‫‪[CH 3 - COO - C 2 H 5 ] ´ [ H 2 O ] = nester ´ n eau  = x 2 f ‬‬ ‫‪ ‬ﻫـ(‬
‫‪[CH 3 - COOH ] ´ [C 2 H 5  - OH ] nacide ´ n alcool  ( 0, 2 - x f  ) 2 ‬‬
‫‪2 ‬‬
‫‪GUEZOURI ‬‬
‫= ‪Q rf ‬‬
‫)‪( 0,134 ‬‬ ‫‪» 4 ‬‬
‫‪Lycée Maraval ‬‬ ‫‪2 ‬‬
‫‪Oran‬‬
‫‪( 0, 2 - 0,134 ) ‬‬
‫‪6 ‬‬
‫ﺑﻜﺎﻟﻮرﻳﺎ ‪ - 2008‬اﻟﻨﻈـﺎم اﻟﺠﺪﻳﺪ‬
‫ﺷﻌﺒﺔ ﻋﻠﻮم اﻟﻄﺒﻴﻌﺔ واﻟﺤﻴﺎة‬ ‫‪-‬‬ ‫ﺗﺼﺤﻴﺢ ﻣﻮﺿﻮﻋﻲ اﻟﻌﻠﻮم اﻟﻔﻴﺰﻳﺎﺋﻴﺔ‬
‫اﻷﺳﺘﺎذ ﻋﺒﺪ اﻟﻘﺎدر ﻗﺰوري – ﺛﺎﻧﻮﻳﺔ ﻣـﺎراﻓﺎل ‪ -‬وهﺮان‬

‫اﻟﻤﻮﺿــﻮع اﻷول‬
‫اﻟﺘﻤﺮﻳﻦ اﻷول )‪ 4‬ﻧﻘﻂ(‬
‫‪ – 1 - Ι‬ﺗﻌﺮﻳﻒ اﻟﺤﻤﺾ ﺣﺴﺐ ﺑﺮوﻧﺸﺘﺪ ‪ :‬اﻟﺤﻤﺾ هﻮ ﻓﺮد آﻴﻤﻴﺎﺋﻲ ﻗﺎدر ﻋﻠﻰ إﻋﻄـﺎء ﺑﺮوﺗﻮﻧﺎ أو أآﺜﺮ ‪. H+‬‬
‫‪ – 2‬اﻟﺜﻨﺎﺋﻴﺘﺎن أﺳﺎس ‪ /‬ﺣﻤﺾ اﻟﻤﺸﺎرآﺘﺎن ﻓﻲ اﻟﺘﻔﺎﻋﻞ هﻤﺎ ) –‪ (CH3COOH / CH3COO‬و )‪( H3O+ / H2O‬‬
‫⎦⎤ ‪⎡⎣CH 3COO − ⎤⎦ × ⎡⎣ H 3O +‬‬
‫= ‪K‬‬ ‫‪f‬‬ ‫‪f‬‬
‫‪ – 3‬ﺛﺎﺑﺖ اﻟﺘﻮازن ‪:‬‬
‫‪[CH 3COOH ]f‬‬
‫‪⎡⎣ H 3O + ⎤⎦ = 10− pH = 10−3 ,7 ≈ 2 ×10−4 mol / L‬‬ ‫‪-1‬‬ ‫‪- ІІ‬‬
‫‪f‬‬

‫‪ - 2‬ﺟﺪول اﻟﺘﻘﺪّم ‪ :‬ﻟﺪﻳﻨﺎ ﻋﺪد ﻣﻮﻻت اﻟﺤﻤﺾ ‪n A = CV = 2 , 7 × 10−3 × 0 ,1 = 2 , 7 × 10−4 mol‬‬

‫‪CH3-COOH‬‬ ‫‪+‬‬ ‫‪H2O‬‬ ‫–‪= CH3-COO‬‬ ‫‪+‬‬ ‫‪H3O+‬‬


‫‪t=0‬‬ ‫‪2,7 × 10–4‬‬ ‫زﻳﺎدة‬ ‫‪0‬‬ ‫‪0‬‬

‫اﻟﺤﺎﻟﺔ اﻻﻧﺘﻘﺎﻟﻴﺔ‬ ‫‪2 , 7 × 10−4 − x‬‬ ‫زﻳﺎدة‬ ‫‪x‬‬ ‫‪x‬‬

‫اﻟﺤﺎﻟﺔ اﻟﻨﻬﺎﺋﻴﺔ‬ ‫‪2 , 7 ×10−4 − x f‬‬ ‫زﻳﺎدة‬ ‫‪xf‬‬ ‫‪xf‬‬

‫اﻟﺘﻘﺪم اﻟﻨﻬﺎﺋﻲ ‪x f = n ( H 3O + ) = ⎡⎣ H 3O + ⎤⎦ ×V = 2 ×10−4 × 0 ,1 = 2 × 10−5 mol :‬‬

‫اﻟﺘﻘﺪم اﻷﻋﻈﻤﻲ ‪ ، 2 , 7 ×10−4 − x max = 0 :‬وﻣﻨﻪ ‪x max = 2 , 7 × 10−4 mol‬‬

‫‪xf‬‬ ‫‪2 ×10−5‬‬


‫‪ ،‬أي ‪7,4 %‬‬ ‫=‪τ‬‬ ‫=‬ ‫‪ - 3‬اﻟﻨﺴﺒﺔ اﻟﻨﻬﺎﺋﻴﺔ ﻟﻠﺘﻘﺪّم ‪= 0 , 074 :‬‬
‫‪x max 2 , 7 × 10−4‬‬
‫ﻧﺴﺘﻨﺘﺞ أن ﺣﻤﺾ اﻹﻳﺜﺎﻧﻮﻳﻚ ﺿﻌﻴﻒ ) آﻤﺎ ﻗﻴﻞ ﻟﻨﺎ ﻓﻲ ﺑﺪاﻳﺔ اﻟﺘﻤﺮﻳﻦ أن اﻟﺘﻔﺎﻋﻞ ﻣﺤﺪود !!(‬
‫أ( ﺣﺴﺐ ﺟﺪول اﻟﺘﻘﺪّم ﻓﺈن ‪⎡⎣CH 3COO − ⎤⎦ = ⎡⎣ H 3O + ⎤⎦ = 2 ×10−4 mol / L‬‬ ‫‪-4‬‬

‫‪2 , 7 × 10−4 − x f‬‬ ‫‪2 , 7 ×10−4 − 2 ×10−5‬‬


‫= ] ‪[CH 3COOH‬‬ ‫=‬ ‫‪= 2 , 5 × 10−3 mol / L‬‬ ‫ﻣﻦ ﺟﺪول اﻟﺘﻘﺪّم ‪:‬‬
‫‪V‬‬ ‫‪0 ,1‬‬

‫⎦⎤ ‪⎡CH 3COO −‬‬ ‫‪2 ×10−4‬‬


‫⎣ ‪pK A = pH − Log‬‬ ‫‪= 3, 7 − Log‬‬ ‫‪≈ 4, 8‬‬ ‫ب(‬
‫] ‪[CH 3COOH‬‬ ‫‪2 , 5 × 10−5‬‬

‫⎦⎤ ‪⎡⎣CH 3COO −‬‬ ‫⎦⎤ ‪⎡⎣CH 3COO −‬‬


‫‪Log‬‬ ‫‪ pH = pK A + Log‬ﻧﺴﺘﻨﺘﺞ ‪= pH − pK A = 3, 7 − 4 , 8 = −1,1‬‬ ‫ﻣﻦ اﻟﻌﻼﻗﺔ‬
‫] ‪[CH 3COOH‬‬ ‫] ‪[CH 3COOH‬‬
‫وهﺬا ﻣﻌﻨﺎﻩ أن ⎦⎤ ‪ [CH 3COOH ] > ⎡⎣CH 3COO −‬ﻷن ﻟﻮﻏﺎرﻳﺘﻢ اﻟﻨﺴﺒﺔ ﺳﺎﻟﺐ ‪.‬‬

‫وﺑﺎﻟﺘﺎﻟﻲ اﻟﻔﺮد اﻟﻜﻴﻤﻴﺎﺋﻲ اﻟﻤﺘﻐﻠﺐ هﻮ ‪. CH 3COOH‬‬

‫‪1‬‬
‫اﻟﺘﻤﺮﻳﻦ اﻟﺜﺎﻧﻲ )‪ 4‬ﻧﻘﻂ(‬
‫‪N‬‬
‫‪.‬‬ ‫أ( زﻣﻦ ﻧﺼﻒ اﻟﻌﻤﺮ هﻮ اﻟﺰﻣﻦ اﻟﻼزم ﻟﻌﻴﻨﺔ ﺗﺤﺘﻮي ﻣﺘﻮﺳﻄﺎ ﻋﻠﻰ ‪ N‬ذرة ﻣﺸﻌﺔ ﻟﻜﻲ ﻳﺼﺒﺢ هﺬا اﻟﻌﺪد‬ ‫‪-1‬‬
‫‪2‬‬

‫) ‪N (t‬‬ ‫) ‪N (t‬‬
‫= ) ‪N (t + t 1/ 2‬‬ ‫أي‬
‫‪N0‬‬ ‫‪2‬‬
‫‪1‬‬
‫) ‪N (t‬‬
‫هﻮ ‪t = 2 , 25 × 103 = 2250s‬‬ ‫ب( ﻣﻦ اﻟﺒﻴﺎن اﻟﺰﻣﻦ اﻟﻤﻮاﻓﻖ ﻟـ ‪= 0 , 5‬‬
‫‪N0‬‬

‫‪0,5‬‬

‫)‪t (s‬‬ ‫‪1‬‬ ‫‪−λ t‬‬


‫‪ ،‬وﺑﺈدﺧﺎل اﻟﻠﻮﻏﺎرﻳﺘﻢ اﻟﻨﻴﺒﻴﺮي ﻋﻠﻰ اﻟﻄﺮﻓﻴﻦ‬ ‫‪ - 2‬أ( ‪= e 1/ 2‬‬
‫‪2250‬‬ ‫‪2‬‬
‫‪0 , 69‬‬
‫=‪λ‬‬ ‫‪ ، ln 2 = λ × t1 / 2‬وﻣﻨﻪ‬
‫‪t1 / 2‬‬
‫‪0 , 69‬‬
‫=‪λ‬‬ ‫‪≈ 3 × 10−4 s −1‬‬ ‫ب(‬
‫‪2250‬‬
‫‪ - 3‬ﺣﺴﺐ اﻟﻨﺘﻴﺠﺔ اﻟﻤﺤﺼّﻞ ﻋﻠﻴﻬﺎ واﻟﺠﺪول اﻟﻤﺮﻓﻖ واﻟﻮرق اﻟﻤﻠﻤﺘﺮي اﻟﺮديء ﻧﻌﺘﺒﺮ ‪ ، 2250s ≈ 2240 s‬وﺑﺎﻟﺘﺎﻟﻲ اﻟﻨﻮاة‬
‫‪38‬‬
‫‪.‬‬ ‫‪17Cl‬‬ ‫هﻲ ﻧﻮاة اﻟﻜﻠﻮر‬ ‫‪A‬‬
‫‪Z X‬‬

‫‪35‬‬
‫‪17Cl‬‬ ‫‪+ 3 01n → 17‬‬
‫‪38‬‬
‫‪Cl‬‬ ‫‪ - 4‬اﻟﻤﻌﺎدﻟﺔ ‪:‬‬

‫(‬ ‫)‬
‫‪E l = 17 × m p + 21m n − m X × 932 , 5 = (17 × 1, 00728 + 21× 1, 00866 − 37 , 9601) × 931, 5‬‬ ‫‪ - 5‬أ(‬

‫‪E l = 321, 8 MeV = 3, 22 × 108 eV‬‬

‫‪E l 321, 8‬‬


‫=‬ ‫‪= 8 , 47MeV = 8, 47 × 106 eV‬‬ ‫ب( ﻃﺎﻗﺔ اﻟﺮﺑﻂ ﻟﻜﻞ ﻧﻮﻳﺔ ‪:‬‬
‫‪A‬‬ ‫‪38‬‬
‫اﻟﺘﻤﺮﻳﻦ اﻟﺜﺎﻟﺚ )‪ 4‬ﻧﻘﻂ(‬
‫‪y‬‬
‫‪ – 1‬ﻣﻌﺎدﻟﺔ اﻟﻤﺴﺎر ‪:‬‬
‫‪GUEZOURI A.‬‬
‫‪Lycée Maraval - Oran‬‬ ‫ﻧﻄﺒّﻖ ﻋﻠﻰ ﺣﺮآﺔ اﻟﻜﺮة اﻟﻘﺎﻧﻮن اﻟﺜﺎﻧﻲ ﻟﻨﻴﻮﺗﻦ ‪:‬‬
‫‪G‬‬ ‫‪G‬‬
‫‪G‬‬
‫‪g‬‬ ‫‪m‬‬ ‫‪∑F = m a‬‬
‫‪G‬‬ ‫‪G‬‬
‫‪G‬‬ ‫‪G G‬‬ ‫‪G‬‬
‫‪v0‬‬ ‫‪ ، P = m a‬وﻣﻨﻪ ﻧﺠﺪ ‪a = g‬‬
‫‪G‬‬ ‫‪G‬‬
‫‪P‬‬ ‫ﻣﺮآﺒﺘﺎ ﺷﻌﺎع اﻟﺘﺴﺎرع ﻓﻲ اﻟﻤﻌﻠﻢ هﻤﺎ ) ‪a ( 0 , − g‬‬
‫‪G‬‬
‫‪α‬‬ ‫ﻣﺮآﺒﺘﺎ ﺷﻌﺎع اﻟﺴﺮﻋﺔ اﻻﺑﺘﺪاﺋﻴﺔ هﻤﺎ ) ‪v0 ( v0 cos α , v0 sin α‬‬
‫‪A‬‬
‫ﺑﻤﺎ أن اﻟﺘﺴﺎرع ﻋﻠﻰ اﻟﻤﺤﻮر ‪ Ox‬ﻣﻌﺪوم ‪ ،‬إذن اﻟﺤﺮآﺔ ﻋﻠﻰ هﺬا اﻟﻤﺤﻮر‬
‫‪G‬‬ ‫‪h0‬‬ ‫ﻣﻨﺘﻈﻤﺔ وﺑﺎﻟﺘﺎﻟﻲ ‪:‬‬
‫‪j‬‬
‫‪O‬‬ ‫‪G‬‬ ‫‪x‬‬ ‫)‪(1‬‬ ‫‪x = v0 cos α t‬‬
‫‪i‬‬

‫‪2‬‬
‫ﺑﻤﺎ أن اﻟﺘﺴﺎرع ﻋﻠﻰ اﻟﻤﺤﻮر ‪ Oy‬ﺛﺎﺑﺖ )‪ ، (– g‬إذن اﻟﺤﺮآﺔ ﻋﻠﻰ هﺬا اﻟﻤﺤﻮر ﻣﺘﻐﻴّﺮة ﺑﺎﻧﺘﻈﺎم ‪ ،‬وﺳﺮﻋﺘﻬﺎ اﻻﺑﺘﺪاﺋﻴﺔ ‪v 0 , y = v 0 sin α‬‬

‫‪1‬‬
‫)‪(2‬‬ ‫‪y = − g t 2 + v 0 sin α t + y 0‬‬ ‫وﺑﺎﻟﺘﺎﻟﻲ ‪:‬‬
‫‪2‬‬
‫‪x‬‬
‫= ‪ ، t‬ﺛﻢ ﻧﻌﻮّض ﻋﺒﺎرة اﻟﺰﻣﻦ ﻓﻲ اﻟﻌﻼﻗﺔ )‪ (2‬وﻧﺠﺪ ﻣﻌﺎدﻟﺔ اﻟﻤﺴﺎر ‪:‬‬ ‫ﻣﻦ اﻟﻌﻼﻗﺔ )‪ (1‬ﻧﺴﺘﺨﺮج‬
‫‪v0 cos α‬‬

‫‪g‬‬ ‫‪1‬‬ ‫‪x2‬‬ ‫‪x‬‬


‫‪z =−‬‬ ‫‪ ، y = − g 2‬وﻣﻨﻪ ‪x + x tg α + y 0 :‬‬
‫‪2‬‬
‫‪+ v 0 sin α‬‬ ‫‪+ y0‬‬
‫‪2 v 0 cos α‬‬
‫‪2‬‬ ‫‪2‬‬
‫‪2 v 0 cos α‬‬
‫‪2‬‬
‫‪v 0 cos α‬‬

‫‪ - 2‬أ( ﻋﻠﻰ اﻟﺸﻜﻞ ‪ ،‬اﻟﻨﻘﻄﺔ اﻟﺘﻲ ﻓﺎﺻﻠﺘﻬﺎ ‪ x = 12m‬ﺗﺮﺗﻴﺒﻬﺎ ‪ ، y = 3m‬وﺑﺎﻟﺘﺎﻟﻲ ‪:‬‬


‫) ‪y (m‬‬
‫‪h2 = 3 − h1 = 3 − 1, 8 = 1, 20m‬‬

‫) ‪(x , y‬‬ ‫ب( ﻣﻦ أﺟﻞ إﻳﺠﺎد اﻟﺴﺮﻋﺔ اﻻﺑﺘﺪاﺋﻴﺔ ‪ v 0‬ﻧﻌﻮّض ﻓﻲ ﻣﻌﺎدﻟﺔ اﻟﻤﺴﺎر ﻗﻴﻤﺘﻴﻦ‬

‫‪(x‬‬ ‫ﻣﺜﻼ اﻟﻨﻘﻄﺔ ) ‪= 18m , y = 0‬‬

‫‪3‬‬ ‫•‬

‫•‪2‬‬ ‫‪h2‬‬
‫•‬
‫•‬ ‫‪M‬‬
‫‪h1‬‬
‫•‬ ‫•‬ ‫) ‪x (m‬‬
‫‪12‬‬ ‫‪18‬‬

‫‪−10‬‬
‫‪× (18 ) + 18 × 0 , 4663 + 2‬‬
‫‪2‬‬
‫= ‪ ، 0‬وﻣﻨﻪ ‪v 0 = 13, 77m / s‬‬
‫)‪( 0 , 9063‬‬
‫‪2‬‬ ‫‪2‬‬
‫‪2v‬‬ ‫‪0‬‬
‫‪GUEZOURI A.‬‬
‫‪Lycée Maraval - Oran‬‬ ‫‪ ، x = v0 cos α t‬وﺑﺎﻟﺘﻌﻮﻳﺾ ‪x = 12 , 48t :‬‬ ‫ﺟـ( ﻟﺪﻳﻨﺎ‬

‫‪1‬‬
‫‪ ، y = − g t 2 + v 0 sin α t + y 0‬وﺑﺎﻟﺘﻌﻮﻳﺾ ‪y = −5t 2 + 5, 82t + 2 :‬‬
‫‪2‬‬
‫‪M (14 , 6 ; 2 ) m‬‬ ‫‪ ، y ≈ 2 m‬وﻣﻨﻪ‬ ‫ﻧﺠﺪ ‪ x = 14 , 6 m‬و‬ ‫ﻧﻌﻮّض ﻓﻲ اﻟﻤﻌﺎدﻟﺘﻴﻦ اﻟﺰﻣﻨﻴﺘﻴﻦ ‪t = 1,17 s‬‬
‫ﻗﻴﻤﺔ اﻟﺴﺮﻋﺔ ﻋﻨﺪ اﻟﻨﻘﻄﺔ ‪: M‬‬
‫اﻟﻄﺮﻳﻘﺔ اﻷوﻟﻰ ‪:‬‬
‫ﻧﻌﺘﺒﺮ اﻻرﺗﻔﺎع ﻣﻦ ﻧﻘﻄﺔ اﻟﻘﺬف إﻟﻰ اﻟﺬروة هﻮ ’‪ ، h‬وهﻮ ﻧﻔﺴﻪ اﻻرﺗﻔﺎع ﻣﻦ ‪ M‬إﻟﻰ اﻟﺬروة ﻷن اﻟﻨﻘﻄﺘﻴﻦ ‪ A‬و ‪ M‬ﺗﻮﺟﺪان ﻋﻠﻰ ﻧﻔﺲ‬
‫اﻟﻤﺴﺘﻮي اﻷﻓﻘﻲ ‪.‬‬
‫‪1‬‬ ‫‪1‬‬
‫‪ ،‬وﻣﻨﻪ ‪:‬‬ ‫ﺑﺘﻄﺒﻴﻖ ﻧﻈﺮﻳﺔ اﻟﻄﺎﻗﺔ اﻟﺤﺮآﻴﺔ ﺑﻴﻦ اﻟﻨﻘﻄﺘﻴﻦ ‪ A‬و ‪mv M2 − mv 20 = mgh' − mgh' : M‬‬
‫‪2‬‬ ‫‪2‬‬
‫‪v M = v 0 = 13, 77m / s‬‬
‫اﻟﻄﺮﻳﻘﺔ اﻟﺜﺎﻧﻴﺔ ‪:‬‬

‫) ‪(v 0 cos α ) + ( − gt + v 0 sin α‬‬


‫‪2‬‬ ‫‪2‬‬
‫= ‪ ، v M = v 2x M + v 2y M‬وﺑﺎﻟﺘﻌﻮﻳﺾ ﻧﺠﺪ ‪v M = 13, 77m / s‬‬

‫‪18‬‬
‫=‪t‬‬ ‫اﻟﻔﺎﺻﻠﺔ ‪ ، x = 18 m‬وﻧﺠﺪ ‪= 1, 44 s‬‬ ‫د( ﻧﻌﻮّض ﻓﻲ اﻟﻤﻌﺎدﻟﺔ اﻟﺰﻣﻨﻴﺔ ‪x = 12 , 48t‬‬
‫‪12 , 48‬‬

‫‪3‬‬
‫اﻟﺘﻤﺮﻳﻦ اﻟﺮاﺑﻊ )‪ 4‬ﻧﻘﻂ(‬
‫‪ – 1‬ﺑﻌﺪ اﻟﻤﺪة اﻟﺰﻣﻨﻴﺔ ‪ Δt = 15 s‬ﻳﻜﻮن اﻟﻨﻈﺎم اﻟﺪاﺋﻢ ﻗﺪ ﺗﺤﻘّﻖ ‪ ،‬وﺑﺎﻟﺘﺎﻟﻲ ﻳﻜﻮن ‪ ، uC = E‬وﺣﺴﺐ ﻗﺎﻧﻮن ﺟﻤﻊ اﻟﺘﻮﺗﺮات ﻓﺈن ‪:‬‬

‫‪ ،‬إذن ‪. i = 0‬‬ ‫‪ ، uC + u R = E‬وﻣﻨﻪ ‪ ، u R = 0‬وﺑﻤﺎ أن ‪u R = Ri‬‬

‫‪[U ] × [ I ] × [T ] = T :‬‬
‫= ] ‪[τ‬‬ ‫] [‬ ‫‪ - 2‬ﺛﺎﺑﺖ اﻟﺰﻣﻦ ‪ ، τ = RC‬وهﻮ ﻣﺘﻨﺎﺳﺐ ﻣﻊ اﻟﺰﻣﻦ ﻷن‬
‫] ‪[ I ] [U‬‬
‫‪E × 63‬‬
‫= ‪ ، uC‬وﺑﺎﻟﺘﺎﻟﻲ ‪τ = 2 , 2 s‬‬ ‫‪= 1, 9 V‬‬ ‫‪ - 3‬ﻣﻦ اﻟﺒﻴﺎن ‪ τ = t‬ﻣﻦ أﺟﻞ‬
‫‪100‬‬
‫‪τ‬‬ ‫‪2, 2‬‬
‫= ‪C‬‬ ‫=‬ ‫‪4‬‬
‫‪= 2 , 2 ×10−4 F = 220μ F‬‬ ‫اﺳﺘﻨﺘﺎج ﻗﻴﻤﺔ ‪: C‬‬
‫‪R‬‬ ‫‪10‬‬
‫) ‪dq (t‬‬
‫= ) ‪i (t‬‬ ‫أ(‬ ‫‪-4‬‬
‫‪dt‬‬
‫) ‪q (t‬‬
‫= ) ‪u c (t‬‬ ‫ب(‬
‫‪C‬‬
‫‪uC + u R = E‬‬ ‫ﺟـ( ﺣﺴﺐ ﻗﺎﻧﻮن ﺟﻤﻊ اﻟﺘﻮﺗﺮات ﻟﺪﻳﻨﺎ‬

‫‪duC‬‬ ‫‪dq‬‬
‫)‪(1‬‬ ‫‪uC + RC‬‬ ‫‪=E‬‬ ‫‪ ،‬وﺑﺎﻟﺘﺎﻟﻲ‬ ‫‪uC + R‬‬ ‫‪=E‬‬
‫‪dt‬‬ ‫‪dt‬‬
‫⎛‬ ‫‪−‬‬
‫‪t‬‬ ‫⎞‬
‫‪u C (t ) = E ⎜ 1 − e‬‬ ‫‪A‬‬
‫ﻟﺪﻳﻨﺎ ⎟⎟‬ ‫‪-5‬‬
‫⎜‬
‫⎝‬ ‫⎠‬
‫‪duC‬‬ ‫‪1‬‬ ‫‪E‬‬
‫‪ ،‬ﺣﻴﺚ أن هﺬﻩ اﻟﻤﻌﺎدﻟﺔ اﻟﺘﻔﺎﺿﻠﻴﺔ ﺣﻠﻬﺎ ﻣﻦ اﻟﺸﻜﻞ ‪:‬‬ ‫‪+‬‬ ‫= ‪uC‬‬ ‫ﻧﻜﺘﺐ اﻟﻤﻌﺎدﻟﺔ اﻟﺘﻔﺎﺿﻠﻴﺔ ﻋﻠﻰ اﻟﺸﻜﻞ‬
‫‪dt‬‬ ‫‪RC‬‬ ‫‪RC‬‬
‫)‪(2‬‬ ‫‪uC = K e α t + B‬‬

‫⎛‬ ‫‪1‬‬ ‫‪⎞ B‬‬ ‫‪E‬‬


‫‪ ، K e α t ⎜ α +‬وهﺬﻩ اﻟﻤﻌﺎدﻟﺔ ﺗﻜﻮن ﻣﺤﻘّﻘﺔ ﻣﻦ أﺟﻞ‬ ‫‪⎟+‬‬ ‫=‬ ‫وﺑﺎﻻﺷﺘﻘﺎق واﻟﺘﻌﻮﻳﺾ ﻓﻲ اﻟﻤﻌﺎدﻟﺔ اﻟﺘﻔﺎﺿﻠﻴﺔ ﻧﺠﺪ ‪:‬‬
‫⎝‬ ‫‪RC‬‬ ‫‪⎠ RC RC‬‬
‫‪1‬‬
‫و ‪ ، B = E‬وﻣﻦ أﺟﻞ ‪ t = 0‬ﻳﻜﻮن ‪ ، uC = 0‬وﺑﺎﻟﺘﺎﻟﻲ ‪ ، K = – E‬وذﻟﻚ ﺑﺎﻟﺘﻌﻮﻳﺾ ﻓﻲ )‪. (2‬‬ ‫‪α =−‬‬
‫‪RC‬‬
‫⎛‬ ‫‪−‬‬
‫‪1‬‬
‫‪t‬‬ ‫⎞‬
‫‪ ، uC = E ⎜1 − e‬وﺑﻤﻘﺎرﻧﺔ هﺬﻩ اﻟﻤﻌﺎدﻟﺔ ﻣﻊ اﻟﻤﻌﺎدﻟﺔ اﻟﻤﻌﻄﺎة ﻧﺠﺪ‬ ‫‪RC‬‬ ‫وﺑﺎﻟﺘﺎﻟﻲ ﻧﻜﺘﺐ ﻋﺒﺎرة اﻟﺘﻮﺗﺮ ﺑﻴﻦ ﻃﺮﻓﻲ اﻟﻤﻜﺜّﻔﺔ ⎟‬
‫⎜‬ ‫⎟‬
‫⎝‬ ‫⎠‬
‫‪ ، A = RC‬وهﻮ ﺛﺎﺑﺖ اﻟﺰﻣﻦ ‪.‬‬
‫ﻣﺪﻟﻮﻟﻪ اﻟﻔﻴﺰﻳﺎﺋﻲ ‪ :‬هﻮ ﻋﺒﺎرة ﻋﻦ ﻣﺆﺷّﺮ ﻟﻤﺪّة ﻣﻜﻮث اﻟﻨﻈﺎم اﻻﻧﺘﻘﺎﻟﻲ ﺧﻼل ﺷﺤﻦ أو ﺗﻔﺮﻳﻎ ﻣﻜﺜﻔﺔ ‪.‬‬
‫أو ‪ :‬اﻟﺰﻣﻦ اﻟﻼزم ﻟﺸﺤﻦ اﻟﻤﻜﺜﻔﺔ إﻟﻰ اﻟﺜﻠﺜﻴﻦ ‪.‬‬
‫أو ‪ :‬اﻟﺰﻣﻦ اﻟﺬي ﻳﻤﺜّﻞ ‪ 20 %‬ﻣﻦ ﻣﺪّة اﻟﺸﺤﻦ ‪.‬‬

‫‪GUEZOURI A.‬‬
‫‪Lycée Maraval - Oran‬‬

‫‪4‬‬
‫اﻟﺘﻤﺮﻳﻦ اﻟﺘﺠﺮﻳﺒﻲ )‪ 4‬ﻧﻘﻂ(‬
‫‪ - 1‬ﺟﺪول اﻟﺘﻘﺪّم ‪ :‬ﻟﺪﻳﻨﺎ ‪n (H2O2) = [H2O2] 0 × VS = 8 × 10–2 × 0,5 = 4 × 10–2 mol‬‬
‫‪2 H2O2‬‬ ‫=‬ ‫‪2 H2O‬‬ ‫‪+ O2‬‬
‫‪t=0‬‬ ‫‪4 × 10–2‬‬ ‫زﻳﺎدة‬ ‫‪0‬‬
‫اﻟﺤﺎﻟﺔ اﻻﻧﺘﻘﺎﻟﻴﺔ‬ ‫‪4 × 10−2 − 2x‬‬ ‫زﻳﺎدة‬ ‫‪x‬‬

‫اﻟﺤﺎﻟﺔ اﻟﻨﻬﺎﺋﻴﺔ‬ ‫‪4 ×10−2 − 2x f‬‬ ‫زﻳﺎدة‬ ‫‪xf‬‬

‫)‪(1‬‬ ‫‪ - 2‬ﻓﻲ اﻟﻠﺤﻈﺔ ‪ t‬ﻳﻜﻮن ﻋﺪد ﻣﻮﻻت ‪ H2O2‬هﻮ ‪n ( H 2O 2 ) = [ H 2O 2 ]0 × V S − 2 x‬‬

‫‪V O2‬‬
‫‪ ،‬وﺑﺎﻟﺘﻌﻮﻳﺾ ﻓﻲ )‪: (1‬‬ ‫= ) ‪n (O 2‬‬ ‫‪=x‬‬ ‫وﻳﻜﻮن ﻋﺪد ﻣﻮﻻت ﺛﻨﺎﺋﻲ اﻷآﺴﺠﻴﻦ‬
‫‪VM‬‬
‫‪VO‬‬
‫)‪(2‬‬ ‫‪n ( H 2O 2 ) = [ H 2O 2 ]0 × V S − 2 × 2‬‬
‫‪VM‬‬
‫‪V O2‬‬
‫× ‪ [ H 2O 2 ] ×V S = [ H 2O 2 ]0 × V S − 2‬وﺑﻘﺴﻤﺔ‬ ‫‪ ، n ( H 2O 2 ) = [ H 2O 2 ] ×V S‬وﺑﺎﻟﺘﻌﻮﻳﺾ ﻓﻲ )‪ (2‬ﻧﺠﺪ‬ ‫وﻟﺪﻳﻨﺎ‬
‫‪VM‬‬

‫‪2 V O2‬‬
‫)‪(3‬‬ ‫‪[ H 2O 2 ] = [ H 2O 2 ]0 −‬‬ ‫×‬ ‫اﻟﻄﺮﻓﻴﻦ ﻋﻠﻰ ‪ VS‬ﻧﺠﺪ ‪:‬‬
‫‪VS VM‬‬
‫‪V O2‬‬
‫‪ ،‬ﺣﻴﺚ ﻧﺴﺘﻌﻤﻞ هﺬﻩ اﻟﻌﻼﻗﺔ ﻹﺗﻤﺎم‬ ‫‪[ H 2O 2 ] = 0 , 08 −‬‬ ‫‪ - 3‬إﺗﻤـﺎم اﻟﺠﺪول ‪ :‬ﺑﺎﻟﺘﻌﻮﻳﺾ اﻟﻌﺪدي ﻓﻲ اﻟﻌﻼﻗﺔ )‪ (3‬ﻧﻜﺘﺐ ‪:‬‬
‫‪6‬‬
‫اﻟﺠﺪول ‪.‬‬
‫)‪t (mn‬‬ ‫‪0‬‬ ‫‪4‬‬ ‫‪8‬‬ ‫‪12‬‬ ‫‪16‬‬ ‫‪20‬‬ ‫‪24‬‬ ‫‪28‬‬ ‫‪32‬‬ ‫‪36‬‬ ‫‪40‬‬

‫) ‪V O 2 ( mL‬‬ ‫‪0‬‬ ‫‪60‬‬ ‫‪114 162 204 234 253 276 288 294 300‬‬

‫‪[¨ H 2O 2 ] ( mol / L ) ×10−2‬‬ ‫‪8,0 7,0‬‬ ‫‪6,1‬‬ ‫‪5,3‬‬ ‫‪4,6‬‬ ‫‪4,1‬‬ ‫‪3,8‬‬ ‫‪3,4‬‬ ‫‪3,2‬‬ ‫‪3,1‬‬ ‫‪3,0‬‬

‫) ‪[ H 2O 2 ] ( mol / L‬‬ ‫ب( اﻟﺘﻤﺜﻴﻞ اﻟﺒﻴﺎﻧﻲ‬

‫• ‪0,05‬‬
‫‪0,04‬‬

‫•‬ ‫)‪t (mn‬‬


‫‪10‬‬ ‫‪20‬‬
‫‪5‬‬
‫‪1 dx‬‬
‫= ‪v‬‬ ‫×‬ ‫ﺟـ( ﻋﺒﺎرة اﻟﺴﺮﻋﺔ اﻟﺤﺠﻤﻴﺔ ﻟﻠﺘﻔﺎﻋﻞ‬
‫‪V S dt‬‬
‫د( ﻧﻌﺒّﺮ ﻋﻦ اﻟﺴﺮﻋﺔ اﻟﺤﺠﻤﻴﺔ ﻟﻠﺘﻔﺎﻋﻞ ﺑﺪﻻﻟﺔ ﺳﺮﻋﺔ إﺧﺘﻔﺎء ‪H2O2‬‬
‫‪x‬‬
‫‪ ،‬وﺑﺎﺷﺘﻘﺎق اﻟﻄﺮﻓﻴﻦ ﺑﺎﻟﻨﺴﺒﺔ ﻟﻠﺰﻣﻦ ﻧﺠﺪ ‪:‬‬ ‫× ‪[ H 2O 2 ] = [ H 2O 2 ]0 − 2‬‬ ‫ﻟﺪﻳﻨﺎ ﻣﻦ اﻟﻌﻼﻗﺔ )‪: (3‬‬
‫‪VS‬‬

‫] ‪1 d [ H 2O 2‬‬ ‫] ‪d [ H 2O 2‬‬ ‫‪2 dx‬‬


‫× ‪v =−‬‬ ‫‪ ،‬وﺑﺎﻟﺘﺎﻟﻲ‬ ‫× ‪=−‬‬ ‫‪= −2 v‬‬
‫‪GUEZOURI A.‬‬ ‫‪2‬‬ ‫‪dt‬‬ ‫‪dt‬‬ ‫‪V S dt‬‬
‫‪Lycée Maraval - Oran‬‬
‫] ‪d [ H 2O 2‬‬
‫ﻳﻤﺜﻞ ﻣﻴﻞ اﻟﻤﻤﺎس ﻓﻲ اﻟﻠﺤﻈﺔ ‪. t‬‬ ‫ﺑﺤﻴﺚ أن‬
‫‪dt‬‬
‫⎞ ‪1 ⎛ 9 × 0 , 005‬‬ ‫‪−4‬‬ ‫‪−1‬‬
‫‪v 16 = − ⎜ −‬‬ ‫‪⎟ ≈ 8 ×10 mol / L. mn : t1 = 16 mn‬‬ ‫اﻟﺴﺮﻋﺔ ﻓﻲ اﻟﻠﺤﻈﺔ‬
‫⎝‪2‬‬ ‫⎠ ‪14 × 2‬‬

‫) ‪[ H 2O 2 ] ( mol / L‬‬

‫•‬
‫• ‪0,05‬‬

‫•‬ ‫•‬

‫•‬ ‫•‬ ‫)‪t (mn‬‬


‫‪10‬‬ ‫‪16‬‬

‫⎞ ‪1 ⎛ 5, 5 × 0 , 005‬‬ ‫‪−4‬‬ ‫‪−1‬‬


‫‪v 24 = − ⎜ −‬‬ ‫‪⎟ ≈ 4 , 3 ×10 mol / L. mn‬‬ ‫‪: t2 = 24 mn‬‬ ‫اﻟﺴﺮﻋﺔ ﻓﻲ اﻟﻠﺤﻈﺔ‬
‫⎝‪2‬‬ ‫⎠ ‪16 × 2‬‬

‫) ‪[ H 2O 2 ] ( mol / L‬‬

‫• ‪0,05‬‬ ‫•‬

‫•‬ ‫•‬

‫•‬ ‫•‬ ‫)‪t (mn‬‬


‫‪10‬‬ ‫‪24‬‬

‫‪6‬‬
‫ﻧﻼﺣﻆ أن اﻟﺴﺮﻋﺔ ﺗﺘﻨﺎﻗﺺ ﻣﻊ ﻣﺮور اﻟﺰﻣﻦ ‪.‬‬
‫‪x max‬‬
‫‪ ،‬وﻟﺤﺴﺎب ﻗﻴﻤﺔ اﻟﺘﻘﺪم اﻷﻋﻈﻤﻲ ﻧﻀﻊ ‪ ، 4 ×10−2 − 2x max = 0‬وﻣﻨﻪ ‪:‬‬ ‫هـ( زﻣﻦ ﻧﺼﻒ اﻟﺘﻔﺎﻋﻞ ﻳﻮاﻓﻖ‬
‫‪2‬‬
‫‪x max‬‬
‫‪ x max = 2 ×10−2 mol‬و ‪= 10−2 mol‬‬
‫‪2‬‬
‫‪x max‬‬
‫‪10−2‬‬
‫× ‪[ H 2O 2 ] = [ H 2O 2 ]0 − 2 × 2 = 0 , 08 − 2‬‬ ‫ﻧﺤﺴﺐ اﻟﺘﺮآﻴﺰ اﻟﻤﻮﻟﻲ ﻟـ ‪ H2O2‬اﻟﻤﻮاﻓﻖ ‪= 4 × 10−2 mol / L :‬‬
‫‪VS‬‬ ‫‪0, 5‬‬

‫ﻧﺴﺘﻨﺘﺞ زﻣﻦ ﻧﺼﻒ اﻟﺘﻔﺎﻋﻞ ﻣﻦ اﻟﺒﻴﺎن ‪) t1 / 2 ≈ 20 mn‬اﻧﻈﺮ ﻟﻠﺒﻴﺎن أﻋﻼﻩ( ‪.‬‬

‫‪ - 4‬ﻣﻦ أﺟﻞ درﺟﺔ اﻟﺤﺮارة ‪ θ ' = 35°C > 12°C‬ﻳﺼﻞ اﻟﺘﻔﺎﻋﻞ إﻟﻰ ﻧﻬﺎﻳﺘﻪ ﻓﻲ ﻣﺪة أﻗﻞ ﻣﻦ اﻟﺴﺎﺑﻖ ﻷن اﻟﺤﺮارة ﺗﻨﺸﻂ اﻟﺘﻔﺎﻋﻞ ‪.‬‬

‫) ‪[ H 2O 2 ] ( mol / L‬‬

‫• ‪0,05‬‬

‫•‬ ‫)‪t (mn‬‬


‫‪10‬‬

‫‪GUEZOURI A.‬‬
‫‪Lycée Maraval - Oran‬‬

‫‪7‬‬
‫‪-‬‬ ‫ ر ‪ - 2008‬ا ـم ا ‬
‫‪.! *1/3‬م ا ‪ */%12‬وا &‪%‬ة‬ ‫‪-‬‬ ‫('&‪ )!"# $%‬ا ‪./‬م ا ‪*%+,%-‬‬
‫ا‪
8‬ذ !‪ 1‬ا در ‪,5‬وري – ;* ‪#‬ـرا‪:‬ل ‪ -‬وهان‬

‫اــع ا ــ

‫ا
 اول )‪( 4‬‬
‫‪ - 1‬أ( ا
 د 
رة )‪ ...‬ر ت ‪ β −‬وإ ت ‪ (.. γ‬ه ‪ ! "#ّ#%‬ا
 ‪ ، β −‬أي إ'ار إ
‪/0#‬ون ‪ ,-‬ا
 اة وإ‪+‬ء‬
‫‪ 1‬اة ا ‪! 56 ,‬ـ
‪3- 4‬ـرة ‪.‬‬
‫‪ 8‬إ'ار ا
 اة
‪ 7‬ت ‪ γ‬ه أن دة ا
 اة ا‪ # , 9‬ن ‪3- 4
! 56‬ـرة ‪ ،‬و ‪':‬اره
‪ 7‬ت ‪ ,- >?@0 γ‬ا
‪ 4=+‬ا
<ا;ة‬

‪ C00‬إ
‪ "0
! B‬ا‪8A‬ــ‪. 48‬‬

‫‪56 Ba + γ‬‬
‫‪→ 137‬‬ ‫‪→ −10e + 137‬‬
‫‪137‬‬ ‫*‬ ‫‪137‬‬ ‫*‬
‫‪56 Ba‬‬ ‫‪EF‬‬ ‫‪55Cs‬‬ ‫ب( ‪56 Ba‬‬

‫‪m‬‬ ‫‪10 −6‬‬


‫× ‪N0 =NA‬‬ ‫× ‪= 6 , 023 ×10 23‬‬ ‫‪= 4 , 39 ×1015‬‬ ‫د ا‪ 4G 1A‬ه ‬ ‫أ(‬ ‫‪-2‬‬
‫‪M‬‬ ‫‪137‬‬
‫)‪(1‬‬ ‫ب( ‪A 0 = λ N 0‬‬

‫‪1‬‬ ‫‪1‬‬
‫‪ ،‬و 
‪L1 (1) 56 KG 0‬‬ ‫=‪λ‬‬ ‫=‬ ‫و
‪ G‬ا
‪ J 3‬ا‪= 7 , 32 × 10−10 s −1 5 I‬‬
‫‪τ‬‬ ‫‪43, 3 × 365 × 24 × 3600‬‬

‫‪A 0 = 7 , 3 ×10−10 × 4 , 39 ×1015 = 3, 2 × 106 Bq‬‬


‫‪0 ,69‬‬
‫‪−‬‬ ‫‪× 0 ,5‬‬
‫‪A = A 0 e −λt = 2 , 22 × 106 e‬‬ ‫‪43 ,3‬‬
‫‪= 3,16 ×106 Bq‬‬ ‫‪ - 3‬أ(‬

‫‪A‬‬ ‫‪3,16 ×106‬‬


‫= ‪N‬‬ ‫=‬ ‫ب( ‪ M1‬د ا‪ 6  4G 1A‬أ"‪= 4 , 32 × 1015 : /‬‬
‫‪λ‬‬ ‫‪7 , 32 × 10−10‬‬

‫‪7 × 1013‬‬
‫د ا‪ 4G 1A‬ا
‪ ، ∆N = ( 4 , 39 − 4 , 32 ) × 1015 = 7 , 0 ×1013 4##%0‬و‪ 41‬ا‪ 4G 1A‬ا
‪ 4##%0‬ه‪= 0 , 016 5‬‬
‫‪4 , 39 × 10‬‬ ‫‪15‬‬

‫أي ! ا
‪ ,- 1,6 % 5‬ا‪. J#ّ#% = 4G 1A‬‬
‫‪− λt‬‬
‫)‪( 2‬‬ ‫‪A1 = A 0 e‬‬ ‫‪1‬‬
‫‪ 56 G
- 4‬ا
?‪ t1 4PM‬ا
‪R‬ط ه ‬
‫‪− λt‬‬
‫)‪( 3‬‬ ‫‪A2 = A0 e‬‬ ‫‪2‬‬ ‫و‪ 4PM
56‬ه ‪ #G t2‬ن ا
‪R‬ط‬

‫‪A‬‬ ‫) ‪λ (t − t‬‬ ‫‪1‬‬


‫‪ ،‬و‪λ (t 2 − t1 ) = ln 100 X-‬‬ ‫= ‪ ، A 2‬و ‪/+
6/W (3) B? (2) E0‬ف ‪= e 2 1 L1‬‬ ‫!‪A1 U‬‬
‫‪0 , 01 A‬‬ ‫‪100‬‬
‫‪∆t = t 2 − t 1 U! ، ∆t = 4 , 6 × τ‬‬ ‫و 
‪5
0‬‬
‫‪∆t = 4 , 6 × 43, 3 = 200 ans‬‬
‫ه‪ YZ‬ا
‪ B? "  ,#G 4L0‬ا‪ 4G 1A‬ا
‪ 4 R‬ا
‪ ]%1 56 /F#0 9 50‬ا
=‪R1 C % J‬ط أ‪ 4G 1‬أ\‪/‬ى ‪.‬‬

‫‪GUEZOURI A.‬‬
‫‪Lycée Maraval - Oran‬‬

‫‪1‬‬
( 4) )> ‫ا
 ا‬
. 4/
‫` ا‬- G‫د‬/W 80 ‫ك‬#0!I‫ = ة ا‬..... :>
‫_ ا‬6‫  ا‬f = k v (1) 4=^
‫ ا‬- 1

. 4/
‫ ` ا‬/- `- G‫د‬/W 80 ‫ك‬#0!I‫ = ة ا‬..... :>
‫_ ا‬6‫  ا‬f = k ' v 2 (2) 4=^
‫ا‬
: , 
513
‫ ن ا‬1
‫_ ا‬+0 (‫ أ‬- 2
: Oz 5
=R
‫ ر ا‬M
‫ ا‬B? 4=^
‫ ا‬YZ‫ط ه‬8: ‫ و‬، P + f + Π = m a
   
O 
f P− f −Π = m a

Π dv

mg − k v − ρ 0 VS g = m
dt
VS 1 dv k  V 
(41

‫ ا‬EL! : VS) = G
‫ و‬، + v = g 1 − ρ0 S 
m ρ dt m  m 

dv k  ρ 
+ v = g 1 − 0  : 4?b%0
‫ ا‬4
‫ ا
 د‬c‫و‬

P dt m  ρ 
z
dv
(1) + B v = A C#R
‫ ا‬B? 4
‫ ا
 د‬0#1 (‫ب‬
dt
. (4;‫ا‬0 ‫ ا‬4/8 ‫ ون‬EL
‫<ول ا‬1) v = 0 ‫ ن‬# t = 0 4PM?
‫ ا‬56 : 
‫ـ(   ر ا‬
. (E0- G/ 4/
‫ ا‬+@- ‫ن‬A ‫م‬P01 4‫ر‬0- 4‫آ‬/!) ‫م‬P01 4/
‫ ر ا‬+0 [ 0 , 0,2 s ] 5-<
‫ل ا‬L
‫ ا‬56
. ( ‫ة‬/ّi0- 4‫آ‬/!) J F /g ‫رع‬0
‫ [ ا‬0,2 , 0,9 s ] 5-<
‫ل ا‬L
‫ ا‬56
( 4P0- 4‫آ‬/M
‫ ) ا‬v l = 2 , 5m / s 4GّM
‫ ا‬4
‫ ا‬5‫" ه‬0=‫ و‬، 40 F 4/
‫ ا‬: t > 0 , 9s C‫ أ‬,-

 ρ   1, 3 
A = g 1 − 0  = 10 × 1 −  = 6 , 83 m / s (‫د‬
2

 ρ   4 , 1 
A 6 , 83 dv
B = = = 2 , 73 s −1 5
0
 ‫ و‬، = 0 ، 40 F 4/
‫ن ا‬A ‫ وم‬- ‫رع‬0
‫ ن ا‬#G E;‫م ا
ا‬P
‫ ا‬56
vl 2, 5 dt
‫ة‬/i
‫ ا‬E
‫ ا‬C‫ أ‬,- CL0
‫ ا‬1 `- _+G (5
‫و‬A‫ ا‬4b/%
‫ ا‬C‫ أ‬,- ‫ )أي‬B ‫ و‬A 50= 4+8‫ م ا‬8/
‫ أن ا
ـن ا‬j!^1 - 3
‫ت‬/
‫ ا‬C‫ أ‬,- 4b/%
‫ ا‬C0@ EF ، v ∈ [ 0 ; 1m / s ] C‫ أ‬,- ‫ أي‬، [ 0 , 0,2 s ] ‫ل‬L
‫ ا‬56 ‫ا‬Z‫ر ه‬0‫ ا‬,#G‫ و‬، EL
‫ ا‬4/

. 1 ‫ة‬/#
‫ا‬
( 4) ? > ‫ا
 ا‬
Y 1 −u uCB Y2 : 5+"
‫<از ا‬0‫ه‬9‫ ا‬E8‫ ر  را‬4G/W - 1
BA

uCB ‫ه‬R1 Y2 C\


‫ ا‬56
R (L , r)
−u BA ‫ أي‬، u AB ‫ه‬R1 Y1 C\
‫ ا‬56
A• • •C
B
. u BA ‫هة‬R
‫رة‬I‫ زر آ] ا‬B? il
‫ ا‬,#G
i
u BA = 2 × 5 = 10V ‫ ا
ن‬,- m001 E;‫م ا
ا‬P
‫ ا‬56 (‫ أ‬- 2
K
E
uCB = 12 − 10 = 2 V X-‫ و‬، u BA + uCB = E G
(‫ب‬
u BA 10
I = = = 1 A X-‫ و‬، u BA = R I (‫ـ‬
R 10

2
63
u BA × ‫_
ـ‬6‫ ا
 ا‬,-<
‫ أو ا‬، τ = 2 ms m001 t = 0  ‫ ا
س
?ن‬4G/W 4+8‫ أ( ا‬- 3
100
E 12
r= − R = − 10 = 2 Ω X-‫ و‬، E = ( R + r ) I : 
‫ب( و ا‬
I 1
L
L = τ × ( R + r ) = 2 × 10−3 × 12 = 24 × 10 −3 H X-‫ و‬، τ = : 
‫ذا ا‬
R +r
L = 24 mH
1 2
EL = LI = 0 , 5 × 0 , 024 × 12 = 1, 2 × 10 −2 J 4 
‫ ا‬56 41<@
‫ ا‬4PA‫ ا‬4=+
‫ ا‬- 4
2
( 4) @‫ا
 ا ا‬
HA(aq) + (Na+ , OH–)(aq) → H2O(l) + (Na+ , A–)(aq) : ‫ة‬/G 
‫ ا‬C% - 1
(C#R
‫ ا
"!
 او
 )ا‬#$
‫ ا‬- 2
‫^ب‬1‫هة ا‬R 
cG 9 K A‫ ا‬X1 ? ?M
‫ أن ا‬E? 1 - 3
‫? ل‬M ‫? ءة‬- 4‫ر‬- 4!M8
‫ م‬G‫هروآ ا
 د‬ (Y‫ّد‬1) ‫ ا
ء‬X
ol1 ‫ا‬Z"
، p6#0
‫ ا‬4+1  o#

ن ا‬
. ‫^ب ا
? ن‬1‫ ر' ا‬,#G 5
0
 ‫ و‬، ‫ول‬A‫ ا‬,- /3‫ أآ‬6% cG B0!
p6#0
‫ وأن  ا‬، G0
 /i0G 9 KM
‫ت ا‬9 - ‫ أن د‬E? 1

EL! ]%1 C 08 ‫ إذن‬، n ( HA ) = n OH − ( ) ‫ ن‬#G

. KM
‫د ا‬1 E
‫ أم‬1‫ّد‬- ‫ اء‬8 588A‫? ل ا‬M
‫ا‬
‫س‬-
pH
pH . G0
‫ ا‬4
! 56 C=‫ ن أ‬# p6#0
‫  ا‬pH 4= ,#

EL! 4!1 ,- ]
‫ و‬pH ‫ ا
ـ‬4!1 ,- /Fr0 p6#0
‫ ا‬4+1 ‫إذن‬
. p6#0
‫ف  ا‬l
‫ ا‬58‫ـ‬8A‫? ل ا‬M
‫ا‬

?M
‫ ا‬B? ‫ ي‬0MG /R
: %‫ ا
)
!
*)( ا
'آ‬+‫ ا
آ‬- 4
: 
‫ا
 او‬
‫ك ا
@^ط‬/M-
5Wi
‫ا‬ X-‫ و‬، C A 1 V A 1 = C B V BE : p6#0
‫ ا‬

C B V BE 5 × 10−2 × 12
GUEZOURI A. C A1 = = = 3, 0 × 10−2 mol / L
Lycée Maraval - Oran V A1 20
: ,-
‫ا
 ا‬

C B V ' BE 5 × 10−2 × 12 , 9
5

‫آ< ا‬/0
‫ ا‬-‫ أ‬، ‫ ا
ّد‬KM?
5

‫آ< ا‬/0
‫ا ا‬Z‫ ه‬، C A 2 = = = 0 , 32 × 10−2 mol / L
VA2 200

C ' A 2 = 0 , 32 × 10−2 × 10 = 3, 2 × 10−2 mol / L "6 C 0


‫ ا‬KM?

. ‫"^ك‬08^
c
' /g ?M
‫ا ا‬Z‫ن ه‬:6 ، X ‫آ< ا
 ح‬/0
‫ ا‬,- /‫ أآ‬Y1‫ و‬,?
‫ ا‬KM
5

‫آ< ا‬/0
‫  أن ا‬
B? 0  413
‫ ا‬4 /L0
‫ ا‬-‫ أ‬، (pH ‫س ا
ـ‬-) 4= p6#0
‫ ا‬4+1 GM ,#G B
‫و‬A‫ ا‬56 ‫ن‬A ، 413
‫ ا‬4 /L0
‫ ا‬,- ‫ق‬
ّ ‫ أد‬B
‫و‬A‫ ا‬4 /L0
‫ ا‬- 5
‫? ل‬M
‫ ا‬EL! ‫ ن‬#G 5
0
 ‫ و‬، ‫ل‬L
‫ا ا‬Z‫ ه‬56 ‫ه‬/! 6 ,#G C ، 4= p6#0
‫ ا‬4+1 GM ,#G 9 ، o#
‫^ب
ن ا‬1‫ل ا‬L- 4G‫رؤ‬
. _=‫ د‬/g p6#0
‫ف  ا‬l
‫ ا‬588A‫ا‬

3
( 4) )1
‫ا
 ا‬
(C#R
‫ ا
 )ا‬/" - 1
,#G U! ، ‫ ب 
<ول‬1A‫ ا‬56 ‫ ى ا
ء‬0- ‫أ‬G ‫ز‬i
‫^ق ا‬+1‫ و ا‬، ‫? ء 
ء‬- ‫ ! ض‬B? X#1‫رّج 
ء و‬- ‫ر‬0\‫ ب ا‬1‫ أ‬v1
‫ ا
ء‬,- XiG/%  4 /L0
‫ ا‬4G"1 56 ‫ز‬i
‫ ا‬, oR#
‫ ا‬,#G . ‫ ب‬1A‫ ا‬56 ‫ ى ا
ء‬0- 4LG‫اءة ر‬/ ‫ز‬i
‫ ا‬EL! ‫ =س‬4PM
C‫ آ‬56
. (,‫ ا
"رو‬5;F ‫ز‬g ‫<ات‬- ,-) 4 =/6 ‫ث‬M6 X0‫ ه‬6 ,- JG/‫  د آ‬G/‫ و‬X6 5=
‫ا‬

Mg (S ) = Mg (2a+q) + 2 e − : ‫آة‬A‫ ا‬- 2


H2
2H +(aq ) + 2 e − = H 2 ( g ) : ‫رــع‬I‫ا‬
H2 ‫ور‬/-
‫آة – إرـع‬A‫ ا‬4
‫ د‬-
Mg(S) + 2 H+(aq) → Mg2+(aq) + H2(g)

(H3O+ , Cl–) ‫ء‬-

Mg
: ‫ّم‬1
‫ول ا‬12 (‫ أ‬- 3
0 , 036
n ( Mg ) = = 1, 5 × 10−3 mol ‫ م‬G<i

دة ا‬4;‫ا‬0 9‫ ا‬4#
‫ا‬
24
Mg (s) + 2 H+ (aq) → Mg2+ (aq) + H2 (g)
t=0 1,5 × 10–3 n0 (H+) 0 0

‫ ل ا‬ 1, 5 × 10−3 − x ( )
n0 H + − 2 x x x

‫ ا‬ 1, 5 × 10−3 − x f n0 (H ) − 2 x
+
f xf xf

. ‫ م‬G<i
‫ ه ا‬M
‫ ا‬C%0
‫ف أن ا‬/ 1 5#
6 ‫دة‬G< ,‫ آ? ر ا
"رو‬K! ‫
 أن‬C= ,G/0
‫ ا‬56 : 56'
V H2
x = n (H 2 ) = ، 4PM
C‫ آ‬56 ,‫ ا
"رو‬5;F ‫دة‬- 4‫ آ‬5‫ ه‬C%0
‫ م ا‬E=
VM
: ‫ول‬1
‫ب( إ)م ا‬

t (mn) 0 2 4 6 8 10 12 14 16 18
V (H2) (mL) 0 12,0 19,2 25,2 28,8 32,4 34,8 36,0 37,2 37,2
x (mol) × 10–4 0 5,0 8,0 10,5 12,0 13,5 14,5 15,0 15,5 15,5

GUEZOURI A.
Lycée Maraval - Oran

4
x = f (t ) : !,8
‫ ا‬9-)
‫ا‬
x (mol) × 10–4

t ( mn )
5

dx 17 × 10−4
v = = = 2 , 83 mol .mn −1 t = 0  ‫ ا
س‬C- Cّ3 : t = 0  C%0
‫ ا‬4/8 (‫ـ‬
dt 6
. B"01‫ إ‬C%0
‫ن ا‬A 5;"
‫م ا‬0
‫ ا‬X%1 ‫ي ه‬Z
‫ وا‬، 5PA‫م ا‬0
‫ ا‬M1 - 4

(14 mn 56 C%0
‫ ا‬5"0G ‫وض‬/%
‫ ا‬,-) . x max = 1, 5 × 10−3 mol X-‫ و‬، 1, 5 × 10−3 − x max = 0

. (4=^
‫_ ا‬+0 c 4‫ در‬B
‫ّدا إ‬- ]
‫? ل‬M
‫ أن ا‬Eg‫ ) ر‬ H +  = 10−1 mol / L C%0
‫ ا‬4G"1 56 G

( aq )

( )
n f H + =  H +  ×V = 0 ,1 × 30 × 10−3 = 3 × 10−3 mol
( aq )
: 5‫ ه‬C%0
‫ ا‬4G"1 56 H+(aq) ‫دة‬- 4‫آ‬

( ) ( )
n 0 H + = 2 × 1, 5 × 10−3 + 3 ×10 −3 = 6 ,0 × 10 −3 mol : X-‫ و‬، n 0 H + − 2 x f = 3 ×10 −3 : ‫م‬0
‫ ول ا‬,- G

H  + ( )=
n0 H + 6 × 10−3
  (a q) = V 30 × 10−3
= 0 , 2mol / L ‫ ه‬,‫ آ? ر ا
"رو‬K! ‫? ل‬M
5;‫ا‬0 9‫ ا‬5

‫آ< ا‬/0
‫ا‬

GUEZOURI A.
Lycée Maraval - Oran

You might also like